Download as pdf or txt
Download as pdf or txt
You are on page 1of 696

Table of Contents

February/March 2023 Variant 2 1


May/June 2023 Variant 1 17
May/June 2023 Variant 2 32
May/June 2023 Variant 3 47

February/March 2022 Variant 2 61


May/June 2022 Variant 1 76
May/June 2022 Variant 2 90
May/June 2022 Variant 3 105
October/November 2022 Variant 1 119
October/November 2022 Variant 2 133
October/November 2022 Variant 3 147

February/March 2021 Variant 2 161


May/June 2021 Variant 1 177
May/June 2021 Variant 2 192
May/June 2021 Variant 3 206
October/November 2021 Variant 1 219
October/November 2021 Variant 2 233
October/November 2021 Variant 3 248

February/March 2020 Variant 2 262


May/June 2020 Variant 1 276
May/June 2020 Variant 2 289
October/November 2020 Variant 1 304
October/November 2020 Variant 2 318
October/November 2020 Variant 3 332
February/March 2019 Variant 2 348
May/June 2019 Variant 2 361
May/June 2019 Variant 1 377
May/June 2019 Variant 2 391
October/November 2019 Variant 3 404
October/November 2019 Variant 1 418
October/November 2019 Variant 2 433
For Examination from 2019 449

February/March 2018 Variant 2 467


May/June 2018 Variant 1 481
May/June 2018 Variant 2 496
May/June 2018 Variant 3 512
October/November 2018 Variant 1 527
October/November 2018 Variant 2 540
October/November 2018 Variant 3 554

February/March 2017 Variant 2 570


May/June 2017 Variant 1 584
May/June 2017 Variant 2 600
May/June 2017 Variant 3 615
October/November 2017 Variant 1 630
October/November 2017 Variant 2 645
October/November 2017 Variant 3 661
For Examination from 2017 677
Combined By Nesrine
2023-2017

Cambridge IGCSE™

COMBINED SCIENCE 0653/22


Paper 2 Multiple Choice (Extended) February/March 2023
45 minutes

You must answer on the multiple choice answer sheet.


*9621705648*

You will need: Multiple choice answer sheet


Soft clean eraser
Soft pencil (type B or HB is recommended)

INSTRUCTIONS
• There are forty questions on this paper. Answer all questions.
• For each question there are four possible answers A, B, C and D. Choose the one you consider correct
and record your choice in soft pencil on the multiple choice answer sheet.
• Follow the instructions on the multiple choice answer sheet.
• Write in soft pencil.
• Write your name, centre number and candidate number on the multiple choice answer sheet in the
spaces provided unless this has been done for you.
• Do not use correction fluid.
• Do not write on any bar codes.
• You may use a calculator.

INFORMATION
• The total mark for this paper is 40.
• Each correct answer will score one mark.
• Any rough working should be done on this question paper.
• The Periodic Table is printed in the question paper.

This document has 16 pages.

IB23 03_0653_22/2RP
© UCLES 2023 [Turn over

1/693
Combined By Nesrine
2
2023-2017

1 Two pieces of potato are cut to have exactly the same mass and shape. The mass is measured
and recorded.

concentrated
water salt solution

potato piece potato piece

One piece of potato is placed in water and the other piece is placed in concentrated salt solution.

They are both left for one hour.

The mass of each piece of potato is then measured again.

What happens to the mass of each piece of potato?

mass of potato
mass of potato
placed in concentrated
placed in water
salt solution

A decreases decreases
B decreases increases
C increases decreases
D increases increases

2 Under which conditions is an enzyme described as being denatured?

A when it is cooled to a low temperature


B when it is turned into a dry powder for storage
C when it is used to catalyse a different reaction
D when the shape of the active site is permanently changed

3 Which two nutrients are needed for healthy bone and tooth development?

A calcium and iron


B fibre and vitamin C
C fibre and vitamin D
D vitamin D and calcium

© UCLES 2023 0653/22/F/M/23

2/693
Combined By Nesrine
3
2023-2017

4 The diagram shows some organs of the human body.

1
3
2

Which two structures carry out both mechanical and chemical digestion?

A 1 and 2 B 1 and 4 C 2 and 3 D 3 and 4

5 Which statement describes a feature of the root hair cells of plants?

A They help the roots to move between soil particles in the ground.
B They have fully permeable cell membranes to improve nitrate ion entry.
C They have large surface areas to increase water uptake by osmosis.
D They have partially permeable cell walls to give the cells more strength.

6 Which statement about all arteries is correct?

A They always contain oxygenated blood.


B They have many valves on their inner walls.
C They have a wide lumen.
D They transport blood away from the heart.

© UCLES 2023 0653/22/F/M/23 [Turn over

3/693
Combined By Nesrine
4
2023-2017

7 The table shows the differences in the composition of carbon dioxide, oxygen and water vapour
for inspired and expired air.

Which row shows the most likely composition for human inspired and expired air?

carbon dioxide % oxygen % water vapour %


inspired expired inspired expired inspired expired

A 4 0.04 21 16 1 1
B 0.04 4 21 16 1 6
C 0.04 4 16 21 6 1
D 4 0.04 16 21 6 6

8 Adrenaline is produced by the adrenal glands in times of ......1...... . It targets vital organs,
increases ......2...... which increases the delivery of oxygen and ......3...... to the brain and
muscles, preparing the body for 'flight or fight'.

Which words complete gaps 1, 2 and 3?

1 2 3

A stress heart rate glucose


B stress breathing rate glycogen
C relaxation breathing rate glucose
D relaxation heart rate glycogen

9 Which chemical is involved in controlling the growth of plant shoots?

A amylase
B auxin
C water
D protease

© UCLES 2023 0653/22/F/M/23

4/693
Combined By Nesrine
5
2023-2017

10 The diagram shows a section through a flower.

P Q

Which row identifies the labelled parts of the flower?

P Q R

A anther ovary stigma


B anther stigma ovary
C stamen carpel sepal
D stamen sepal carpel

11 Where does fertilisation take place?

B
C

12 A woodland consists of trees, other plants and animals.

Which term describes this woodland and the interaction of its living things with each other and
their environment?

A ecosystem
B food web
C habitat
D trophic level

© UCLES 2023 0653/22/F/M/23 [Turn over

5/693
Combined By Nesrine
6
2023-2017

13 The concentration of nitrate ions in a lake increases.

Why does this result in a decrease in the number of fish in the lake?

A There is a decrease in the decomposition of producers.


B There is a decrease in the growth of producers.
C There is an increase in aerobic respiration by decomposers.
D There is an increase in dissolved oxygen.

14 The structures of three molecules are shown.

water ethanol methane

How many atoms are in each molecule?

water ethanol methane

A 2 3 2
B 2 4 5
C 3 3 2
D 3 9 5

15 Which elements react together to give positive ions and negative ions that all have the same
electronic structure as argon?

A calcium and chlorine


B calcium and fluorine
C magnesium and chlorine
D magnesium and fluorine

© UCLES 2023 0653/22/F/M/23

6/693
Combined By Nesrine
7
2023-2017

16 The diagram shows an experiment to electrolyse dilute sulfuric acid.

negative positive
electrode electrode

Which statement explains why bubbles form at the positive electrode?

A Oxide ions lose electrons to form oxygen.


B Hydroxide ions lose electrons to form hydrogen.
C Hydroxide ions lose electrons to form oxygen and water.
D Hydrogen ions gain electrons to form hydrogen.

17 The reaction between aqueous lead(II) nitrate and dilute sulfuric acid produces insoluble
lead(II) sulfate.

Which ions do not change state in this reaction?

A NO3– and H+
B NO3– and SO42–
C Pb2+ and H+
D Pb2+ and SO42–

18 The energy level diagram for an exothermic reaction is shown.

Which arrow represents the activation energy for this reaction?

A
energy C D
B

progress of reaction

© UCLES 2023 0653/22/F/M/23 [Turn over

7/693
Combined By Nesrine
8
2023-2017

19 Which statement explains the effect of temperature on the rate of a reaction?

A At a higher temperature, more particles have sufficient energy to overcome the activation
energy.
B At a higher temperature, the particles collide less frequently.
C At a lower temperature, the particles collide with more energy and so more bonds are
broken.
D At a lower temperature, the particles have a lower concentration.

20 The equation for one of the reactions that occurs in a blast furnace is shown.

Fe2O3 + 3CO → 2Fe + 3CO2

Which row identifies the oxidising agent in this reaction and explains its role as an oxidising
agent?

oxidising agent explanation

A carbon monoxide causes iron(III) oxide to gain oxygen


B carbon monoxide causes iron(III) oxide to lose oxygen
C iron(III) oxide causes carbon monoxide to gain oxygen
D iron(III) oxide causes carbon monoxide to lose oxygen

21 Which solid reacts with sulfuric acid to produce a gas?

A copper
B copper carbonate
C copper oxide
D copper sulfate

© UCLES 2023 0653/22/F/M/23

8/693
Combined By Nesrine
9
2023-2017

22 Some water is added to a coloured, powdered mixture. After shaking, a blue solution and a white
solid are seen.

water blue solution


coloured,
powdered white solid
mixture

What does the powder contain?

A sodium chloride and copper(II) oxide

B sodium chloride and copper(II) sulfate

C barium sulfate and copper(II) oxide

D barium sulfate and copper(II) sulfate

23 Which row describes a noble gas?

type of particle reactivity

A diatomic high
B diatomic low
C monatomic high
D monatomic low

24 Why is potassium more reactive than sodium?

A Potassium accepts electrons more readily than sodium.


B Potassium forms positive ions more readily than sodium.
C Sodium accepts electrons more readily than potassium.
D Sodium forms positive ions more readily than potassium.

25 Which colour change is seen when water is added to anhydrous cobalt(II) chloride?

A white to blue
B pink to blue
C blue to white
D blue to pink

© UCLES 2023 0653/22/F/M/23 [Turn over

9/693
Combined By Nesrine
10
2023-2017

26 Which statements about air pollutants are correct?

1 Sulfur dioxide can damage buildings.


2 Oxides of nitrogen are harmful to health.
3 Carbon monoxide is a poisonous gas.
4 Carbon monoxide can damage buildings.

A 1, 2 and 3 B 1 and 2 only C 2, 3 and 4 D 3 and 4 only

27 Which statement about the homologous series of alkenes is correct?

A They are all saturated hydrocarbons.


B They all have the same physical properties.
C Their molecules have the same ratio of carbon atoms to hydrogen atoms.
D They all have the same molecular formula.

28 A measuring cylinder on a balance contains 40 cm3 of water. The reading on the balance is 30 g.

A stone is lowered into the water in the measuring cylinder until it is completely submerged.

The level of the water in the measuring cylinder is now at the 66 cm3 mark. The density of the
stone is 2.0 g / cm3.

What is the reading on the balance now?

A 43 g B 52 g C 82 g D 162 g

© UCLES 2023 0653/22/F/M/23

10/693
Combined By Nesrine
11
2023-2017

29 The force acting on a spring is gradually increased from 0 N.

The spring eventually passes its limit of proportionality.

Which graph shows how the extension of the spring changes as the force increases?

A B

extension extension

0 0
0 force 0 force

C D

extension extension

0 0
0 force 0 force

30 Which object has a resultant force acting on it?

A a book at rest on a table


B a car travelling up a hill in a straight line at constant speed
C a football moving upwards freely after being kicked
D a parachutist descending vertically at constant speed

© UCLES 2023 0653/22/F/M/23 [Turn over

11/693
Combined By Nesrine
12
2023-2017

31 An object is falling in a vacuum.

As the object falls, it passes through points P, Q and R.

object

Q
vacuum

Which statement describes the total quantity of energy of the object as it falls?

A It is greatest at point P.
B It is greatest at point Q.
C It is greatest at point R.
D It is the same at points P, Q and R.

32 Four students have different masses.

The students take different times to run the same distance up the same hill. The gravitational field
strength is 10 N / kg.

Which student produces the greatest power?

mass of time taken


student / kg /s

A 50 11
B 55 11
C 60 16
D 85 16

33 Which type of power station produces greenhouse gases when generating electricity?

A coal-fired
B geothermal
C hydroelectric
D wind-powered

© UCLES 2023 0653/22/F/M/23

12/693
Combined By Nesrine
13
2023-2017

34 A glass test-tube contains cold water. The diagram shows a small block of ice trapped at the
bottom of the test-tube by a metal weight.

The top of the test-tube is heated at the position shown. Very soon, the water at the top of the
test-tube is boiling, but the ice at the bottom has just started to melt.

glass test-tube

water

heat
ice
metal weight

What does this experiment show about thermal conduction?

A Conduction can only occur upwards.


B Conduction cannot occur in a liquid.
C Glass is a good thermal conductor.
D Water is a bad thermal conductor.

35 In which states of matter can convection occur?

in a solid in a liquid in a gas

A no no yes
B no yes yes
C yes no no
D yes yes no

36 Visible light and sound are both waves.

Which row describes the nature of these waves?

visible light sound

A longitudinal longitudinal
B longitudinal transverse
C transverse longitudinal
D transverse transverse

© UCLES 2023 0653/22/F/M/23 [Turn over

13/693
Combined By Nesrine
14
2023-2017

37 A beam that consists of red and blue light strikes a glass prism.

As the beam enters the prism, it splits into a red ray and a blue ray, as shown.

glass prism
red and blue light

red

blue

Which light refracts more and which light slows down more as it enters the prism?

light that light that


refracts more slows down more

A blue blue
B blue red
C red blue
D red red

38 Which circuit is used to measure the current in a lamp and the potential difference (p.d.) across
the lamp?

A B

A V

C D

A V

© UCLES 2023 0653/22/F/M/23

14/693
Combined By Nesrine
15
2023-2017

39 The diagram shows a circuit containing four resistors and four ammeters.

Which ammeter has the smallest reading?

A A A D

10 Ω 40 Ω
B
30 Ω
A

20 Ω
A
C

40 When a computer is switched on, the current increases quickly to 3.1 A and then decreases
slowly to a steady value of 1.0 A when the computer is in use.

The cable connecting the computer to the power supply can safely carry a current of 10.0 A.

The circuit contains a fuse.

Which fuse rating is used to provide suitable protection?

A 1A B 3A C 5A D 13 A

Permission to reproduce items where third-party owned material protected by copyright is included has been sought and cleared where possible. Every
reasonable effort has been made by the publisher (UCLES) to trace copyright holders, but if any items requiring clearance have unwittingly been included, the
publisher will be pleased to make amends at the earliest possible opportunity.

To avoid the issue of disclosure of answer-related information to candidates, all copyright acknowledgements are reproduced online in the Cambridge
Assessment International Education Copyright Acknowledgements Booklet. This is produced for each series of examinations and is freely available to download
at www.cambridgeinternational.org after the live examination series.

Cambridge Assessment International Education is part of Cambridge Assessment. Cambridge Assessment is the brand name of the University of Cambridge
Local Examinations Syndicate (UCLES), which is a department of the University of Cambridge.

© UCLES 2023 0653/22/F/M/23

15/693
The Periodic Table of Elements
Group

© UCLES 2023
I II III IV V VI VII VIII
1 2

H He
hydrogen helium
Key 1 4
3 4 atomic number 5 6 7 8 9 10

Li Be atomic symbol B C N O F Ne
lithium beryllium name boron carbon nitrogen oxygen fluorine neon
7 9 relative atomic mass 11 12 14 16 19 20
11 12 13 14 15 16 17 18
Na Mg Al Si P S Cl Ar
sodium magnesium aluminium silicon phosphorus sulfur chlorine argon
23 24 27 28 31 32 35.5 40
19 20 21 22 23 24 25 26 27 28 29 30 31 32 33 34 35 36
K Ca Sc Ti V Cr Mn Fe Co Ni Cu Zn Ga Ge As Se Br Kr
potassium calcium scandium titanium vanadium chromium manganese iron cobalt nickel copper zinc gallium germanium arsenic selenium bromine krypton
39 40 45 48 51 52 55 56 59 59 64 65 70 73 75 79 80 84
37 38 39 40 41 42 43 44 45 46 47 48 49 50 51 52 53 54

Rb Sr Y Zr Nb Mo Tc Ru Rh Pd Ag Cd In Sn Sb Te I Xe
rubidium strontium yttrium zirconium niobium molybdenum technetium ruthenium rhodium palladium silver cadmium indium tin antimony tellurium iodine xenon
16

85 88 89 91 93 96 – 101 103 106 108 112 115 119 122 128 127 131

16/693
55 56 57–71 72 73 74 75 76 77 78 79 80 81 82 83 84 85 86

0653/22/F/M/23
lanthanoids
Cs Ba Hf Ta W Re Os Ir Pt Au Hg Tl Pb Bi Po At Rn
caesium barium hafnium tantalum tungsten rhenium osmium iridium platinum gold mercury thallium lead bismuth polonium astatine radon
133 137 178 181 184 186 190 192 195 197 201 204 207 209 – – –
87 88 89–103 104 105 106 107 108 109 110 111 112 113 114 115 116 117 118
actinoids
Fr Ra Rf Db Sg Bh Hs Mt Ds Rg Cn Nh Fl Mc Lv Ts Og
francium radium rutherfordium dubnium seaborgium bohrium hassium meitnerium darmstadtium roentgenium copernicium nihonium flerovium moscovium livermorium tennessine oganesson
– – – – – – – – – – – – – – – – –

57 58 59 60 61 62 63 64 65 66 67 68 69 70 71
lanthanoids La Ce Pr Nd Pm Sm Eu Gd Tb Dy Ho Er Tm Yb Lu
lanthanum cerium praseodymium neodymium promethium samarium europium gadolinium terbium dysprosium holmium erbium thulium ytterbium lutetium
139 140 141 144 – 150 152 157 159 163 165 167 169 173 175
89 90 91 92 93 94 95 96 97 98 99 100 101 102 103
actinoids Ac Th Pa U Np Pu Am Cm Bk Cf Es Fm Md No Lr
actinium thorium protactinium uranium neptunium plutonium americium curium berkelium californium einsteinium fermium mendelevium nobelium lawrencium
– 232 231 238 – – – – – – – – – – –
2023-2017

The volume of one mole of any gas is 24 dm3 at room temperature and pressure (r.t.p.).
Combined By Nesrine
Combined By Nesrine
2023-2017

Cambridge IGCSE™

COMBINED SCIENCE 0653/21


Paper 2 Multiple Choice (Extended) May/June 2023
45 minutes

You must answer on the multiple choice answer sheet.


*9384631348*

You will need: Multiple choice answer sheet


Soft clean eraser
Soft pencil (type B or HB is recommended)

INSTRUCTIONS
• There are forty questions on this paper. Answer all questions.
• For each question there are four possible answers A, B, C and D. Choose the one you consider correct
and record your choice in soft pencil on the multiple choice answer sheet.
• Follow the instructions on the multiple choice answer sheet.
• Write in soft pencil.
• Write your name, centre number and candidate number on the multiple choice answer sheet in the
spaces provided unless this has been done for you.
• Do not use correction fluid.
• Do not write on any bar codes.
• You may use a calculator.

INFORMATION
• The total mark for this paper is 40.
• Each correct answer will score one mark.
• Any rough working should be done on this question paper.
• The Periodic Table is printed in the question paper.

This document has 16 pages. Any blank pages are indicated.

IB23 06_0653_21/2RP
© UCLES 2023 [Turn over

17/693
Combined By Nesrine
2
2023-2017

1 Which characteristic of living organisms involves chemical reactions in cells that break down
nutrient molecules and release energy?

A excretion
B nutrition
C respiration
D sensitivity

2 Which structures are present in an animal cell?

cell
cell wall cytoplasm nucleus
membrane

A     key
B     = present
C     = not present
D    

3 Which row describes the movement of water by osmosis?

from a region of to a region of through

A low water potential high water potential a partially permeable membrane


B low water potential high water potential a totally permeable membrane
C high water potential low water potential a totally permeable membrane
D high water potential low water potential a partially permeable membrane

4 Which combination of pH and temperature does not denature a protease enzyme from the
stomach?

temperature
pH
/ °C

A 3 37
B 3 60
C 10 37
D 10 60

© UCLES 2023 0653/21/M/J/23

18/693
Combined By Nesrine
3
2023-2017

5 Four test-tubes are set up as shown.

In which test-tube is there an increase in oxygen concentration after 4 hours?

A B C D
black
polythene
to keep
water out light
light light light light
water plant
water snail

6 What is the purpose of chemical digestion?

A to absorb minerals including calcium and iron


B to pass food out as faeces
C to break down large nutrient molecules into smaller molecules
D to secrete enzymes

7 Which row matches the adaptation of a root hair cell to its function?

adaptation function

A large surface area uptake of water and glucose


B large surface area uptake of water and ions
C small surface area uptake of water and glucose
D small surface area uptake of water and ions

© UCLES 2023 0653/21/M/J/23 [Turn over

19/693
Combined By Nesrine
4
2023-2017

8 The diagram shows a section through the heart.

Which vessel is the pulmonary vein?

D A

B
C

9 Which molecule contains the energy that is released in aerobic respiration?

A C6H12O6 B CO2 C H2O D O2

10 What are features of sexual reproduction?

fusion
nature of offspring
of nuclei

A no genetically different
B yes genetically identical
C no genetically identical
D yes genetically different

© UCLES 2023 0653/21/M/J/23

20/693
Combined By Nesrine
5
2023-2017

11 The diagrams show four different flowers.

1 2 3 4

Which flowers are wind pollinated?

A 1 and 3 B 1 and 4 C 2 and 3 D 2 and 4

12 Which letter represents the secondary consumer in the food chain shown?

A B C D

13 The flow chart shows some of the steps in the process of eutrophication.

increased availability of nitrates and other ions



step X

increased decomposition after death of producers

step Y

reduction in amount of dissolved oxygen

death of fish requiring dissolved oxygen in water

What is happening at step Y?

A increased aerobic respiration by decomposers


B decreased aerobic respiration by decomposers
C increased growth of aquatic plants
D decreased growth of aquatic plants

© UCLES 2023 0653/21/M/J/23 [Turn over

21/693
Combined By Nesrine
6
2023-2017

14 Which diagram represents particles in a gaseous element?

A B C D

15 What is the relative mass of a proton and the relative charge on a proton?

relative mass relative charge

A 0.0005 +1
B 0.0005 –1
C 1 –1
D 1 +1

16 Sodium reacts with chlorine to form sodium chloride.

Which statement describes a change that occurs during this reaction?

A Each chlorine atom loses one proton.


B Each sodium atom loses one electron.
C The mass number of each chlorine atom increases.
D The atomic number of sodium decreases.

17 Sodium burns in oxygen forming sodium oxide.

An equation for this reaction is shown.

xNa + yO2 → zNa2O

What are x, y and z?

x y z

A 2 1 2
B 2 2 1
C 4 1 2
D 4 2 2

© UCLES 2023 0653/21/M/J/23

22/693
Combined By Nesrine
7
2023-2017

18 Which statement about the electrolysis of molten lead(II) bromide is correct?

A Lead anions move to the anode where they gain electrons.


B Lead anions move to the cathode where they lose electrons.
C Lead cations move to the anode where they lose electrons.
D Lead cations move to the cathode where they gain electrons.

19 Which process is endothermic?

A the reaction of petrol with air in a car engine

B Cl –Cl → Cl + Cl
C the reaction of potassium with water

D C + O2 → CO2

20 The equation for the reaction of zinc oxide with carbon monoxide is shown.

ZnO + CO → Zn + CO2

Which statement explains the role of carbon monoxide in this reaction?

A It is the oxidising agent and it is oxidised.


B It is the oxidising agent and it is reduced.
C It is the reducing agent and it is oxidised.
D It is the reducing agent and it is reduced.

21 Dilute hydrochloric acid is tested with universal indicator and with calcium carbonate.

Which row shows the pH and describes the reaction with calcium carbonate?

pH reaction with calcium carbonate

A 2 a colourless gas is given off


B 2 no reaction
C 10 a colourless gas is given off
D 10 no reaction

© UCLES 2023 0653/21/M/J/23 [Turn over

23/693
Combined By Nesrine
8
2023-2017

22 The results of two tests on a solution of substance R are shown.

test result
aqueous sodium red-brown precipitate formed,
hydroxide added insoluble in excess

dilute nitric acid added


followed by aqueous white precipitate formed
silver nitrate added

What is R?

A iron(II) carbonate

B iron(III) carbonate

C iron(II) chloride

D iron(III) chloride

23 The character of the elements changes from metallic to non-metallic across a period of the
Periodic Table.

Which statement explains this change?

A Metal atoms need to gain electrons to form metal ions.


B It becomes more difficult for elements to lose electrons across a period.
C Non-metal atoms lose electrons more easily than metal atoms.
D Atoms get bigger across a period.

24 Copper oxide and excess carbon are mixed together.

The mass before heating is 12.2 g.

The mixture is heated strongly and allowed to cool.

The mass after heating is 10.4 g.

Why does the mass change?

A Carbon forms carbon dioxide which then combines with the copper oxide.
B Carbon reduces the copper oxide and leaves the test-tube as carbon dioxide.
C Copper oxide loses oxygen, turns into copper and the carbon remains unchanged.
D Carbon oxidises the copper oxide and leaves the test-tube as carbon dioxide.

© UCLES 2023 0653/21/M/J/23

24/693
Combined By Nesrine
9
2023-2017

25 Which statement about both carbon dioxide and methane is correct?

A Increased concentrations of carbon dioxide and methane in the air cause an enhanced
greenhouse effect.
B Methane and carbon dioxide are hydrocarbons.
C Respiration increases the concentration of carbon dioxide and methane in the air.
D The combustion of fossil fuels increases the concentration of carbon dioxide and methane in
the air.

26 Why are alkanes members of the same homologous series?

A They are all hydrocarbons.


B They have similar physical properties.
C They have the same general formula.
D They all undergo combustion to give carbon dioxide and water.

27 What are the products of cracking large alkane molecules?

A smaller alkanes only


B smaller alkenes only
C smaller alkanes, alkenes and hydrogen
D smaller alkanes and hydrogen only

28 The graph shows the motion of a cyclist over a period of 30 s.

7
speed
m/s

0
0 20 30
time / s

Which distance does the cyclist travel?

A 90 m B 105 m C 115 m D 120 m

© UCLES 2023 0653/21/M/J/23 [Turn over

25/693
Combined By Nesrine
10
2023-2017

29 A solid cuboid block of metal has density ρ.

The diagram shows its dimensions.

y
x

Which expression is used to calculate the mass of the block?

ρ ρ
A B C ρxy D ρxyz
xy xyz

30 A spring that obeys Hooke’s law is fixed at one end.

When the spring is pulled by a force of 30 N, it has a stretched length of 14 cm.

When the spring is pulled by a force of 48 N, it has a stretched length of 20 cm.

What is the spring constant of the spring?

A 2.1 N / cm B 2.4 N / cm C 3.0 N / cm D 8.0 N / cm

31 A 3.0 kW electric kettle is switched on for 30 seconds.

Which row gives the equation that defines power and gives the energy transferred in the kettle?

equation energy / J

A P = ∆E 90
t

B P = ∆E 90 000
t
C P = ∆Et 0.10
D P = ∆Et 100

© UCLES 2023 0653/21/M/J/23

26/693
Combined By Nesrine
11
2023-2017

32 Solar cells mounted on a boat produce electrical energy to power the motor.

solar cells

Which resource does this energy come from?

A hydroelectric energy
B light energy
C tidal energy
D wind energy

33 The molecules in a liquid are close together.

What are other features of the molecules in a liquid?

A They are arranged in a regular pattern but change positions with each other.
B They are arranged in a regular pattern and vibrate about fixed positions.
C They are arranged randomly and change positions with each other.
D They are arranged randomly and vibrate about fixed positions.

34 What is a method of thermal conduction in a metal?

A electrons moving through the metal, hitting distant molecules


B electrons vibrating and passing on energy to other electrons
C molecules moving through the metal, hitting other molecules
D protons moving through the metal, hitting distant molecules

© UCLES 2023 0653/21/M/J/23 [Turn over

27/693
Combined By Nesrine
12
2023-2017

35 The diagram represents a wave in air. Molecules are closer together in region P than they are in
region Q.

region P region Q

What are the names of regions P and Q, and which type of wave is represented?

region P region Q type of wave

A compression rarefaction longitudinal


B compression rarefaction transverse
C rarefaction compression longitudinal
D rarefaction compression transverse

36 The diagram shows an object placed at position O near a thin converging lens.

F1 and F2 are the principal focuses of the lens.

A student draws four rays that leave the top of the object and pass through the lens.

Which labelled ray is not correct?

F1 O F2 B

37 The unit of electric charge is the coulomb (C).

Which combination of other units is equivalent to the coulomb?

A A/s B As C V/A D VA

© UCLES 2023 0653/21/M/J/23

28/693
Combined By Nesrine
13
2023-2017

38 Which circuit is used to determine the resistance of resistor R?

A B

R R
A A V

C D

R V
V R

A
A

39 Which arrangement of resistors has the smallest combined resistance?

A B

2.0 : 3.0 :
4.0 : 2.0 :
3.0 : 4.0 :

C D

2.0 :

3.0 : 2.0 : 3.0 : 4.0 :

4.0 :

© UCLES 2023 0653/21/M/J/23 [Turn over

29/693
Combined By Nesrine
14
2023-2017

40 The diagram shows two resistors connected in parallel to a battery.

Currents I1 and I2 are labelled.

Current I1 is greater than current I2.

I1

I2

Which calculation gives the current in the battery?

( I1 + I2 )
A I1 + I2 B I1 – I2 C I1 × I2 D
2

© UCLES 2023 0653/21/M/J/23

30/693
The Periodic Table of Elements
Group

© UCLES 2023
I II III IV V VI VII VIII
1 2

H He
hydrogen helium
Key 1 4
3 4 atomic number 5 6 7 8 9 10

Li Be atomic symbol B C N O F Ne
lithium beryllium name boron carbon nitrogen oxygen fluorine neon
7 9 relative atomic mass 11 12 14 16 19 20
11 12 13 14 15 16 17 18
Na Mg Al Si P S Cl Ar
sodium magnesium aluminium silicon phosphorus sulfur chlorine argon
23 24 27 28 31 32 35.5 40
19 20 21 22 23 24 25 26 27 28 29 30 31 32 33 34 35 36
K Ca Sc Ti V Cr Mn Fe Co Ni Cu Zn Ga Ge As Se Br Kr
potassium calcium scandium titanium vanadium chromium manganese iron cobalt nickel copper zinc gallium germanium arsenic selenium bromine krypton
39 40 45 48 51 52 55 56 59 59 64 65 70 73 75 79 80 84
37 38 39 40 41 42 43 44 45 46 47 48 49 50 51 52 53 54

Rb Sr Y Zr Nb Mo Tc Ru Rh Pd Ag Cd In Sn Sb Te I Xe
rubidium strontium yttrium zirconium niobium molybdenum technetium ruthenium rhodium palladium silver cadmium indium tin antimony tellurium iodine xenon
16

85 88 89 91 93 96 – 101 103 106 108 112 115 119 122 128 127 131

31/693
55 56 57–71 72 73 74 75 76 77 78 79 80 81 82 83 84 85 86

0653/21/M/J/23
lanthanoids
Cs Ba Hf Ta W Re Os Ir Pt Au Hg Tl Pb Bi Po At Rn
caesium barium hafnium tantalum tungsten rhenium osmium iridium platinum gold mercury thallium lead bismuth polonium astatine radon
133 137 178 181 184 186 190 192 195 197 201 204 207 209 – – –
87 88 89–103 104 105 106 107 108 109 110 111 112 113 114 115 116 117 118
actinoids
Fr Ra Rf Db Sg Bh Hs Mt Ds Rg Cn Nh Fl Mc Lv Ts Og
francium radium rutherfordium dubnium seaborgium bohrium hassium meitnerium darmstadtium roentgenium copernicium nihonium flerovium moscovium livermorium tennessine oganesson
– – – – – – – – – – – – – – – – –

57 58 59 60 61 62 63 64 65 66 67 68 69 70 71
lanthanoids La Ce Pr Nd Pm Sm Eu Gd Tb Dy Ho Er Tm Yb Lu
lanthanum cerium praseodymium neodymium promethium samarium europium gadolinium terbium dysprosium holmium erbium thulium ytterbium lutetium
139 140 141 144 – 150 152 157 159 163 165 167 169 173 175
89 90 91 92 93 94 95 96 97 98 99 100 101 102 103
actinoids Ac Th Pa U Np Pu Am Cm Bk Cf Es Fm Md No Lr
actinium thorium protactinium uranium neptunium plutonium americium curium berkelium californium einsteinium fermium mendelevium nobelium lawrencium
– 232 231 238 – – – – – – – – – – –
2023-2017

The volume of one mole of any gas is 24 dm3 at room temperature and pressure (r.t.p.).
Combined By Nesrine
Combined By Nesrine
2023-2017

Cambridge IGCSE™

COMBINED SCIENCE 0653/22


Paper 2 Multiple Choice (Extended) May/June 2023
45 minutes

You must answer on the multiple choice answer sheet.


*2547608433*

You will need: Multiple choice answer sheet


Soft clean eraser
Soft pencil (type B or HB is recommended)

INSTRUCTIONS
 There are forty questions on this paper. Answer all questions.
 For each question there are four possible answers A, B, C and D. Choose the one you consider correct
and record your choice in soft pencil on the multiple choice answer sheet.
 Follow the instructions on the multiple choice answer sheet.
 Write in soft pencil.
 Write your name, centre number and candidate number on the multiple choice answer sheet in the
spaces provided unless this has been done for you.
 Do not use correction fluid.
 Do not write on any bar codes.
 You may use a calculator.

INFORMATION
 The total mark for this paper is 40.
 Each correct answer will score one mark.
 Any rough working should be done on this question paper.
 The Periodic Table is printed in the question paper.

This document has 16 pages. Any blank pages are indicated.

IB23 06_0653_22/3RP
© UCLES 2023 [Turn over

32/693
Combined By Nesrine
2
2023-2017

1 Which process removes toxic materials from an organism?

A digestion
B egestion
C excretion
D respiration

2 The diagram shows a cell as seen with a microscope.

Which label is correct?

B
chloroplast

A
vacuole

C
cell membrane

D
cytoplasm

3 The activity of an enzyme-catalysed reaction is altered by changes in temperature.

What occurs when the temperature rises above the temperature at which the enzyme works
best?

A The shape of the substrate molecule no longer fits the active site of the enzyme molecule.
B The increasing temperature causes the substrate molecules to break down.
C The concentration of the substrate increases and that of the product decreases.
D The kinetic energy of the substrate particles decreases.

© UCLES 2023 0653/22/M/J/23

33/693
Combined By Nesrine
3
2023-2017

4 In plants, photosynthesis takes place in the leaf.

During photosynthesis, ......P...... transfers ......Q...... energy into ......R...... energy. This is used
for the synthesis of ......S...... .

Which row correctly completes gaps P, Q, R and S?

P Q R S

A chlorophyll chemical light carbohydrate


B chlorophyll light chemical carbohydrate
C glucose chemical light chlorophyll
D glucose light chemical chlorophyll

5 What can be caused by a diet containing too little vitamin C?

A anaemia
B coronary heart disease
C rickets
D scurvy

6 In which order does food pass through parts of the alimentary canal?

A oesophagus  anus  large intestine

B small intestine  oesophagus  stomach

C small intestine  large intestine  anus

D stomach  large intestine  small intestine

7 The table shows two processes that are involved in transpiration.

What happens to the rate of these processes in high humidity?

diffusion of evaporation of
water vapour water from surfaces
through stomata of mesophyll cells

A rate decreases rate increases


B rate decreases rate decreases
C rate increases rate increases
D rate increases rate decreases

© UCLES 2023 0653/22/M/J/23 [Turn over

34/693
Combined By Nesrine
4
2023-2017

8 A sample of blood is taken from a person who often gets infections.

The blood is also slow to clot.

Which blood components are likely to be at a lower level than normal?

1 platelets
2 red blood cells
3 white blood cells

A 1, 2 and 3 B 1 and 2 only C 1 and 3 only D 2 and 3 only

9 In respiration, glucose is broken down to release energy.

Which row states how humans could use this energy?

keep a constant muscle protein


growth
body temperature contraction synthesis

A     key
B     = true
C     = false
D    

10 What happens when adrenaline is released into the blood?

blood glucose
pulse rate
concentration

A increases increases
B increases decreases
C decreases increases
D decreases decreases

11 Which row describes asexual reproduction?

number of offspring genetically


parents involved identical to each other

A 1 yes
B 1 no
C 2 yes
D 2 no

© UCLES 2023 0653/22/M/J/23

35/693
Combined By Nesrine
5
2023-2017

12 The diagram shows part of a food web.

snake
jaguar

macaw monkey frog


fruit bat tapir

water weevil
banana fruit
water weed

Which row gives the number of each type of consumer?

primary secondary tertiary

A 2 2 0
B 2 5 3
C 5 1 0
D 5 3 1

13 Eutrophication of fresh water occurs because of a series of events in the water.

The list describes these events.

1 increased aerobic respiration by decomposers


2 increased availability of nitrate and other ions
3 increased decomposition after death of producers
4 increased growth of producers
5 reduction in amount of dissolved oxygen in the water

Which order of these events results in the death of fish and other aquatic organisms?

A 21453

B 24315

C 42315

D 45213

© UCLES 2023 0653/22/M/J/23 [Turn over

36/693
Combined By Nesrine
6
2023-2017

14 The formulae of three substances are shown.

substance formula

methane CH4
water H2O
oxygen O2

Which statement is correct?

A Methane is made from five different types of atom.


B Methane, water and oxygen are molecules.
C Only methane and water are molecules.
D Oxygen is made from two different types of atom.

15 What is the definition of nucleon number?

A the number of protons in an atom


B the number of electrons in an atom
C the total number of electrons and neutrons in an atom
D the total number of neutrons and protons in an atom

16 Which structure represents an ionic compound?

A B C D

+ – H H H

Li F H C C C H Cl Cl

H H H

17 Aqueous lead(II) nitrate, Pb(NO3)2, reacts with aqueous potassium iodide to make a precipitate of
lead(II) iodide.

What is the ionic equation for this reaction?

A Pb+ + I–  PbI

B Pb2+ + 2I–  PbI2

C Pb(NO3)2 + I–  PbI + 2NO3–

D Pb2+ + 2NO3– + 2I–  PbI2 + 2NO3–

© UCLES 2023 0653/22/M/J/23

37/693
Combined By Nesrine
7
2023-2017

18 Which statement about the electrolysis of ionic substances is correct?

A Negatively charged ions move to the cathode.


B At the anode, ions lose electrons.
C The anions gain electrons during electrolysis.
D The cations are negatively charged.

19 The energy level diagram for dissolving solid ammonium nitrate in water is shown.

products

energy
reactants

progress of reaction

Which statement about this process is correct?

A Activation energy is given out causing an overall increase in temperature.


B Energy is taken in to form new bonds at the start of the reaction.
C During the reaction, the temperature of the water decreases because the reaction takes in
energy.
D The products have a higher energy than the reactants because the reaction is exothermic.

20 Reducing agents are ......1...... in a reaction.

Reducing agents cause the other substance in the reaction to ......2...... oxygen.

Which words complete gaps 1 and 2?

1 2

A oxidised gain
B oxidised lose
C reduced gain
D reduced lose

© UCLES 2023 0653/22/M/J/23 [Turn over

38/693
Combined By Nesrine
8
2023-2017

21 Dilute sulfuric acid reacts with aqueous potassium hydroxide.

What are the products of this reaction?

potassium potassium carbon


water
hydroxide sulfate dioxide

A     key
B     = yes
C     = no
D    

22 The results of two tests on solid P are shown.

test result
1 add aqueous gas given off that turns
sodium hydroxide to solid moist red litmus paper blue
2 dissolve solid in water, white precipitate formed
add dilute aqueous silver nitrate

What is P?

A aluminium carbonate
B aluminium sulfate
C ammonium chloride
D ammonium nitrate

23 Which electronic structure is that of a metal?

A 2,8,3 B 2,8,4 C 2,8,6 D 2,8,7

24 Why are gold alloys, rather than pure gold, used to make jewellery?

A Alloys are better electrical conductors.


B Alloys are less likely to corrode.
C Alloys are harder.
D Alloys are less dense.

© UCLES 2023 0653/22/M/J/23

39/693
Combined By Nesrine
9
2023-2017

25 What is an effect of increasing the amount of carbon dioxide in the atmosphere?

A increased acid rain


B increased climate change
C increased damage to buildings
D increased health problems

26 Which statements about the members of an homologous series are correct?

1 They have similar chemical properties.


2 They have the same boiling points.
3 They have the same general formula.

A 1, 2 and 3 B 1 and 2 only C 1 and 3 only D 2 and 3 only

27 Which equation represents cracking?

A C6H14  2C3H6 + H2

B C3H8 + 5O2  3CO2 + 4H2O

C nCH2=CH2  poly(ethene)

D CH2=CH2 + Br2  CH2BrCH2Br

© UCLES 2023 0653/22/M/J/23 [Turn over

40/693
Combined By Nesrine
10
2023-2017

28 Which speed–time graph represents the motion of an object that travels a distance of 24 m?

A B

speed 4.0 speed


m/s m / s 3.0

0 0
0 6.0 0 8.0
time / s time / s

C D
4.0
speed speed
m/s m/s

1.0
0 0
0 3.0 0 12
time / s time / s

29 A vehicle is taken from the Earth to the Moon where the gravitational field strength is smaller.

How do the mass and the weight of the vehicle on the Moon compare with their values on the
Earth?

A smaller mass and smaller weight


B smaller mass and the same weight
C the same mass and smaller weight
D the same mass and the same weight

30 Which form of energy is not a form of potential energy?

A chemical
B elastic
C gravitational
D sound

31 A rock of mass 2000 kg has a kinetic energy of 64 000 J.

What is the speed of the rock?

A 5.7 m / s B 8.0 m / s C 32 m / s D 64 m / s

© UCLES 2023 0653/22/M/J/23

41/693
Combined By Nesrine
11
2023-2017

32 A circuit contains two lamps and four ammeters. The readings on the ammeters are I1, I2, I3
and I4, as shown.

reading = I1 A A reading = I4
reading = I2
A

A
reading = I3

Which equation is correct?

A I1 = I4 = (I2 + I3)

B (I1 + I4) = (I2 + I3)

C I1 = I2 = I3 = I4

D I2 = I3 = (I1 + I4)

33 What happens as a liquid starts to evaporate?

A The mass of the remaining liquid increases.


B The mass of the remaining liquid is constant.
C The temperature of the remaining liquid decreases.
D The temperature of the remaining liquid increases.

34 The temperature of air next to a heater increases. This causes a convection current.

Which row describes what happens to the density of the air next to the heater and states the
direction of movement of this air?

direction of
density
movement
of air
of air

A decreases downwards
B decreases upwards
C increases downwards
D increases upwards

© UCLES 2023 0653/22/M/J/23 [Turn over

42/693
Combined By Nesrine
12
2023-2017

35 The diagram represents a sound wave travelling in air.

wave direction

1 2 3 4 5 6

Which numbered points are at the centre of a compression and which numbered points are at the
centre of a rarefaction?

centre of a centre of a
compression rarefaction

A 1 and 5 2 and 4
B 1 and 5 3 and 6
C 3 and 6 1 and 5
D 3 and 6 2 and 4

36 The diagram shows the change in direction of light as it moves from medium 1 into medium 2.

medium 1
medium 2

Why does this change of direction happen?

A Light is a longitudinal wave in medium 1 but a transverse wave in medium 2.


B Light is a transverse wave in medium 1 but a longitudinal wave in medium 2.
C The frequency of the light changes as it moves from medium 1 into medium 2.
D The speed of the light changes as it moves from medium 1 into medium 2.

© UCLES 2023 0653/22/M/J/23

43/693
Combined By Nesrine
13
2023-2017

37 A student uses a thin converging lens as a magnifying glass to view an object.

Where is the object placed?

A as far away as possible from the lens


B at a distance from the lens that is slightly greater than the focal length of the lens
C at a distance from the lens that is less than the focal length of the lens
D between the lens and the student’s eye

38 A battery is connected to two identical lamps X and Y in parallel.

0.50 A
X

The current in the battery is 0.50 A.

How much charge flows through lamp Y in 10 s?

A 0.025 C B 0.050 C C 2.5 C D 5.0 C

© UCLES 2023 0653/22/M/J/23 [Turn over

44/693
Combined By Nesrine
14
2023-2017

39 The diagram shows a cell connected to a resistor and two meters, X and Y.

The circuit is used when determining the resistance of the resistor.

X
Y

What are the quantities measured by meters X and Y, and what are their correct units?

meter X meter Y
quantity unit quantity unit

A current A p.d. V
B current V p.d. A
C p.d. A current V
D p.d. V current A

40 An electrical appliance with a resistance of 600  is connected to a 240 V supply.

Which fuse rating is appropriate to protect the appliance and the wires from overheating if a fault
occurs?

A 0.04 A B 0.5 A C 5A D 13 A

© UCLES 2023 0653/22/M/J/23

45/693
The Periodic Table of Elements
Group

© UCLES 2023
I II III IV V VI VII VIII
1 2

H He
hydrogen helium
Key 1 4
3 4 atomic number 5 6 7 8 9 10

Li Be atomic symbol B C N O F Ne
lithium beryllium name boron carbon nitrogen oxygen fluorine neon
7 9 relative atomic mass 11 12 14 16 19 20
11 12 13 14 15 16 17 18
Na Mg Al Si P S Cl Ar
sodium magnesium aluminium silicon phosphorus sulfur chlorine argon
23 24 27 28 31 32 35.5 40
19 20 21 22 23 24 25 26 27 28 29 30 31 32 33 34 35 36
K Ca Sc Ti V Cr Mn Fe Co Ni Cu Zn Ga Ge As Se Br Kr
potassium calcium scandium titanium vanadium chromium manganese iron cobalt nickel copper zinc gallium germanium arsenic selenium bromine krypton
39 40 45 48 51 52 55 56 59 59 64 65 70 73 75 79 80 84
37 38 39 40 41 42 43 44 45 46 47 48 49 50 51 52 53 54

Rb Sr Y Zr Nb Mo Tc Ru Rh Pd Ag Cd In Sn Sb Te I Xe
rubidium strontium yttrium zirconium niobium molybdenum technetium ruthenium rhodium palladium silver cadmium indium tin antimony tellurium iodine xenon
16

85 88 89 91 93 96 – 101 103 106 108 112 115 119 122 128 127 131

46/693
55 56 57–71 72 73 74 75 76 77 78 79 80 81 82 83 84 85 86

0653/22/M/J/23
lanthanoids
Cs Ba Hf Ta W Re Os Ir Pt Au Hg Tl Pb Bi Po At Rn
caesium barium hafnium tantalum tungsten rhenium osmium iridium platinum gold mercury thallium lead bismuth polonium astatine radon
133 137 178 181 184 186 190 192 195 197 201 204 207 209 – – –
87 88 89–103 104 105 106 107 108 109 110 111 112 113 114 115 116 117 118
actinoids
Fr Ra Rf Db Sg Bh Hs Mt Ds Rg Cn Nh Fl Mc Lv Ts Og
francium radium rutherfordium dubnium seaborgium bohrium hassium meitnerium darmstadtium roentgenium copernicium nihonium flerovium moscovium livermorium tennessine oganesson
– – – – – – – – – – – – – – – – –

57 58 59 60 61 62 63 64 65 66 67 68 69 70 71
lanthanoids La Ce Pr Nd Pm Sm Eu Gd Tb Dy Ho Er Tm Yb Lu
lanthanum cerium praseodymium neodymium promethium samarium europium gadolinium terbium dysprosium holmium erbium thulium ytterbium lutetium
139 140 141 144 – 150 152 157 159 163 165 167 169 173 175
89 90 91 92 93 94 95 96 97 98 99 100 101 102 103
actinoids Ac Th Pa U Np Pu Am Cm Bk Cf Es Fm Md No Lr
actinium thorium protactinium uranium neptunium plutonium americium curium berkelium californium einsteinium fermium mendelevium nobelium lawrencium
– 232 231 238 – – – – – – – – – – –
2023-2017

The volume of one mole of any gas is 24 dm3 at room temperature and pressure (r.t.p.).
Combined By Nesrine
Combined By Nesrine
2023-2017

Cambridge IGCSE™

COMBINED SCIENCE 0653/23


Paper 2 Multiple Choice (Extended) May/June 2023
45 minutes

You must answer on the multiple choice answer sheet.


*0523095116*

You will need: Multiple choice answer sheet


Soft clean eraser
Soft pencil (type B or HB is recommended)

INSTRUCTIONS
 There are forty questions on this paper. Answer all questions.
 For each question there are four possible answers A, B, C and D. Choose the one you consider correct
and record your choice in soft pencil on the multiple choice answer sheet.
 Follow the instructions on the multiple choice answer sheet.
 Write in soft pencil.
 Write your name, centre number and candidate number on the multiple choice answer sheet in the
spaces provided unless this has been done for you.
 Do not use correction fluid.
 Do not write on any bar codes.
 You may use a calculator.

INFORMATION
 The total mark for this paper is 40.
 Each correct answer will score one mark.
 Any rough working should be done on this question paper.
 The Periodic Table is printed in the question paper.

This document has 16 pages. Any blank pages are indicated.

IB23 06_0653_23/4RP
© UCLES 2023 [Turn over

47/693
Combined By Nesrine
2
2023-2017

1 What is a characteristic of all living things?

A egestion
B ingestion
C nutrition
D photosynthesis

2 Which structure is found only in plant cells?

A cell membrane
B cytoplasm
C large vacuole
D nucleus

3 The graph shows how the rate of an enzyme-controlled reaction varies with temperature.

At which labelled point does the enzyme have the least kinetic energy?

B
5

rate of reaction 3 A C
/ arbitrary units
2

1
D
0
temperature

4 In plants, which energy transfer occurs in chlorophyll during photosynthesis?

A chemical to light
B heat to chemical
C light to chemical
D chemical to heat

© UCLES 2023 0653/23/M/J/23

48/693
Combined By Nesrine
3
2023-2017

5 Which row matches the part of the alimentary canal to its function?

part of the
function of part
alimentary canal

A anus absorption
B oesophagus digestion
C mouth ingestion
D small intestines egestion

6 The list gives two ways in which an environment changes.

1 humidity increases
2 temperature increases

Which changes cause an increase in the rate of transpiration of plants?

1 2

A   key
B   = increase in rate of transpiration
C   = decrease in rate of transpiration
D  

7 From which chamber does the blood leave the heart to travel to the organs of the body?

D A

B
C

© UCLES 2023 0653/23/M/J/23 [Turn over

49/693
Combined By Nesrine
4
2023-2017

8 Which processes require energy?

1 growth
2 maintenance of body temperature
3 protein synthesis

A 1, 2 and 3 B 1 only C 2 and 3 only D 3 only

9 What is caused by the secretion of adrenaline?

blood glucose
pulse rate pupil size
concentration

A decreases decreases increases


B decreases increases decreases
C increases decreases decreases
D increases increases increases

10 Which definition of asexual reproduction is correct?

A production of genetically different offspring from one parent


B production of genetically different offspring from two parents
C production of genetically identical offspring from one parent
D production of genetically identical offspring from two parents

11 Which row shows the correct information about a human female gamete?

contains an
can swim
energy store

A  
B  
C  
D  

© UCLES 2023 0653/23/M/J/23

50/693
Combined By Nesrine
5
2023-2017

12 The diagram shows part of a food web.

eagle

lizard hawk snake

rabbit
grasshopper mouse

grass carrot
plants

Which row shows the numbers of different types of consumers present in this food web?

primary secondary tertiary quaternary


consumers consumers consumers consumers

A 2 3 3 1
B 2 2 1 0
C 3 3 1 2
D 3 3 2 1

13 Eutrophication typically occurs as the result of nitrates and other ions accumulating in bodies of
water. Eutrophication involves the five processes listed.

1 death of organisms that require dissolved oxygen


2 increased aerobic respiration by decomposers
3 increased decomposition after death of producers
4 increased growth of producer organisms
5 reduction of amount of dissolved oxygen in water

Which sequence of processes is correct?

A 13245

B 43251

C 12435

D 45321

© UCLES 2023 0653/23/M/J/23 [Turn over

51/693
Combined By Nesrine
6
2023-2017

14 The diagrams show four different mixtures of gases.

Which diagram represents a mixture containing only elements?

A B C D key

different
types of
atom

15 The nucleon number of an atom of chlorine is 35.

Which statement about this atom is correct?

A It contains the same number of neutrons as electrons.


B It contains the same number of protons as neutrons.
C It contains the same number of protons as electrons.
D The numbers of protons, neutrons and electrons are all different from each other.

16 Which dot-and-cross diagram represents lithium fluoride, LiF?

A B

+ –
Li F
Li F

C D

+ –

Li F Li F

© UCLES 2023 0653/23/M/J/23

52/693
Combined By Nesrine
7
2023-2017

17 Aqueous sodium sulfate reacts with aqueous barium chloride to make barium sulfate and
sodium chloride.

What is the ionic equation for this reaction?

A Ba2+(aq) + SO42–(aq)  BaSO4(aq)

B Ba2+(aq) + SO42–(aq)  BaSO4(s)

C Na+(aq) + Cl –(aq)  NaCl (s)

D Na+(aq) + Cl –(aq)  NaCl (aq)

18 Dilute sulfuric acid is electrolysed using inert electrodes.

The concentration of two ions decreases during this process.

What are these ions?

A hydrogen ions and oxide ions


B hydrogen ions and hydroxide ions
C hydroxide ions and sulfate ions
D oxide ions and sulfate ions

19 Substances P and Q react together.

P + Q  R + S

The energy level diagram for this reaction is shown.

R + S

energy
Y
P + Q

progress of reaction

Which statement about this reaction is correct?

A Arrow Y represents the activation energy.


B The energy given out forming the bonds in R and S is less than the energy used to break the
bonds in P and Q.
C The reaction is exothermic.
D The temperature increases when P reacts with Q because R and S have more energy than P
and Q.

© UCLES 2023 0653/23/M/J/23 [Turn over

53/693
Combined By Nesrine
8
2023-2017

20 Iron displaces copper from copper oxide.

Fe + CuO  FeO + Cu

Magnesium displaces iron from iron oxide.

Mg + FeO  Fe + MgO

Which statement about these reactions is correct?

A Copper oxide and iron oxide are being oxidised.


B Iron is a stronger oxidising agent than copper.
C Magnesium and iron are being reduced.
D Magnesium is a stronger reducing agent than iron.

21 Which two substances both react with dilute sulfuric acid to make the salt magnesium sulfate?

A magnesium carbonate and magnesium chloride


B magnesium chloride and magnesium nitrate
C magnesium oxide and magnesium carbonate
D magnesium oxide and magnesium nitrate

22 The results of two tests on a white solid are shown.

test result

1 add aqueous sodium hydroxide white precipitate formed


2 add dilute hydrochloric acid colourless gas formed

What is the white solid?

A iron(II) carbonate

B iron(II) chloride

C zinc carbonate
D zinc chloride

23 Which statement explains how a greater number of outer-shell electrons affects the metallic
character of an element?

A The element is more metallic because electrons are lost less easily.
B The element is more metallic because electrons are lost more easily.
C The element is less metallic because electrons are lost less easily.
D The element is less metallic because electrons are lost more easily.

© UCLES 2023 0653/23/M/J/23

54/693
Combined By Nesrine
9
2023-2017

24 Copper oxide is heated with carbon as shown.

copper oxide
and carbon
crucible

tripod

Bunsen burner

Which statement about this experiment is correct?

A A pink-brown solid is formed.


B Carbon is placed underneath the copper oxide so that the air can react with the hot copper
oxide.
C Carbon reacts with the air to form carbon dioxide which then reacts with the copper oxide.
D Copper is more reactive than carbon.

25 Some gases in air are listed.

1 carbon dioxide
2 oxygen
3 methane
4 sulfur dioxide

Which gases cause an enhanced greenhouse effect when their concentration in the air
increases?

A 1 and 2 B 1 and 3 C 2 and 4 D 3 and 4

26 Which statement about all alkanes is correct?

A They contain the same number of carbon atoms.


B They have different chemical properties.
C They have the same general formula.
D They have the same melting point.

© UCLES 2023 0653/23/M/J/23 [Turn over

55/693
Combined By Nesrine
10
2023-2017

27 Which structure represents a molecule that is not formed during cracking of large alkane
molecules?

A B C D
H H H
O
H C H C C H H
H H
H H H

28 The diagram shows the speed–time graph for a moving object.

speed 20
m/s

10

0
0 30
time / s

What is the distance travelled by the object in 30 s?

A 150 m B 300 m C 450 m D 600 m

29 Which statement about mass and weight is correct?


A Mass and weight are different types of force.
B The mass of an object depends on the strength of the gravitational field in which it is placed.
C The mass of an object is the same on the Moon as it is on the Earth.
D The unit of weight is the kilogram.

30 The diagram shows an extension–load graph for a spring.

Which labelled point shows the limit of proportionality for the spring?

D
extension

B
A

0
0 load

© UCLES 2023 0653/23/M/J/23

56/693
Combined By Nesrine
11
2023-2017

31 A block of metal has a mass of 2.0 kg. The area of contact between the block and a horizontal
surface is 100 cm2.

The gravitational field strength is 10 N / kg.

What is the pressure on the surface due to the block?

A 0.020 N / cm2 B 0.20 N / cm2 C 5.0 N / cm2 D 50 N / cm2

32 An object has speed v and kinetic energy E.

What is the mass of the object?


E 2E E 2E
A B C D
v v v2 v2

33 The generator in a power station is rotated by a turbine. Steam from boiling water rotates the
turbine.

Which energy source is used to produce electricity in this way?

A hydroelectric energy
B nuclear fission
C tidal energy
D wind energy

34 Which statement describes the molecules in a gas?

A They are close together and move about quickly.


B They are close together and move about slowly.
C They are far apart and move about quickly.
D They are far apart and move about slowly.

35 A student investigates the rate of evaporation of water.

Which changes produce the greatest increase in the rate of evaporation of the water?

A doubling its temperature and doubling its surface area


B doubling its temperature and halving its surface area
C halving its temperature and doubling its surface area
D halving its temperature and halving its surface area

© UCLES 2023 0653/23/M/J/23 [Turn over

57/693
Combined By Nesrine
12
2023-2017

36 Two resistors P and Q are connected in parallel to a cell, as shown.

The resistance of resistor P is greater than the resistance of resistor Q.

Which row gives the relationship between the currents in P and Q, and between the potential
differences across P and Q?

potential
current
difference (p.d.)

A P greater than Q P different to Q


B P greater than Q P the same as Q
C Q greater than P P different to Q
D Q greater than P P the same as Q

37 A sound wave passes through air.

Which type of wave is a sound wave and in which direction do the air particles vibrate?

type of wave direction of vibration

A longitudinal parallel to wave direction


B longitudinal perpendicular to wave direction
C transverse parallel to wave direction
D transverse perpendicular to wave direction

38 The speed of light c in a vacuum is 3.0  108 m / s.

Which row relates other speeds to c ?

speed of light speed of infrared


in glass waves in a vacuum

A equal to c equal to c
B equal to c different from c
C different from c equal to c
D different from c different from c

© UCLES 2023 0653/23/M/J/23

58/693
Combined By Nesrine
13
2023-2017

39 A variable power supply is connected to a resistor and there is a current in the resistor.

variable
power supply

current

The potential difference (p.d.) across the resistor is decreased.

The temperature of the resistor does not change.

What happens to the current in the resistor and what happens to the resistance of the resistor?

current resistance

A decreases increases
B decreases stays the same
C increases decreases
D increases stays the same

40 An electric heater has a label stating this information: 240 V, 2400 W, 10 A.

What is written on a fuse with a rating that is appropriate for use with this heater?

A 5A B 13 A C 230 V D 250 V

© UCLES 2023 0653/23/M/J/23

59/693
The Periodic Table of Elements
Group

© UCLES 2023
I II III IV V VI VII VIII
1 2

H He
hydrogen helium
Key 1 4
3 4 atomic number 5 6 7 8 9 10

Li Be atomic symbol B C N O F Ne
lithium beryllium name boron carbon nitrogen oxygen fluorine neon
7 9 relative atomic mass 11 12 14 16 19 20
11 12 13 14 15 16 17 18
Na Mg Al Si P S Cl Ar
sodium magnesium aluminium silicon phosphorus sulfur chlorine argon
23 24 27 28 31 32 35.5 40
19 20 21 22 23 24 25 26 27 28 29 30 31 32 33 34 35 36
K Ca Sc Ti V Cr Mn Fe Co Ni Cu Zn Ga Ge As Se Br Kr
potassium calcium scandium titanium vanadium chromium manganese iron cobalt nickel copper zinc gallium germanium arsenic selenium bromine krypton
39 40 45 48 51 52 55 56 59 59 64 65 70 73 75 79 80 84
37 38 39 40 41 42 43 44 45 46 47 48 49 50 51 52 53 54

Rb Sr Y Zr Nb Mo Tc Ru Rh Pd Ag Cd In Sn Sb Te I Xe
rubidium strontium yttrium zirconium niobium molybdenum technetium ruthenium rhodium palladium silver cadmium indium tin antimony tellurium iodine xenon
16

85 88 89 91 93 96 – 101 103 106 108 112 115 119 122 128 127 131

60/693
55 56 57–71 72 73 74 75 76 77 78 79 80 81 82 83 84 85 86

0653/23/M/J/23
lanthanoids
Cs Ba Hf Ta W Re Os Ir Pt Au Hg Tl Pb Bi Po At Rn
caesium barium hafnium tantalum tungsten rhenium osmium iridium platinum gold mercury thallium lead bismuth polonium astatine radon
133 137 178 181 184 186 190 192 195 197 201 204 207 209 – – –
87 88 89–103 104 105 106 107 108 109 110 111 112 113 114 115 116 117 118
actinoids
Fr Ra Rf Db Sg Bh Hs Mt Ds Rg Cn Nh Fl Mc Lv Ts Og
francium radium rutherfordium dubnium seaborgium bohrium hassium meitnerium darmstadtium roentgenium copernicium nihonium flerovium moscovium livermorium tennessine oganesson
– – – – – – – – – – – – – – – – –

57 58 59 60 61 62 63 64 65 66 67 68 69 70 71
lanthanoids La Ce Pr Nd Pm Sm Eu Gd Tb Dy Ho Er Tm Yb Lu
lanthanum cerium praseodymium neodymium promethium samarium europium gadolinium terbium dysprosium holmium erbium thulium ytterbium lutetium
139 140 141 144 – 150 152 157 159 163 165 167 169 173 175
89 90 91 92 93 94 95 96 97 98 99 100 101 102 103
actinoids Ac Th Pa U Np Pu Am Cm Bk Cf Es Fm Md No Lr
actinium thorium protactinium uranium neptunium plutonium americium curium berkelium californium einsteinium fermium mendelevium nobelium lawrencium
– 232 231 238 – – – – – – – – – – –
2023-2017

The volume of one mole of any gas is 24 dm3 at room temperature and pressure (r.t.p.).
Combined By Nesrine
Combined By Nesrine
2023-2017

Cambridge IGCSE™

COMBINED SCIENCE 0653/22


Paper 2 Multiple Choice (Extended) February/March 2022
45 minutes

You must answer on the multiple choice answer sheet.


*3062057543*

You will need: Multiple choice answer sheet


Soft clean eraser
Soft pencil (type B or HB is recommended)

INSTRUCTIONS
 There are forty questions on this paper. Answer all questions.
 For each question there are four possible answers A, B, C and D. Choose the one you consider correct
and record your choice in soft pencil on the multiple choice answer sheet.
 Follow the instructions on the multiple choice answer sheet.
 Write in soft pencil.
 Write your name, centre number and candidate number on the multiple choice answer sheet in the
spaces provided unless this has been done for you.
 Do not use correction fluid.
 Do not write on any bar codes.
 You may use a calculator.

INFORMATION
 The total mark for this paper is 40.
 Each correct answer will score one mark.
 Any rough working should be done on this question paper.
 The Periodic Table is printed in the question paper.

This document has 16 pages.

IB22 03_0653_22/3RP
© UCLES 2022 [Turn over

61/693
Combined By Nesrine
2
2023-2017

1 What is the function of the cell membrane?

A to control which substances move in and out of the cell


B to hold the DNA of the cell
C to hold the chlorophyll of the cell
D to store nutrients or waste products

2 The diagram shows three plant cells, P, Q and R.

cell P cell Q

highest medium
water water
potential potential

lowest
water
potential

cell R

What is the correct net movement of water by osmosis?

A B C D

cell P cell Q cell P cell Q cell P cell Q cell P cell Q

cell R cell R cell R cell R

© UCLES 2022 0653/22/F/M/22

62/693
Combined By Nesrine
3
2023-2017

3 The graph shows the rate of an enzyme-controlled reaction at different temperatures.

rate of reaction

X Y

temperature

Which row is correct at point X?

kinetic energy enzyme


of substrate denatured

A higher than at Y no
B higher than at Y yes
C lower than at Y no
D lower than at Y yes

4 The diagram shows a section through a leaf.

Which row correctly identifies the labelled parts of the leaf section?

X Y Z

A cuticle vascular bundle palisade mesophyll


B palisade mesophyll vascular bundle spongy mesophyll
C palisade mesophyll cuticle spongy mesophyll
D spongy mesophyll cuticle vascular bundle

© UCLES 2022 0653/22/F/M/22 [Turn over

63/693
Combined By Nesrine
4
2023-2017

5 Which row is correct for chemical digestion?

food enzyme substances produced

A fat protease fatty acids and glycerol


B fat lipase amino acids
C protein lipase fatty acids and glycerol
D protein protease amino acids

6 The diagram shows a plant in a container of water. The layer of oil stops the water in the
container from evaporating.

oil

container water

The initial mass of the container and its contents is 296 g.

After two hours, the mass of the container and its contents is 292 g.

What is the rate of transpiration in this time?

A 148 g of water per hour


B 146 g of water per hour
C 4 g of water per hour
D 2 g of water per hour

© UCLES 2022 0653/22/F/M/22

64/693
Combined By Nesrine
5
2023-2017

7 The diagram shows two different types of cell which line the trachea in the gas exchange system.

Which row describes the roles of X and Y?

X Y

A produces mucus traps pathogens


B produces mucus moves pathogens towards the mouth
C moves pathogens towards the mouth traps pathogens
D moves pathogens towards the mouth moves pathogens towards the mouth

8 In the equation for respiration shown, the components have been represented by numbers.

1 + 2  3 + 4

Each component has been given a letter, as shown.

W = carbon dioxide, X = glucose, Y = oxygen, Z = water

Which letter should be inserted into each position in the equation?

1 2 3 4

A W Y X Z
B W Z X Y
C X Y W Z
D X Z W Y

© UCLES 2022 0653/22/F/M/22 [Turn over

65/693
Combined By Nesrine
6
2023-2017

9 The shoot of a plant grows towards the light.

plant shoot light

Which diagram shows the correct distribution of auxin in this shoot?

A B C D
key
= auxin

10 Which features are correct for a wind-pollinated flower?

nectar petals

A absent small
B absent large
C present small
D present large

11 During pregnancy, what protects the embryo against toxins?

A amniotic fluid
B amniotic sac
C placenta
D umbilical cord

12 A human eats vegetables.

Which row describes the position of the human in the food chain?

consumer trophic level

A primary first
B primary second
C secondary first
D secondary second

© UCLES 2022 0653/22/F/M/22

66/693
Combined By Nesrine
7
2023-2017

13 Which labelled box represents plants in the carbon cycle?

carbon dioxide in
the atmosphere

A B C

14 A mixture contains two liquids.

One liquid has a boiling point of 120 C.

The other liquid has a boiling point of 160 C.

They are separated by fractional distillation.

water

heat heat heat


P Q R

Which apparatus is used to separate the two liquids?

A P and Q B P only C Q only D R only

© UCLES 2022 0653/22/F/M/22 [Turn over

67/693
Combined By Nesrine
8
2023-2017

15 Iodine is a non-metal.

It is a solid at room temperature.

What is a property of iodine?

A It can be stretched into a wire.


B It is brittle.
C It is a good conductor of electricity.
D It is a good conductor of heat.

16 The atomic number of argon is 18.

The mass number of argon is 40.

How many protons, neutrons and electrons are in an argon atom?

protons neutrons electrons

A 18 22 18
B 18 22 22
C 22 18 18
D 22 18 22

17 Which dot-and-cross diagram represents the bonding in nitrogen?

A B C D

N N N N N N N N

18 What happens at the anode during the electrolysis of molten aluminium oxide?

A Aluminium ions gain electrons to form aluminium atoms.


B Aluminium ions lose electrons to form aluminium atoms.
C Oxide ions gain electrons to form oxygen molecules.
D Oxide ions lose electrons to form oxygen molecules.

© UCLES 2022 0653/22/F/M/22

68/693
Combined By Nesrine
9
2023-2017

19 Steam condenses to form liquid water.

Which row shows the type of reaction and the energy level diagram for this change?

type of reaction energy level diagram

liquid water

energy
A endothermic
steam

progress of reaction

steam

energy
B endothermic liquid water

progress of reaction

steam

energy
C exothermic liquid water

progress of reaction

liquid water

energy
D exothermic
steam

progress of reaction

20 In which reaction is carbon dioxide not formed?

A adding hydrochloric acid to calcium


B adding hydrochloric acid to calcium carbonate
C burning coal in air
D burning methane in air

© UCLES 2022 0653/22/F/M/22 [Turn over

69/693
Combined By Nesrine
10
2023-2017

21 Which statements about elements in Group I of the Periodic Table are correct?

1 They become less reactive going down the group.


2 Sodium forms positive ions more easily than lithium.
3 Their melting points increase going down the group.
4 Rubidium is more dense than sodium.

A 1 and 2 B 1 and 3 C 2 and 4 D 3 and 4

22 Which statement about noble gases is correct?

A Argon is used in lamps because it gives out a bright light when it is heated.
B Helium is used to fill balloons because it is more dense than air.
C Krypton forms diatomic molecules because it is an unreactive gas.
D Neon is unreactive because it has a full outer shell of electrons.

23 X, Y and Z are three metals.

When Z is heated with the oxide of X, the metal X is formed.

When X is added to a solution of Y2+ ions, no reaction takes place.

What is the order of reactivity of the metals?

least most
reactive reactive

A X Y Z
B Y X Z
C Y Z X
D Z Y X

24 Which statement describes the correct order in which two reactions in a blast furnace occur?

A Carbon dioxide is reduced, then carbon monoxide is oxidised.


B Carbon monoxide is oxidised, then carbon dioxide is reduced.
C Carbon monoxide is reduced, then carbon is oxidised.
D Iron oxide is reduced, then carbon is oxidised.

© UCLES 2022 0653/22/F/M/22

70/693
Combined By Nesrine
11
2023-2017

25 Which statement about carbon dioxide is correct?

A Carbon dioxide is the only greenhouse gas.


B Carbon dioxide makes up approximately 4% of clean air.
C Increased concentrations of carbon dioxide enhance the greenhouse effect.
D The only source of carbon dioxide in the air is from motor vehicles.

26 Refinery gas contains methane.

Which statement about methane is correct?

A Its boiling point is high because it has strong forces of attraction between molecules.
B Its boiling point is high because it has weak forces of attraction between molecules.
C Its boiling point is low because it has strong forces of attraction between molecules.
D Its boiling point is low because it has weak forces of attraction between molecules.

27 Which description identifies the monomer that is used to form poly(ethene) by addition
polymerisation?

A saturated alkane
B saturated alkene
C unsaturated alkane
D unsaturated alkene

28 A boy walks for 120 s. The graph shows how the speed of the boy varies with time.

2
speed
m/s

0
0 20 40 60 80 100 120
time / s

What is the distance travelled by the boy while his speed is increasing?

A 20 m B 40 m C 80 m D 120 m

© UCLES 2022 0653/22/F/M/22 [Turn over

71/693
Combined By Nesrine
12
2023-2017

29 The diagram shows an extension–load graph for a spring.

20
extension / cm
15

10

0
0 1 2 3 4
load / N

What is the spring constant k of the spring?

A 0.20 N / cm B 5.0 N / cm C 40 N / cm D 80 N / cm

30 A brick of mass m has an area A in contact with the ground.

The gravitational force on unit mass is g.

Which expression gives the pressure on the ground due to the brick?
m Ag mg
A mgA B C D
Ag m A

31 An object of mass m is travelling at speed v at a constant height h above the ground.

Which expressions give the kinetic energy and the gravitational potential energy of the object?

gravitational
kinetic energy
potential energy
mg
A 1
2
mv 2
h
B 1
2
mv 2 mgh
mg
C 1
2
(mv)2
h
D 1
2
(mv)2 mgh

32 A motor is used to lift a load of 3000 N through a vertical distance of 40 m in 2.0 minutes.

How much useful power does the motor produce?

A 1000 W B 9000 W C 60 000 W D 240 000 W

© UCLES 2022 0653/22/F/M/22

72/693
Combined By Nesrine
13
2023-2017

33 Electricity is generated in different power stations that use coal, hydroelectric dams, nuclear
fission or geothermal resources.

How is a hydroelectric power station different from the other three types of power station?

A It is the only power station that uses steam as part of the process.
B It is the only power station that does not use steam as part of the process.
C It is the only power station that uses a renewable form of energy.
D It is the only power station that does not use a renewable form of energy.

34 A gas is trapped in a container.

Which properties of the molecules of the gas determine its temperature and its pressure?

temperature determined by pressure determined by

A the separation of the molecules how often the molecules collide with the container
B the separation of the molecules the forces between the molecules
C the speed of the molecules how often the molecules collide with the container
D the speed of the molecules the forces between the molecules

35 A wave travels through a substance from point X to point Y. The arrows show the direction in
which particles of the substance vibrate.

X Y

Which row states the type of wave involved and gives an example of this type of wave?

type of wave example

A longitudinal radio
B longitudinal sound
C transverse radio
D transverse sound

36 What is not part of the electromagnetic spectrum?

A gamma-radiation
B microwaves
C sound waves
D X-rays

© UCLES 2022 0653/22/F/M/22 [Turn over

73/693
Combined By Nesrine
14
2023-2017

37 An electrically charged student produces soap bubbles. When he holds his hand near the
bubbles, they move away quickly from his hand.

electrically
charged soap bubbles
student move away
quickly

insulator

For this movement of the bubbles to happen, which statement is correct?

A The bubbles must be negatively charged.


B The bubbles must be positively charged.
C The bubbles must have the opposite charge to the charge on the student.
D The bubbles must have the same charge as the charge on the student.

38 Which two changes together must cause the resistance of a wire to increase?

change in
change in
cross-sectional
length of wire
area of wire

A decrease decrease
B decrease increase
C increase decrease
D increase increase

© UCLES 2022 0653/22/F/M/22

74/693
Combined By Nesrine
15
2023-2017

39 The diagram shows a circuit with three ammeters. The readings on the ammeters are X, Y and Z.

A X

Y
A

Z
A

Which set of readings on the ammeters is possible?

X/A Y/A Z/A

A 2 3 5
B 3 2 5
C 3 3 3
D 5 2 3

40 A worker uses an electric drill continuously for 1.0 hour.

The current in the drill is 5.0 A when connected to the 250 V mains.

How much electrical energy is transferred by the drill?

A 1.25 kJ B 180 kJ C 4500 kJ D 5400 kJ

Permission to reproduce items where third-party owned material protected by copyright is included has been sought and cleared where possible. Every
reasonable effort has been made by the publisher (UCLES) to trace copyright holders, but if any items requiring clearance have unwittingly been included, the
publisher will be pleased to make amends at the earliest possible opportunity.

To avoid the issue of disclosure of answer-related information to candidates, all copyright acknowledgements are reproduced online in the Cambridge
Assessment International Education Copyright Acknowledgements Booklet. This is produced for each series of examinations and is freely available to download
at www.cambridgeinternational.org after the live examination series.

Cambridge Assessment International Education is part of Cambridge Assessment. Cambridge Assessment is the brand name of the University of Cambridge
Local Examinations Syndicate (UCLES), which is a department of the University of Cambridge.

© UCLES 2022 0653/22/F/M/22

75/693
Combined By Nesrine
2023-2017

Cambridge IGCSE™

COMBINED SCIENCE 0653/21


Paper 2 Multiple Choice (Extended) May/June 2022
45 minutes

You must answer on the multiple choice answer sheet.


*8065714787*

You will need: Multiple choice answer sheet


Soft clean eraser
Soft pencil (type B or HB is recommended)

INSTRUCTIONS
 There are forty questions on this paper. Answer all questions.
 For each question there are four possible answers A, B, C and D. Choose the one you consider correct
and record your choice in soft pencil on the multiple choice answer sheet.
 Follow the instructions on the multiple choice answer sheet.
 Write in soft pencil.
 Write your name, centre number and candidate number on the multiple choice answer sheet in the
spaces provided unless this has been done for you.
 Do not use correction fluid.
 Do not write on any bar codes.
 You may use a calculator.

INFORMATION
 The total mark for this paper is 40.
 Each correct answer will score one mark.
 Any rough working should be done on this question paper.
 The Periodic Table is printed in the question paper.

This document has 16 pages. Any blank pages are indicated.

IB22 06_0653_21/2RP
© UCLES 2022 [Turn over

76/693
Combined By Nesrine
2
2023-2017

1 What is the outermost layer of an animal cell and a plant cell?

animal cell plant cell

A cell membrane cell membrane


B cell membrane cell wall
C cell wall cell membrane
D cell wall cell wall

2 The plant cell in the diagram is in a concentrated salt solution.

Which arrow represents osmosis?

D
C

3 The enzyme salivary amylase starts digesting starchy foods in the mouth.

This stops when the food reaches the stomach.

Why does this happen?

A The acid in the stomach slows down all reactions.


B The shape of the active site of the enzyme is altered by the low pH.
C The kinetic energy of molecules is reduced by acids.
D The shape of the substrate molecules is changed.

4 Which foods are rich in carbohydrate?

1 eggs
2 meat
3 potatoes
4 rice

A 1 and 2 B 1 and 4 C 2 and 3 D 3 and 4

© UCLES 2022 0653/21/M/J/22

77/693
Combined By Nesrine
3
2023-2017

5 Which row is correct for a protease enzyme?

where secreted molecule acted on end product

A stomach protein amino acids


B pancreas protein fatty acid and glycerol
C stomach lipids amino acids
D pancreas lipids fatty acid and glycerol

6 The diagram shows the circulation of blood through the heart, lungs and other organs.

lungs
4 1

3 heart 2

other
organs

Which row gives a correct comparison of oxygen concentration in the blood in two of the
numbered vessels?

lower oxygen higher oxygen


concentration concentration

A 1 2
B 1 3
C 3 4
D 4 2

7 Which row shows the features of an efficient gas exchange surface in mammals?

alveoli wall blood supply surface area

A thick low large


B thick high small
C thin high large
D thin low small

© UCLES 2022 0653/21/M/J/22 [Turn over

78/693
Combined By Nesrine
4
2023-2017

8 Physical activity affects our rate and depth of breathing.

What happens during increased physical activity?

rate of breathing depth of breathing

A decreases decreases
B decreases increases
C increases decreases
D increases increases

9 Which responses occur as a result of adrenaline secretion?

increased decreased increased


breathing rate pupil diameter pulse rate

A    key
B     = does occur
C     = does not occur
D   

10 A plant seedling is pinned horizontally onto a damp board inside a light-proof box.

The diagrams show the seedling at the start of the experiment and after 72 hours.

start of experiment after 72 hours

light-proof box
root

shoot
pin damp board

Which response is shown by the root and the shoot?

root shoot

A gravitropism gravitropism
B gravitropism phototropism
C phototropism gravitropism
D phototropism phototropism

© UCLES 2022 0653/21/M/J/22

79/693
Combined By Nesrine
5
2023-2017

11 During sexual intercourse the penis transfers sperm cells to the vagina.

What is the pathway for sperm cells from their site of production to the vagina?

A sperm ducts  testes  urethra  vagina

B testes  sperm ducts  urethra  vagina

C testes  urethra  sperm ducts  vagina

D urethra  testes  sperm ducts  vagina

12 The diagram shows part of a food web.

fox eagle

stoat

rabbit
small bird
caterpillar
ant
beetle
locust

grass flowering plant tree

Which animal is a quaternary consumer only?

A eagle
B fox
C locust
D small bird

13 In the process of eutrophication, what causes the increased growth of producers?

A increased carbon dioxide availability


B increased decomposition
C increased nitrate ion availability
D increased oxygen availability

© UCLES 2022 0653/21/M/J/22 [Turn over

80/693
Combined By Nesrine
6
2023-2017

14 Which row describes a chemical change?

test result

A one end of a piece of aluminium is heated the other end gets hot
B calcium carbonate is heated carbon dioxide is made
C a piece of iron is heated it becomes more malleable
D a beaker of water is heated steam is made

15 What describes a solvent?

A a solid that dissolves in a liquid


B the amount of solid that dissolves in a liquid
C the liquid in which a solid dissolves
D the mixture formed when a solid dissolves in a liquid

16 The formula of sodium phosphate is Na3PO4.

The formula of aluminium chloride is Al Cl 3.

What is the formula of aluminium phosphate?

A Al PO4 B Al (PO4)3 C Al 2(PO4)3 D Al 3PO4

17 Magnesium reacts with dilute hydrochloric acid.

Which statement explains why the rate of this reaction increases when the concentration of the
acid is increased?

A A greater proportion of the particles have the minimum energy to react.


B The particles are closer together and the particles collide more frequently.
C The particles collide more frequently and more of the particles have the minimum energy to
react.
D The particles collide more frequently and the activation energy of the reaction is reduced.

18 In which equation is the underlined substance acting as an oxidising agent?

A CaCO3 + 2HCl  CaCl 2 + CO2 + H2O

B Fe2O3 + 3CO  2Fe + 3CO2

C 2Mg + O2  2MgO

D ZnO + C  Zn + CO

© UCLES 2022 0653/21/M/J/22

81/693
Combined By Nesrine
7
2023-2017

19 Three powders are added to dilute sulfuric acid, as shown.

magnesium magnesium magnesium


powder oxide carbonate
powder powder

dilute dilute dilute


sulfuric acid sulfuric acid sulfuric acid

Which powders react to produce water?

magnesium magnesium
magnesium
oxide carbonate

A    key
B     = does produce water
C     = does not produce water
D   

20 Elements X, Y and Z are in Group I of the Periodic Table.

Some information about these elements is shown.

melting point density


/ C g / cm3

X 1.53
Y 98
Z 63 0.86

Which row correctly identifies elements X, Y and Z?

X Y Z

A potassium sodium rubidium


B rubidium potassium sodium
C rubidium sodium potassium
D sodium rubidium potassium

© UCLES 2022 0653/21/M/J/22 [Turn over

82/693
Combined By Nesrine
8
2023-2017

21 The results of two tests on substance Q are shown.

test result
add dilute hydrochloric acid bubbles of colourless gas, R,
to solid Q which turns limewater milky
add aqueous sodium hydroxide
green precipitate
to a solution of Q

Which cation is present in Q and what is gas R?

cation present in Q gas R

A iron(II) carbon dioxide


B iron(II) chlorine
C iron(III) carbon dioxide
D iron(III) chlorine

22 The equation for the reaction between zinc and aqueous iron(II) sulfate is shown.

Zn(s) + FeSO4(aq)  ZnSO4(aq) + Fe(s)

Which statements about this reaction are correct?

1 Zinc atoms give electrons to iron ions.


2 Iron atoms have a greater tendency to form positive ions than zinc atoms.
3 Zinc displaces iron because it is more reactive than iron.

A 1 and 2 only B 1 and 3 only C 2 and 3 only D 1, 2 and 3

23 Iron is extracted from hematite in the blast furnace.

Coke and hematite are added at the top of the blast furnace, and hot air enters at the bottom.

Which statements are correct?

1 Coke burns to produce high temperatures.


2 Carbon monoxide is formed by the reaction of carbon with carbon dioxide.

3 Hematite contains iron(III) oxide which is oxidised by carbon monoxide.


4 The oxygen needed for the combustion of the coke comes from the hematite.

A 1 and 2 B 1 and 3 C 2 and 3 D 3 and 4

© UCLES 2022 0653/21/M/J/22

83/693
Combined By Nesrine
9
2023-2017

24 Which statement about a chemical test for water is correct?

A Anhydrous cobalt(II) chloride turns blue.

B Anhydrous cobalt(II) chloride turns white.

C Anhydrous copper(II) sulfate turns blue.

D Anhydrous copper(II) sulfate turns white.

25 Alkenes are hydrocarbons that belong to the same homologous series.

What are the general properties of a homologous series?

1 same general formula


2 same melting point
3 similar chemical properties

A 1 and 2 only B 1 and 3 only C 2 and 3 only D 1, 2 and 3

26 Methane, ethane and propane are all alkanes. Their formulae are shown.

methane, CH4
ethane, C2H6
propane, C3H8

Which statement is not correct?

A All three compounds are hydrocarbons.


B All three compounds burn.
C Methane is the main constituent of natural gas.
D Propane burns completely to form carbon dioxide and hydrogen.

27 Which substance rapidly turns aqueous bromine from orange to colourless?

A ethane
B ethanol
C ethene
D methane

© UCLES 2022 0653/21/M/J/22 [Turn over

84/693
Combined By Nesrine
10
2023-2017

28 Which statement about forces is always correct?

A A resultant force is needed to keep an object moving at constant speed in a straight line.
B Air resistance acting on an object falling in still air causes its speed to increase.
C Friction on an object sliding along rough ground acts in the opposite direction to its motion.
D No forces act on any object that is at rest.

29 A table of mass 20 kg is supported on four legs. The area of contact between each leg and the
ground is 1.0 × 10–3 m2.

The value of the gravitational field strength g is 10 N / kg.

How much pressure is exerted on the ground by each leg?

A 5000 Pa B 20 000 Pa C 50 000 Pa D 200 000 Pa

30 The diagram shows an extension–load graph for a spring.

Which labelled point is the limit of proportionality of the spring?

extension D

A
0
0 load

31 A boy of mass 80 kg is running at a speed of 4.0 m / s.

What is his kinetic energy?

A 160 J B 320 J C 640 J D 1280 J

© UCLES 2022 0653/21/M/J/22

85/693
Combined By Nesrine
2023-2017
11

32 A mechanic cannot remove a large steel nut from a steel bolt because it is too tight.

bolt
nut

What does the mechanic do to help remove the nut?

A cool the nut and heat the bolt


B heat the bolt only
C heat the nut and the bolt through the same temperature rise
D heat the nut only

33 A metal rod is heated at one end.

Thermal energy moves from the hotter end to the colder end.

How do molecules and free electrons transfer thermal energy along the rod?

molecules free electrons


A move from the hotter end move from the hotter end
to the colder end to the colder end
B move from the hotter end pass kinetic energy
to the colder end to neighbouring electrons
C pass kinetic energy move from the hotter end
to neighbouring molecules to the colder end
D pass kinetic energy pass kinetic energy
to neighbouring molecules to neighbouring electrons

34 Light travels at a speed of 3.0  108 m / s in a vacuum.

A radio station transmits radio waves at a frequency of 9.1  107 Hz.

What is the wavelength of the radio waves?

A 0.30 m B 0.33 m C 3.0 m D 3.3 m

35 Which region of the electromagnetic spectrum is used in remote controllers to control a


television?

A microwaves
B infrared
C ultraviolet
D visible light

© UCLES 2022 0653/21/M/J/22 [Turn over

86/693
www.xtrapapers.com
Combined By Nesrine
12
2023-2017

36 Where does sound travel at the greatest speed?

A in a gas
B in a liquid
C in a solid
D in a vacuum

37 Two balloons X and Y are suspended by insulating threads. They are each held near a negatively
charged balloon. The balloons hang as shown.

insulating insulating
threads threads

– – ––
– – – – ––
– – X – – Y
– – – – – –
– – – –

What is the charge on balloon X and what is the charge on balloon Y?

balloon X balloon Y

A negative negative
B negative positive
C positive negative
D positive positive

© UCLES 2022 0653/21/M/J/22

87/693
Combined By Nesrine
13
2023-2017

38 A circuit contains two lamps and a variable resistor.

lamp 1 lamp 2

The resistance of the variable resistor is increased.

What happens to the brightness of lamp 1 and what happens to the brightness of lamp 2?

brightness of lamp 1 brightness of lamp 2

A decreases decreases
B decreases increases
C no change decreases
D no change increases

© UCLES 2022 0653/21/M/J/22 [Turn over

88/693
Combined By Nesrine
2023-2017
14

39 The diagram shows a circuit that includes three resistors, a battery and three voltmeters.

Readings V1, V2 and V3 on the voltmeters, and currents I1, I2 and I3, are labelled.

reading = V1
V

I1

I2
V V
reading = V2 reading = V3

I3

Which row gives the relationships between the currents and between the voltages?

currents voltages

A I1 = I2 + I3 V1 = V2 + V3
B I1 = I2 + I3 V1 + V2 = V3
C I1 + I2 = I3 V1 = V2 + V3
D I1 + I2 = I3 V1 + V2 = V3

40 There is a current I in a resistor when there is a potential difference (p.d.) V across it.

Which quantity is equal to the product IV ?

A the charge passing through the resistor


B the energy transferred in the resistor
C the power transferred in the resistor
D the resistance of the resistor

© UCLES 2022 0653/21/M/J/22

89/693
Combined By Nesrine
2023-2017

Cambridge IGCSE™

COMBINED SCIENCE 0653/22


Paper 2 Multiple Choice (Extended) May/June 2022
45 minutes

You must answer on the multiple choice answer sheet.


*9353435643*

You will need: Multiple choice answer sheet


Soft clean eraser
Soft pencil (type B or HB is recommended)

INSTRUCTIONS
 There are forty questions on this paper. Answer all questions.
 For each question there are four possible answers A, B, C and D. Choose the one you consider correct
and record your choice in soft pencil on the multiple choice answer sheet.
 Follow the instructions on the multiple choice answer sheet.
 Write in soft pencil.
 Write your name, centre number and candidate number on the multiple choice answer sheet in the
spaces provided unless this has been done for you.
 Do not use correction fluid.
 Do not write on any bar codes.
 You may use a calculator.

INFORMATION
 The total mark for this paper is 40.
 Each correct answer will score one mark.
 Any rough working should be done on this question paper.
 The Periodic Table is printed in the question paper.

This document has 16 pages.

IB22 06_0653_22/2RP
© UCLES 2022 [Turn over

90/693
Combined By Nesrine
2
2023-2017

1 What is the outermost layer of an animal cell and a plant cell?

animal cell plant cell

A cell membrane cell membrane


B cell membrane cell wall
C cell wall cell membrane
D cell wall cell wall

2 The diagram shows a plant palisade mesophyll cell.

What will happen to structure X if this cell is immersed in distilled water or concentrated salty
water?

structure X
structure X
in concentrated
in distilled water
salty water

A shrink shrink
B shrink swell
C swell swell
D swell shrink

3 The enzyme salivary amylase starts digesting starchy foods in the mouth.

This stops when the food reaches the stomach.

Why does this happen?

A The acid in the stomach slows down all reactions.


B The shape of the active site of the enzyme is altered by the low pH.
C The kinetic energy of molecules is reduced by acids.
D The shape of the substrate molecules is changed.

© UCLES 2022 0653/22/M/J/22

91/693
Combined By Nesrine
3
2023-2017

4 Which two nutrients does a pregnant woman need in greater amounts to help her baby develop
bones and blood?

A calcium and iron


B calcium and vitamin D
C carbohydrate and iron
D carbohydrate and vitamin D

5 Which row is correct for mechanical digestion?

substance being
broken down using product of breakdown
broken down

A large food molecules enzymes small pieces of food


B large food molecules teeth small food molecules
C large pieces of food enzymes small food molecules
D large pieces of food teeth small pieces of food

6 What is a role of root hair cells?

A to decrease surface area, to decrease loss of water


B to decrease surface area, to increase uptake of water
C to increase surface area, to decrease loss of water
D to increase surface area, to increase uptake of water

7 The table shows two components of tobacco smoke and their possible effects on the body.

effects on the body


component in
tobacco smoke decreased oxygen increased
absorption by blood blood pressure

1 carbon monoxide  
2 carbon monoxide  
3 nicotine  
4 nicotine  

Which rows show the correct effects of each component?

A 1 and 3 B 1 and 4 C 2 and 3 D 2 and 4

© UCLES 2022 0653/22/M/J/22 [Turn over

92/693
Combined By Nesrine
4
2023-2017

8 Physical activity affects our rate and depth of breathing.

What happens during increased physical activity?

rate of breathing depth of breathing

A decreases decreases
B decreases increases
C increases decreases
D increases increases

9 Some examples of responses in the body are listed.

1 decreased pupil diameter


2 increased breathing rate
3 increased pulse rate

Which responses are caused by the secretion of adrenaline?

A 1, 2 and 3 B 1 and 2 only C 1 and 3 only D 2 and 3 only

10 Some examples of how parts of a plant grow are listed.

1 grow away from gravity


2 grow away from the direction of light
3 grow towards gravity
4 grow towards the direction of light

Which growth responses are due to gravitropism?

A 1, 2 and 4 B 1 only C 1 and 3 D 3 only

© UCLES 2022 0653/22/M/J/22

93/693
Combined By Nesrine
5
2023-2017

11 Which row is correct for a wind-pollinated flower?

pollen shape position of stigma

A outside of flower

B inside of flower

C inside of flower

D outside of flower

12 During sexual intercourse the penis transfers sperm cells to the vagina.

What is the pathway for sperm cells from their site of production to the vagina?

A sperm ducts  testes  urethra  vagina

B testes  sperm ducts  urethra  vagina

C testes  urethra  sperm ducts  vagina

D urethra  testes  sperm ducts  vagina

13 What is an ecosystem?

A a habitat containing organisms interacting together, in a given area


B a unit containing all of the organisms and their environment, interacting together, in a given
area
C an environment containing some organisms, interacting together
D the positions of organisms in a food web, interacting together, with the environment, in a
given area

© UCLES 2022 0653/22/M/J/22 [Turn over

94/693
Combined By Nesrine
6
2023-2017

14 Three changes are listed.

1 Dilute hydrochloric acid is reacted with aqueous sodium hydroxide.


2 The mixture formed is then heated until all of the water is evaporated.
3 The solid that is formed is then heated until it melts.

Which row describes changes 1, 2 and 3?

1 2 3

A chemical chemical physical


B chemical physical physical
C physical physical chemical
D physical chemical chemical

15 Substance Z exists as molecules that contain only one type of atom.

What is Z?

A a compound
B a mixture
C an element
D a noble gas

16 Which statement about the electrolysis of molten lead(II) bromide using carbon electrodes is
correct?
A Bromide ions gain electrons at the anode.
B Bromide ions lose electrons at the anode.
C Lead ions gain electrons at the anode.
D Lead ions lose electrons at the anode.

© UCLES 2022 0653/22/M/J/22

95/693
Combined By Nesrine
2023-2017
7

17 Zinc reacts with dilute hydrochloric acid to form hydrogen which is collected in a gas syringe.

Zn(s) + 2HCl (aq)  ZnCl 2(aq) + H2(g)

Which statement is correct?

A Larger pieces of zinc react faster than the same mass of smaller pieces because they have a
larger total surface area.
B When a catalyst is added, the time taken to collect 20 cm3 of hydrogen is reduced because
fewer particles have the activation energy.
C Hydrogen is produced faster when the acid is more concentrated because a larger proportion
of the particles have the activation energy.
D Raising the temperature reduces the time taken to collect 20 cm3 of hydrogen because more
particles have the activation energy.

18 Three powders are added to dilute sulfuric acid, as shown.

magnesium magnesium magnesium


powder oxide carbonate
powder powder

dilute dilute dilute


sulfuric acid sulfuric acid sulfuric acid

Which powders react to produce water?

magnesium magnesium
magnesium
oxide carbonate

A    key
B     = does produce water
C     = does not produce water
D   

© UCLES 2022 0653/22/M/J/22 [Turn over

96/693
Combined By Nesrine
2023-2017
8

19 Magnesium nitrate is produced by reacting magnesium oxide with dilute nitric acid.

Which process is used to produce a pure sample of magnesium nitrate crystals?

A Add excess dilute nitric acid to magnesium oxide, filter and boil the filtrate to dryness.
B Add excess dilute nitric acid to magnesium oxide, filter and evaporate the filtrate to the point
of crystallisation.
C Add excess magnesium oxide to dilute nitric acid, filter and boil the filtrate to dryness.
D Add excess magnesium oxide to dilute nitric acid, filter and evaporate the filtrate to the point
of crystallisation.

20 The results of two tests on substance Q are shown.

test result
add dilute hydrochloric acid bubbles of colourless gas, R,
to solid Q which turns limewater milky
add aqueous sodium hydroxide
green precipitate
to a solution of Q

Which cation is present in Q and what is gas R?

cation present in Q gas R

A iron(II) carbon dioxide


B iron(II) chlorine
C iron(III) carbon dioxide
D iron(III) chlorine

21 Indium is an element in the Periodic Table.

Which row describes the electronic structure and character of indium?

number of outer
character
shell electrons

A 3 metal
B 3 non-metal
C 5 metal
D 5 non-metal

© UCLES 2022 0653/22/M/J/22

97/693
Combined By Nesrine
9
2023-2017

22 Which statements about the reactivity series of metals are correct?

1 Iron is higher in the reactivity series than copper because it cannot be extracted
from its oxide using carbon.
2 Sodium is higher in the reactivity series than copper because it has a greater
tendency to form positive ions.
3 Magnesium is higher in the reactivity series than zinc because it can displace zinc
ions from aqueous solution.

A 1 and 2 only B 1 and 3 only C 2 and 3 only D 1, 2 and 3

23 Which equations represent reactions that take place in the blast furnace?

1 C + O2  CO2

2 2CO2  2CO + O2

3 2FeO + C  2Fe + CO2

4 Fe2O3 + 3CO  2Fe + 3CO2

A 1 and 2 B 1 and 4 C 2 and 3 D 3 and 4

24 Which colour change is seen when water is added to anhydrous cobalt(II) chloride?

A blue to pink
B blue to white
C pink to blue
D white to blue

25 Which statement about homologous series is correct?

A Alkanes and alkenes have the same general formula.


B Alkenes contain only double bonds.
C Alkanes and alkenes have similar chemical properties.
D Ethene, C2H4, and propene, C3H6, are members of the same homologous series.

© UCLES 2022 0653/22/M/J/22 [Turn over

98/693
Combined By Nesrine
10
2023-2017

26 Methane, ethane and propane are all alkanes. Their formulae are shown.

methane, CH4
ethane, C2H6
propane, C3H8

Which statement is not correct?

A All three compounds are hydrocarbons.


B All three compounds burn.
C Methane is the main constituent of natural gas.
D Propane burns completely to form carbon dioxide and hydrogen.

27 Which substance rapidly turns aqueous bromine from orange to colourless?

A ethane
B ethanol
C ethene
D methane

28 A student investigates a spring that obeys Hooke’s law.

The student suspends loads with different weights from the spring and measures the length of the
spring for each weight.

Lo is the length of the spring when there is no load on it.

Lw is the length of the spring when there is a load of weight W on it.

The graph shows the student’s results.

0
0 W

Which quantity is plotted on the y-axis?


Lw
A Lw – Lo B Lw + Lo C Lw D
Lo

© UCLES 2022 0653/22/M/J/22

99/693
Combined By Nesrine
2023-2017
11

29 A load of mass m is moved to the top of a slope of length p and vertical height q.

load

q
p
load

Which expression gives the gravitational potential energy gained by the load?

A mgp B mgq C mp D mq

30 Motor X does 300 J of work in 10 s.

Motor Y is twice as powerful as motor X.

Which row gives possible values for the work done and the time taken for motor Y?

work done / J time taken / s

A 300 5
B 300 20
C 600 5
D 600 20

31 Which group of energy sources consists of only renewable sources?

A geothermal, nuclear, solar


B geothermal, solar, wind
C nuclear, solar, wind
D oil, geothermal, solar

32 Air is trapped in a sealed glass bottle that has a fixed volume.

The temperature of the air in the bottle decreases.

Which statement describes what happens to the air in the bottle?

A The average separation of the molecules decreases and the pressure decreases.
B The average separation of the molecules decreases but the pressure remains the same.
C The average separation of the molecules remains the same but the pressure decreases.
D The average separation of the molecules remains the same and the pressure remains the
same.

© UCLES 2022 0653/22/M/J/22 [Turn over

100/693
Combined By Nesrine
2023-2017
12

33 A mechanic cannot remove a large steel nut from a steel bolt because it is too tight.

bolt
nut

What does the mechanic do to help remove the nut?

A cool the nut and heat the bolt


B heat the bolt only
C heat the nut and the bolt through the same temperature rise
D heat the nut only

34 Light travels at a speed of 3.0  108 m / s in a vacuum.

A radio station transmits radio waves at a frequency of 9.1  107 Hz.

What is the wavelength of the radio waves?

A 0.30 m B 0.33 m C 3.0 m D 3.3 m

35 Where does sound travel at the greatest speed?

A in a gas
B in a liquid
C in a solid
D in a vacuum

© UCLES 2022 0653/22/M/J/22

101/693
Combined By Nesrine
13
2023-2017

36 Two balloons X and Y are suspended by insulating threads. They are each held near a negatively
charged balloon. The balloons hang as shown.

insulating insulating
threads threads

– – ––
– – – – ––
– – X – – Y
– – – – – –
– – – –

What is the charge on balloon X and what is the charge on balloon Y?

balloon X balloon Y

A negative negative
B negative positive
C positive negative
D positive positive

37 A 1.0 m length of resistance wire with a cross-sectional area of 0.032 mm2 has a resistance of 15 .

Which other wire, made from the same material, also has a resistance of 15 ?

cross-sectional
length / m
area / mm2

A 0.50 0.0080
B 0.50 0.064
C 2.0 0.0080
D 2.0 0.064

© UCLES 2022 0653/22/M/J/22 [Turn over

102/693
Combined By Nesrine
14
2023-2017

38 A circuit contains two lamps and a variable resistor.

lamp 1 lamp 2

The resistance of the variable resistor is increased.

What happens to the brightness of lamp 1 and what happens to the brightness of lamp 2?

brightness of lamp 1 brightness of lamp 2

A decreases decreases
B decreases increases
C no change decreases
D no change increases

39 Two identical resistors R1 and R2 are connected to a 3.0 V battery as shown. The switch in the
circuit is open.

3.0 V

R1

R2

The switch is now closed.

What happens?

A The current in the battery halves.


B The current in the battery stays the same.
C The potential difference across R1 stays the same.
D The potential difference across R2 becomes 1.5 V.

© UCLES 2022 0653/22/M/J/22

103/693
Combined By Nesrine
2023-2017
15

40 A cell produces a potential difference (p.d.) V across a resistor of resistance R.

There is a current I in the resistor.

V
I

resistance = R

Which expression gives the energy transferred in the resistor in a time t ?


IV IR
A B IVt C D IRt
t t

Permission to reproduce items where third-party owned material protected by copyright is included has been sought and cleared where possible. Every
reasonable effort has been made by the publisher (UCLES) to trace copyright holders, but if any items requiring clearance have unwittingly been included, the
publisher will be pleased to make amends at the earliest possible opportunity.

To avoid the issue of disclosure of answer-related information to candidates, all copyright acknowledgements are reproduced online in the Cambridge
Assessment International Education Copyright Acknowledgements Booklet. This is produced for each series of examinations and is freely available to download
at www.cambridgeinternational.org after the live examination series.

Cambridge Assessment International Education is part of Cambridge Assessment. Cambridge Assessment is the brand name of the University of Cambridge
Local Examinations Syndicate (UCLES), which is a department of the University of Cambridge.

© UCLES 2022 0653/22/M/J/22

104/693
Combined By Nesrine
2023-2017

Cambridge IGCSE™

COMBINED SCIENCE 0653/23


Paper 2 Multiple Choice (Extended) May/June 2022
45 minutes

You must answer on the multiple choice answer sheet.


*5903761286*

You will need: Multiple choice answer sheet


Soft clean eraser
Soft pencil (type B or HB is recommended)

INSTRUCTIONS
 There are forty questions on this paper. Answer all questions.
 For each question there are four possible answers A, B, C and D. Choose the one you consider correct
and record your choice in soft pencil on the multiple choice answer sheet.
 Follow the instructions on the multiple choice answer sheet.
 Write in soft pencil.
 Write your name, centre number and candidate number on the multiple choice answer sheet in the
spaces provided unless this has been done for you.
 Do not use correction fluid.
 Do not write on any bar codes.
 You may use a calculator.

INFORMATION
 The total mark for this paper is 40.
 Each correct answer will score one mark.
 Any rough working should be done on this question paper.
 The Periodic Table is printed in the question paper.

This document has 16 pages. Any blank pages are indicated.

IB22 06_0653_23/2RP
© UCLES 2022 [Turn over

105/693
Combined By Nesrine
2
2023-2017

1 What is the outermost layer of an animal cell and a plant cell?

animal cell plant cell

A cell membrane cell membrane


B cell membrane cell wall
C cell wall cell membrane
D cell wall cell wall

2 What is the definition of diffusion?

A the downward movement of particles in the atmosphere


B the movement of particles down a concentration gradient
C the movement of molecules against a concentration gradient
D the movement of particles from a hotter to a cooler region

3 The enzyme salivary amylase starts digesting starchy foods in the mouth.

This stops when the food reaches the stomach.

Why does this happen?

A The acid in the stomach slows down all reactions.


B The shape of the active site of the enzyme is altered by the low pH.
C The kinetic energy of molecules is reduced by acids.
D The shape of the substrate molecules is changed.

4 Stomata are found in the leaves of plants.

What is their main function?

A gas exchange
B structural support
C transport of food molecules
D transport of water

© UCLES 2022 0653/23/M/J/22

106/693
Combined By Nesrine
3
2023-2017

5 The graph shows the daily energy requirements for two people, P and Q.

daily energy
requirements

P Q

Which statements could describe P and Q?

1 P is a mother who is breast-feeding her baby and Q is a mother who is


bottle-feeding her baby.
2 P is a 20-year-old adult male and Q is a 20-year-old adult female.
3 P is an office worker and Q is a professional cyclist.

A 1 and 2 only B 1 and 3 only C 2 and 3 only D 1, 2 and 3

6 The diagram shows the digestion of fat.

......1...... glycerol
......2...... and
......3......

large piece small pieces


of fat of fat

Which row completes gaps 1, 2 and 3 in the diagram?

1 2 3

A chemical digestion chemical digestion amino acids


B chemical digestion mechanical digestion fatty acids
C mechanical digestion mechanical digestion amino acids
D mechanical digestion chemical digestion fatty acids

7 Which statement about root hair cells is correct?

A They are present in large numbers to increase the absorption of water.


B They are only present in young seedlings before major roots grow.
C They are branched to help prevent the wind dislodging a plant.
D They have a large surface area to allow carbon dioxide uptake.

© UCLES 2022 0653/23/M/J/22 [Turn over

107/693
Combined By Nesrine
4
2023-2017

8 Physical activity affects our rate and depth of breathing.

What happens during increased physical activity?

rate of breathing depth of breathing

A decreases decreases
B decreases increases
C increases decreases
D increases increases

9 Some statements about adrenaline are listed.

1 It has one target organ.


2 It is a hormone.
3 It is produced by a gland.
4 It is transported in the blood.

Which statements are correct?

A 1, 2 and 3 only
B 1, 2 and 4 only
C 2, 3 and 4 only
D 1, 2, 3 and 4

© UCLES 2022 0653/23/M/J/22

108/693
Combined By Nesrine
5
2023-2017

10 The diagram shows a seed germinating in soil.

soil surface

shoot

root

Which tropic responses are taking place in the shoot and root while they are still underground?

shoot root

A gravitropism gravitropism
B gravitropism phototropism
C phototropism gravitropism
D phototropism phototropism

11 The diagram shows a typical wind-pollinated flower.

Which structure is the stigma?

© UCLES 2022 0653/23/M/J/22 [Turn over

109/693
Combined By Nesrine
6
2023-2017

12 During sexual intercourse the penis transfers sperm cells to the vagina.

What is the pathway for sperm cells from their site of production to the vagina?

A sperm ducts  testes  urethra  vagina

B testes  sperm ducts  urethra  vagina

C testes  urethra  sperm ducts  vagina

D urethra  testes  sperm ducts  vagina

13 The diagram shows a food chain from an ocean ecosystem.

microscopic plants  krill  small fish  tuna  shark

What is the trophic level of the tuna?

A primary consumer
B quaternary consumer
C secondary consumer
D tertiary consumer

14 When solid sodium carbonate is added to dilute hydrochloric acid, it dissolves and carbon dioxide
is given off.

Which statement is correct?

A This is a chemical change because sodium carbonate dissolves.


B This is a chemical change because the acid reacts with sodium carbonate.
C This is a physical change because sodium carbonate dissolves.
D This is a physical change because the acid reacts with sodium carbonate.

15 Which statement about non-metallic elements is correct?

A They are hard.


B They are malleable.
C They conduct electricity.
D They have low densities.

© UCLES 2022 0653/23/M/J/22

110/693
Combined By Nesrine
7
2023-2017

16 Which statement about the movement of particles during the electrolysis of dilute sulfuric acid is
correct?

A Anions move to the negative electrode and lose electrons.


B Electrons travel through the electrolyte from the cathode to the anode.
C Electrons travel through the external circuit from the anode to the cathode.
D Positive ions move to the anode and gain electrons.

17 Which row describes what happens to the frequency of collisions between reacting particles and
the energy of these collisions when the concentration of the reactants is decreased?

frequency of energy of
collisions collisions

A decreases decreases
B decreases no change
C increases decreases
D increases no change

18 The equation for a reaction that occurs in the blast furnace is shown.

Fe2O3 + 3CO  2Fe + 3CO2

Which statement about the reaction is correct?

A Fe2O3 is the oxidising agent.


B CO2 is the reducing agent.
C CO is reduced.
D Fe2O3 is oxidised.

© UCLES 2022 0653/23/M/J/22 [Turn over

111/693
Combined By Nesrine
8
2023-2017

19 Three powders are added to dilute sulfuric acid, as shown.

magnesium magnesium magnesium


powder oxide carbonate
powder powder

dilute dilute dilute


sulfuric acid sulfuric acid sulfuric acid

Which powders react to produce water?

magnesium magnesium
magnesium
oxide carbonate

A    key
B     = does produce water
C     = does not produce water
D   

20 Salt X is produced in the reaction between solid Y and acid Z.

The following method is used to prepare crystals of salt X.

● Solid Y is added to acid Z until no further reaction occurs.


● Any unreacted solid Y is removed by filtration.
● The filtrate is evaporated to the point of crystallisation and left to cool.
● Salt X crystallises.

Which row shows the substances that can be used to produce a salt by using this method?

solid Y acid Z salt X

A insoluble copper dilute hydrochloric acid soluble copper(II) chloride


B insoluble lead carbonate dilute sulfuric acid insoluble lead sulfate
C soluble sodium hydroxide dilute hydrochloric acid soluble sodium chloride
D insoluble zinc oxide dilute sulfuric acid soluble zinc sulfate

© UCLES 2022 0653/23/M/J/22

112/693
Combined By Nesrine
9
2023-2017

21 The results of two tests on substance Q are shown.

test result
add dilute hydrochloric acid bubbles of colourless gas, R,
to solid Q which turns limewater milky
add aqueous sodium hydroxide
green precipitate
to a solution of Q

Which cation is present in Q and what is gas R?

cation present in Q gas R

A iron(II) carbon dioxide


B iron(II) chlorine
C iron(III) carbon dioxide
D iron(III) chlorine

22 In which mixture is the aqueous metal ion displaced by the metal?

A Cu2+ and Zn B Fe2+ and Cu C Mg2+ and Zn D Zn2+ and Fe

23 Iron is extracted from iron(III) oxide in the blast furnace.

Which reaction produces the heat to maintain a high temperature in the furnace?

A calcium carbonate  calcium oxide + carbon dioxide

B carbon + oxygen  carbon dioxide

C iron(III) oxide + carbon monoxide  iron + carbon dioxide

D silicon dioxide + calcium oxide  calcium silicate

24 Which statement about the treatment of the water supply is correct?

A After filtration and chlorination, the water contains no impurities.


B Chlorine is added to remove dissolved impurities.
C Water is filtered and chlorinated to remove solids and kill bacteria.
D Water is filtered to remove dissolved impurities.

© UCLES 2022 0653/23/M/J/22 [Turn over

113/693
Combined By Nesrine
10
2023-2017

25 Which statement best describes the members of a homologous series?

A They have different general formulae and different chemical properties.


B They have different general formulae and similar chemical properties.
C They have the same general formula and different chemical properties.
D They have the same general formula and similar chemical properties.

26 Methane, ethane and propane are all alkanes. Their formulae are shown.

methane, CH4
ethane, C2H6
propane, C3H8

Which statement is not correct?

A All three compounds are hydrocarbons.


B All three compounds burn.
C Methane is the main constituent of natural gas.
D Propane burns completely to form carbon dioxide and hydrogen.

27 Which substance rapidly turns aqueous bromine from orange to colourless?

A ethane
B ethanol
C ethene
D methane

28 A climber climbs a mountain.

The gravitational field strength at the top of the mountain is less than it is at the bottom.

How do the mass and the weight of the climber at the top compare with the mass and the weight
at the bottom?

mass at top compared weight at top compared


with mass at bottom with weight at bottom

A less less
B less the same
C the same less
D the same the same

© UCLES 2022 0653/23/M/J/22

114/693
Combined By Nesrine
11
2023-2017

29 A stone is placed on a balance. The reading on the balance is shown.

stone

52.8 g
balance

The stone is lowered carefully into a measuring cylinder that contains 50 cm3 of water. The level
of the water in the measuring cylinder rises to the 72 cm3 mark.

What is the density of the stone?

A 0.42 g / cm3 B 0.73 g / cm3 C 1.36 g / cm3 D 2.40 g / cm3

30 A man lifts four heavy boxes from the ground onto a high shelf, one at a time.

When does he develop the greatest power?

A lifting a box of mass 20 kg in 3.0 s


B lifting a box of mass 20 kg in 4.0 s
C lifting a box of mass 30 kg in 3.0 s
D lifting a box of mass 30 kg in 4.0 s

31 The volume of a gas decreases and the temperature of the gas increases.

Which row describes the changes to the separation and to the speed of the gas molecules?

separation speed

A decreases decreases
B decreases increases
C increases decreases
D increases increases

© UCLES 2022 0653/23/M/J/22 [Turn over

115/693
Combined By Nesrine
2023-2017
12

32 A mechanic cannot remove a large steel nut from a steel bolt because it is too tight.

bolt
nut

What does the mechanic do to help remove the nut?

A cool the nut and heat the bolt


B heat the bolt only
C heat the nut and the bolt through the same temperature rise
D heat the nut only

33 What is the main method of thermal energy transfer in liquids?

A conduction
B convection
C absorption
D radiation

34 Light travels at a speed of 3.0  108 m / s in a vacuum.

A radio station transmits radio waves at a frequency of 9.1  107 Hz.

What is the wavelength of the radio waves?

A 0.30 m B 0.33 m C 3.0 m D 3.3 m

35 A student uses a converging lens as a magnifying glass to view an insect.

Which labelled point is a possible position for the image of the head of the insect?

converging lens
A
head
B
C

insect D

student’s eye

© UCLES 2022 0653/23/M/J/22

116/693
Combined By Nesrine
13
2023-2017

36 Where does sound travel at the greatest speed?

A in a gas
B in a liquid
C in a solid
D in a vacuum

37 Two balloons X and Y are suspended by insulating threads. They are each held near a negatively
charged balloon. The balloons hang as shown.

insulating insulating
threads threads

– – ––
– – – – ––
– – X – – Y
– – – – – –
– – – –

What is the charge on balloon X and what is the charge on balloon Y?

balloon X balloon Y

A negative negative
B negative positive
C positive negative
D positive positive

© UCLES 2022 0653/23/M/J/22 [Turn over

117/693
Combined By Nesrine
14
2023-2017

38 A circuit contains two lamps and a variable resistor.

lamp 1 lamp 2

The resistance of the variable resistor is increased.

What happens to the brightness of lamp 1 and what happens to the brightness of lamp 2?

brightness of lamp 1 brightness of lamp 2

A decreases decreases
B decreases increases
C no change decreases
D no change increases

39 A torch (flashlight) contains a 3.0 V battery. When the torch is switched on the current in the
battery is 0.50 A.

How much energy is transferred by the battery in 1.0 minute?

A 1.5 J B 6.0 J C 90 J D 360 J

40 What is the purpose of a fuse in an electric circuit?

A to disconnect the circuit if the current becomes too large


B to increase the voltage if the current becomes too small
C to prevent someone cutting the insulation of the wiring
D to stop water getting into the circuit

© UCLES 2022 0653/23/M/J/22

118/693
Combined By Nesrine
2023-2017

Cambridge IGCSE™

COMBINED SCIENCE 0653/21


Paper 2 Multiple Choice (Extended) October/November 2022
45 minutes

You must answer on the multiple choice answer sheet.


*8139398620*

You will need: Multiple choice answer sheet


Soft clean eraser
Soft pencil (type B or HB is recommended)

INSTRUCTIONS
 There are forty questions on this paper. Answer all questions.
 For each question there are four possible answers A, B, C and D. Choose the one you consider correct
and record your choice in soft pencil on the multiple choice answer sheet.
 Follow the instructions on the multiple choice answer sheet.
 Write in soft pencil.
 Write your name, centre number and candidate number on the multiple choice answer sheet in the
spaces provided unless this has been done for you.
 Do not use correction fluid.
 Do not write on any bar codes.
 You may use a calculator.

INFORMATION
 The total mark for this paper is 40.
 Each correct answer will score one mark.
 Any rough working should be done on this question paper.
 The Periodic Table is printed in the question paper.

This document has 16 pages. Any blank pages are indicated.

IB22 11_0653_21/2RP
© UCLES 2022 [Turn over

119/693
Combined By Nesrine
2
2023-2017

1 What are characteristics of all living organisms?

A breathing, excretion, nutrition


B excretion, growth, nutrition
C reproduction, respiration, germination
D secretion, growth, sensitivity

2 Which row describes a correct structural adaptation for red blood cells and for cells lining the
trachea?

red blood cells cells lining the trachea

A nucleus absent cilia present


B nucleus present cilia present
C nucleus absent small surface area
D nucleus present small surface area

3 A student tests samples of four different foods.

Which row is the correct result for a sample containing only fat and starch?

Benedict’s biuret ethanol iodine


solution test emulsion solution

A blue purple clear blue-black


B blue blue cloudy blue-black
C red purple clear brown
D red blue cloudy brown

© UCLES 2022 0653/21/O/N/22

120/693
Combined By Nesrine
2023-2017
3

4 Which graph correctly shows the effect of varying light intensity on the rate of photosynthesis?

A B

rate of rate of
photosynthesis photosynthesis

0 0
0 light intensity 0 light intensity

C D

rate of rate of
photosynthesis photosynthesis

0 0
0 light intensity 0 light intensity

5 Which row correctly matches the type of digestion to its effect on the food particles?

type of digestion makes smaller makes soluble

A chemical no yes
B chemical yes yes
C mechanical no no
D mechanical yes yes

© UCLES 2022 0653/21/O/N/22 [Turn over

121/693
Combined By Nesrine
4
2023-2017

6 A student investigates factors affecting the rate of transpiration in a plant.

Which row shows the correct effects on the rates of transpiration?

increase temperature decrease humidity

A decreases decreases
B decreases increases
C increases decreases
D increases increases

7 Which structures in the circulatory system ensure the one-way flow of blood?

A arteries
B atria
C valves
D ventricles

8 When at rest, a student measures his rate of breathing and the volume of air inspired.

volume of air inspired


number of breaths
per minute
per minute
/ dm3

12 6.0

He then runs 400 m and immediately measures his breathing again.

Which set of results does he obtain?

volume of air inspired


number of breaths
per minute
per minute
/ dm3

A 12 6.0
B 12 12.5
C 30 6.0
D 30 12.5

© UCLES 2022 0653/21/O/N/22

122/693
Combined By Nesrine
5
2023-2017

9 What is the word equation for aerobic respiration?

A carbon dioxide + chlorophyll  glucose + oxygen

B carbon dioxide + glucose  oxygen + water

C glucose + oxygen  carbon dioxide + water

D oxygen + light energy  carbon dioxide + water

10 Which row shows the correct changes when the hormone adrenaline is secreted?

blood breathing pulse


glucose rate rate

A decreases decreases decreases


B increases decreases increases
C decreases increases decreases
D increases increases increases

11 Light shines on a shoot tip from the direction shown.

light
X Y

After three days, the shoot tip has bent towards the light.

What is the reason for this change?

A Auxin moves away from the light causing cell elongation in area Y.
B Auxin moves away from the light preventing cell elongation in area Y.
C Auxin moves towards the light causing cell elongation in area X.
D Auxin moves towards the light preventing cell elongation in area X.

© UCLES 2022 0653/21/O/N/22 [Turn over

123/693
Combined By Nesrine
6
2023-2017

12 Which environmental conditions are necessary for seeds to germinate?

suitable
oxygen sunlight water
temperature

A yes yes yes no


B yes yes no yes
C no no yes yes
D no yes no yes

13 What is the function of amniotic fluid?

A to cushion the fetus


B to remove carbon dioxide
C to supply nutrients
D to supply oxygen

14 Which particles are present in the nuclei of hydrogen atoms, 11H?

A electrons and neutrons


B electrons only
C protons and neutrons
D protons only

15 Which statement explains the difference in boiling point between ionic and covalent compounds?

A The boiling point of covalent compounds is higher because covalent bonds are stronger than
ionic bonds.
B The boiling point of covalent compounds is higher because the attractive forces between
covalent molecules are stronger than the attractive forces between ions.
C The boiling point of ionic compounds is higher because the attractive forces between ions
are stronger than covalent bonds.
D The boiling point of ionic compounds is higher because the attractive forces between ions
are stronger than the attractive forces between covalent molecules.

© UCLES 2022 0653/21/O/N/22

124/693
Combined By Nesrine
7
2023-2017

16 Aqueous sodium hydroxide is added to aqueous aluminium sulfate.

A white precipitate of aluminium hydroxide is formed.

The ionic equation for this reaction is shown.

Al 3+(aq) + 3OH–(aq)  Al (OH)3(s)

What is the full symbol equation for this reaction?

A Al SO4(aq) + 3NaOH(aq)  Al (OH)3(s) + Na3SO4(aq)

B Al 2(SO4)3(aq) + NaOH(aq)  Al (OH)3(s) + Na2SO4(aq)

C Al 2(SO4)3(aq) + 6NaOH(aq)  2Al (OH)3(s) + 3Na2SO4(aq)

D Al 3(SO4)2(aq) + 9NaOH(aq)  3Al (OH)3(s) + 2Na2SO4(aq)

17 Concentrated aqueous sodium chloride is electrolysed using inert electrodes.

Which statement is correct?

A Chloride ions lose electrons at the cathode.


B Hydrogen ions gain electrons at the cathode.
C Oxide ions gain electrons at the anode.
D Sodium ions gain electrons at the cathode.

18 Which statements explain why the rate of a reaction increases when the temperature is
increased?

1 More of the colliding molecules have enough energy to react.


2 The molecules are closer together, so they collide more frequently.
3 The molecules are further apart, so they collide less frequently.
4 The molecules are moving faster, so they collide more frequently.

A 1 and 2 B 1 and 4 C 2 and 3 D 3 and 4

© UCLES 2022 0653/21/O/N/22 [Turn over

125/693
Combined By Nesrine
2023-2017
8

19 The word equation represents the reaction between substance J and hydrochloric acid.

substance J + hydrochloric acid  magnesium chloride + hydrogen

What is substance J?

A magnesium
B magnesium carbonate
C magnesium hydroxide
D magnesium oxide

20 Which pair of gases can be identified using damp litmus paper and limewater?

A carbon dioxide and hydrogen


B chlorine and carbon dioxide
C chlorine and oxygen
D hydrogen and chlorine

21 Aqueous potassium halides are mixed with aqueous halogens as listed.

1 potassium bromide + iodine


2 potassium chloride + bromine
3 potassium iodide + chlorine
4 potassium iodide + bromine

Which mixtures produce a chemical reaction?

A 1 and 2 B 1 and 4 C 2 and 3 D 3 and 4

22 Which properties are shown by transition elements?

1 They form coloured compounds.


2 They have low melting points.
3 They have low densities.
4 They can act as catalysts.

A 1 and 2 B 1 and 4 C 2 and 3 D 3 and 4

© UCLES 2022 0653/21/O/N/22

126/693
Combined By Nesrine
9
2023-2017

23 Which statement about reactions in the blast furnace is not correct?

A Carbon is oxidised by oxygen.


B Carbon is oxidised by carbon dioxide.
C Carbon dioxide is reduced by iron oxide.
D Iron oxide is reduced by carbon monoxide.

24 Damp air is passed through a tube containing blue copper(II) sulfate and blue cobalt(II) chloride.

damp air

blue blue
copper(II) sulfate cobalt(II) chloride

What is observed?

copper(II) sulfate cobalt(II) chloride

A turns white turns pink


B turns white no change
C no change turns pink
D no change no change

25 Naphtha and diesel oil are two fractions obtained from the fractional distillation of petroleum.

Which statement about naphtha and diesel oil is correct?

A Molecules in naphtha are smaller than molecules in diesel oil.


B Naphtha has a higher boiling point range than diesel oil.
C Naphtha is less volatile than diesel oil.
D Naphtha is removed from the fractionating column at a lower level than diesel oil.

26 The formula of the hydrocarbon octane is C8H18.

What are the products of the complete combustion of octane?

A carbon and hydrogen


B carbon and water
C carbon dioxide and water
D carbon monoxide and water

© UCLES 2022 0653/21/O/N/22 [Turn over

127/693
Combined By Nesrine
10
2023-2017

27 Which process is an example of thermal decomposition?

A cracking an alkane

B electrolysis of molten lead(II) bromide


C extraction of iron in the blast furnace
D fractional distillation of petroleum

28 A pendulum swings repeatedly from P to Q and back to P.

P Q

A stop-watch is used to find the period of the pendulum.

Which method gives the most accurate value for the period?

A timing how long it takes to go from P to Q


B timing how long it takes to go from P to Q to P
C timing how long it takes to go from P to Q to P 10 times, and dividing this time by 10
D timing how long it takes to go from P to Q to P 10 times, and multiplying this time by 10

29 When a spring is stretched by a force of 20 N, its length increases from 3.2 cm to 5.4 cm.

What is the spring constant of the spring?

A 0.11 N / cm B 0.27 N / cm C 3.7 N / cm D 9.1 N / cm

© UCLES 2022 0653/21/O/N/22

128/693
Combined By Nesrine
11
2023-2017

30 The diagram shows a tank containing a liquid.

tank

liquid

The base of the tank is rectangular and has dimensions 0.20 m by 0.50 m.

The mass of the liquid is 60 kg and its depth is 0.10 m.

Gravitational field strength g = 10 N / kg.

What is the pressure exerted on the base of the tank by the liquid?

A 60 Pa B 600 Pa C 6000 Pa D 60 000 Pa

31 A piece of scientific equipment is taken from the Earth to a distant planet.

Which row describes the properties of the equipment on the distant planet?

mass weight

A   key
B    = the same as on Earth
C    = different on each planet
D  

32 For which energy resource is energy stored as gravitational potential energy?

A geothermal energy
B hydroelectric energy
C nuclear fission
D wind energy

© UCLES 2022 0653/21/O/N/22 [Turn over

129/693
Combined By Nesrine
12
2023-2017

33 Air is trapped in a cylinder by a piston. The piston is moved so that the volume of the trapped air
decreases. The pressure of the trapped air increases but the temperature of the trapped air does
not change.

trapped air piston

cylinder

What happens to the average speed of the air particles and what happens to the average
distance between them?

average speed average distance


of particles between particles

A does not change decreases


B does not change increases
C increases decreases
D increases increases

34 In which states of matter is thermal energy transferred because of changes in the density of the
medium?

A solids and liquids only


B liquids and gases only
C gases and solids only
D solids, liquids and gases

35 Light takes 500 s to travel from the Sun to the Earth.

What is the distance from the Sun to the Earth?

A 1.7  105 m

B 6.0  105 m

C 3.0  108 m

D 1.5  1011 m

© UCLES 2022 0653/21/O/N/22

130/693
Combined By Nesrine
13
2023-2017

36 A sound wave passes through air.

The diagram shows the arrangement of the air particles at one moment.

Three regions are labelled X, Y and Z.

X Y Z

In which region is there a rarefaction and which distance is equal to the wavelength of the sound
wave?

rarefaction wavelength

A X XY
B X XZ
C Y XY
D Y XZ

37 In which circuit is there a current of 2.0 A?

A B C D
6.0 V 0.50 V 3.0 V 1.5 V

3.0 : 1.5 : 6.0 : 0.50 :

38 The resistance of a wire depends on its length and on its diameter.

Which row shows two changes that both increase the resistance of the wire?

change to length change to diameter

A decrease decrease
B decrease increase
C increase decrease
D increase increase

© UCLES 2022 0653/21/O/N/22 [Turn over

131/693
Combined By Nesrine
14
2023-2017

39 A 20 V power supply provides a current of 5.0 A for 1.0 minute.

How much energy does the power supply transfer?

A 4.0 J B 100 J C 240 J D 6000 J

40 Why is the electricity supply to a mains circuit fitted with a fuse?

A to increase the current in the circuit


B to increase the resistance of the circuit
C to maintain a constant current in the circuit
D to prevent overheating of the cables in the circuit

© UCLES 2022 0653/21/O/N/22

132/693
Combined By Nesrine
2023-2017

Cambridge IGCSE™

COMBINED SCIENCE 0653/22


Paper 2 Multiple Choice (Extended) October/November 2022
45 minutes

You must answer on the multiple choice answer sheet.


*9053236720*

You will need: Multiple choice answer sheet


Soft clean eraser
Soft pencil (type B or HB is recommended)

INSTRUCTIONS
 There are forty questions on this paper. Answer all questions.
 For each question there are four possible answers A, B, C and D. Choose the one you consider correct
and record your choice in soft pencil on the multiple choice answer sheet.
 Follow the instructions on the multiple choice answer sheet.
 Write in soft pencil.
 Write your name, centre number and candidate number on the multiple choice answer sheet in the
spaces provided unless this has been done for you.
 Do not use correction fluid.
 Do not write on any bar codes.
 You may use a calculator.

INFORMATION
 The total mark for this paper is 40.
 Each correct answer will score one mark.
 Any rough working should be done on this question paper.
 The Periodic Table is printed in the question paper.

This document has 16 pages. Any blank pages are indicated.

IB22 11_0653_22/2RP
© UCLES 2022 [Turn over

133/693
Combined By Nesrine
2
2023-2017

1 What are characteristics of all living organisms?

A breathing, excretion, nutrition


B excretion, growth, nutrition
C reproduction, respiration, germination
D secretion, growth, sensitivity

2 Which row describes a correct structural adaptation for red blood cells and for cells lining the
trachea?

red blood cells cells lining the trachea

A nucleus absent cilia present


B nucleus present cilia present
C nucleus absent small surface area
D nucleus present small surface area

3 Food tests are carried out on a biscuit.

The results of the food tests are shown.

test for colour observed

fat white emulsion


protein blue
reducing sugar orange
starch blue-black

Which biological molecules are present in the biscuit?

reducing
fat protein starch
sugar

A    
B    
C    
D    

© UCLES 2022 0653/22/O/N/22

134/693
Combined By Nesrine
3
2023-2017

4 Which substance in leaves traps light energy for use in photosynthesis?

A carbohydrate
B carbon
C carbon dioxide
D chlorophyll

5 Which types of malnutrition could lead to constipation and scurvy?

constipation scurvy

A excess of fibre lack of vitamin C


B excess of fibre lack of vitamin D
C lack of fibre lack of vitamin C
D lack of fibre lack of vitamin D

6 Where is amylase active in the alimentary canal?

stomach small intestine

A  
B  
C  
D  

© UCLES 2022 0653/22/O/N/22 [Turn over

135/693
Combined By Nesrine
2023-2017
4

7 A student tests her exhaled breath by blowing through a straw into some limewater.

straw

limewater

Which statements are correct about this test?

colour of limewater colour of limewater


what the test shows
at start of test at end of test
A colourless milky white carbon dioxide is
present in the exhaled breath
B colourless milky white water vapour is present in
the exhaled breath
C milky white colourless carbon dioxide is
present in the exhaled breath
D milky white colourless water vapour is present in
the exhaled breath

8 What is the word equation for aerobic respiration?

A carbon dioxide + chlorophyll  glucose + oxygen

B carbon dioxide + glucose  oxygen + water

C glucose + oxygen  carbon dioxide + water

D oxygen + light energy  carbon dioxide + water

9 What are two effects of the secretion of adrenaline on the human body?

A decreased blood glucose concentration and decreased pulse rate


B decreased blood glucose concentration and increased pulse rate
C increased blood glucose concentration and decreased pulse rate
D increased blood glucose concentration and increased pulse rate

© UCLES 2022 0653/22/O/N/22

136/693
Combined By Nesrine
5
2023-2017

10 Light shines on a shoot tip from the direction shown.

light
X Y

After three days, the shoot tip has bent towards the light.

What is the reason for this change?

A Auxin moves away from the light causing cell elongation in area Y.
B Auxin moves away from the light preventing cell elongation in area Y.
C Auxin moves towards the light causing cell elongation in area X.
D Auxin moves towards the light preventing cell elongation in area X.

11 What are two features of sexual reproduction?

feature 1 feature 2

A fusion of two identical nuclei requires two different parents


B fusion of two zygotes offspring are genetically identical
C offspring are genetically different fusion of two different nuclei
D only requires a single parent development from a single zygote

12 Which row is correct for the female gamete?

released in can move


large numbers by itself

A  
B  
C  
D  

© UCLES 2022 0653/22/O/N/22 [Turn over

137/693
Combined By Nesrine
6
2023-2017

13 The diagram shows a food web.

foxes

lizards

kestrels
carnivorous
insects spiders stoats

herbivorous voles rabbits


insects

vegetation

Which organisms in this web are quaternary consumers?

A carnivorous insects and foxes


B foxes and lizards
C kestrels and stoats
D lizards and stoats

14 An atom of aluminium and an atom of fluorine are represented as shown.

27 Al 19F
13 9

Which statement is not correct?

A The aluminium atom contains four more electrons than the fluorine atom.
B The aluminium atom contains four more protons than the fluorine atom.
C The aluminium atom contains eight more neutrons than the fluorine atom.
D The aluminium atom contains eight more nucleons than the fluorine atom.

© UCLES 2022 0653/22/O/N/22

138/693
Combined By Nesrine
7
2023-2017

15 Which row describes and explains the difference in melting points between ionic and covalent
compounds?

melting point reason


A ionic compounds have ionic bonds are stronger
higher melting points than covalent bonds
B ionic compounds have attractive forces between ions are stronger
higher melting points than attractive forces between molecules
C ionic compounds have ionic bonds are weaker
lower melting points than covalent bonds
D ionic compounds have attractive forces between ions are weaker
lower melting points than attractive forces between molecules

16 Potassium phosphate is an ionic compound used in fertilisers.

Phosphate ions have the symbol PO43–.

What is the formula for potassium phosphate?

A KPO4 B K(PO4)3 C K2PO4 D K3PO4

17 Which equation represents the process that occurs at the cathode during the electrolysis of
concentrated aqueous sodium chloride?

A 2O2–  O2 + 4e–

B 2Cl –  Cl 2 + 2e–

C Na+ + e–  Na

D 2H+ + 2e–  H2

© UCLES 2022 0653/22/O/N/22 [Turn over

139/693
Combined By Nesrine
2023-2017
8

18 The reaction between two aqueous reactants, P and Q, is carried out in two different beakers.

1000 cm3
500 cm3

beaker 1 beaker 2

The temperature and the number of particles of P and Q are the same in both beakers.

Which statements about the collisions between the reacting particles in the two beakers must be
correct?

1 The average energy of the collisions is greater in beaker 2.


2 The frequency of the collisions is greater in beaker 2.
3 The proportion of the collisions that result in a reaction is greater in beaker 2.

A 1 only B 2 only C 1 and 3 D 2 and 3

19 The word equation represents the reaction between substance J and hydrochloric acid.

substance J + hydrochloric acid  magnesium chloride + hydrogen

What is substance J?

A magnesium
B magnesium carbonate
C magnesium hydroxide
D magnesium oxide

20 Which pair of gases can be identified using damp litmus paper and limewater?

A carbon dioxide and hydrogen


B chlorine and carbon dioxide
C chlorine and oxygen
D hydrogen and chlorine

© UCLES 2022 0653/22/O/N/22

140/693
Combined By Nesrine
9
2023-2017

21 Which statement about the elements in Group VII is correct?

A Bromine reacts with potassium chloride to make chlorine.


B Chlorine is the least reactive element in Group VII.
C Chlorine reacts with potassium iodide to make iodine.
D Potassium bromide reacts with all of the elements in Group VII.

22 Element X has a high density and conducts electricity when solid and when molten.

Where in the Periodic Table is element X placed?

A Group 0
B Group I
C halogens
D transition elements

23 Which metal cannot be extracted from its ore by heating with carbon?

A Al B Cu C Fe D Zn

24 A few drops of liquid X are added to a white solid.

The white solid turns blue.

Which statements are correct?

1 The white solid is copper(II) sulfate.


2 Liquid X is water.

3 Liquid X turns blue cobalt(II) chloride paper pink.

A 1 and 2 only B 1 and 3 only C 2 and 3 only D 1, 2 and 3

25 Bitumen and gasoline are fractions obtained from petroleum by fractional distillation.

Which statement explains why the boiling range of the bitumen fraction is higher than the boiling
range of the gasoline fraction?

A It contains smaller molecules.


B It leaves the fractional distillation column at the bottom.
C Its molecules have greater forces of attraction.
D Its molecules have stronger covalent bonds.

© UCLES 2022 0653/22/O/N/22 [Turn over

141/693
Combined By Nesrine
10
2023-2017

26 The formula of the hydrocarbon octane is C8H18.

What are the products of the complete combustion of octane?

A carbon and hydrogen


B carbon and water
C carbon dioxide and water
D carbon monoxide and water

27 Which process is an example of thermal decomposition?

A cracking an alkane

B electrolysis of molten lead(II) bromide


C extraction of iron in the blast furnace
D fractional distillation of petroleum

28 The diagram shows a speed–time graph for a car.

24
speed
m/s

0
0 8.0
time / s

What is the distance travelled by the car between time = 0 and time = 8.0 s?

A 96 m B 120 m C 144 m D 192 m

© UCLES 2022 0653/22/O/N/22

142/693
Combined By Nesrine
11
2023-2017

29 A spring is stretched by a force F. The graph shows how the length l of the spring changes
with F.

14
l / cm
12

10

0
0 1.0 2.0 3.0 4.0 5.0
F/N

What is the spring constant of the spring?

A 0.42 N / cm B 0.63 N / cm C 1.6 N / cm D 2.4 N / cm

30 A piece of scientific equipment is taken from the Earth to a distant planet.

Which row describes the properties of the equipment on the distant planet?

mass weight

A   key
B    = the same as on Earth
C    = different on each planet
D  

31 Which statement about water is correct?

A It boils at 0 C and melts at 100 C.

B It boils at 0 C and melts at –100 C.

C It boils at 100 C and melts at –100 C.

D It boils at 100 C and melts at 0 C.

© UCLES 2022 0653/22/O/N/22 [Turn over

143/693
Combined By Nesrine
12
2023-2017

32 The volume of a gas is increased but its temperature remains the same.

What happens to the molecules of the gas?

A They move closer together.


B They move further apart.
C They move more quickly.
D They move more slowly.

33 Which row compares how well a dull, black surface and a shiny, white surface emit and absorb
thermal radiation?

emitting absorbing
thermal radiation thermal radiation

A dull, black is better dull, black is better


B dull, black is better shiny, white is better
C shiny, white is better dull, black is better
D shiny, white is better shiny, white is better

34 The diagram represents a water wave that is moving at a speed of 6.0 m / s.

1.2 m

0.80 m distance

What is the frequency of the wave?

A 3.0 Hz B 4.8 Hz C 5.0 Hz D 7.5 Hz

35 Which statement about sound is not correct?

A A sound wave of frequency 2000 Hz can be heard by a healthy human ear.


B Sound waves can travel through a vacuum.
C The loudness of a sound depends on the amplitude of the sound wave.
D The pitch of a sound depends on the frequency of the sound wave.

© UCLES 2022 0653/22/O/N/22

144/693
Combined By Nesrine
13
2023-2017

36 A circuit consists of a resistor, a switch and a battery. The switch is closed.

Which expression is used to calculate the charge that passes through the resistor?

A charge = current  voltage across the resistor

current
B charge =
voltage across the resistor

C charge = current  time for which the switch is closed

current
D charge =
time for which the switch is closed

37 In which circuit is there a current of 2.0 A?

A B C D
6.0 V 0.50 V 3.0 V 1.5 V

3.0 : 1.5 : 6.0 : 0.50 :

38 The resistance of a wire depends on its length and on its diameter.

Which row shows two changes that both increase the resistance of the wire?

change to length change to diameter

A decrease decrease
B decrease increase
C increase decrease
D increase increase

39 A 20 V power supply provides a current of 5.0 A for 1.0 minute.

How much energy does the power supply transfer?

A 4.0 J B 100 J C 240 J D 6000 J

© UCLES 2022 0653/22/O/N/22 [Turn over

145/693
Combined By Nesrine
14
2023-2017

40 Why is the electricity supply to a mains circuit fitted with a fuse?

A to increase the current in the circuit


B to increase the resistance of the circuit
C to maintain a constant current in the circuit
D to prevent overheating of the cables in the circuit

© UCLES 2022 0653/22/O/N/22

146/693
Combined By Nesrine
2023-2017

Cambridge IGCSE™

COMBINED SCIENCE 0653/23


Paper 2 Multiple Choice (Extended) October/November 2022
45 minutes

You must answer on the multiple choice answer sheet.


*6478710885*

You will need: Multiple choice answer sheet


Soft clean eraser
Soft pencil (type B or HB is recommended)

INSTRUCTIONS
 There are forty questions on this paper. Answer all questions.
 For each question there are four possible answers A, B, C and D. Choose the one you consider correct
and record your choice in soft pencil on the multiple choice answer sheet.
 Follow the instructions on the multiple choice answer sheet.
 Write in soft pencil.
 Write your name, centre number and candidate number on the multiple choice answer sheet in the
spaces provided unless this has been done for you.
 Do not use correction fluid.
 Do not write on any bar codes.
 You may use a calculator.

INFORMATION
 The total mark for this paper is 40.
 Each correct answer will score one mark.
 Any rough working should be done on this question paper.
 The Periodic Table is printed in the question paper.

This document has 16 pages. Any blank pages are indicated.

IB22 11_0653_23/2RP
© UCLES 2022 [Turn over

147/693
Combined By Nesrine
2
2023-2017

1 What are characteristics of all living organisms?

A breathing, excretion, nutrition


B excretion, growth, nutrition
C reproduction, respiration, germination
D secretion, growth, sensitivity

2 Which row describes a correct structural adaptation for red blood cells and for cells lining the
trachea?

red blood cells cells lining the trachea

A nucleus absent cilia present


B nucleus present cilia present
C nucleus absent small surface area
D nucleus present small surface area

3 From which kind of molecule are enzymes made?

A glucose
B glycogen
C fat
D protein

4 A student investigates the effect of changing the light intensity on the rate of photosynthesis.

Which environmental conditions need to be kept constant in this investigation?

A carbon dioxide concentration, light intensity and temperature


B carbon dioxide concentration and temperature only
C carbon dioxide concentration and light intensity only
D light intensity and temperature only

5 Which condition is caused by a lack of vitamin D?

A anaemia
B constipation
C rickets
D scurvy

© UCLES 2022 0653/23/O/N/22

148/693
Combined By Nesrine
3
2023-2017

6 Which statement about digestion is correct?

A Chemical digestion occurs in the liver.


B Chemical digestion only occurs in the mouth.
C Mechanical digestion occurs in the large intestine.
D Mechanical digestion occurs in the mouth and stomach.

7 Which row shows the conditions that lead to the slowest rate of transpiration of a plant?

humidity temperature
of air of air
/% / C

A 30 10
B 70 20
C 30 20
D 70 10

8 The diagram shows a section through the heart.

valve 1
valve 4

valve 3 valve 2

When valve 1 is open, which other valves are open and which are closed?

valve 2 valve 3 valve 4

A closed closed open


B closed open closed
C open closed open
D open open closed

© UCLES 2022 0653/23/O/N/22 [Turn over

149/693
Combined By Nesrine
4
2023-2017

9 The diagram shows the cross-section of an alveolus in the lung.

air flow

blood flow

blood flow

Which statement is correct?


A Carbon dioxide levels are higher at Z than at X.
B Carbon dioxide levels are higher at X than at Y.
C Carbon dioxide moves by diffusion from Y into the blood.
D Carbon dioxide moves by osmosis from the blood into Y.

10 What is the word equation for aerobic respiration?

A carbon dioxide + chlorophyll  glucose + oxygen

B carbon dioxide + glucose  oxygen + water

C glucose + oxygen  carbon dioxide + water

D oxygen + light energy  carbon dioxide + water

11 How does the body respond to being frightened?

decreased
increased increased widened
blood glucose
breathing rate pulse rate pupils
concentration

A    
B    
C    
D    

© UCLES 2022 0653/23/O/N/22

150/693
Combined By Nesrine
5
2023-2017

12 Light shines on a shoot tip from the direction shown.

light
X Y

After three days, the shoot tip has bent towards the light.

What is the reason for this change?

A Auxin moves away from the light causing cell elongation in area Y.
B Auxin moves away from the light preventing cell elongation in area Y.
C Auxin moves towards the light causing cell elongation in area X.
D Auxin moves towards the light preventing cell elongation in area X.

13 The diagram shows the human male reproductive system.

Which label is correct?

D
urethra
transports sperm

A
prostate gland
produces sperm

B
scrotum keeps
testis at body
temperature
C
testis
produces gametes

© UCLES 2022 0653/23/O/N/22 [Turn over

151/693
Combined By Nesrine
6
2023-2017

23
14 A sodium atom is represented by 11 Na.

Which row shows the number of electrons, protons and neutrons in this atom?

electrons protons neutrons

A 10 11 12
B 11 11 12
C 11 12 11
D 12 12 23

15 Which dot-and-cross diagram represents the bonding in a molecule of nitrogen?

A B C D

N N N N N N N N

16 Dilute hydrochloric acid reacts with aqueous sodium carbonate to form sodium chloride,
carbon dioxide and water.

What is the ionic equation for this reaction?

A CO32–(aq) + 2H+(aq)  CO2(g) + H2O(l)

B CO32–(aq) + 2Na+(aq) + 2H+(aq) + 2Cl –(aq)  2Na+(aq) + 2Cl –(aq)

C Cl –(aq) + Na+(aq)  NaCl (aq)

D Na2CO3(aq) + 2HCl (aq)  2NaCl (aq) + CO2(g) + H2O(l)

17 Concentrated aqueous sodium chloride is electrolysed using inert electrodes.

Which statement about this process is correct?

A Chloride ions lose electrons at the cathode.


B Hydrogen ions gain electrons at the cathode.
C Oxide ions lose electrons at the anode.
D Sodium ions gain electrons at the anode.

© UCLES 2022 0653/23/O/N/22

152/693
Combined By Nesrine
7
2023-2017

18 Excess limestone is added to 50 cm3 1 mol / dm3 hydrochloric acid.

The volume of gas produced is measured over time.

The results produce line X on the graph.

Which line is produced when excess limestone is added to 50 cm3 0.5 mol / dm3 hydrochloric acid
at the same temperature?

A X B

volume
C D
of gas

0
0 time

19 The word equation represents the reaction between substance J and hydrochloric acid.

substance J + hydrochloric acid  magnesium chloride + hydrogen

What is substance J?

A magnesium
B magnesium carbonate
C magnesium hydroxide
D magnesium oxide

20 Which pair of gases can be identified using damp litmus paper and limewater?

A carbon dioxide and hydrogen


B chlorine and carbon dioxide
C chlorine and oxygen
D hydrogen and chlorine

© UCLES 2022 0653/23/O/N/22 [Turn over

153/693
Combined By Nesrine
8
2023-2017

21 Fluorine is at the top of Group VII in the Periodic Table.

Which statements about fluorine are correct?

1 It is a solid at room temperature.


2 It has a dark colour.
3 It is a very reactive element.
4 It exists as diatomic molecules.

A 1 and 2 B 1 and 3 C 2 and 4 D 3 and 4

22 What are properties of transition elements?

1 They can act as catalysts.


2 They form coloured compounds.
3 They have high densities.

A 1 and 2 only B 1 and 3 only C 2 and 3 only D 1, 2 and 3

23 Iron is extracted from iron ore in a blast furnace.

Which substance is not one of the reactants added to the blast furnace?

A carbon
B carbon dioxide
C hematite
D oxygen

24 Which statements about clean air are correct?

1 It consists of 78% nitrogen.


2 It contains a small amount of argon.
3 It contains a small amount of carbon monoxide.
4 It is mostly a compound of nitrogen and oxygen.

A 1 and 2 B 1 and 4 C 2 and 3 D 3 and 4

© UCLES 2022 0653/23/O/N/22

154/693
Combined By Nesrine
9
2023-2017

25 Petroleum is separated into fractions by fractional distillation.

Which row describes the properties of the molecules in a single fraction?

boiling points number of carbon atoms


of the molecules in the molecules

A same same
B same similar
C similar same
D similar similar

26 The formula of the hydrocarbon octane is C8H18.

What are the products of the complete combustion of octane?

A carbon and hydrogen


B carbon and water
C carbon dioxide and water
D carbon monoxide and water

27 Which process is an example of thermal decomposition?

A cracking an alkane

B electrolysis of molten lead(II) bromide


C extraction of iron in the blast furnace
D fractional distillation of petroleum

© UCLES 2022 0653/23/O/N/22 [Turn over

155/693
Combined By Nesrine
10
2023-2017

28 The diagrams show two distance–time graphs and two speed–time graphs.

Which graph represents the motion of an object that is moving with a constant acceleration that is
greater than zero?

A B

distance distance

0 0
0 time 0 time

C D

speed speed

0 0
0 time 0 time

29 A man has a mass of 76 kg and an average density of 950 kg / m3.

The man steps into a bath that is completely full of water. Water spills over the edge of the bath
as the man lies down slowly and becomes completely submerged.

What is the volume of water that spills over the edge of the bath?

A 0.072 m3 B 0.080 m3 C 12.5 m3 D 72.2 m3

30 An aircraft is flying forwards at a steady speed in a straight line.

Which statement about the resultant force on the aircraft is correct?

A The resultant force is a backwards force caused by air resistance.


B The resultant force is a forwards force caused by the engines.
C The resultant force is a downwards force caused by the weight of the aircraft.
D The resultant force is zero because all the forces on the aircraft cancel.

© UCLES 2022 0653/23/O/N/22

156/693
Combined By Nesrine
11
2023-2017

31 A piece of scientific equipment is taken from the Earth to a distant planet.

Which row describes the properties of the equipment on the distant planet?

mass weight

A   key
B    = the same as on Earth
C    = different on each planet
D  

32 A liquid in a beaker evaporates as air moves over it.

movement of air

liquid

Which change increases the rate of evaporation?

A decreasing the speed of the air over the beaker


B decreasing the temperature of the liquid in the beaker
C increasing the quantity of liquid in the beaker
D increasing the width of the beaker

33 A hot object is placed in a vacuum. It loses thermal energy by radiation.

What is this radiation?

A infrared waves
B microwaves
C ultraviolet waves
D X-rays

34 The crests of a wave on the sea reach the beach at a rate of 6.0 crests every 60 seconds. The
distance between one crest and the next is 20 m.

What is the speed of the wave?

A 0.30 m / s B 2.0 m / s C 120 m / s D 200 m / s

© UCLES 2022 0653/23/O/N/22 [Turn over

157/693
Combined By Nesrine
12
2023-2017

35 Sound travels at different speeds in different substances.

What are possible values for the speed of sound in air, in water and in steel?

speed in air speed in water speed in steel


m/s m/s m/s
A 330 6000 1500
B 330 1500 6000
C 6000 1500 330
D 6000 330 1500

36 In which circuit is there a current of 2.0 A?

A B C D
6.0 V 0.50 V 3.0 V 1.5 V

3.0 : 1.5 : 6.0 : 0.50 :

37 The resistance of a wire depends on its length and on its diameter.

Which row shows two changes that both increase the resistance of the wire?

change to length change to diameter

A decrease decrease
B decrease increase
C increase decrease
D increase increase

38 A resistor of resistance 30  and a resistor of resistance 60  are connected in parallel.

What is their combined resistance?

A 0.050  B 20  C 45  D 90 

© UCLES 2022 0653/23/O/N/22

158/693
Combined By Nesrine
13
2023-2017

39 A 20 V power supply provides a current of 5.0 A for 1.0 minute.

How much energy does the power supply transfer?

A 4.0 J B 100 J C 240 J D 6000 J

40 Why is the electricity supply to a mains circuit fitted with a fuse?

A to increase the current in the circuit


B to increase the resistance of the circuit
C to maintain a constant current in the circuit
D to prevent overheating of the cables in the circuit

© UCLES 2022 0653/23/O/N/22

159/693
The Periodic Table of Elements
Group
I II III IV V VI VII VIII

© UCLES 2022
1 2

H He
hydrogen helium
Key 1 4
3 4 atomic number 5 6 7 8 9 10

Li Be atomic symbol B C N O F Ne
lithium beryllium name boron carbon nitrogen oxygen fluorine neon
7 9 relative atomic mass 11 12 14 16 19 20
11 12 13 14 15 16 17 18
Na Mg Al Si P S Cl Ar
sodium magnesium aluminium silicon phosphorus sulfur chlorine argon
23 24 27 28 31 32 35.5 40
19 20 21 22 23 24 25 26 27 28 29 30 31 32 33 34 35 36
K Ca Sc Ti V Cr Mn Fe Co Ni Cu Zn Ga Ge As Se Br Kr
potassium calcium scandium titanium vanadium chromium manganese iron cobalt nickel copper zinc gallium germanium arsenic selenium bromine krypton
39 40 45 48 51 52 55 56 59 59 64 65 70 73 75 79 80 84
37 38 39 40 41 42 43 44 45 46 47 48 49 50 51 52 53 54

Rb Sr Y Zr Nb Mo Tc Ru Rh Pd Ag Cd In Sn Sb Te I Xe
rubidium strontium yttrium zirconium niobium molybdenum technetium ruthenium rhodium palladium silver cadmium indium tin antimony tellurium iodine xenon
85 88 89 91 93 96 – 101 103 106 108 112 115 119 122 128 127 131
16

55 56 57–71 72 73 74 75 76 77 78 79 80 81 82 83 84 85 86
lanthanoids

160/693
Cs Ba Hf Ta W Re Os Ir Pt Au Hg Tl Pb Bi Po At Rn

0653/23/O/N/22
caesium barium hafnium tantalum tungsten rhenium osmium iridium platinum gold mercury thallium lead bismuth polonium astatine radon
133 137 178 181 184 186 190 192 195 197 201 204 207 209 – – –
87 88 89–103 104 105 106 107 108 109 110 111 112 114 116
actinoids
Fr Ra Rf Db Sg Bh Hs Mt Ds Rg Cn Fl Lv
francium radium rutherfordium dubnium seaborgium bohrium hassium meitnerium darmstadtium roentgenium copernicium flerovium livermorium
– – – – – – – – – – – – –

57 58 59 60 61 62 63 64 65 66 67 68 69 70 71
lanthanoids La Ce Pr Nd Pm Sm Eu Gd Tb Dy Ho Er Tm Yb Lu
lanthanum cerium praseodymium neodymium promethium samarium europium gadolinium terbium dysprosium holmium erbium thulium ytterbium lutetium
139 140 141 144 – 150 152 157 159 163 165 167 169 173 175
89 90 91 92 93 94 95 96 97 98 99 100 101 102 103
actinoids Ac Th Pa U Np Pu Am Cm Bk Cf Es Fm Md No Lr
actinium thorium protactinium uranium neptunium plutonium americium curium berkelium californium einsteinium fermium mendelevium nobelium lawrencium
– 232 231 238 – – – – – – – – – – –
2023-2017

The volume of one mole of any gas is 24 dm3 at room temperature and pressure (r.t.p.).
Combined By Nesrine
Combined By Nesrine
2023-2017

Cambridge IGCSE™

COMBINED SCIENCE 0653/22


Paper 2 Multiple Choice (Extended) February/March 2021
45 minutes

You must answer on the multiple choice answer sheet.


*8785854026*

You will need: Multiple choice answer sheet


Soft clean eraser
Soft pencil (type B or HB is recommended)

INSTRUCTIONS
 There are forty questions on this paper. Answer all questions.
 For each question there are four possible answers A, B, C and D. Choose the one you consider correct
and record your choice in soft pencil on the multiple choice answer sheet.
 Follow the instructions on the multiple choice answer sheet.
 Write in soft pencil.
 Write your name, centre number and candidate number on the multiple choice answer sheet in the
spaces provided unless this has been done for you.
 Do not use correction fluid.
 Do not write on any bar codes.
 You may use a calculator.

INFORMATION
 The total mark for this paper is 40.
 Each correct answer will score one mark.
 Any rough working should be done on this question paper.
 The Periodic Table is printed in the question paper.

This document has 20 pages. Any blank pages are indicated.

IB21 03_0653_22/3RP
© UCLES 2021 [Turn over

161/693
Combined By Nesrine
2
2023-2017

1 What are the characteristics of living organisms?

sensitivity
excretion growth movement nutrition reproduction respiration
/ response

A       
B       
C       
D       

2 The diagram shows a plant cell.

Which structures are also found in an animal cell?

A 1 and 3 B 1 and 4 C 2 and 3 D 2 and 4

3 An animal cell is placed in a solution with a water potential lower than its cytoplasm.

Which statement correctly describes the movement of water across the cell membrane?

A equal movement in and out of the cell


B net movement into the cell
C net movement out of the cell
D no movement in or out of the cell

© UCLES 2021 0653/22/F/M/21

162/693
Combined By Nesrine
3
2023-2017

4 Which graph shows the effects of pH on the activity of an enzyme?

A B

enzyme enzyme
activity activity

pH pH

C D

enzyme enzyme
activity activity

pH pH

5 The diagram shows a section of a leaf.

Which letter is the epidermis?

A
B

6 What does a lack of iron in the diet cause?

A bleeding from the gums


B constipation
C less oxygen transported to cells
D weakening of bones

© UCLES 2021 0653/22/F/M/21 [Turn over

163/693
Combined By Nesrine
4
2023-2017

7 Which diagram represents the digestion of food molecules in the alimentary canal?

8 Which statement explains why the rate of transpiration changes on a hot dry day?

A a decrease in diffusion of water vapour through the stomata


B a decrease in evaporation of water vapour through the stomata
C an increase in evaporation of water from the mesophyll cells
D a decrease in diffusion of water from the mesophyll cells

9 What are the functions of the cilia and mucus in the gas exchange system of mammals?

cilia mucus

A make mucus trap pathogens


B make mucus move cilia
C move mucus trap pathogens
D move mucus move cilia

© UCLES 2021 0653/22/F/M/21

164/693
Combined By Nesrine
5
2023-2017

10 The table shows some data recorded by a scientist about a student.

pulse rate
pupil size in eye
/ beats per minute

68

The scientist then frightens the student with a sudden loud noise.

Which row shows the results immediately after the loud noise?

pulse rate
pupil size in eye
/ beats per minute

A 60

B 80

C 60

D 80

© UCLES 2021 0653/22/F/M/21 [Turn over

165/693
Combined By Nesrine
6
2023-2017

11 The diagram shows a flower.

X Y

Which row is correct for the flower illustrated?

X Y

A captures insect-carried pollen produces rough pollen grains


B captures wind-carried pollen produces smooth pollen grains
C produces rough pollen grains captures insect-carried pollen
D produces smooth pollen grains captures wind-carried pollen

12 Materials are exchanged between a mother and her fetus across the placenta.

Which row shows the overall direction of movement of these materials?

mother to fetus fetus to mother

A amino acids glucose


B amino acids urea
C carbon dioxide glucose
D carbon dioxide urea

13 What is the name given to the position of an organism in a food chain?

A ecosystem
B energy content
C trophic level
D tropism

© UCLES 2021 0653/22/F/M/21

166/693
Combined By Nesrine
7
2023-2017

14 Which methods of separation depend on the substances in a mixture having different boiling
points?

A crystallisation and distillation


B evaporation and filtration
C fractional distillation and chromatography
D fractional distillation and distillation

15 Which row describes an element and a compound?

element compound
A contains more than contains elements
one type of atom chemically combined
B contains more than contains elements
one type of atom mixed together
C contains only one contains elements
type of atom chemically combined
D contains only one contains elements
type of atom mixed together

16 Potassium chloride is a solid.

Hydrogen chloride is a gas.

Which statement explains why potassium chloride has a much higher boiling point than
hydrogen chloride?

A Covalent bonding is stronger than ionic bonding.


B Covalent bonds are stronger than the attractive forces between molecules.
C Ionic bonding is stronger than covalent bonding.
D Ionic bonds are stronger than the attractive forces between molecules.

17 Ammonium phosphate has the formula (NH4)3PO4.

Which row shows the formulae of ammonium ions and phosphate ions?

ammonium phosphate

A NH4+ PO4–
B NH43+ PO43–
C NH4+ PO43–
D NH43+ PO4–

© UCLES 2021 0653/22/F/M/21 [Turn over

167/693
Combined By Nesrine
8
2023-2017

18 Which process occurs during electrolysis?

A Anions gain electrons at the anode.


B Anions lose electrons at the anode.
C Cations gain electrons at the anode.
D Cations lose electrons at the cathode.

19 Which energy level diagram represents an exothermic reaction with the greatest activation
energy?

A B

energy energy

progress of reaction progress of reaction

C D

energy energy

progress of reaction progress of reaction

© UCLES 2021 0653/22/F/M/21

168/693
Combined By Nesrine
9
2023-2017

20 Zinc reacts with dilute hydrochloric acid.

Which row explains the effect of increasing the temperature on this reaction?

frequency of collisions number of particles


between reacting possessing the minimum
particles energy for the reaction

A decreases increases
B decreases stays the same
C increases increases
D increases stays the same

21 Copper is extracted from its ore by heating with carbon.

What is the role of the carbon in this process?

A fuel
B electrolyte
C oxidising agent
D reducing agent

22 A mixture of ammonium carbonate and ammonium chloride is heated with aqueous


sodium hydroxide.

Which gas is produced?

A ammonia
B carbon dioxide
C chlorine
D hydrogen chloride

© UCLES 2021 0653/22/F/M/21 [Turn over

169/693
Combined By Nesrine
10
2023-2017

23 The diagram shows what happens when damp red litmus paper is placed into two different
gases, X and Y.

gas gas
X Y

red litmus paper red litmus paper


turns blue turns white

What are gases X and Y?

X Y

A ammonia carbon dioxide


B ammonia chlorine
C chlorine ammonia
D chlorine carbon dioxide

24 Different aqueous solutions of halogens and halides are mixed in four beakers.

Which mixture produces a colourless solution?

A Cl 2(aq) and Cl –(aq)


B Br2(aq) and Br –(aq)
C Cl 2(aq) and Br –(aq)
D Br2(aq) and Cl –(aq)

25 Which change shows the presence of water?

A Anhydrous copper(II) sulfate turns white.

B Anhydrous copper(II) sulfate turns pink.

C Cobalt(II) chloride paper turns pink.

D Cobalt(II) chloride paper turns blue.

© UCLES 2021 0653/22/F/M/21

170/693
Combined By Nesrine
11
2023-2017

26 A petrol car engine takes in clean air and lets out exhaust gases.

exhaust
clean air in gases out

Which gas has a higher concentration in the exhaust gases than in clean air?

A argon
B carbon dioxide
C nitrogen
D oxygen

27 Which statement about hydrocarbons is not correct?

A Alkenes are made by cracking alkanes.


B Butene decolourises aqueous bromine.
C C2H4 is used to make poly(ethene).
D The general formula of alkanes is CnH2n.

28 Graph 1 is a distance–time graph. Graph 2 is a speed–time graph.

distance speed

0 0
0 time 0 time
graph 1 graph 2

Which of these graphs represents a car that is moving at constant speed?

A graph 1 only
B graph 2 only
C both graphs
D neither graph

© UCLES 2021 0653/22/F/M/21 [Turn over

171/693
Combined By Nesrine
12
2023-2017

29 An object has mass 1.0 kg and weight 10 N on the Earth.

It is moved to another planet where the gravitational field strength is smaller.

What are its mass and weight now?

mass / kg weight / N

A less than 1.0 less than 10


B less than 1.0 10
C 1.0 less than 10
D 1.0 10

30 A measuring cylinder contains 20 cm3 of oil. The measuring cylinder is placed on a balance and
the reading on the balance is 100 g.

Extra oil is added to the measuring cylinder. The volume increases to 70 cm3 and the reading on
the balance increases to 140 g.

measuring 70 cm3
cylinder

20 cm3 oil
oil

100 g 140 g
balance

before extra oil is added after extra oil is added

What is the density of the oil?

A 0.50 g / cm3 B 0.80 g / cm3 C 1.25 g / cm3 D 2.00 g / cm3

© UCLES 2021 0653/22/F/M/21

172/693
Combined By Nesrine
13
2023-2017

31 Diagram 1 shows a load hanging on a spring.

Diagram 2 shows the load pulled down.

diagram 1 diagram 2

When the load is pulled down, what happens to the gravitational potential energy of the load and
the elastic potential (strain) energy of the spring?

gravitational potential elastic potential


energy of load energy of spring

A decreases decreases
B decreases increases
C increases decreases
D increases increases

32 A load of mass 50 kg is lifted vertically by 8.0 m in 20 s.

The gravitational field strength g is 10 N / kg.

How much power is developed in lifting the load?

A 125 W B 200 W C 1600 W D 8000 W

© UCLES 2021 0653/22/F/M/21 [Turn over

173/693
Combined By Nesrine
14
2023-2017

33 A metal bar is heated at one end.

metal bar
heat

What is the main method by which thermal energy reaches the other end of the bar?

A Free electrons at the heated end gain kinetic energy and move along the bar.
B Free electrons at the heated end move apart and set up a convection current along the bar.
C Molecules at the heated end gain kinetic energy and move along the bar.
D Molecules at the heated end move apart and set up a convection current along the bar.

34 How is thermal energy transferred from the Sun to the Earth through the vacuum of space?

A by conduction, convection and radiation


B by conduction only
C by convection only
D by radiation only

35 Which waves are longitudinal?

A light waves
B microwaves
C sound waves
D X-rays

36 A thin converging lens is used as a magnifying glass.

The focal length of the lens is f.

Where is the object placed?

A on the opposite side of the lens to the eye and at a distance from the lens between f and 2f
B on the opposite side of the lens to the eye and at a distance from the lens less than f
C on the same side of the lens as the eye and at a distance from the lens between f and 2f
D on the same side of the lens as the eye and at a distance from the lens less than f

37 There is a current of 5.0 A in a resistor.

How much electric charge passes through the resistor in 30 minutes?

A 6.0 C B 150 C C 360 C D 9000 C

© UCLES 2021 0653/22/F/M/21

174/693
Combined By Nesrine
15
2023-2017

38 A battery is connected to a resistor.

current

Which changes to the resistance of the resistor, and to the potential difference (p.d.) across the
resistor, must produce a smaller current?

resistance p.d.

A decrease decrease
B decrease increase
C increase decrease
D increase increase

39 A piece of wire has electrical resistance.

The wire is stretched so that it becomes longer and thinner.

What happens to its resistance?

A It could increase or decrease depending on how much it is stretched.


B It does not change because the effect of its smaller diameter cancels the effect of its greater
length.
C It must decrease.
D It must increase.

© UCLES 2021 0653/22/F/M/21 [Turn over

175/693
Combined By Nesrine
16
2023-2017

40 The diagrams show four arrangements of resistors.

Which arrangement has the smallest total resistance?

A B
4Ω

2Ω 2Ω

4Ω

C D
4Ω 4Ω 4Ω

© UCLES 2021 0653/22/F/M/21

176/693
Combined By Nesrine
2023-2017

Cambridge IGCSE™

COMBINED SCIENCE 0653/21


Paper 2 Multiple Choice (Extended) May/June 2021
45 minutes

You must answer on the multiple choice answer sheet.


*2888690449*

You will need: Multiple choice answer sheet


Soft clean eraser
Soft pencil (type B or HB is recommended)

INSTRUCTIONS
 There are forty questions on this paper. Answer all questions.
 For each question there are four possible answers A, B, C and D. Choose the one you consider correct
and record your choice in soft pencil on the multiple choice answer sheet.
 Follow the instructions on the multiple choice answer sheet.
 Write in soft pencil.
 Write your name, centre number and candidate number on the multiple choice answer sheet in the
spaces provided unless this has been done for you.
 Do not use correction fluid.
 Do not write on any bar codes.
 You may use a calculator.

INFORMATION
 The total mark for this paper is 40.
 Each correct answer will score one mark.
 Any rough working should be done on this question paper.
 The Periodic Table is printed in the question paper.

This document has 16 pages.

IB21 06_0653_21/2RP
© UCLES 2021 [Turn over

177/693
Combined By Nesrine
2
2023-2017

1 Which row correctly matches a named cell with its feature and function?

cell feature function

A ciliated cell flagellum absorbs water


B palisade mesophyll cell chloroplasts transports oxygen
C red blood cell large surface area phagocytosis
D sperm cell flagellum reproduction

2 The diagrams represent four similar animal cells immersed in blood plasma.

The black dots represent molecules of dissolved oxygen.

Which cell will have oxygen molecules diffusing into it most rapidly?

A B C D

3 Which row shows the elements that occur in all proteins?

carbon hydrogen nitrogen oxygen

A yes yes yes no


B yes yes no yes
C yes no yes yes
D yes yes yes yes

4 What is the balanced equation for photosynthesis?

A CO2 + H2O  C6H12O6 + O2

B CO2 + 6H2O  6C6H12O6 + O2

C 6CO2 + 6H2O  C6H12O6 + 6O2

D 6CO2 + 12H2O  6C6H12O6 + 6O2

© UCLES 2021 0653/21/M/J/21

178/693
Combined By Nesrine
3
2023-2017

5 Which description of mechanical digestion is correct?

A breakdown of food into smaller pieces with chemical change to the food molecules
B breakdown of food into smaller pieces without chemical change to the food molecules
C breakdown of large insoluble food molecules into small soluble food molecules
D breakdown of large soluble food molecules into small soluble food molecules

6 The diagram shows a cross-section of a root hair cell.

Y
X

Which row identifies the part of the cell with the larger surface area and the correct function?

part of cell function

A X water and glucose uptake


B X water and ion uptake
C Y water and glucose uptake
D Y water and ion uptake

7 What is the maximum number of carbon dioxide molecules produced when four glucose
molecules are used in aerobic respiration?

A 6 B 12 C 24 D 48

8 What are effects of increased adrenaline production in humans?

A increased rate of breathing and increased pulse rate


B increased rate of breathing and narrower pupils
C slower pulse rate and narrower pupils
D slower pulse rate and wider pupils

© UCLES 2021 0653/21/M/J/21 [Turn over

179/693
Combined By Nesrine
4
2023-2017

9 The diagram shows a parent plant.

parent plant

Which offspring has been produced by asexual reproduction from this plant?

A B C D

10 The diagram shows a flower.

What is structure X?

A anther of an insect-pollinated flower


B anther of a wind-pollinated flower
C stigma of an insect-pollinated flower
D stigma of a wind-pollinated flower

© UCLES 2021 0653/21/M/J/21

180/693
Combined By Nesrine
5
2023-2017

11 Materials are exchanged between the blood of the mother and embryo in the placenta.

lining of
maternal
uterus
blood

placenta

capillary network
in placenta
1 2

embryo

Which row shows the blood vessels with higher concentrations of carbon dioxide and glucose?

higher concentration higher concentration


of carbon dioxide of glucose

A 1 1
B 2 2
C 2 1
D 1 2

© UCLES 2021 0653/21/M/J/21 [Turn over

181/693
Combined By Nesrine
6
2023-2017

12 The diagram shows a food web.

eagle
fox

small bird

goat

ant
caterpillar

locust beetle

grass wildflower tree

Which row states the number of species of each category in this food web?

number of primary number of secondary number of tertiary


consumer species consumer species consumer species

A 6 1 1
B 5 2 1
C 5 2 2
D 3 4 3

13 Which process takes carbon dioxide out of the air?

A combustion
B decomposition
C photosynthesis
D plant respiration

© UCLES 2021 0653/21/M/J/21

182/693
Combined By Nesrine
7
2023-2017

14 A fixed mass of argon gas in a sealed container is heated.

The pressure inside the container increases.

Which statement explains why the pressure increases?

A There is an increase in the number of gaseous particles inside the container.


B There is an increase in the number of collisions per second between the particles of gas and
the walls of the container.
C The particles of gas have less energy and collide with the wall of the container more
frequently.
D There is a decrease in the space that the particles have to move in.

15 What is an example of a physical change?

A carbon dioxide turning limewater milky

B the crystallisation of copper(II) sulfate from solution

C the electrolysis of molten lead(II) bromide

D the thermal decomposition of calcium carbonate

16 Which row about elements and compounds is correct?

elements compounds

A are metals only contain ionic or covalent bonds


B are non-metals only contain covalent bonds only
C are metals or non-metals contain ionic bonds only
D are metals or non-metals contain ionic or covalent bonds

© UCLES 2021 0653/21/M/J/21 [Turn over

183/693
Combined By Nesrine
8
2023-2017

17 The symbols and charges of some ions are shown.

● aluminium, Al 3+
● phosphate, PO43–
● sodium, Na+
● sulfate, SO42–

What are the formulae of aluminium sulfate and sodium phosphate?

aluminium sodium
sulfate phosphate

A Al 2(SO4)3 Na3PO4
B Al 2(SO4)3 Na(PO4)3
C Al 3(SO4)2 Na3PO4
D Al 3(SO4)2 Na(PO4)3

18 Two substances are electrolysed separately using inert electrodes.

Which two substances form the same product at one of the electrodes?

A molten sodium bromide and concentrated aqueous sodium chloride


B molten sodium bromide and dilute sulfuric acid
C molten aluminium oxide and concentrated aqueous sodium chloride
D molten aluminium oxide and dilute sulfuric acid

19 An energy diagram for a reaction is shown.

energy

progress of reaction

Which statement about the reaction is not correct?

A Activation energy is needed to start the reaction.


B The overall energy change is endothermic.
C The surroundings increase in temperature during the reaction.
D The reaction could be a combustion reaction.

© UCLES 2021 0653/21/M/J/21

184/693
Combined By Nesrine
9
2023-2017

20 In the reaction between an acid and a metal, the rate of reaction decreases as the reaction
proceeds.

A student suggests three reasons why the rate of this reaction decreases.

1 The concentration of the acid decreases as it gets used up.


2 The energy needed to break bonds is used up as the products form.
3 The surface area of the metal decreases as it gets smaller.

Which reasons are correct?

A 1, 2 and 3 B 1 and 2 only C 1 and 3 only D 2 and 3 only

21 The solubilities of some copper compounds are shown.

copper compound solubility in water

copper nitrate soluble


copper sulfate soluble
copper oxide insoluble
copper hydroxide insoluble

Which method is used to make copper sulfate?

A Mix aqueous copper hydroxide and dilute sulfuric acid.


B Mix aqueous copper nitrate with aqueous sodium sulfate and filter off solid copper sulfate.
C Mix excess aqueous copper nitrate with dilute sulfuric acid and filter off unreacted
copper nitrate.
D Mix excess solid copper oxide with dilute sulfuric acid and filter off unreacted copper oxide.

22 Which two substances form a white precipitate when they are mixed?

A barium chloride and hydrochloric acid


B barium chloride and nitric acid
C silver nitrate and hydrochloric acid
D silver nitrate and nitric acid

© UCLES 2021 0653/21/M/J/21 [Turn over

185/693
Combined By Nesrine
10
2023-2017

23 There are eight elements in Period 3 of the Periodic Table.

Na Mg Al Si P S Cl Ar

Which statement about the elements in this period is correct?

A The elements become less metallic across the period.


B The most metallic elements are at both ends of the period.
C The most metallic elements are in the middle of the period.
D There is no pattern in metallic character across the period.

24 The colours of concentrated aqueous solutions of Group VII elements are shown.

colour of
element
aqueous solution

fluorine colourless
chlorine pale yellow
bromine orange
iodine red-brown

Concentrated aqueous chlorine is added to colourless aqueous potassium bromide.

What is the colour of the mixture?

A colourless
B pale yellow
C orange
D red-brown

25 In the blast furnace, carbon reacts with carbon dioxide to form carbon monoxide.

C + CO2  2CO

Why is this reaction essential in the extraction of iron from hematite?

A Carbon monoxide is needed to reduce iron oxide.


B Carbon monoxide neutralises impurities to make slag.
C The reaction removes carbon impurities from the iron ore.
D The reaction prevents the release of greenhouse gases.

© UCLES 2021 0653/21/M/J/21

186/693
Combined By Nesrine
11
2023-2017

26 Which gases may contribute to climate change when their concentrations in the air increase?

1 carbon dioxide
2 methane
3 sulfur dioxide

A 1 and 2 only B 1 and 3 only C 2 and 3 only D 1, 2 and 3

27 Which type of compound contains only carbon and hydrogen?

A carbohydrate
B carbonate
C hydrocarbon
D hydroxide

28 A boy runs up some stairs.

Which two physical quantities are used to calculate the power he develops?

A his mass and his acceleration


B his mass and the time taken
C the work done and the time taken
D the work done and the vertical distance moved

29 Diagram 1 is a distance–time graph.

Diagram 2 and diagram 3 are speed–time graphs.

distance speed speed

0 0 0
0 time 0 time 0 time
diagram 1 diagram 2 diagram 3

Which of the diagrams represents the motion of an object moving with a non-zero constant
acceleration?

A 1 and 3 B 1 only C 2 only D 3 only

© UCLES 2021 0653/21/M/J/21 [Turn over

187/693
Combined By Nesrine
12
2023-2017

30 The diagram shows the load–extension graph for a spring.

Two points on the graph are labelled X and Y.

Y
load / N
X
10

0
0 20
extension / cm

What is the spring constant of the spring and which labelled point is the limit of proportionality?

spring constant limit of


N/cm proportionality

A 0.50 X
B 0.50 Y
C 2.0 X
D 2.0 Y

31 Work W is done when a force F moves an object a distance d in the direction of the force.

Which equation gives the distance d ?

A d=F+W B d=FW C d= F D d= W
W F

32 Which row contains a renewable and a non-renewable energy resource in the correct column?

renewable non-renewable

A geothermal wind
B geothermal coal
C oil wind
D oil coal

© UCLES 2021 0653/21/M/J/21

188/693
Combined By Nesrine
13
2023-2017

33 Cold water evaporates as molecules leave it.

Which molecules leave the water and from which part of the water do they leave?

molecules that where they


leave the water leave from

A least energetic the surface only


B least energetic throughout the water
C most energetic the surface only
D most energetic throughout the water

34 Copper is a good conductor of heat.

Plastic is a bad conductor of heat.

Which statement explains this difference?

A Electrons can move freely in copper but cannot move freely in plastic.
B Electrons can move freely in plastic but cannot move freely in copper.
C Molecules can move freely in copper but cannot move freely in plastic.
D Molecules can move freely in plastic but cannot move freely in copper.

35 Which statement about waves is correct?

A In a longitudinal wave, the vibration of the particles is perpendicular to the direction of the
wave.
B In a sound wave, the vibration of the particles is parallel to the direction of the wave.
C Radio waves are longitudinal waves.
D Sound waves are transverse waves.

36 A student uses a converging lens with a focal length F as a magnifying glass.

What is the distance between the object and the lens?

A less than F
B between F and 2F
C 2F
D greater than 2F

© UCLES 2021 0653/21/M/J/21 [Turn over

189/693
Combined By Nesrine
14
2023-2017

37 There is a current of 20 mA in a resistor for a time t.

During this time, a charge of 600 C passes through the resistor.

What is the value of t ?

A 3.3  10–5 s B 12 s C 30 s D 3.0  104 s

38 The diagram represents a circuit that includes a battery, an ammeter, a voltmeter and a variable
resistor.

What happens to the readings on the meters as the resistance of the variable resistor is
increased?

ammeter reading voltmeter reading

A decreases decreases
B decreases stays constant
C increases decreases
D increases stays constant

39 A lamp is labelled 12 V, 25 W.

How much electrical energy does the lamp transfer in 4.0 minutes when it is operating at its
normal brightness?

A 100 J B 1200 J C 6000 J D 72 000 J

© UCLES 2021 0653/21/M/J/21

190/693
Combined By Nesrine
15
2023-2017

40 An air conditioner and a television are both connected to the same electrical circuit.

power air
television
supply conditioner

The current in the air conditioner is 9.0 A and the current in the television is 2.0 A.

Several different fuses are available.

Which fuse should be connected at X?

A 1A B 3A C 7A D 13 A

Permission to reproduce items where third-party owned material protected by copyright is included has been sought and cleared where possible. Every
reasonable effort has been made by the publisher (UCLES) to trace copyright holders, but if any items requiring clearance have unwittingly been included, the
publisher will be pleased to make amends at the earliest possible opportunity.

To avoid the issue of disclosure of answer-related information to candidates, all copyright acknowledgements are reproduced online in the Cambridge
Assessment International Education Copyright Acknowledgements Booklet. This is produced for each series of examinations and is freely available to download
at www.cambridgeinternational.org after the live examination series.

Cambridge Assessment International Education is part of the Cambridge Assessment Group. Cambridge Assessment is the brand name of the University of
Cambridge Local Examinations Syndicate (UCLES), which itself is a department of the University of Cambridge.

© UCLES 2021 0653/21/M/J/21

191/693
Combined By Nesrine
2023-2017

Cambridge IGCSE™

COMBINED SCIENCE 0653/22


Paper 2 Multiple Choice (Extended) May/June 2021
45 minutes

You must answer on the multiple choice answer sheet.


*2755644340*

You will need: Multiple choice answer sheet


Soft clean eraser
Soft pencil (type B or HB is recommended)

INSTRUCTIONS
 There are forty questions on this paper. Answer all questions.
 For each question there are four possible answers A, B, C and D. Choose the one you consider correct
and record your choice in soft pencil on the multiple choice answer sheet.
 Follow the instructions on the multiple choice answer sheet.
 Write in soft pencil.
 Write your name, centre number and candidate number on the multiple choice answer sheet in the
spaces provided unless this has been done for you.
 Do not use correction fluid.
 Do not write on any bar codes.
 You may use a calculator.

INFORMATION
 The total mark for this paper is 40.
 Each correct answer will score one mark.
 Any rough working should be done on this question paper.
 The Periodic Table is printed in the question paper.

This document has 16 pages. Any blank pages are indicated.

IB21 06_0653_22/2RP
© UCLES 2021 [Turn over

192/693
Combined By Nesrine
2
2023-2017

1 Which row correctly identifies the function of a ciliated cell in the bronchus of a healthy human?

substance being moved direction of movement

A air towards bronchioles


B air towards trachea
C mucus towards bronchioles
D mucus towards trachea

2 The diagrams represent four similar animal cells immersed in blood plasma.

The black dots represent molecules of dissolved oxygen.

Which cell will have oxygen molecules diffusing into it most rapidly?

A B C D

3 Which name is given to biological catalysts?

A antibodies
B enzymes
C hormones
D platelets

4 Which row is correct for photosynthesis?

cells where
substrates products
photosynthesis occurs

A C6H12O6 + 6O2 6H2O + 6CO2 palisade mesophyll


B C6H12O6 + 6O2 6H2O + 6CO2 upper epidermis
C 6H2O + 6CO2 C6H12O6 + 6O2 palisade mesophyll
D 6H2O + 6CO2 C6H12O6 + 6O2 upper epidermis

© UCLES 2021 0653/22/M/J/21

193/693
Combined By Nesrine
3
2023-2017

5 Which disease is associated with malnutrition?

A AIDS
B COPD
C lung cancer
D scurvy

6 Which part of the alimentary canal carries out digestion and absorption?

A
C

7 The diagram shows a cross-section of a root hair cell.

Y
X

Which row identifies the part of the cell with the larger surface area and the correct function?

part of cell function

A X water and glucose uptake


B X water and ion uptake
C Y water and glucose uptake
D Y water and ion uptake

© UCLES 2021 0653/22/M/J/21 [Turn over

194/693
Combined By Nesrine
4
2023-2017

8 What is the maximum number of carbon dioxide molecules produced when four glucose
molecules are used in aerobic respiration?

A 6 B 12 C 24 D 48

9 Which graph shows the correct changes in blood glucose concentration and pulse rate shortly
after adrenaline is released into the blood stream?

A B

key
blood glucose
pulse rate

time time

C D

time time

10 A shoot tip receives light from one direction only, as shown.

Which diagram shows how auxin will distribute and how the shoot will respond?

A B C D

auxin

© UCLES 2021 0653/22/M/J/21

195/693
Combined By Nesrine
5
2023-2017

11 The diagram shows the human female reproductive system.

Where does fertilisation usually take place?

C
A

12 The diagram shows a food web.

cat

sparrowhawk

thrush robin
frog

sparrow slug caterpillar

plant

Which row shows the correct organism for each trophic level?

trophic level 1 trophic level 2 trophic level 3

A cat sparrow plant


B caterpillar robin sparrowhawk
C plant sparrow cat
D sparrowhawk robin caterpillar

13 Which process takes carbon dioxide out of the air?

A combustion
B decomposition
C photosynthesis
D plant respiration

© UCLES 2021 0653/22/M/J/21 [Turn over

196/693
Combined By Nesrine
6
2023-2017

14 Which statement about the particles is correct?


1
A 1H has the same number of protons as neutrons.

B 2
1 H+ has the same number of electrons as neutrons.

C OH– contains more protons than electrons.


D NH3 has the same number of protons as electrons.

15 What is an example of a physical change?

A carbon dioxide turning limewater milky

B the crystallisation of copper(II) sulfate from solution

C the electrolysis of molten lead(II) bromide

D the thermal decomposition of calcium carbonate

16 Which substances are mixtures?

1 air
2 brass
3 sodium chloride

A 1 and 2 only B 1 and 3 only C 2 and 3 only D 1, 2 and 3

17 Which products are formed when molten sodium chloride is electrolysed using inert electrodes?

at the anode at the cathode

A chlorine hydrogen
B chlorine sodium
C oxygen hydrogen
D oxygen sodium

© UCLES 2021 0653/22/M/J/21

197/693
Combined By Nesrine
7
2023-2017

18 The energy level diagrams for reaction X and for reaction Y are shown.

reaction X reaction Y

energy energy

progress of reaction progress of reaction

Which statement about the reactions is correct?

A Reaction X has a greater activation energy than reaction Y.


B Reaction X is endothermic and reaction Y is exothermic.
C The overall energy change in reaction X is much greater than in reaction Y.
D The temperature increases during reaction X and decreases during reaction Y.

19 In the reaction between an acid and a metal, the rate of reaction decreases as the reaction
proceeds.

A student suggests three reasons why the rate of this reaction decreases.

1 The concentration of the acid decreases as it gets used up.


2 The energy needed to break bonds is used up as the products form.
3 The surface area of the metal decreases as it gets smaller.

Which reasons are correct?

A 1, 2 and 3 B 1 and 2 only C 1 and 3 only D 2 and 3 only

20 Which statements about redox reactions are correct?

1 An oxidising agent is reduced in a reaction.


2 A reducing agent is oxidised in a reaction.
3 An oxidising agent gains oxygen in a reaction.
4 A reducing agent loses oxygen in a reaction.

A 1 and 2 B 1 and 3 C 2 and 4 D 3 and 4

© UCLES 2021 0653/22/M/J/21 [Turn over

198/693
Combined By Nesrine
8
2023-2017

21 Excess insoluble solid copper carbonate is mixed with dilute nitric acid.

Aqueous copper nitrate is formed.

Which row shows the processes used to obtain pure solid copper nitrate from the reaction
mixture?

process 1 process 2 process 3


A filter the mixture dry the solid on warm in an oven
the filter paper
B filter the mixture heat the solution filter the mixture
and crystallise and dry
C heat the solution cool to crystallise filter the mixture
and dry
D heat the solution filter the mixture dry the solid

22 Which two substances form a white precipitate when they are mixed?

A barium chloride and hydrochloric acid


B barium chloride and nitric acid
C silver nitrate and hydrochloric acid
D silver nitrate and nitric acid

23 Which statement describes how the elements change across a period in the Periodic Table from
left to right?

A They change from elements to compounds.


B They change from metals to non-metals.
C They change from gases to solids.
D They change from non-metals to metals.

24 Which equation represents a correct displacement reaction involving halogens?

A 2NaBr + I2  Br2 + 2NaI

B 2NaCl + Br2  Cl 2 + 2NaBr

C 2NaF + I2  F2 + 2NaI

D 2NaI + Cl 2  I2 + 2NaCl

© UCLES 2021 0653/22/M/J/21

199/693
Combined By Nesrine
9
2023-2017

25 Element X is a metal.

X is more reactive than aluminium.

Which method is used to obtain X?

A electrolysis of a molten salt of X


B electrolysis of an aqueous solution of a salt of X
C heating the oxide of X with carbon
D heating the oxide of X with hydrogen

26 Which statement about greenhouse gases is correct?

A They are gases in Group VIII of the Periodic Table.


B They cause acid rain.
C They contribute to climate change.
D They make up most of the atmosphere.

27 Which type of compound contains only carbon and hydrogen?

A carbohydrate
B carbonate
C hydrocarbon
D hydroxide

28 Which row shows apparatus used to measure length, time and volume?

length time volume

A measuring cylinder metre rule stop-clock


B measuring cylinder stop-clock metre rule
C metre rule measuring cylinder stop-clock
D metre rule stop-clock measuring cylinder

© UCLES 2021 0653/22/M/J/21 [Turn over

200/693
Combined By Nesrine
10
2023-2017

29 Diagram 1 is a distance–time graph.

Diagram 2 and diagram 3 are speed–time graphs.

distance speed speed

0 0 0
0 time 0 time 0 time
diagram 1 diagram 2 diagram 3

Which of the diagrams represents the motion of an object moving with a non-zero constant
acceleration?

A 1 and 3 B 1 only C 2 only D 3 only

30 An athlete of mass 62 kg jumps through a vertical height of 1.25 m.

As he moves upwards, all his initial kinetic energy is transferred to gravitational potential energy.

The gravitational field strength g is 10 N / kg.

What is the initial speed of the athlete?

A 2.5 m / s B 3.5 m / s C 5.0 m / s D 12.4 m / s

31 What is the main source of the energy released from the Sun?

A fission of helium nuclei to form hydrogen nuclei


B fusion of hydrogen nuclei to form helium nuclei
C hydrogen atoms combining to form hydrogen molecules
D hydrogen atoms reacting with oxygen atoms to form water molecules

32 Cold water evaporates as molecules leave it.

Which molecules leave the water and from which part of the water do they leave?

molecules that where they


leave the water leave from

A least energetic the surface only


B least energetic throughout the water
C most energetic the surface only
D most energetic throughout the water

© UCLES 2021 0653/22/M/J/21

201/693
Combined By Nesrine
11
2023-2017

33 A metal rod with a wooden handle is placed with the end of the metal rod in a flame.

metal rod wooden handle

How does heat pass through the metal and how does heat pass through the wood?

heat passes through the metal heat passes through the wood
A by movement of electrons by molecular vibrations only
and by molecular vibrations
B by movement of electrons by movement of electrons
and by molecular vibrations and by molecular vibrations
C by molecular vibrations only by molecular vibrations only

D by movement of electrons only by movement of electrons


and by molecular vibrations

34 Which equation relates wave speed v, frequency f and wavelength ?

A v = f B v= f C v 2 = f D v2 = f
 

© UCLES 2021 0653/22/M/J/21 [Turn over

202/693
Combined By Nesrine
12
2023-2017

35 The diagram shows rays of light from an object being reflected by a plane mirror.

plane mirror

object Y

At which labelled point is the image formed, and is the image real or virtual?

image real or virtual

A at X real
B at X virtual
C at Y real
D at Y virtual

36 The table shows the speed of sound in three different substances X, Y and Z. One substance is a
solid, one is a liquid and one is a gas.

speed of sound
substance
m/ s

X 3600
Y 1500
Z 267

Which row shows the states of the three substances?

solid liquid gas

A X Y Z
B X Z Y
C Z X Y
D Z Y X

© UCLES 2021 0653/22/M/J/21

203/693
Combined By Nesrine
13
2023-2017

37 The diagram represents a circuit that includes a battery, an ammeter, a voltmeter and a variable
resistor.

What happens to the readings on the meters as the resistance of the variable resistor is
increased?

ammeter reading voltmeter reading

A decreases decreases
B decreases stays constant
C increases decreases
D increases stays constant

38 Four copper wires have different lengths and different cross-sectional areas.

Which wire has the smallest resistance?

length / cm cross-sectional area / mm2

A 50 0.025
B 50 0.050
C 100 0.025
D 100 0.050

39 A lamp is labelled 12 V, 25 W.

How much electrical energy does the lamp transfer in 4.0 minutes when it is operating at its
normal brightness?

A 100 J B 1200 J C 6000 J D 72 000 J

© UCLES 2021 0653/22/M/J/21 [Turn over

204/693
Combined By Nesrine
14
2023-2017

40 An air conditioner and a television are both connected to the same electrical circuit.

power air
television
supply conditioner

The current in the air conditioner is 9.0 A and the current in the television is 2.0 A.

Several different fuses are available.

Which fuse should be connected at X?

A 1A B 3A C 7A D 13 A

© UCLES 2021 0653/22/M/J/21

205/693
Combined By Nesrine
2023-2017

Cambridge IGCSE™

COMBINED SCIENCE 0653/23


Paper 2 Multiple Choice (Extended) May/June 2021
45 minutes

You must answer on the multiple choice answer sheet.


*0986887174*

You will need: Multiple choice answer sheet


Soft clean eraser
Soft pencil (type B or HB is recommended)

INSTRUCTIONS
 There are forty questions on this paper. Answer all questions.
 For each question there are four possible answers A, B, C and D. Choose the one you consider correct
and record your choice in soft pencil on the multiple choice answer sheet.
 Follow the instructions on the multiple choice answer sheet.
 Write in soft pencil.
 Write your name, centre number and candidate number on the multiple choice answer sheet in the
spaces provided unless this has been done for you.
 Do not use correction fluid.
 Do not write on any bar codes.
 You may use a calculator.

INFORMATION
 The total mark for this paper is 40.
 Each correct answer will score one mark.
 Any rough working should be done on this question paper.
 The Periodic Table is printed in the question paper.

This document has 16 pages. Any blank pages are indicated.

IB21 06_0653_23/2RP
© UCLES 2021 [Turn over

206/693
Combined By Nesrine
2
2023-2017

1 Which row links a specialised cell to its correct function?

specialised cell function

A ciliated cell photosynthesis


B palisade cell movement of mucus
C red blood cell blood clotting
D sperm cell reproduction

2 The diagrams represent four similar animal cells immersed in blood plasma.

The black dots represent molecules of dissolved oxygen.

Which cell will have oxygen molecules diffusing into it most rapidly?

A B C D

3 The diagram shows the effect of increasing the pH of an enzyme-controlled reaction.

rate of
reaction

pH

What is happening at point X?

1 denaturation
2 greatest number of enzyme–substrate complexes
3 increased kinetic energy

A 1 only B 2 only C 1 and 3 D 2 and 3

© UCLES 2021 0653/23/M/J/21

207/693
Combined By Nesrine
3
2023-2017

4 The leaves of plants produce carbohydrates during photosynthesis.

How are these carbohydrates used by the plants?

to make other
for respiration for storage
substances

A   
B   
C   
D   

5 A person has a low level of haemoglobin.

Which row identifies the blood cell that transports oxygen and the nutrient the person is
deficient in?

type of blood cell nutrient deficiency

A red calcium
B red iron
C white calcium
D white iron

6 Most food molecules need to be digested to allow them to be absorbed into the blood.

Which row shows the type of digestion and the change needed to allow absorption to happen?

type of digestion change to food molecules

A chemical large molecules to small, insoluble molecules


B chemical large molecules to small, soluble molecules
C mechanical large molecules to small, soluble molecules
D mechanical large molecules to small, insoluble molecules

© UCLES 2021 0653/23/M/J/21 [Turn over

208/693
Combined By Nesrine
4
2023-2017

7 The diagram shows a cross-section of a root hair cell.

Y
X

Which row identifies the part of the cell with the larger surface area and the correct function?

part of cell function

A X water and glucose uptake


B X water and ion uptake
C Y water and glucose uptake
D Y water and ion uptake

8 What is the maximum number of carbon dioxide molecules produced when four glucose
molecules are used in aerobic respiration?

A 6 B 12 C 24 D 48

9 A plant in a pot was placed on its side for four days.

shoot root

start after four days

Which row describes the gravitropic response in the root and shoot?

root shoot

A positive negative
B negative positive
C negative negative
D positive positive

© UCLES 2021 0653/23/M/J/21

209/693
Combined By Nesrine
5
2023-2017

10 During human reproduction an egg fuses with a sperm.

Sometimes the zygote splits into two and produces twins.

Which row describes the formation of these twins?

original zygote produced by twins

A asexual reproduction genetically identical


B sexual reproduction genetically identical
C asexual reproduction genetically different
D sexual reproduction genetically different

11 The diagram shows a wind-pollinated flower.

Which label identifies a stigma?

A D

12 In which food chain does the final consumer receive the most energy from the producer?
primary secondary tertiary quaternary
A producer    
consumer consumer consumer consumer
primary secondary tertiary
B producer   
consumer consumer consumer
primary secondary
C producer  
consumer consumer
primary
D producer 
consumer

13 Which process takes carbon dioxide out of the air?

A combustion
B decomposition
C photosynthesis
D plant respiration

© UCLES 2021 0653/23/M/J/21 [Turn over

210/693
Combined By Nesrine
6
2023-2017

14 The melting point and boiling point of oxygen and nitrogen are shown.

melting point boiling point


/ C / C

oxygen –219 –183


nitrogen –210 –196

A sealed flask contains a mixture of oxygen and nitrogen.

Which diagram shows the arrangement of oxygen and nitrogen particles at –190 C?

A B C D

key
= nitrogen molecules
= oxygen molecules

15 During a chromatography investigation, colour X moves 4.5 cm up the chromatography paper


from the base line.

The Rf value of colour X is 0.59.

What is the distance moved by the solvent in this experiment?

A 2.7 cm B 4.5 cm C 7.6 cm D 10.3 cm

16 What is an example of a physical change?

A carbon dioxide turning limewater milky

B the crystallisation of copper(II) sulfate from solution

C the electrolysis of molten lead(II) bromide

D the thermal decomposition of calcium carbonate

17 Water has the chemical formula H2O.

Which statement is correct?

A Pure water is a mixture because it contains hydrogen and oxygen.


B Pure water is an element because it contains only one type of molecule.
C Salt water is a compound because it contains salt and water.
D Salt water is a mixture because it contains salt and water.

© UCLES 2021 0653/23/M/J/21

211/693
Combined By Nesrine
7
2023-2017

18 When water boils it changes from a liquid to a gas.

Which statement about this process is correct?

A It is endothermic because it requires energy to break covalent bonds.


B It is endothermic because energy is needed to break attractive forces between molecules.
C It is exothermic because it requires energy to break attractive forces between atoms.
D It is exothermic because energy is given out when bonds form.

19 In the reaction between an acid and a metal, the rate of reaction decreases as the reaction
proceeds.

A student suggests three reasons why the rate of this reaction decreases.

1 The concentration of the acid decreases as it gets used up.


2 The energy needed to break bonds is used up as the products form.
3 The surface area of the metal decreases as it gets smaller.

Which reasons are correct?

A 1, 2 and 3 B 1 and 2 only C 1 and 3 only D 2 and 3 only

20 Iron is extracted from its oxide using carbon monoxide. The equation is shown.

Fe2O3 + 3CO  2Fe + 3CO2

Which row identifies the reducing agent and explains how it acts as a reducing agent?

reducing agent explanation

A Fe2O3 it loses mass to become Fe


B Fe2O3 it loses oxygen to become Fe
C CO it gains mass to become CO2
D CO it removes oxygen from Fe2O3

21 Substances that react together to make zinc salts are listed.

1 zinc carbonate and hydrochloric acid


2 zinc oxide and sulfuric acid
3 zinc and nitric acid
4 zinc hydroxide and hydrochloric acid

Which substances produce water when they react?

A 1, 2 and 3 B 1, 2 and 4 C 1 and 2 only D 3 and 4

© UCLES 2021 0653/23/M/J/21 [Turn over

212/693
Combined By Nesrine
8
2023-2017

22 Which two substances form a white precipitate when they are mixed?

A barium chloride and hydrochloric acid


B barium chloride and nitric acid
C silver nitrate and hydrochloric acid
D silver nitrate and nitric acid

23 Aqueous chlorine is added to aqueous sodium bromide.

aqueous chlorine

aqueous sodium bromide

Which statement about the reaction is correct?

A The solution turns orange because bromine is formed.


B The solution turns orange because bromide ions are reduced.
C The solution remains colourless because bromine is less reactive than chlorine.
D The solution remains colourless because chlorine is reduced.

24 How does the character of the elements change across a period of the Periodic Table from left to
right?

A acidic to basic
B basic to acidic
C metallic to non-metallic
D non-metallic to metallic

© UCLES 2021 0653/23/M/J/21

213/693
Combined By Nesrine
9
2023-2017

25 Four metals, W, X, Y and Z, are added to aqueous solutions of their salts.

W displaces Y.

Y displaces X.

Z displaces Y but does not displace W.

Which row shows the reactivity order of the metals?

least most
reactive reactive

A X Y Z W
B X Z Y W
C W Y Z X
D W Z Y X

26 Which statement about greenhouse gases is correct?

A Greenhouse gases cause acid rain.


B The combustion of fossil fuels produces greenhouse gases.
C Nitrogen is a greenhouse gas.
D Greenhouse gases are removed from the atmosphere by respiration.

27 Which type of compound contains only carbon and hydrogen?

A carbohydrate
B carbonate
C hydrocarbon
D hydroxide

28 Which change cannot be caused by a force acting on an object?

A change of mass
B change of motion
C change of shape
D change of size

© UCLES 2021 0653/23/M/J/21 [Turn over

214/693
Combined By Nesrine
10
2023-2017

29 Diagram 1 is a distance–time graph.

Diagram 2 and diagram 3 are speed–time graphs.

distance speed speed

0 0 0
0 time 0 time 0 time
diagram 1 diagram 2 diagram 3

Which of the diagrams represents the motion of an object moving with a non-zero constant
acceleration?

A 1 and 3 B 1 only C 2 only D 3 only

30 A student does 10 J of work when lifting an object through a vertical distance of 2.0 m.

What is the size of the force that the student exerts on the object?

A 0.20 N B 5.0 N C 12 N D 20 N

31 Which source of energy is non-renewable?

A chemical energy stored in fossil fuels


B energy stored in waves
C energy stored in water behind a hydroelectric dam
D wind energy

32 Cold water evaporates as molecules leave it.

Which molecules leave the water and from which part of the water do they leave?

molecules that where they


leave the water leave from

A least energetic the surface only


B least energetic throughout the water
C most energetic the surface only
D most energetic throughout the water

© UCLES 2021 0653/23/M/J/21

215/693
Combined By Nesrine
11
2023-2017

33 A heater creates a convection current in a room.

What happens to air as it is heated?

A It contracts and its density decreases.


B It contracts and its density increases.
C It expands and its density decreases.
D It expands and its density increases.

34 Which row gives an example of a transverse wave and a longitudinal wave?

transverse longitudinal

A light wave radio wave


B radio wave sound wave
C sound wave light wave
D sound wave radio wave

35 Which diagram shows a converging lens being used to produce the largest virtual image?

(Every point labelled F is a principal focus.)

A B

object object

F F F F

C D

object object

F F F F

© UCLES 2021 0653/23/M/J/21 [Turn over

216/693
Combined By Nesrine
12
2023-2017

36 The speed of sound in air is approximately 330 m / s.

The speed of sound in water is approximately 1500 m / s.

What is a possible speed of sound in solid iron?

A 120 m / s B 330 m / s C 1200 m / s D 5100 m / s

37 The diagram represents a circuit that includes a battery, an ammeter, a voltmeter and a variable
resistor.

What happens to the readings on the meters as the resistance of the variable resistor is
increased?

ammeter reading voltmeter reading

A decreases decreases
B decreases stays constant
C increases decreases
D increases stays constant

38 Which combination of resistors has a combined resistance of 4.0 ?

A B C D
1.0 : 4.0 : 6.0 : 20 :

3.0 : 4.0 : 12 : 24 :

39 A lamp is labelled 12 V, 25 W.

How much electrical energy does the lamp transfer in 4.0 minutes when it is operating at its
normal brightness?

A 100 J B 1200 J C 6000 J D 72 000 J

© UCLES 2021 0653/23/M/J/21

217/693
Combined By Nesrine
13
2023-2017

40 An air conditioner and a television are both connected to the same electrical circuit.

power air
television
supply conditioner

The current in the air conditioner is 9.0 A and the current in the television is 2.0 A.

Several different fuses are available.

Which fuse should be connected at X?

A 1A B 3A C 7A D 13 A

© UCLES 2021 0653/23/M/J/21

218/693
Combined By Nesrine
2023-2017

Cambridge IGCSE™

COMBINED SCIENCE 0653/21


Paper 2 Multiple Choice (Extended) October/November 2021
45 minutes

You must answer on the multiple choice answer sheet.


*0826651995*

You will need: Multiple choice answer sheet


Soft clean eraser
Soft pencil (type B or HB is recommended)

INSTRUCTIONS
 There are forty questions on this paper. Answer all questions.
 For each question there are four possible answers A, B, C and D. Choose the one you consider correct
and record your choice in soft pencil on the multiple choice answer sheet.
 Follow the instructions on the multiple choice answer sheet.
 Write in soft pencil.
 Write your name, centre number and candidate number on the multiple choice answer sheet in the
spaces provided unless this has been done for you.
 Do not use correction fluid.
 Do not write on any bar codes.
 You may use a calculator.

INFORMATION
 The total mark for this paper is 40.
 Each correct answer will score one mark.
 Any rough working should be done on this question paper.
 The Periodic Table is printed in the question paper.

This document has 16 pages. Any blank pages are indicated.

IB21 11_0653_21/3RP
© UCLES 2021 [Turn over

219/693
Combined By Nesrine
2
2023-2017

1 Movement is a characteristic of all living organisms.

Which two other characteristics of living organisms provide the energy for movement?

A excretion and nutrition


B growth and sensitivity
C nutrition and respiration
D respiration and growth

2 What are all living organisms made of?

A cells
B chloroplasts
C muscles
D organs

3 Which statement about enzymes is correct?

A They are denatured at high temperatures.


B They all have an optimum pH of 7.

C They all have an optimum temperature of 10 C.


D They are made of carbohydrates.

4 Which letters from the list represent the balanced equation for photosynthesis?

P C6H12O6 T H2O
Q 6C6H12O6 U 6H2O
R CO2 V O2
S 6CO2 W 6O2

A P + U  R + V

B Q + T  S + U

C R + T  W + P

D U + S  P + W

© UCLES 2021 0653/21/O/N/21

220/693
Combined By Nesrine
3
2023-2017

5 What is an effect of iron deficiency in the diet?

A anaemia
B constipation
C coronary heart disease
D scurvy

6 The following paragraph is a description of the digestion of fats.

Large pieces of fat are broken down into smaller pieces of fat by ……1…… digestion. These
smaller pieces of fat can then be broken down by the enzyme ……2…… . This is ……3……
digestion. During this process, the larger molecules are broken down into smaller, ……4……
molecules.

Which row correctly completes gaps 1, 2, 3 and 4?

1 2 3 4

A chemical lipase mechanical soluble


B chemical protease mechanical insoluble
C mechanical lipase chemical soluble
D mechanical protease chemical insoluble

7 Which row correctly describes double circulation in mammals?

pressure of blood pressure of blood type of blood


from heart to body from heart to lungs from heart to lungs

A high high oxygenated


B high low deoxygenated
C low high deoxygenated
D low low oxygenated

© UCLES 2021 0653/21/O/N/21 [Turn over

221/693
Combined By Nesrine
4
2023-2017

8 What causes the change in breathing seen at X?

3
X

volume / dm3

0
0 5 10 15 20 25 30 35 40 45 50 55
time / s

A decreased oxygen in the blood


B decreased lactic acid in the blood
C increased carbon dioxide in the blood
D increased sweating

9 A plant shoot is illuminated from one side only.

What collects on the shaded side of the plant shoot?

A auxin
B chlorophyll
C glucose
D starch

10 Which part of a flower is not required for pollination?

A anther
B sepal
C stamen
D stigma

© UCLES 2021 0653/21/O/N/21

222/693
Combined By Nesrine
5
2023-2017

11 The diagram represents the human placenta.

P
blood of blood of
fetus Q mother

placenta

P and Q show the net movement of substances.

Which row identifies substances that travel in the directions of P and Q?

in direction P in direction Q

A blood urea
B oxygen carbon dioxide
C excretory products glucose
D amino acids toxins

12 The diagram represents four organisms in a food chain.

T  U  V  W

Which organisms are consumers?

A T, U and V B T, U and W C T, V and W D U, V and W

13 Carbon dioxide levels in the atmosphere have risen by 30% in the last 60 years.

Which actions have contributed to this increase?

1 burning fossil fuels


2 deforestation
3 extinction of species

A 1 only B 1 and 2 only C 2 and 3 only D 1, 2 and 3

14 Which statement describes the change when water becomes ice at 0 C?

A The particles collide with each other more frequently.


B The particles have more kinetic energy.
C The process is endothermic.
D The process is exothermic.

© UCLES 2021 0653/21/O/N/21 [Turn over

223/693
Combined By Nesrine
6
2023-2017

15 Which statement explains why ionic compounds have higher melting points than covalent
compounds?

A Attractive forces are stronger between ions than between molecules.


B Ionic bonds are stronger than covalent bonds.
C Ions are formed by the transfer of electrons from one atom to another.
D The atoms in covalent molecules share electrons.

16 Aluminium sulfate is made when aluminium hydroxide, Al (OH)3, reacts with dilute sulfuric acid,
H2SO4.

What is the formula of aluminium sulfate?

A Al SO4 B Al 2SO4 C Al 2(SO4)3 D Al 3(SO4)2

17 Hydrogen peroxide decomposes to form water and oxygen.

Which changes in temperature and in concentration both reduce the rate of this reaction?

temperature of concentration of
hydrogen peroxide hydrogen peroxide

A decrease decrease
B decrease increase
C increase decrease
D increase increase

18 Magnesium reacts with copper oxide.

The equation for this reaction is shown.

Mg + CuO  MgO + Cu

Which substance is acting as an oxidising agent in this reaction?

A Cu B CuO C Mg D MgO

19 Which word equation correctly describes a reaction of dilute sulfuric acid?

A sulfuric acid + zinc  zinc sulfate + water

B sulfuric acid + zinc carbonate  zinc sulfate + carbon dioxide

C sulfuric acid + zinc hydroxide  zinc sulfate + water

D sulfuric acid + zinc oxide  zinc sulfate + hydrogen

© UCLES 2021 0653/21/O/N/21

224/693
Combined By Nesrine
7
2023-2017

20 A piece of damp blue litmus paper is placed in a gas.

The litmus paper turns red and then turns white.

What is the gas?

A carbon dioxide
B chlorine
C hydrogen
D oxygen

21 Elements in Group I and Group VII of the Periodic Table are listed.

Group I Group VII

Li F
Na Cl
K Br
Rb I

Group I elements react with Group VII elements.

Which compound is formed most vigorously?

A LiF B LiI C RbF D RbI

22 Which part of the Periodic Table contains elements that are used as catalysts?

A Group I
B Group VII
C noble gases
D transition metals

23 Brass is an alloy.

What is brass?

A a compound containing two metallic elements


B a compound containing two non-metallic elements
C a mixture containing two metallic elements
D a mixture containing two non-metallic elements

© UCLES 2021 0653/21/O/N/21 [Turn over

225/693
Combined By Nesrine
8
2023-2017

24 Four metals E, F, G and H are mixed with solutions of metal salts.

The results are shown.

metal metal salt result

H E chloride no reaction
E F chloride reacts
E G chloride reacts
F H chloride no reaction
G H chloride reacts

What is the order of reactivity of these metals, from most to least reactive?

A EHGF

B EGHF

C FHGE

D FGHE

25 Carbon is used in the production of iron in a blast furnace.

A student suggests four reasons why carbon is added to the blast furnace.

1 It is an oxidising agent.
2 It burns to produce high temperatures.
3 It removes impurities by forming slag.
4 It reacts with carbon dioxide to form carbon monoxide.

Which reasons are correct?

A 1 and 2 B 1 and 4 C 2 and 3 D 2 and 4

© UCLES 2021 0653/21/O/N/21

226/693
Combined By Nesrine
9
2023-2017

26 A student measures the masses of three unpainted and three painted iron nails.

The student places the nails into separate beakers of water.

water water
unpainted painted
iron nails iron nails

After one week, the student removes the nails from the beakers, dries them and measures the
masses again.

Which row about the masses of the iron nails is correct?

mass of mass of
unpainted iron nails painted iron nails

A decreased decreased
B decreased unchanged
C increased increased
D increased unchanged

27 The fractional distillation of petroleum is shown.

refinery gas

gasoline

paraffin

light gas oil

diesel

lubricating oil

petroleum

bitumen

Which fraction contains molecules that have the largest attractive forces?

A bitumen
B diesel
C gasoline
D refinery gas

© UCLES 2021 0653/21/O/N/21 [Turn over

227/693
Combined By Nesrine
10
2023-2017

28 The diagram shows the speed–time graph for an object that is accelerating.

8
speed
m/s 6

0
0 1 2 3 4
time / s

What is the acceleration of the object and what is the distance it travels in 4.0 s?

acceleration distance
m / s2 /m

A 1.5 20
B 1.5 32
C 2.0 20
D 2.0 32

29 A ball of mass m is thrown vertically upwards with an initial speed v.

The gravitational field strength is g.

What is the kinetic energy of the ball when it has risen through a height h above its starting point?

A 1
2
(mv)2 + mgh

B 1
2
(mv)2 – mgh

C 1
2
mv2 + mgh

D 1
2
mv2 – mgh

30 A gas loses energy and changes state to become a liquid.

How do the forces between the molecules and the distances between the molecules change?

forces between distances between


molecules molecules

A decrease decrease
B decrease increase
C increase decrease
D increase increase

© UCLES 2021 0653/21/O/N/21

228/693
Combined By Nesrine
11
2023-2017

31 A student cooks a potato in a fire. The student holds the potato using a metal rod.

potato

metal rod

Which transfer of thermal energy is caused mainly by radiation?

A from the fire to the air above the fire


B from the fire to the student’s face
C from the inside of the potato to the student’s hand
D from the outside of the potato to the inside of the potato

32 A microwave oven uses microwaves with a frequency of 2.5  109 Hz.

What is the wavelength of these microwaves?

A 0.0075 m B 0.12 m C 7.5 m D 12 m

33 A ray of light passes through a glass window.

Which path does it take?

A
air glass air
B
C
D

ray of
light

© UCLES 2021 0653/21/O/N/21 [Turn over

229/693
Combined By Nesrine
12
2023-2017

34 The diagram shows a thin converging lens with focal length f.

The lens forms a magnified, upright image of an object.

At which point is the object placed?

lens

f f f

A B C D

35 Sound travels at different speeds in air, glass and water.

Which list shows these three materials in the order of increasing speed of sound (slowest to
fastest)?

A air  water  glass

B glass  water  air

C water  air  glass

D water  glass  air

36 There is a current of 4.0 A in a resistor.

How much charge passes through the resistor in 8.0 s?

A 0.50 C B 2.0 C C 12 C D 32 C

© UCLES 2021 0653/21/O/N/21

230/693
Combined By Nesrine
13
2023-2017

37 A circuit contains a battery connected to a resistor.

Which values of electromotive force (e.m.f.) and resistance produce the smallest current in the
circuit?

e.m.f. / V resistance / 

A 6.0 10
B 6.0 20
C 24 80
D 24 160

38 Four wires are made from the same material but have different lengths and diameters.

Which wire has the smallest resistance?

length diameter
/ cm / mm

A 50 0.10
B 50 0.20
C 100 0.10
D 100 0.20

39 The diagrams show four circuits, each containing an ammeter and two lamps with different
resistances.

Which circuit shows an ammeter with a reading equal to the current in each lamp?

A B C D

A
A A
A

© UCLES 2021 0653/21/O/N/21 [Turn over

231/693
Combined By Nesrine
14
2023-2017

40 What is the purpose of a fuse in an electric circuit?

A It acts as an extra resistor in the circuit.


B It keeps the current at a steady value.
C It keeps the voltage at a steady value.
D It protects the circuit from a current that is too large.

© UCLES 2021 0653/21/O/N/21

232/693
Combined By Nesrine
2023-2017

Cambridge IGCSE™

COMBINED SCIENCE 0653/22


Paper 2 Multiple Choice (Extended) October/November 2021
45 minutes

You must answer on the multiple choice answer sheet.


*1081777105*

You will need: Multiple choice answer sheet


Soft clean eraser
Soft pencil (type B or HB is recommended)

INSTRUCTIONS
 There are forty questions on this paper. Answer all questions.
 For each question there are four possible answers A, B, C and D. Choose the one you consider correct
and record your choice in soft pencil on the multiple choice answer sheet.
 Follow the instructions on the multiple choice answer sheet.
 Write in soft pencil.
 Write your name, centre number and candidate number on the multiple choice answer sheet in the
spaces provided unless this has been done for you.
 Do not use correction fluid.
 Do not write on any bar codes.
 You may use a calculator.

INFORMATION
 The total mark for this paper is 40.
 Each correct answer will score one mark.
 Any rough working should be done on this question paper.
 The Periodic Table is printed in the question paper.

This document has 16 pages.

IB21 11_0653_22/3RP
© UCLES 2021 [Turn over

233/693
Combined By Nesrine
2
2023-2017

1 Movement is a characteristic of all living organisms.

Which two other characteristics of living organisms provide the energy for movement?

A excretion and nutrition


B growth and sensitivity
C nutrition and respiration
D respiration and growth

2 The diagrams show three different specialised cells.

1 2 3

not to scale

Which row shows the correct functions of cells 1, 2 and 3?

1 2 3

A absorbs water transports oxygen moves mucus


B absorbs water transports oxygen absorbs digested food
C transports oxygen absorbs water moves mucus
D transports oxygen absorbs water absorbs digested food

3 A biological molecule is analysed and found to contain carbon, oxygen, hydrogen and nitrogen.

What is this biological molecule?

A fat
B glucose
C protein
D starch

© UCLES 2021 0653/22/O/N/21

234/693
Combined By Nesrine
3
2023-2017

4 The graph shows the effect of pH on the activity of an enzyme.

Where on the graph would collisions between enzyme and substrate be most effective?

B
enzyme
activity
D

pH

5 Which letters from the list represent the balanced equation for photosynthesis?

P C6H12O6 T H2O
Q 6C6H12O6 U 6H2O
R CO2 V O2
S 6CO2 W 6O2

A P + U  R + V

B Q + T  S + U

C R + T  W + P

D U + S  P + W

6 During pregnancy, a woman is told she is iron-deficient.

Which food could she eat to increase the iron content in her diet?

A cheese
B fruit
C milk
D red meat

© UCLES 2021 0653/22/O/N/21 [Turn over

235/693
Combined By Nesrine
4
2023-2017

7 The diagram shows part of the alimentary canal and associated structures.

stomach

bile duct

Which row correctly identifies structure X, an enzyme secreted by structure X and the action of
this enzyme?

structure X enzyme action of enzyme

A liver amylase converts proteins to amino acids


B pancreas amylase converts starch to simple sugars
C liver protease converts proteins to amino acids
D pancreas protease converts starch to simple sugars

© UCLES 2021 0653/22/O/N/21

236/693
Combined By Nesrine
5
2023-2017

8 The graph shows the rate and depth of breathing of a student at rest.

volume of air
in the lungs

0 time / s 15

Which graph shows the rate and depth of breathing of the student immediately after five minutes
of physical activity?

A B

volume of air volume of air


in the lungs in the lungs

0 time / s 15 0 time / s 15

C D

volume of air volume of air


in the lungs in the lungs

0 time / s 15 0 time / s 15

9 A plant shoot is illuminated from one side only.

What collects on the shaded side of the plant shoot?

A auxin
B chlorophyll
C glucose
D starch

© UCLES 2021 0653/22/O/N/21 [Turn over

237/693
Combined By Nesrine
6
2023-2017

10 Which row is correct for sexual reproduction?

gametes are offspring genetically


formed identical to parents

A no no
B yes no
C no yes
D yes yes

11 Which row correctly describes features of human egg cells and sperm cells?

egg cells sperm cells

A energy stores present enzymes present


B enzymes present energy stores present
C produced in large numbers flagellum present
D flagellum present produced in large numbers

12 The diagram represents four organisms in a food chain.

T  U  V  W

Which organisms are consumers?

A T, U and V B T, U and W C T, V and W D U, V and W

13 During eutrophication, what is the main reason for the increased growth of producers?

A increased availability of carbon dioxide


B increased availability of nitrate
C increased availability of oxygen
D increased availability of water

14 Which dot-and-cross diagram represents the bonding in a molecule of carbon dioxide?

A B C D

O C O O C O O C O O C O

© UCLES 2021 0653/22/O/N/21

238/693
Combined By Nesrine
7
2023-2017

15 Copper forms two different ions, Cu2+ and Cu+.

Copper forms two different oxides.

What are the formulae of these two oxides?

A CuO2 and Cu2O


B Cu2O2 and CuO
C Cu2O2 and CuO2
D CuO and Cu2O

16 Which statements about bond breaking and bond forming are correct?

1 Bond breaking is endothermic.


2 Bond breaking is exothermic.
3 Bond forming is endothermic.
4 Bond forming is exothermic.

A 1 and 3 B 1 and 4 C 2 and 3 D 2 and 4

17 Hydrogen peroxide decomposes to form water and oxygen.

Which changes in temperature and in concentration both reduce the rate of this reaction?

temperature of concentration of
hydrogen peroxide hydrogen peroxide

A decrease decrease
B decrease increase
C increase decrease
D increase increase

© UCLES 2021 0653/22/O/N/21 [Turn over

239/693
Combined By Nesrine
8
2023-2017

18 Aluminium reacts with iron oxide to produce iron.

The equation is shown.

2Al + Fe2O3  Al 2O3 + 2Fe

Which row identifies the oxidising agent and the reducing agent?

oxidising agent reducing agent

A Fe Al
B Fe Al 2O3
C Fe2O3 Al
D Fe2O3 Al 2O3

19 Ammonia dissolves in water.

Which test shows that the solution has a pH of 9?

A Blue litmus paper stays blue.


B Red litmus paper turns blue.
C Universal indicator paper turns green.
D Universal indicator paper turns blue.

20 A piece of damp blue litmus paper is placed in a gas.

The litmus paper turns red and then turns white.

What is the gas?

A carbon dioxide
B chlorine
C hydrogen
D oxygen

21 Fluorine is an element in Group VII of the Periodic Table.

Which statement about fluorine is correct?

A Fluorine is a metal with a low melting point.


B Fluorine is a gas and is less reactive than bromine.
C Fluorine molecules are diatomic.
D Chlorine displaces fluorine from its compounds.

© UCLES 2021 0653/22/O/N/21

240/693
Combined By Nesrine
9
2023-2017

22 Which statement about transition elements is not correct?

A They can act as catalysts.


B They can be metals or non-metals.
C They have high densities.
D They have high melting points.

23 Brass is an alloy.

What is brass?

A a compound containing two metallic elements


B a compound containing two non-metallic elements
C a mixture containing two metallic elements
D a mixture containing two non-metallic elements

24 Which two substances react together?

A aluminium and aqueous magnesium sulfate

B copper and aqueous iron(II) sulfate


C iron and aqueous zinc sulfate
D zinc and aqueous copper sulfate

25 Which row shows how copper can be obtained from copper oxide?

heat copper oxide electrolysis of


with carbon molten copper oxide

A  
B  
C  
D  

© UCLES 2021 0653/22/O/N/21 [Turn over

241/693
Combined By Nesrine
10
2023-2017

26 Magnesium carbonate reacts with dilute hydrochloric acid.

Calcium carbonate decomposes when heated.

Which gas is produced in both reactions?

A carbon dioxide
B carbon monoxide
C chlorine
D hydrogen

27 The fractional distillation of petroleum is shown.

refinery gas

gasoline

paraffin

light gas oil

diesel

lubricating oil

petroleum

bitumen

Which fraction contains molecules that have the largest attractive forces?

A bitumen
B diesel
C gasoline
D refinery gas

© UCLES 2021 0653/22/O/N/21

242/693
Combined By Nesrine
11
2023-2017

28 A car accelerates from rest at a constant rate. It then moves with constant speed and finally
comes to rest with non-constant deceleration.

Which diagram shows the speed–time graph for the car?

A B

speed speed

0 0
0 time 0 time

C D

speed speed

0 0
0 time 0 time

29 Four planets have different gravitational field strengths.

An object has a mass of 50 kg.

Which gravitational field strength causes the object to have a weight of 450 N?

gravitational field strength


N / kg

A 4.5
B 5.0
C 9.0
D 10.0

30 Which process is the source of the energy released from the Sun?

A chemical reactions
B geothermal heating
C nuclear fission
D nuclear fusion

© UCLES 2021 0653/22/O/N/21 [Turn over

243/693
Combined By Nesrine
12
2023-2017

31 Which statements about liquids and gases are correct?

1 Molecules in gases are further apart than molecules in liquids.


2 Molecules in liquids and gases are arranged randomly.
3 When a liquid evaporates, the temperature of the remaining liquid decreases.

A 1 and 2 only B 1 and 3 only C 2 and 3 only D 1, 2 and 3

32 The bottom of a container of water is heated.

water
Q

convection current

A convection current forms and water rises from P to Q.

Which statement is correct?

A Water at P expands and decreases in density.


B Water at P expands and increases in density.
C Water at Q expands and decreases in density.
D Water at Q expands and increases in density.

33 A microwave oven uses microwaves with a frequency of 2.5  109 Hz.

What is the wavelength of these microwaves?

A 0.0075 m B 0.12 m C 7.5 m D 12 m

© UCLES 2021 0653/22/O/N/21

244/693
Combined By Nesrine
13
2023-2017

34 A ray of light passes through a glass window.

Which path does it take?

A
air glass air
B
C
D

ray of
light

35 A thin converging lens is used as a magnifying glass.

The focal length of the lens is 5.0 cm.

How far from the lens is the object placed?

A less than 5.0 cm


B between 5.0 cm and 10 cm
C 10 cm
D more than 10 cm

36 A lightning strike transfers 20 C of charge in 5.0  10–4 s.

What is the average current during the lightning strike?

A 2.5  10–5 A B 1.0  10–2 A C 1.0  102 A D 4.0  104 A

© UCLES 2021 0653/22/O/N/21 [Turn over

245/693
Combined By Nesrine
14
2023-2017

37 A circuit contains a battery connected to a resistor.

Which values of electromotive force (e.m.f.) and resistance produce the smallest current in the
circuit?

e.m.f. / V resistance / 

A 6.0 10
B 6.0 20
C 24 80
D 24 160

38 Four wires are made from the same material but have different lengths and diameters.

Which wire has the smallest resistance?

length diameter
/ cm / mm

A 50 0.10
B 50 0.20
C 100 0.10
D 100 0.20

39 The diagrams show four circuits, each containing an ammeter and two lamps with different
resistances.

Which circuit shows an ammeter with a reading equal to the current in each lamp?

A B C D

A
A A
A

© UCLES 2021 0653/22/O/N/21

246/693
Combined By Nesrine
15
2023-2017

40 The diagram shows a circuit with four labelled components.

One component breaks the circuit automatically when the current becomes too large.

Which component does this?

D B

Permission to reproduce items where third-party owned material protected by copyright is included has been sought and cleared where possible. Every
reasonable effort has been made by the publisher (UCLES) to trace copyright holders, but if any items requiring clearance have unwittingly been included, the
publisher will be pleased to make amends at the earliest possible opportunity.

To avoid the issue of disclosure of answer-related information to candidates, all copyright acknowledgements are reproduced online in the Cambridge
Assessment International Education Copyright Acknowledgements Booklet. This is produced for each series of examinations and is freely available to download
at www.cambridgeinternational.org after the live examination series.

Cambridge Assessment International Education is part of the Cambridge Assessment Group. Cambridge Assessment is the brand name of the University of
Cambridge Local Examinations Syndicate (UCLES), which itself is a department of the University of Cambridge.

© UCLES 2021 0653/22/O/N/21

247/693
Combined By Nesrine
2023-2017

Cambridge IGCSE™

COMBINED SCIENCE 0653/23


Paper 2 Multiple Choice (Extended) October/November 2021
45 minutes

You must answer on the multiple choice answer sheet.


*8904809872*

You will need: Multiple choice answer sheet


Soft clean eraser
Soft pencil (type B or HB is recommended)

INSTRUCTIONS
 There are forty questions on this paper. Answer all questions.
 For each question there are four possible answers A, B, C and D. Choose the one you consider correct
and record your choice in soft pencil on the multiple choice answer sheet.
 Follow the instructions on the multiple choice answer sheet.
 Write in soft pencil.
 Write your name, centre number and candidate number on the multiple choice answer sheet in the
spaces provided unless this has been done for you.
 Do not use correction fluid.
 Do not write on any bar codes.
 You may use a calculator.

INFORMATION
 The total mark for this paper is 40.
 Each correct answer will score one mark.
 Any rough working should be done on this question paper.
 The Periodic Table is printed in the question paper.

This document has 16 pages. Any blank pages are indicated.

IB21 11_0653_23/3RP
© UCLES 2021 [Turn over

248/693
Combined By Nesrine
2
2023-2017

1 Movement is a characteristic of all living organisms.

Which two other characteristics of living organisms provide the energy for movement?

A excretion and nutrition


B growth and sensitivity
C nutrition and respiration
D respiration and growth

2 Which row correctly describes a feature of a specialised cell?

specialised cell feature

A egg cell energy store


B palisade cell cilia
C red blood cell cell wall
D root hair cell chloroplasts

3 Which small molecules are used to make proteins?

A amino acids
B fatty acids
C glucose
D glycerol

4 What is a suitable range for investigating the effect of temperature on the activity of an enzyme
from a human body?

A 0 C to 30 C

B 20 C to 60 C

C 40 C to 60 C

D 50 C to 100 C

© UCLES 2021 0653/23/O/N/21

249/693
Combined By Nesrine
3
2023-2017

5 Which letters from the list represent the balanced equation for photosynthesis?

P C6H12O6 T H2O
Q 6C6H12O6 U 6H2O
R CO2 V O2
S 6CO2 W 6O2

A P + U  R + V

B Q + T  S + U

C R + T  W + P

D U + S  P + W

6 Which type of digestion causes the breakdown of large, insoluble molecules into small, soluble
molecules?

A chemical
B hormonal
C mechanical
D physical

7 Which conditions cause plants to lose most mass by transpiration?

humidity temperature

A high high
B high low
C low high
D low low

© UCLES 2021 0653/23/O/N/21 [Turn over

250/693
Combined By Nesrine
4
2023-2017

8 The diagram shows part of the gas exchange system in humans.

X
Y

What are the structures labelled X and Y?

X Y

A bronchiole trachea
B bronchus trachea
C trachea bronchiole
D trachea bronchus

9 A plant shoot is illuminated from one side only.

What collects on the shaded side of the plant shoot?

A auxin
B chlorophyll
C glucose
D starch

10 What is a characteristic of insect-pollinated flowers?

A anthers hanging outside the flower


B hairy or sticky stigmas
C large quantities of smooth, light pollen
D no scent or nectar

© UCLES 2021 0653/23/O/N/21

251/693
Combined By Nesrine
5
2023-2017

11 Which comparison between human female and male gametes is correct?

eggs sperm

A have a flagellum have no flagellum


B move a short distance move a long distance
C produced in greater numbers produced in fewer numbers
D smaller size larger size

12 The diagram represents four organisms in a food chain.

T  U  V  W

Which organisms are consumers?

A T, U and V B T, U and W C T, V and W D U, V and W

13 The eutrophication of water has a number of stages.

1 2 3
increased growth dissolved oxygen available
of producers decreases and nitrate / ion levels
aquatic organisms increase
die

4 5
increased aerobic death
respiration by of producers
decomposers

What is the correct order of the stages?

A 13542

B 12543

C 31542

D 31425

14 How many electrons are shared by the atoms in a nitrogen molecule, N2?

A 2 B 4 C 6 D 8

© UCLES 2021 0653/23/O/N/21 [Turn over

252/693
Combined By Nesrine
6
2023-2017

15 The formula of magnesium chloride is MgCl 2.

The formula of sodium phosphide is Na3P.

What is the formula of magnesium phosphide?

A MgP B MgP2 C Mg2P3 D Mg3P2

16 An energy level diagram for a reaction is shown.

X
energy

progress of reaction

Which statement describes and explains energy change X?

A Energy is given out as bonds break.


B Energy is given out as bonds form.
C Energy is taken in as bonds break.
D Energy is taken in as bonds form.

17 Hydrogen peroxide decomposes to form water and oxygen.

Which changes in temperature and in concentration both reduce the rate of this reaction?

temperature of concentration of
hydrogen peroxide hydrogen peroxide

A decrease decrease
B decrease increase
C increase decrease
D increase increase

© UCLES 2021 0653/23/O/N/21

253/693
Combined By Nesrine
7
2023-2017

18 Iron oxide reacts with aluminium.

Fe2O3 + 2Al  2Fe + Al 2O3

Which row identifies the oxidising agent and reducing agent in the reaction?

oxidising agent reducing agent

A aluminium oxide aluminium


B aluminium oxide iron
C iron(III) oxide aluminium
D iron(III) oxide iron

19 Which statement describes an acid?

A It has a pH less than 7.


B It reacts with calcium carbonate to form a white precipitate.
C It reacts with hydrochloric acid to form a salt and water.
D It turns universal indicator blue.

20 A piece of damp blue litmus paper is placed in a gas.

The litmus paper turns red and then turns white.

What is the gas?

A carbon dioxide
B chlorine
C hydrogen
D oxygen

© UCLES 2021 0653/23/O/N/21 [Turn over

254/693
Combined By Nesrine
8
2023-2017

21 Some properties of noble gases are shown.

melting boiling density


point / C point / C g / cm3

helium –272 –269 0.0002


neon
argon –189
krypton –152 0.0059
xenon –112 –108 0.0097

What are the properties of neon?

melting boiling density


point / C point / C g / cm3

A –251 –274 0.0004


B –178 –174 0.0041
C –249 –246 0.0008
D –240 –236 0.0062

22 P, Q, R and S are four metals.

P is soft.

Q reacts violently with water.

R has a high melting point.

S forms blue compounds.

Which metals are transition elements?

A P and Q B P and R C Q and S D R and S

23 Brass is an alloy.

What is brass?

A a compound containing two metallic elements


B a compound containing two non-metallic elements
C a mixture containing two metallic elements
D a mixture containing two non-metallic elements

© UCLES 2021 0653/23/O/N/21

255/693
Combined By Nesrine
9
2023-2017

24 The results of mixing metal X with aqueous metal ions are shown.

X + Zn2+  X2+ + Zn

X + Cu2+  X2+ + Cu

X + Mg2+  Mg2+ + X

What is the position of X in the reactivity series?

most least
reactive reactive

A X Mg Zn Cu
B Mg X Zn Cu
C Mg Zn X Cu
D Mg Zn Cu X

25 Which substance reduces iron(III) oxide in the blast furnace?

A carbon dioxide
B carbon monoxide
C limestone
D oxygen

26 Which statements about the rusting of iron are correct?

1 It requires oxygen and water.


2 It is prevented by coating with another metal.
3 Painted iron nails do not rust.

A 1 and 2 only B 1 and 3 only C 2 and 3 only D 1, 2 and 3

© UCLES 2021 0653/23/O/N/21 [Turn over

256/693
Combined By Nesrine
10
2023-2017

27 The fractional distillation of petroleum is shown.

refinery gas

gasoline

paraffin

light gas oil

diesel

lubricating oil

petroleum

bitumen

Which fraction contains molecules that have the largest attractive forces?

A bitumen
B diesel
C gasoline
D refinery gas

28 A distance–time graph and a speed–time graph are plotted for a moving vehicle.

Which feature gives the acceleration of the vehicle?

A the area under the distance–time graph


B the area under the speed–time graph
C the gradient of the distance–time graph
D the gradient of the speed–time graph

29 A container is filled to the top with water. An object is slowly lowered into the water until it is
completely submerged. The water that overflows from the container is collected.

The mass of the object is 84 kg. The volume of water collected is 0.12 m3.

What is the density of the object?

A 1.4 kg / m3 B 10 kg / m3 C 84 kg / m3 D 700 kg / m3

© UCLES 2021 0653/23/O/N/21

257/693
Combined By Nesrine
11
2023-2017

30 A spring that obeys Hooke’s Law has unstretched length l.

A load F is suspended from the spring, and the spring extends by an amount x.

Which equation is used to define the spring constant k ?


F
A k = Fx B k= C k= F D k= x
(l + x ) x F

31 A force pushes an object in a straight line.

Which expression gives the work done by the force?

A force  distance moved

B force  time taken

C force  distance moved

D force  time taken

32 Water in a beaker evaporates quickly.

Which statements about the evaporation of the water from the beaker are correct?

1 Evaporation happens at all temperatures between 0 C and 100 C.


2 The more-energetic water molecules escape from the surface of the water.
3 The temperature of the water remaining in the beaker decreases.

A 1 and 2 only B 1 and 3 only C 2 and 3 only D 1, 2 and 3

33 A gas is heated.

Which statement explains how thermal energy is transferred by convection in the gas?

A The heated gas expands, becomes less dense and falls.


B The heated gas expands, becomes less dense and rises.
C The heated gas expands, becomes more dense and falls.
D The heated gas expands, becomes more dense and rises.

34 A microwave oven uses microwaves with a frequency of 2.5  109 Hz.

What is the wavelength of these microwaves?

A 0.0075 m B 0.12 m C 7.5 m D 12 m

© UCLES 2021 0653/23/O/N/21 [Turn over

258/693
Combined By Nesrine
12
2023-2017

35 A ray of light passes through a glass window.

Which path does it take?

A
air glass air
B
C
D

ray of
light

36 The diagram shows a thin converging lens used as a magnifying glass. Each principal focus of
the lens is labelled.

image
principal focus
object

principal focus

The object is moved to the right, closer to the lens.

What happens to the image?

A It moves to the left and becomes larger.


B It moves to the left and becomes smaller.
C It moves to the right and becomes larger.
D It moves to the right and becomes smaller.

37 There is a potential difference of 4.0 V across a resistor of resistance 2.0 .

How much charge passes through the resistor in 10 s?

A 0.80 C B 5.0 C C 20 C D 80 C

© UCLES 2021 0653/23/O/N/21

259/693
Combined By Nesrine
13
2023-2017

38 A circuit contains a battery connected to a resistor.

Which values of electromotive force (e.m.f.) and resistance produce the smallest current in the
circuit?

e.m.f. / V resistance / 

A 6.0 10
B 6.0 20
C 24 80
D 24 160

39 Four wires are made from the same material but have different lengths and diameters.

Which wire has the smallest resistance?

length diameter
/ cm / mm

A 50 0.10
B 50 0.20
C 100 0.10
D 100 0.20

40 The diagrams show four circuits, each containing an ammeter and two lamps with different
resistances.

Which circuit shows an ammeter with a reading equal to the current in each lamp?

A B C D

A
A A
A

© UCLES 2021 0653/23/O/N/21

260/693
The Periodic Table of Elements
Group
I II III IV V VI VII VIII

© UCLES 2021
1 2

H He
hydrogen helium
Key 1 4
3 4 atomic number 5 6 7 8 9 10

Li Be atomic symbol B C N O F Ne
lithium beryllium name boron carbon nitrogen oxygen fluorine neon
7 9 relative atomic mass 11 12 14 16 19 20
11 12 13 14 15 16 17 18
Na Mg Al Si P S Cl Ar
sodium magnesium aluminium silicon phosphorus sulfur chlorine argon
23 24 27 28 31 32 35.5 40
19 20 21 22 23 24 25 26 27 28 29 30 31 32 33 34 35 36
K Ca Sc Ti V Cr Mn Fe Co Ni Cu Zn Ga Ge As Se Br Kr
potassium calcium scandium titanium vanadium chromium manganese iron cobalt nickel copper zinc gallium germanium arsenic selenium bromine krypton
39 40 45 48 51 52 55 56 59 59 64 65 70 73 75 79 80 84
37 38 39 40 41 42 43 44 45 46 47 48 49 50 51 52 53 54

Rb Sr Y Zr Nb Mo Tc Ru Rh Pd Ag Cd In Sn Sb Te I Xe
rubidium strontium yttrium zirconium niobium molybdenum technetium ruthenium rhodium palladium silver cadmium indium tin antimony tellurium iodine xenon
85 88 89 91 93 96 – 101 103 106 108 112 115 119 122 128 127 131
16

55 56 57–71 72 73 74 75 76 77 78 79 80 81 82 83 84 85 86
lanthanoids

261/693
Cs Ba Hf Ta W Re Os Ir Pt Au Hg Tl Pb Bi Po At Rn

0653/23/O/N/21
caesium barium hafnium tantalum tungsten rhenium osmium iridium platinum gold mercury thallium lead bismuth polonium astatine radon
133 137 178 181 184 186 190 192 195 197 201 204 207 209 – – –
87 88 89–103 104 105 106 107 108 109 110 111 112 114 116
actinoids
Fr Ra Rf Db Sg Bh Hs Mt Ds Rg Cn Fl Lv
francium radium rutherfordium dubnium seaborgium bohrium hassium meitnerium darmstadtium roentgenium copernicium flerovium livermorium
– – – – – – – – – – – – –

57 58 59 60 61 62 63 64 65 66 67 68 69 70 71
lanthanoids La Ce Pr Nd Pm Sm Eu Gd Tb Dy Ho Er Tm Yb Lu
lanthanum cerium praseodymium neodymium promethium samarium europium gadolinium terbium dysprosium holmium erbium thulium ytterbium lutetium
139 140 141 144 – 150 152 157 159 163 165 167 169 173 175
89 90 91 92 93 94 95 96 97 98 99 100 101 102 103
actinoids Ac Th Pa U Np Pu Am Cm Bk Cf Es Fm Md No Lr
actinium thorium protactinium uranium neptunium plutonium americium curium berkelium californium einsteinium fermium mendelevium nobelium lawrencium
– 232 231 238 – – – – – – – – – – –
2023-2017

The volume of one mole of any gas is 24 dm3 at room temperature and pressure (r.t.p.).
Combined By Nesrine
Combined By Nesrine
2023-2017

Cambridge IGCSE™

COMBINED SCIENCE 0653/22


Paper 2 Multiple Choice (Extended) February/March 2020
45 minutes

You must answer on the multiple choice answer sheet.


*5868561160*

You will need: Multiple choice answer sheet


Soft clean eraser
Soft pencil (type B or HB is recommended)

INSTRUCTIONS
• There are forty questions on this paper. Answer all questions.
• For each question there are four possible answers A, B, C and D. Choose the one you consider correct
and record your choice in soft pencil on the multiple choice answer sheet.
• Follow the instructions on the multiple choice answer sheet.
• Write in soft pencil.
• Write your name, centre number and candidate number on the multiple choice answer sheet in the
spaces provided unless this has been done for you.
• Do not use correction fluid.
• Do not write on any bar codes.
• You may use a calculator.

INFORMATION
• The total mark for this paper is 40.
• Each correct answer will score one mark. A mark will not be deducted for a wrong answer.
• Any rough working should be done on this question paper.
• The Periodic Table is printed in the question paper.

This document has 16 pages. Blank pages are indicated.

IB20 03_0653_22/5RP
© UCLES 2020 [Turn over

262/693
Combined By Nesrine
2
2023-2017

1 Which row shows the features of a plant cell?

cell membrane cell wall


surrounding the surrounding the vacuole present
cell wall cell membrane

A   
B   
C   
D   

2 When an apple is cut, the cut surface quickly turns brown. This is due to enzyme action.

Which action destroys the enzyme?

A brushing the cut surface with a strong sugar solution


B cutting the apple into smaller pieces
C placing the cut apple in boiling water
D placing the cut apple in cold water

3 Which vitamin and which mineral would a doctor recommend increasing in the diet of a patient
with scurvy and anemia?

A vitamin C and calcium


B vitamin C and iron
C vitamin D and calcium
D vitamin D and iron

4 Animals break down large, insoluble molecules into small, soluble molecules in the alimentary
canal.

What is this process?

A chemical digestion
B chemical ingestion
C mechanical digestion
D mechanical ingestion

© UCLES 2020 0653/22/F/M/20

263/693
Combined By Nesrine
3
2023-2017

5 The table shows how humidity may affect the rate of diffusion of water vapour from a plant to the
surrounding air.

Which row will result in the highest rate of transpiration?

diffusion gradient
humidity
for water

A high low
B high high
C low low
D low high

6 Which diagram shows how blood circulates in mammals?

A B C D
lungs heart heart heart

heart lungs
body lungs body lungs
body body

7 Which component of tobacco smoke increases the risk of lung cancer?

A carbon dioxide
B carbon monoxide
C nicotine
D tar

8 What is the equation for aerobic respiration?

A carbon dioxide + water → glucose + oxygen

B glucose + oxygen → carbon dioxide + water

C glucose + water → carbon dioxide + oxygen

D oxygen + water → carbon dioxide + glucose

© UCLES 2020 0653/22/F/M/20 [Turn over

264/693
Combined By Nesrine
4
2023-2017

9 When an athlete prepares for the start of a sprint race, excitement causes the concentration of
adrenaline in the blood to increase.

What effects does adrenaline have on the blood glucose concentration and the heart rate of the
athlete?

blood glucose
heart rate
concentration

A decreases decreases
B decreases increases
C increases decreases
D increases increases

10 Which row shows the responses of a young shoot to gravity and light?

gravity light

A negatively gravitropic negatively phototropic


B negatively gravitropic positively phototropic
C positively gravitropic negatively phototropic
D positively gravitropic positively phototropic

© UCLES 2020 0653/22/F/M/20

265/693
Combined By Nesrine
5
2023-2017

11 The diagram shows a section through a flower.

What are the correct labels and functions for parts X and Y of the flower?

X Y
label function label function

A petal attracts insects anther produces pollen grains


B petal protects flower ovary produces pollen grains
C sepal attracts insects anther contains egg cells
D sepal protects flower ovary contains egg cells

12 What gives the human embryo protection from mechanical shock?

A amniotic fluid
B amniotic sac
C placenta
D umbilical cord

13 What is an undesirable effect of overuse of fertilisers in agriculture?

A acid rain
B deforestation
C eutrophication
D global warming

© UCLES 2020 0653/22/F/M/20 [Turn over

266/693
Combined By Nesrine
6
2023-2017

14 What happens to water molecules when water is heated to 100 °C in a beaker?

A They gain energy and escape from the beaker.


B They gain energy and move more slowly.
C They lose energy and escape from the beaker.
D They lose energy and move more slowly.

15 A dye contains four different coloured components.

The dye is separated by chromatography.

The table shows the colour and Rf values of the four coloured components.

colour Rf value

blue 0.50
green 0.52
red 0.74
yellow 0.36

Which two dyes are furthest apart from each other on the final chromatogram?

A blue and green


B blue and yellow
C green and red
D red and yellow

16 A mixture contains hydrogen, helium, neon and oxygen.

What does this mixture contain?

A elements and compounds


B elements only
C molecules and compounds
D molecules only

© UCLES 2020 0653/22/F/M/20

267/693
Combined By Nesrine
7
2023-2017

17 Some information about a sodium ion is shown.

proton nucleon number of number of number of


particle
number number protons neutrons electrons

Na+ 11 23 11 X Y

What are the values of X and Y?

X Y

A 11 10
B 11 11
C 12 10
D 12 11

18 Which diagram represents a molecule of methanol?

A B

H H

H C O H H C O H

H H

C D

H H

H C O H H C O H

H H

© UCLES 2020 0653/22/F/M/20 [Turn over

268/693
Combined By Nesrine
8
2023-2017

19 Aqueous sodium sulfate reacts with aqueous barium chloride to make barium sulfate and
sodium chloride.

What is the ionic equation for this reaction?

A Ba2+(aq) + SO42–(aq) → BaSO4(aq)

B Ba2+(aq) + SO42–(aq) → BaSO4(s)

C Na+(aq) + Cl –(aq) → NaCl (s)

D Na+(aq) + Cl –(aq) → NaCl (aq)

20 Which energy level diagram identifies the activation energy and the energy change for an
exothermic reaction?

A B
key
1 = activation energy
1
2 = energy change
1 for the reaction

2 2

C D

2
2 1

21 Which process is a redox reaction?

A combustion of methane
B decomposition of calcium carbonate
C neutralisation of dilute hydrochloric acid by copper oxide
D precipitation by the addition of aqueous silver nitrate to aqueous chloride ions

© UCLES 2020 0653/22/F/M/20

269/693
Combined By Nesrine
9
2023-2017

22 A solution of compound X produces a dark green precipitate when aqueous sodium hydroxide is
added.

What is X?

A copper(II) chloride

B copper(II) sulfate

C iron(II) sulfate

D iron(III) chloride

23 Rubidium is a very reactive Group I metal.

It is kept in a sealed box surrounded by a gas.

gas

rubidium

Which gas does not react with rubidium?

A chlorine
B neon
C oxygen
D water vapour

24 Which ionic equation represents a more reactive metal displacing a less reactive metal?

A Cu + Mg2+ → Cu2+ + Mg

B Mg + Ca2+ → Mg2+ + Ca

C Zn + Mg2+ → Zn2+ + Mg

D Zn + Cu2+ → Zn2+ + Cu

25 Why is carbon used to extract some metals from their oxide ores?

A It oxidises the ore by removing oxygen.


B It prevents the oxygen of the air reacting with the ore.
C It reacts with impurities in the ore.
D It reduces the ore by removing oxygen.

© UCLES 2020 0653/22/F/M/20 [Turn over

270/693
Combined By Nesrine
10
2023-2017

26 Which statement describes the structure of sodium chloride?

A It is composed of a regular arrangement of alternating positive and negative ions.


B It is composed of negatively charged sodium ions joined to positively charged chloride ions.
C It is composed of oppositely charged ions held together by strong covalent bonds.
D It is composed of sodium atoms joined to chlorine atoms by shared pairs of electrons.

27 What is formed during the complete combustion of a hydrocarbon?

A carbon dioxide and water


B carbon dioxide and hydrogen
C carbon monoxide and carbon dioxide
D carbon monoxide and water

28 The diagrams show two distance–time graphs and two speed–time graphs.

Which graph represents the motion of an object that is moving with constant acceleration?

A B

distance distance

0 0
0 time 0 time

C D

speed speed

0 0
0 time 0 time

© UCLES 2020 0653/22/F/M/20

271/693
Combined By Nesrine
11
2023-2017

29 A measuring cylinder contains 60 cm3 of water.

A solid object of mass 120 g is lowered into the water until it is completely submerged.

The new reading on the measuring cylinder is 80 cm3.

What is the density of the object?

A 0.50 g / cm3 B 1.5 g / cm3 C 2.0 g / cm3 D 6.0 g / cm3

30 A heavy bag of flour is dragged against friction along a horizontal floor at a constant speed of
12 m / s for 2.0 s.

The energy input required is 3000 J.

What is the force due to friction?

A 125 N B 500 N C 18 000 N D 72 000 N

31 A toy car rolls from rest down a slope and on to a horizontal bench.

The car stops before it reaches the end of the bench.

What energy changes take place during this journey?

A gravitational potential → kinetic → elastic potential

B gravitational potential → kinetic → thermal and sound

C kinetic → gravitational potential → elastic potential

D kinetic → gravitational potential → thermal and sound

32 Which statement about a tidal energy power station is correct?

A It creates no environmental impact when being built.


B It does not work at night.
C It does not work when there is no wind.
D It supplies energy at predictable times.

33 Which row describes the forces between molecules in a solid and the motion of the molecules in
a solid?

forces motion

A strong free to change places


B strong vibration only
C weak free to change places
D weak vibration only

© UCLES 2020 0653/22/F/M/20 [Turn over

272/693
Combined By Nesrine
12
2023-2017

34 A student moves one end of a long rope up and down through a short distance. A wave travels
along the rope in the direction shown.

moved up and down wave direction along rope


a short distance

The student now moves the rope up and down through a larger distance. He also moves it up
and down more times in each minute.

Which row shows the effects of these two changes?

amplitude of frequency of
the wave the wave

A decreases decreases
B decreases increases
C increases decreases
D increases increases

35 A student determines the speed of sound in air. She measures the time between making a sound
and hearing the echo from a cliff.

cliff

student

She uses the equation: speed = distance .


time
Which type of sound does she make and which distance does she use in her calculation?

type of sound distance used

A continuous sound 2 × distance to cliff

continuous sound 1 × distance to cliff


B
2
C short, sharp sound 2 × distance to cliff

short, sharp sound 1 × distance to cliff


D
2

© UCLES 2020 0653/22/F/M/20

273/693
Combined By Nesrine
13
2023-2017

36 A polythene rod is rubbed with a cloth. The rod becomes positively charged.

What has happened to the rod?

A It has gained electrons.


B It has gained protons.
C It has lost electrons.
D It has lost protons.

37 In the circuit shown, the cell has an electromotive force (e.m.f.) of 1.5 V and the total resistance of
the circuit is 12 Ω.

What is the total charge that flows through the cell in 2.0 minutes?

A 0.25 C B 15 C C 36 C D 2160 C

38 In each of four circuits a lamp is connected to a battery using connecting wires that have
resistance.

The wires are all made from the same metal but have different lengths and thicknesses.

The lamps are all identical and the batteries are all identical.

In which circuit does the lamp shine most brightly?

length of diameter of
connecting connecting
wires / cm wires / mm

A 10 0.25
B 10 0.50
C 20 0.25
D 20 0.50

© UCLES 2020 0653/22/F/M/20 [Turn over

274/693
Combined By Nesrine
14
2023-2017

39 The diagram shows a circuit containing four resistors and four ammeters.

Which ammeter has the smallest reading?

A A A D

10 Ω 40 Ω
B
30 Ω
A

20 Ω
A
C

40 An electric oven is connected to the mains supply using insulated copper wires. The wires
become very warm.

Which change reduces the amount of heat produced in the connecting wires?

A Use thicker copper wires.


B Use thinner copper wires.
C Use thicker insulation.
D Use thinner insulation.

© UCLES 2020 0653/22/F/M/20

275/693
Combined By Nesrine
2023-2017

Cambridge IGCSE™

COMBINED SCIENCE 0653/21


Paper 2 Multiple Choice (Extended) May/June 2020
45 minutes

You must answer on the multiple choice answer sheet.


*8938744342*

You will need: Multiple choice answer sheet


Soft clean eraser
Soft pencil (type B or HB is recommended)

INSTRUCTIONS
• There are forty questions on this paper. Answer all questions.
• For each question there are four possible answers A, B, C and D. Choose the one you consider correct
and record your choice in soft pencil on the multiple choice answer sheet.
• Follow the instructions on the multiple choice answer sheet.
• Write in soft pencil.
• Write your name, centre number and candidate number on the multiple choice answer sheet in the
spaces provided unless this has been done for you.
• Do not use correction fluid.
• Do not write on any bar codes.
• You may use a calculator.

INFORMATION
• The total mark for this paper is 40.
• Each correct answer will score one mark. A mark will not be deducted for a wrong answer.
• Any rough working should be done on this question paper.
• The Periodic Table is printed in the question paper.

This document has 16 pages. Blank pages are indicated.

IB20 06_0653_21/2RP
© UCLES 2020 [Turn over

276/693
Combined By Nesrine
2
2023-2017

1 The cytoplasm of a plant cell contains a 15% sugar solution. The plant cell is placed in sugar
solutions of different concentrations.

In which solution would there be a net diffusion of water out of the cell?

A 5% sugar solution
B 10% sugar solution
C 15% sugar solution
D 20% sugar solution

2 Which row matches the adaptation of a root hair cell to its function?

adaptation function

A large surface area uptake of water and glucose


B large surface area uptake of water and ions
C small surface area uptake of water and glucose
D small surface area uptake of water and ions

3 Which condition could result from a shortage of fibre in the diet?

A constipation
B obesity
C scurvy
D starvation

4 Which row correctly matches the enzyme to the products?

enzyme products

A lipase amino acids only


B lipase glycerol and fatty acids
C protease fatty acids only
D protease glycerol and amino acids

© UCLES 2020 0653/21/M/J/20

277/693
Combined By Nesrine
3
2023-2017

5 Which features are found in a typical animal cell?

cell
cell wall chloroplast cytoplasm nucleus vacuole
membrane

A      
B      
C      
D      

6 Which feature of red blood cells allows them to transport oxygen?

A contain haemoglobin
B large size
C surface hairs
D thick cell membrane

7 How does auxin cause a plant shoot to bend to the right?

A Cells elongate more on the left side of the shoot than on the right side.
B Cells elongate more on the right side of the shoot than on the left side.
C Cells shrink on the left side of the shoot.
D Cells shrink on the right side of the shoot.

8 Four people have the same resting pulse rate and the same blood glucose concentration. The
table shows their pulse rates and blood glucose concentrations later on the same day.

Which person has the highest concentration of adrenaline in their blood?

blood glucose
pulse rate / beats
concentration
per minute
/mg per dm3

A 70 65
B 70 100
C 120 65
D 120 100

© UCLES 2020 0653/21/M/J/20 [Turn over

278/693
Combined By Nesrine
4
2023-2017

9 The diagram shows the root of a plant exposed to light and gravity, and the same root a day later.

light

gravity

Light does not influence the growth of roots in this plant.

Which row shows how the root has responded?

gravitropism phototropism

A grows away from the stimulus no response


B grows towards the stimulus no response
C no response grows away from the stimulus
D no response grows towards the stimulus

10 The diagram shows pollen grains from two different plants.

1 2

How are the two pollen grains dispersed?

pollen grain 1 pollen grain 2

A insect insect
B insect wind
C wind insect
D wind wind

© UCLES 2020 0653/21/M/J/20

279/693
Combined By Nesrine
5
2023-2017

11 Which part of the male reproductive system in humans produces sperm?

A penis
B scrotum
C testes
D urethra

12 What is defined as all of the organisms and their environment interacting together in a given
area?

A ecosystem
B food chain
C food web
D trophic levels

13 The list describes six events involved in the eutrophication of fresh water.

P death of organisms that need dissolved oxygen in water


Q increased aerobic respiration by decomposers
R increased availability of nitrate and other ions
S increased decomposition after death of producers
T increased growth of producers
U reduction in dissolved oxygen in water

What is the normal sequence of events leading to eutrophication of a body of fresh water?

A Q → S → R → U → T → P

B R → T → S → Q → U → P

C Q → T → S → R → U → P

D R → S → Q → U → T → P

14 In which change of state do the particles gain kinetic energy and remain tightly packed?

A gas to liquid
B liquid to gas
C liquid to solid
D solid to liquid

© UCLES 2020 0653/21/M/J/20 [Turn over

280/693
Combined By Nesrine
6
2023-2017

15 Salt, sand and water are stirred together in a beaker.

The salt dissolves in the water.

What does the beaker contain?

A a mixture of a solution and a solid


B a mixture of three elements
C only one compound and one solid
D only one compound containing three elements

16 Which ion is formed from a metal?

A Cl – B H+ C Na+ D NH4+

17 Sodium burns in oxygen forming sodium oxide.

An equation for this reaction is shown.

xNa + yO2 → zNa2O

What are x, y and z?

x y z

A 2 1 1
B 2 2 1
C 4 1 2
D 4 2 2

© UCLES 2020 0653/21/M/J/20

281/693
Combined By Nesrine
7
2023-2017

18 Molten silver chloride is electrolysed using inert electrodes.

anode cathode

molten
silver chloride

The cathode attracts ......1...... in the electrolyte where they are ......2...... .

Which words complete the gaps?

1 2

A chloride ions oxidised


B chloride ions reduced
C silver ions reduced
D silver ions oxidised

19 Solid sodium carbonate is added to vinegar in a beaker and stirred.

sodium
carbonate

beaker vinegar

watch glass

water

The water in the watch glass freezes.

Which statement about the reaction explains why the water freezes?

A It is a redox reaction.
B It is an endothermic reaction.
C It is catalysed by sodium carbonate.
D It is thermal decomposition.

© UCLES 2020 0653/21/M/J/20 [Turn over

282/693
Combined By Nesrine
8
2023-2017

20 The rate of a reaction increases when the temperature or the concentration of the reactants
increases.

Which row explains why the rate of reaction increases?

number of particles
activation collisions
change with energy greater than
energy per second
the activation energy

A increase in increases increases stays the same


concentration
B increase in stays the same stays the same increases
concentration
C increase in stays the same increases stays the same
temperature
D increase in stays the same increases increases
temperature

21 Hot carbon reacts with carbon dioxide to form carbon monoxide.

C(s) + CO2(g) → 2CO(g)

Which statements are correct?

1 Carbon is being oxidised.


2 The reducing agent is carbon.
3 The oxidising agent is carbon monoxide.
4 Carbon dioxide is being reduced.

A 1, 2 and 3 B 1, 2 and 4 C 2 and 4 only D 3 and 4

22 Copper(II) sulfate is prepared by reacting copper(II) oxide with dilute sulfuric acid.

CuO(s) + H2SO4(aq) → CuSO4(aq) + H2O(I)

Which statement is correct?

A Excess copper(II) oxide is used because it can be easily removed by filtration.

B Excess copper(II) oxide is used because it can be easily removed by reacting with more
sulfuric acid.
C Excess sulfuric acid is used because it can be easily removed by evaporation.

D Excess sulfuric acid is used because unreacted copper(II) oxide would contaminate the
product.

© UCLES 2020 0653/21/M/J/20

283/693
Combined By Nesrine
9
2023-2017

23 Solution X is mixed with nitric acid and aqueous barium nitrate.

A white precipitate is formed.

Which ion is present in solution X?


A carbonate
B chloride
C nitrate
D sulfate

24 Properties of some Group II elements are shown.

atomic melting reaction


number point / °C with cold water

Mg 12
Ca 20 850 reacts slowly
Sr 38 reacts quickly
Ba 56 714

Group II elements show similar trends in melting point and reactivity as Group I elements.

Which statement is correct?

A Barium reacts violently with cold water.


B Magnesium burns brightly when added to cold water.
C Magnesium has the lowest melting point.
D Strontium is the most reactive.

25 Copper can be made from copper oxide by reacting it with carbon at a high temperature.

Why is carbon used?

A It does not react with copper.


B It is a conductor of electricity.
C It is a high melting point solid.
D It is more reactive than copper.

26 Which volume of air contains about 20 cm3 of oxygen?

A 25 cm3 B 50 cm3 C 80 cm3 D 100 cm3

© UCLES 2020 0653/21/M/J/20 [Turn over

284/693
Combined By Nesrine
10
2023-2017

27 Petroleum is separated into useful fractions by fractional distillation.

Which statement about the fractions is correct?

A All the fractions are used as fuels.


B Bitumen has the strongest attractive forces between molecules.
C Gasoline contains the largest molecules.
D Refinery gas is the least volatile.

28 A car travels at various speeds during a short journey.

The table shows the distances travelled and the times taken during each of four stages
P, Q, R and S.

stage P Q R S

distance travelled / km 1.8 3.6 2.7 2.7


time taken / minutes 2.0 2.0 4.0 3.0

During which two stages is the car travelling at the same average speed?

A P and Q B P and S C Q and R D R and S

29 A solid, rectangular block of wood has length 4.0 cm, width 5.0 cm and height 6.0 cm.

The mass of the block is 90 g.

What is the density of the wood?

A 0.75 g / cm3 B 1.3 g / cm3 C 4.5 g / cm3 D 6.0 g / cm3

30 An object is travelling in a straight line at constant speed.

Which statement describes the resultant force on the object?

A It acts in the opposite direction to the motion of the object.


B It acts in the same direction as the motion of the object.
C It is constant, but not zero.
D It is zero.

© UCLES 2020 0653/21/M/J/20

285/693
Combined By Nesrine
11
2023-2017

31 A spring obeys Hooke’s law. A load of 10 N hangs from the spring and causes the spring to
extend by 12 mm.

Two springs, identical to the first one, are now joined as shown. A load of 5.0 N is hung from the
springs.

springs

5.0 N

What is the total extension of the combination of the two springs?

A 3.0 mm B 6.0 mm C 12 mm D 24 mm

32 A force of 4.0 N acts on a body for 6.0 s. The body moves a distance of 15 m in the direction of the
force.

How much energy is transferred?

A 10 J B 24 J C 60 J D 360 J

33 Oxygen in a steel cylinder is easily compressed, but steel cannot be compressed.

How is this difference explained?

A The forces between the atoms in steel are greater than those between the molecules in
oxygen.
B The kinetic energy of the atoms in steel is greater than that of oxygen molecules.
C The mass of an oxygen molecule is different from the mass of an atom in steel.
D The atoms in steel are further apart than the molecules in oxygen.

© UCLES 2020 0653/21/M/J/20 [Turn over

286/693
Combined By Nesrine
12
2023-2017

34 End X of an iron rod is held in a fire.

X
fire iron rod

The other end Y of the rod becomes warm by thermal conduction through the rod.

One process of conduction involves atoms at end X vibrating faster.

This vibration is passed on to atoms at end Y.

How does this happen?

A The atoms collide with their neighbouring atoms and transfer energy.
B The atoms move along the rod, taking energy with them.
C The atoms emit infrared radiation which travels through the rod.
D The atoms produce an electric current in the rod.

35 The sound from a drum is loud and has a low pitch.

Which row describes the amplitude and the frequency of the sound wave?

amplitude frequency

A large high
B large low
C small high
D small low

36 A thin converging lens has a focal length of 5.0 cm.

An object is placed different distances from the lens.

For which distance does the lens act as a magnifying glass?

A 15 cm B 10 cm C 6.0 cm D 3.0 cm

© UCLES 2020 0653/21/M/J/20

287/693
Combined By Nesrine
13
2023-2017

37 How are sound waves transmitted in air?

A by compressions and crests


B by compressions and rarefactions
C by crests and rarefactions
D by crests and troughs

38 What is the definition of electrical current?

A the rate of flow of charge


B the rate of flow of energy
C the rate of flow of power
D the rate of flow of voltage

39 A 12 V power supply is connected to a 6.0 Ω resistor. This causes a current in the resistor.

How much thermal energy is produced in the resistor in 5.0 minutes?

A 120 J B 600 J C 7200 J D 21 600 J

40 The charger for a laptop computer is connected by a cable to the mains supply through a plug.

The plug contains a 13 A fuse. The cable is designed to carry a current of 2 A.

A fault develops and the current in the cable increases to 5 A.

laptop
plug computer

connecting cable

What is a possible danger caused by this larger current?

A A large amount of electrical energy is wasted.


B Somebody receives an electric shock.
C The fuse blows and starts a fire.
D The cable overheats and starts a fire.

© UCLES 2020 0653/21/M/J/20

288/693
Combined By Nesrine
2023-2017

Cambridge IGCSE™

COMBINED SCIENCE 0653/22


Paper 2 Multiple Choice (Extended) May/June 2020
45 minutes

You must answer on the multiple choice answer sheet.


*6383492179*

You will need: Multiple choice answer sheet


Soft clean eraser
Soft pencil (type B or HB is recommended)

INSTRUCTIONS
• There are forty questions on this paper. Answer all questions.
• For each question there are four possible answers A, B, C and D. Choose the one you consider correct
and record your choice in soft pencil on the multiple choice answer sheet.
• Follow the instructions on the multiple choice answer sheet.
• Write in soft pencil.
• Write your name, centre number and candidate number on the multiple choice answer sheet in the
spaces provided unless this has been done for you.
• Do not use correction fluid.
• Do not write on any bar codes.
• You may use a calculator.

INFORMATION
• The total mark for this paper is 40.
• Each correct answer will score one mark. A mark will not be deducted for a wrong answer.
• Any rough working should be done on this question paper.
• The Periodic Table is printed in the question paper.

This document has 16 pages. Blank pages are indicated.

IB20 06_0653_22/3RP
© UCLES 2020 [Turn over

289/693
Combined By Nesrine
2
2023-2017

1 The diagram shows a palisade mesophyll cell from a leaf.

The features of the cell are numbered.

Which features are found only in plant cells?

A 1, 2 and 3 B 1, 5 and 6 C 2, 4 and 5 D 3, 4 and 6

2 Which row about osmosis is correct?

molecules details of permeability


that move movement of membrane

A solute from a concentrated fully


solution to a dilute solution
B solute from a dilute solution partially
to a concentrated solution
C water from a concentrated fully
solution to a dilute solution
D water from a dilute solution partially
to a concentrated solution

3 Which row matches the adaptation of a root hair cell to its function?

adaptation function

A large surface area uptake of water and glucose


B large surface area uptake of water and ions
C small surface area uptake of water and glucose
D small surface area uptake of water and ions

© UCLES 2020 0653/22/M/J/20

290/693
Combined By Nesrine
3
2023-2017

4 The graph shows the activity of an enzyme as temperature increases.

enzyme
activity

X
temperature

What happens to the enzyme as the temperature increases above X?

A The enzyme becomes too hot and dies.


B The enzyme denatures.
C The enzyme is used up.
D The enzyme activity increases.

5 What are the correct substrate and products for lipase?

substrate products

A fat amino acids


B fat fatty acids and glycerol
C protein amino acids
D protein fatty acids and glycerol

6 How does auxin cause a plant shoot to bend to the right?

A Cells elongate more on the left side of the shoot than on the right side.
B Cells elongate more on the right side of the shoot than on the left side.
C Cells shrink on the left side of the shoot.
D Cells shrink on the right side of the shoot.

© UCLES 2020 0653/22/M/J/20 [Turn over

291/693
Combined By Nesrine
4
2023-2017

7 Which component of tobacco smoke reduces the ability of haemoglobin to carry oxygen?

A carbon monoxide
B nicotine
C smoke particles
D tar

8 Which substances are the products of photosynthesis?

A carbon dioxide and glucose


B glucose and oxygen
C oxygen and water
D water and carbon dioxide

9 Four people have the same resting pulse rate and the same blood glucose concentration. The
table shows their pulse rates and blood glucose concentrations later on the same day.

Which person has the highest concentration of adrenaline in their blood?

blood glucose
pulse rate / beats
concentration
per minute
/mg per dm3

A 70 65
B 70 100
C 120 65
D 120 100

© UCLES 2020 0653/22/M/J/20

292/693
Combined By Nesrine
5
2023-2017

10 The diagram shows the root of a plant exposed to light and gravity, and the same root a day later.

light

gravity

Light does not influence the growth of roots in this plant.

Which row shows how the root has responded?

gravitropism phototropism

A grows away from the stimulus no response


B grows towards the stimulus no response
C no response grows away from the stimulus
D no response grows towards the stimulus

11 Which diagram of a flower is correctly labelled?

A B
anther
stigma stigma

anther

C D
anther stigma anther

stigma

© UCLES 2020 0653/22/M/J/20 [Turn over

293/693
Combined By Nesrine
6
2023-2017

12 Which row contains correct adaptive features for both sperm and egg cells?

sperm egg

A energy store presence of enzymes


B flagellum jelly coat
C jelly coat energy store
D presence of enzymes flagellum

13 Which stage of eutrophication must be reached to cause fish to die?

A increased growth of decomposers


B increased levels of nitrates
C increased growth of plants
D reduction of dissolved oxygen

14 Which process uses Rf values to identify the components of a mixture?

A chromatography
B crystallisation
C distillation
D filtration

15 In which experiment is a compound formed?

A B
copper oxide
filter paper + water

filter funnel
ink

C D

air aqueous
hydrogen sodium chloride
burning

heat

© UCLES 2020 0653/22/M/J/20

294/693
Combined By Nesrine
7
2023-2017

16 A magnesium ion, Mg2+, is formed from a magnesium atom, Mg.

Which row about the numbers of protons and neutrons in the magnesium ion and in the
magnesium atom is correct?

number of protons number of neutrons

A larger in Mg2+ than in Mg same in Mg and Mg2+


B same in Mg and Mg2+ same in Mg and Mg2+
C same in Mg and Mg2+ smaller in Mg2+ than in Mg
D smaller in Mg2+ than in Mg larger in Mg2+ than in Mg

17 Aluminium displaces copper from an aqueous solution of its ions.

xAl + yCu2+ → xAl 3+ + yCu

Which values of x and y balance the equation?

x y

A 1 2
B 2 1
C 2 3
D 3 2

© UCLES 2020 0653/22/M/J/20 [Turn over

295/693
Combined By Nesrine
8
2023-2017

18 Solid sodium carbonate is added to vinegar in a beaker and stirred.

sodium
carbonate

beaker vinegar

watch glass

water

The water in the watch glass freezes.

Which statement about the reaction explains why the water freezes?

A It is a redox reaction.
B It is an endothermic reaction.
C It is catalysed by sodium carbonate.
D It is thermal decomposition.

19 Which statements about redox reactions are correct?

1 The oxidising agent oxidises another substance.


2 The reducing agent is oxidised.
3 The substance that loses oxygen has been oxidised.

A 1 and 2 only B 1 and 3 only C 2 and 3 only D 1, 2 and 3

20 Copper(II) sulfate is prepared by reacting copper(II) oxide with dilute sulfuric acid.

CuO(s) + H2SO4(aq) → CuSO4(aq) + H2O(I)

Which statement is correct?

A Excess copper(II) oxide is used because it can be easily removed by filtration.

B Excess copper(II) oxide is used because it can be easily removed by reacting with more
sulfuric acid.
C Excess sulfuric acid is used because it can be easily removed by evaporation.

D Excess sulfuric acid is used because unreacted copper(II) oxide would contaminate the
product.

© UCLES 2020 0653/22/M/J/20

296/693
Combined By Nesrine
9
2023-2017

21 Solution X is mixed with nitric acid and aqueous barium nitrate.

A white precipitate is formed.

Which ion is present in solution X?


A carbonate
B chloride
C nitrate
D sulfate

22 Which row shows the relationship between the group number of an element, its number of outer
shell electrons and its metallic / non-metallic character?

number of outer metallic / non-metallic


group number
shell electrons character

A I 1 non-metal
B III 5 metal
C V 5 non-metal
D VII 1 metal

23 Astatine is an element found at the bottom of Group VII in the Periodic Table.

Which row shows the properties of astatine?

melting point reacts with


/ °C iodide ions

A –7 no
B 302 no
C –7 yes
D 302 yes

© UCLES 2020 0653/22/M/J/20 [Turn over

297/693
Combined By Nesrine
10
2023-2017

24 Atoms and ions of metals P, Q and R take part in two reactions.

The equations for these reactions are shown.

P + 2Q+ → P2+ + 2Q

R + P2+ → R2+ + P

Which statements are correct?

1 P is more reactive than R.


2 P is less reactive than Q.
3 R is more reactive than Q.
4 R loses electrons most readily.

A 1 and 2 B 1 and 3 C 2 and 4 D 3 and 4

25 Copper can be made from copper oxide by reacting it with carbon at a high temperature.

Why is carbon used?

A It does not react with copper.


B It is a conductor of electricity.
C It is a high melting point solid.
D It is more reactive than copper.

26 Which row about a gas in clean air is correct?

name of gas percentage of gas

A nitrogen 20
B nitrogen 50
C oxygen 20
D oxygen 50

27 In which list are all of the hydrocarbons in the same homologous series?

A CH4, C2H6, C3H8, C5H10


B C2H4, C2H6, C3H6, C4H8
C C2H4, C3H6, C4H8, C5H10
D C2H6, C3H6, C4H8, C5H10

© UCLES 2020 0653/22/M/J/20

298/693
Combined By Nesrine
11
2023-2017

28 Which distance–time graph represents a body that is moving with changing speed?

A B

distance distance

0 0
0 time 0 time

C D

distance distance

0 0
0 time 0 time

29 A car travels at various speeds during a short journey.

The table shows the distances travelled and the times taken during each of four stages
P, Q, R and S.

stage P Q R S

distance travelled / km 1.8 3.6 2.7 2.7


time taken / minutes 2.0 2.0 4.0 3.0

During which two stages is the car travelling at the same average speed?

A P and Q B P and S C Q and R D R and S

© UCLES 2020 0653/22/M/J/20 [Turn over

299/693
Combined By Nesrine
12
2023-2017

30 A spring obeys Hooke’s law. A load of 10 N hangs from the spring and causes the spring to
extend by 12 mm.

Two springs, identical to the first one, are now joined as shown. A load of 5.0 N is hung from the
springs.

springs

5.0 N

What is the total extension of the combination of the two springs?

A 3.0 mm B 6.0 mm C 12 mm D 24 mm

31 A force F acts on a body of mass m for a time t. In this time, the speed of the body increases from
speed u to speed v and the body travels a distance d.

Which expression gives the work done by the force on the body?

A F×d B F×t C F×u D F×v

32 Four objects P, Q, R and S are moving.

The table shows the mass and speed of each object.

speed
mass / kg
m/ s

P 1.0 4.0
Q 2.0 1.0
R 1.0 2.0
S 4.0 1.0

Which two objects have equal kinetic energy?

A P and R B P and S C Q and R D R and S

© UCLES 2020 0653/22/M/J/20

300/693
Combined By Nesrine
13
2023-2017

33 Which action increases the rate of evaporation of a liquid?

A cooling the liquid


B covering the liquid
C increasing the surface area of the liquid
D reducing any draught over the liquid

34 Conduction of heat in metals involves the movement of atoms and electrons.

Which row describes the movement of the atoms and the electrons?

atoms electrons

A move freely move freely


B move freely vibrate about fixed positions
C vibrate about fixed positions move freely
D vibrate about fixed positions vibrate about fixed positions

35 Four students suggest values for the speed of electromagnetic waves in a vacuum.

The students use two different units.

Which value is correct?

A 300 m / s
B 300 km / s

C 3.0 × 105 m / s

D 3.0 × 105 km / s

36 The sound from a drum is loud and has a low pitch.

Which row describes the amplitude and the frequency of the sound wave?

amplitude frequency

A large high
B large low
C small high
D small low

© UCLES 2020 0653/22/M/J/20 [Turn over

301/693
Combined By Nesrine
14
2023-2017

37 What is the unit of charge?

A ampere
B coulomb
C ohm
D volt

38 An electric circuit contains two resistors connected to a cell.

One resistor is labelled R. The switch is open.

The switch is now closed.

What happens to the potential difference (p.d.) across resistor R and what happens to the current
in resistor R?

A The p.d. decreases and the current increases.


B The p.d. decreases and the current remains the same.
C The p.d. remains the same and the current increases.
D The p.d. remains the same and the current remains the same.

39 A 12 V power supply is connected to a 6.0 Ω resistor. This causes a current in the resistor.

How much thermal energy is produced in the resistor in 5.0 minutes?

A 120 J B 600 J C 7200 J D 21 600 J

© UCLES 2020 0653/22/M/J/20

302/693
Combined By Nesrine
15
2023-2017

40 The charger for a laptop computer is connected by a cable to the mains supply through a plug.

The plug contains a 13 A fuse. The cable is designed to carry a current of 2 A.

A fault develops and the current in the cable increases to 5 A.

laptop
plug computer

connecting cable

What is a possible danger caused by this larger current?

A A large amount of electrical energy is wasted.


B Somebody receives an electric shock.
C The fuse blows and starts a fire.
D The cable overheats and starts a fire.

Permission to reproduce items where third-party owned material protected by copyright is included has been sought and cleared where possible. Every
reasonable effort has been made by the publisher (UCLES) to trace copyright holders, but if any items requiring clearance have unwittingly been included, the
publisher will be pleased to make amends at the earliest possible opportunity.

To avoid the issue of disclosure of answer-related information to candidates, all copyright acknowledgements are reproduced online in the Cambridge
Assessment International Education Copyright Acknowledgements Booklet. This is produced for each series of examinations and is freely available to download
at www.cambridgeinternational.org after the live examination series.

Cambridge Assessment International Education is part of the Cambridge Assessment Group. Cambridge Assessment is the brand name of the University of
Cambridge Local Examinations Syndicate (UCLES), which itself is a department of the University of Cambridge.

© UCLES 2020 0653/22/M/J/20

303/693
Combined By Nesrine
2023-2017

Cambridge IGCSE™

COMBINED SCIENCE 0653/21


Paper 2 Multiple Choice (Extended) October/November 2020
45 minutes

You must answer on the multiple choice answer sheet.


*6376377871*

You will need: Multiple choice answer sheet


Soft clean eraser
Soft pencil (type B or HB is recommended)

INSTRUCTIONS
 There are forty questions on this paper. Answer all questions.
 For each question there are four possible answers A, B, C and D. Choose the one you consider correct
and record your choice in soft pencil on the multiple choice answer sheet.
 Follow the instructions on the multiple choice answer sheet.
 Write in soft pencil.
 Write your name, centre number and candidate number on the multiple choice answer sheet in the
spaces provided unless this has been done for you.
 Do not use correction fluid.
 Do not write on any bar codes.
 You may use a calculator.

INFORMATION
 The total mark for this paper is 40.
 Each correct answer will score one mark. A mark will not be deducted for a wrong answer.
 Any rough working should be done on this question paper.
 The Periodic Table is printed in the question paper.

This document has 16 pages. Blank pages are indicated.

IB20 11_0653_21/3RP
© UCLES 2020 [Turn over

304/693
Combined By Nesrine
2
2023-2017

1 The diagram shows a cell.

What is the function of X?

A contains the genetic information


B controls substances entering and leaving the cell
C maintains the shape of the cell
D photosynthesis

2 What is the function of ciliated cells in the bronchi?

A absorption of oxygen
B movement of mucus
C production of mucus
D transport of oxygen

3 The diagram shows how the activity of an enzyme changes with temperature.

enzyme activity
/ arbitrary units

X temperature / °C

This enzyme works in the human body.

What is the most likely value of temperature X?

A 10 C B 40 C C 70 C D 100 C

© UCLES 2020 0653/21/O/N/20

305/693
Combined By Nesrine
3
2023-2017

4 What is necessary for photosynthesis?

1 carbon dioxide
2 chlorophyll
3 glucose
4 light
5 oxygen
6 water

A 1, 2, 4 and 6
B 1, 3, 4 and 6
C 2, 3, 4 and 5
D 3, 4, 5 and 6

5 Deficiencies in vitamin D and in iron can cause diseases.

Which statement is correct?

A Vitamin D deficiency can cause anaemia.


B Vitamin D deficiency can cause rickets.
C Iron deficiency can cause rickets.
D Iron deficiency can cause scurvy.

6 Which enzymes are secreted from the pancreas?

1 amylase
2 lipase
3 protease

A 1 and 2 only B 1 and 3 only C 2 and 3 only D 1, 2 and 3

© UCLES 2020 0653/21/O/N/20 [Turn over

306/693
Combined By Nesrine
4
2023-2017

7 The graph shows the uptake of water by root hair cells over many hours during a day.

water uptake
by root hair cells

0
0 time

What could have caused the change in the rate of uptake at T?

A decrease in temperature
B decrease in humidity
C increase in light intensity
D increase in temperature

8 How does mucus benefit the gas exchange system?

A It absorbs carbon monoxide before it reaches the alveoli.


B It prevents friction between the air and the trachea.
C It removes the nicotine in cigarette smoke.
D It traps pathogens.

9 Which statement about adrenaline is correct?

A It is produced by a gland.
B It is transported in the red blood cells.
C It only has one target organ.
D It reduces the size of the pupils.

© UCLES 2020 0653/21/O/N/20

307/693
Combined By Nesrine
5
2023-2017

10 Which row shows the correct descriptions for the anther and stigma of a wind-pollinated flower?

anther stigma stigma


position position description

A inside flower inside flower smooth


B exposed exposed feathery
C exposed inside flower smooth
D inside flower exposed feathery

11 Which row describes asexual reproduction?

number of a zygote is offspring identical


parents produced to the parent

A 1 no yes
B 1 yes no
C 2 no yes
D 2 yes no

© UCLES 2020 0653/21/O/N/20 [Turn over

308/693
Combined By Nesrine
6
2023-2017

12 The diagram shows a placenta and umbilical cord.

X key
umbilical
artery X movement
of oxygen
from fetus
to fetus X
mother’s
umbilical blood
vein X

Which row is correct?

high oxygen low oxygen


name of
concentration concentration
process X
present present

A umbilical artery umbilical vein diffusion


B umbilical artery umbilical vein osmosis
C umbilical vein umbilical artery diffusion
D umbilical vein umbilical artery osmosis

13 Eutrophication results in the death of aquatic organisms.

What is a stage in this process?

A reduced aerobic respiration by decomposers


B reduced decomposition after death of producers
C reduced growth of producers
D reduced levels of dissolved oxygen

14 Which term describes ammonia, NH3?

A element
B ion
C atom
D molecule

© UCLES 2020 0653/21/O/N/20

309/693
Combined By Nesrine
7
2023-2017

15 Two different dyes are analysed using chromatography.

Each dye produces only one coloured spot on the chromatogram.

The Rf values of the coloured spots are shown.

coloured
Rf value
spot

red 0.2
blue 0.4

The two different dyes are then mixed together to make a purple dye.

What is observed on the chromatogram of the purple dye?

A one spot with Rf value 0.3


B one spot with Rf value 0.6
C two spots with Rf values 0.2 and 0.4
D three spots with Rf values 0.2, 0.3 and 0.4

16 Which statement describes a mixture?

A It contains molecules made from the same type of atom.


B It contains only one type of atom.
C It contains two different types of atom joined by chemical bonds.
D It contains two different types of atom that can be separated by physical processes.

17 Aqueous lead(II) nitrate, Pb(NO3)2, reacts with potassium iodide to make a precipitate of
lead(II) iodide.

What is the ionic equation for this reaction?

A Pb+ + I–  PbI

B Pb2+ + 2I–  PbI2

C Pb(NO3)2 + I–  PbI + 2NO3–

D Pb2+ + 2NO3– + 2I–  PbI2 + 2NO3–

18 Which statement about the electrolysis of a molten metal halide is correct?

A Cations move to the anode.


B Electrons flow through the electrolyte.
C Ions gain protons at the cathode.
D Ions lose electrons at the anode.

© UCLES 2020 0653/21/O/N/20 [Turn over

310/693
Combined By Nesrine
8
2023-2017

19 The energy level diagram for an endothermic reaction is shown.

energy

progress of reaction

Which statement about this reaction is correct?

A The activation energy is the energy required to break bonds.


B The energy required to break bonds is less than the energy released on making new bonds.
C The activation energy is less than the energy change for the reaction.
D The final products have less energy than the reactants.

20 Iron can be obtained from iron(III) oxide by heating with aluminium powder.

The equation is shown.

2Al + Fe2O3  Al 2O3 + 2Fe

What is the oxidising agent?

A Al B Fe2O3 C Al 2O3 D Fe

21 Which substances react with dilute sulfuric acid to make copper sulfate?

1 copper
2 copper carbonate
3 copper hydroxide
4 copper nitrate

A 1 and 2 B 1 and 4 C 2 and 3 D 3 and 4

© UCLES 2020 0653/21/O/N/20

311/693
Combined By Nesrine
9
2023-2017

22 Acid X reacts with metal Y.

A colourless gas is given off and a pale green solution is produced.

Two tests are carried out on the solution.

test reagent(s) added result

1 aqueous silver nitrate and nitric acid white precipitate


2 aqueous sodium hydroxide green precipitate

What are acid X and metal Y?

acid metal

A hydrochloric iron
B hydrochloric zinc
C sulfuric iron
D sulfuric zinc

23 Rubidium and sodium are elements in Group I of the Periodic Table.

The atomic number of sodium is 11, and the atomic number of rubidium is 37.

Rubidium has a ……1…… melting point and a ……2…… density than sodium. The reactivity of
rubidium is ……3…… than the reactivity of sodium.

Which row completes gaps 1, 2 and 3?

1 2 3

A higher lower lower


B lower lower higher
C lower higher higher
D higher higher lower

24 Ammonia, NH3, can be made by combining the gases nitrogen, N2, and hydrogen, H2.

This reaction is slow.

When element Y is added, the rate of reaction increases.

What is Y?

A Al B Fe C Rb D I2

© UCLES 2020 0653/21/O/N/20 [Turn over

312/693
Combined By Nesrine
10
2023-2017

25 Which method is used to extract copper from copper(II) oxide?

A dissolving copper(II) oxide in hydrochloric acid and then filtering

B dissolving copper(II) oxide in water and then filtering

C heating the copper(II) oxide

D heating the copper(II) oxide mixed with carbon

26 Which statement describes a hydrocarbon?

A a compound that burns to form carbon dioxide and hydrogen


B a compound that contains carbon and hydrogen only
C a compound that only contains ionic bonds
D a compound that reacts easily with metals

27 What can be produced when naphtha is cracked?

A alkanes, alkenes and hydrogen


B alkanes and alkenes only
C alkanes and hydrogen only
D alkenes only

28 What does the area under a speed–time graph represent?

A acceleration
B average speed
C distance travelled
D maximum speed

29 A satellite of mass 20 kg is in orbit around the Earth.

At the height of the satellite’s orbit, the gravitational field strength is one quarter of its strength on
the surface of the Earth.

The gravitational field strength on the surface of the Earth is 10 N / kg.

What is the weight of the satellite as it orbits the Earth?

A 0N B 20 N C 50 N D 200 N

© UCLES 2020 0653/21/O/N/20

313/693
Combined By Nesrine
11
2023-2017

30 A raindrop falls vertically at a constant speed.

What is the resultant force on the raindrop as it falls?

A It is equal to the air pressure on the drop.


B It is equal to the air resistance on the drop.
C It is equal to the weight of the drop.
D It is zero.

31 An apple falls to the ground.

Which form of energy decreases as the apple falls?

A chemical potential
B gravitational potential
C kinetic
D sound

32 A builder drops a brick from a height of 15 m above the ground.

The gravitational field strength g is 10 N / kg.

What is the speed of the brick as it hits the ground?

A 12 m / s B 17 m / s C 150 m / s D 300 m / s

33 The molecules in a substance vibrate about fixed positions.

The substance is now cooled.

Which row gives the state of the substance and the effect of cooling on the distance between its
molecules?

state of effect on distance


substance between molecules

A solid decreases
B solid increases
C liquid decreases
D liquid increases

© UCLES 2020 0653/21/O/N/20 [Turn over

314/693
Combined By Nesrine
12
2023-2017

34 In which states of matter can convection occur?

in a solid in a liquid in a gas

A no no yes
B no yes yes
C yes no no
D yes yes no

35 The diagram shows a section of a rope.

Four wave crests pass a point on the rope every second.

Each wave crest travels 80 cm in one second.

wave crest
20 cm
direction
5.0 cm of wave

What is the speed of the wave?

A 4.0 cm / s B 5.0 cm / s C 20 cm / s D 80 cm / s

36 A converging lens is used as a magnifying glass.

Where is the image formed and what is the nature of the image?

position of image nature

A on the opposite side of the lens to the object real


B on the opposite side of the lens to the object virtual
C on the same side of the lens as the object real
D on the same side of the lens as the object virtual

© UCLES 2020 0653/21/O/N/20

315/693
Combined By Nesrine
13
2023-2017

37 The diagram represents a wave in air. Molecules are closer together in region P than they are in
region Q.

region P region Q

What are the names of regions P and Q, and which type of wave is represented?

region P region Q type of wave

A compression rarefaction longitudinal


B compression rarefaction transverse
C rarefaction compression longitudinal
D rarefaction compression transverse

38 A power supply causes a current in a circuit.

The electromotive force (e.m.f.) of the power supply and the resistance of the circuit are both
changed.

Which pair of changes must result in a smaller current in the circuit?

e.m.f. resistance

A decreased decreased
B decreased increased
C increased decreased
D increased increased

39 There is a current of 0.25 A in a wire.

How long does it take for 120 C of charge to pass a point in the wire?

A 0.50 minutes
B 8.0 minutes
C 30 minutes
D 480 minutes

© UCLES 2020 0653/21/O/N/20 [Turn over

316/693
Combined By Nesrine
14
2023-2017

40 The diagram shows an electric circuit.

8.0 V

2.0 A

4.0 Ω

The battery of electromotive force (e.m.f.) 8.0 V produces a current of 2.0 A in a 4.0  resistor.

How much power is delivered to the resistor?

A 0.25 W B 4.0 W C 16 W D 64 W

© UCLES 2020 0653/21/O/N/20

317/693
Combined By Nesrine
2023-2017

Cambridge IGCSE™

COMBINED SCIENCE 0653/22


Paper 2 Multiple Choice (Extended) October/November 2020
45 minutes

You must answer on the multiple choice answer sheet.


*8188494618*

You will need: Multiple choice answer sheet


Soft clean eraser
Soft pencil (type B or HB is recommended)

INSTRUCTIONS
 There are forty questions on this paper. Answer all questions.
 For each question there are four possible answers A, B, C and D. Choose the one you consider correct
and record your choice in soft pencil on the multiple choice answer sheet.
 Follow the instructions on the multiple choice answer sheet.
 Write in soft pencil.
 Write your name, centre number and candidate number on the multiple choice answer sheet in the
spaces provided unless this has been done for you.
 Do not use correction fluid.
 Do not write on any bar codes.
 You may use a calculator.

INFORMATION
 The total mark for this paper is 40.
 Each correct answer will score one mark. A mark will not be deducted for a wrong answer.
 Any rough working should be done on this question paper.
 The Periodic Table is printed in the question paper.

This document has 16 pages. Blank pages are indicated.

IB20 11_0653_22/3RP
© UCLES 2020 [Turn over

318/693
Combined By Nesrine
2
2023-2017

1 The diagram shows a cell from an animal’s liver.

In what way does this cell differ from a typical animal cell?

A It contains a central vacuole.


B It contains cytoplasm.
C It contains two nuclei.
D It has a cell wall.

2 The diagram shows how the activity of an enzyme changes with temperature.

enzyme activity
/ arbitrary units

X temperature / °C

This enzyme works in the human body.

What is the most likely value of temperature X?

A 10 C B 40 C C 70 C D 100 C

3 Some undigested food passes out of the digestive system as faeces.

What is this process?

A absorption
B digestion
C egestion
D ingestion

© UCLES 2020 0653/22/O/N/20

319/693
Combined By Nesrine
3
2023-2017

4 What is a function of the hydrochloric acid produced in the stomach?

A to help absorption of all food in the stomach


B to kill bacteria in the ingested food
C to prevent chemical digestion
D to prevent the stomach contents being too acidic

5 The graph shows the uptake of water by root hair cells over many hours during a day.

water uptake
by root hair cells

0
0 time

What could have caused the change in the rate of uptake at T?

A decrease in temperature
B decrease in humidity
C increase in light intensity
D increase in temperature

6 How does mucus benefit the gas exchange system?

A It absorbs carbon monoxide before it reaches the alveoli.


B It prevents friction between the air and the trachea.
C It removes the nicotine in cigarette smoke.
D It traps pathogens.

© UCLES 2020 0653/22/O/N/20 [Turn over

320/693
Combined By Nesrine
4
2023-2017

7 Which row about the effects of adrenaline in humans is correct?

blood glucose
pulse rate
concentration

A increases decreases
B increases increases
C stays the same decreases
D stays the same increases

8 The following are features of human gametes.

1 have a jelly coat


2 have energy stores
3 have flagella
4 motile
5 produced in large numbers

What are features of human male gametes?

A 1 and 2 only B 1, 2 and 4 C 2, 3 and 5 D 3, 4 and 5

9 Which row describes asexual reproduction?

number of a zygote is offspring identical


parents produced to the parent

A 1 no yes
B 1 yes no
C 2 no yes
D 2 yes no

10 On which part of a flower is pollen deposited during pollination?

A ovary
B stamen
C stigma
D style

© UCLES 2020 0653/22/O/N/20

321/693
Combined By Nesrine
5
2023-2017

11 The diagram shows a fetus in the uterus.

Which letter identifies the umbilical cord?

D
A

C B

12 Which is an effect of the process of eutrophication of water?

A increased fish population


B increased growth of algae
C decreased availability of nitrates and other ions in the water
D increased levels of dissolved carbon dioxide in the water

13 The diagram shows the trophic levels of a food chain.

producer herbivore carnivore carnivore


  
1st trophic 2nd trophic 3rd trophic 4th trophic
level level level level

Why do most food chains not have more than four trophic levels?

A There are too many carnivores in the 3rd trophic level.


B There are too many herbivores in the 2nd trophic level.
C There is no energy transferred from the 2nd trophic level to the 3rd trophic level.
D There is not enough energy available to be transferred to a 5th trophic level.

© UCLES 2020 0653/22/O/N/20 [Turn over

322/693
Combined By Nesrine
6
2023-2017

14 Which diagram represents particles in a gaseous element?

A B C D

15 Which statement describes a mixture?

A It contains molecules made from the same type of atom.


B It contains only one type of atom.
C It contains two different types of atom joined by chemical bonds.
D It contains two different types of atom that can be separated by physical processes.

16 Which statement about the formation of ions is correct?

A Metal atoms gain electrons to form cations and non-metal atoms lose electrons to form
anions.
B Metal atoms gain electrons to form anions and non-metal atoms lose electrons to form
cations.
C Metal atoms lose electrons to form cations and non-metal atoms gain electrons to form
anions.
D Metal atoms lose electrons to form anions and non-metal atoms gain electrons to form
cations.

17 The formula of a magnesium ion is Mg2+.

The formula of a nitride ion is N3–.

What is the formula of magnesium nitride?

A MgN B Mg2N2 C Mg2N3 D Mg3N2

18 Which statement describes the movement of electrons during electrolysis?

A They move from the anode to the cathode through the external circuit.
B They move from the anode to the cathode through the electrolyte.
C They move from the cathode to the anode through the external circuit.
D They move from the cathode to the anode through the electrolyte.

© UCLES 2020 0653/22/O/N/20

323/693
Combined By Nesrine
7
2023-2017

19 The energy level diagram for a reaction is shown.

products
Y
energy
reactants
X

progress of reaction

Which statement about the reaction is correct?

A The activation energy for this reaction is equal to the value of (Z – Y).
B The energy released by this reaction is equal to the value of (Y – X).
C The energy used to break bonds is more than the energy released in forming bonds.
D The overall energy change for this reaction is equal to the value of (Z – X).

20 What are the effects of increasing the temperature of a reaction?

frequency of number of particles


particle collisions having activation energy

A less more
B less same
C more more
D more same

21 Dilute hydrochloric acid is tested with universal indicator and with calcium carbonate.

Which row shows the results?

pH reaction with calcium carbonate

A 2 a colourless gas is given off


B 2 no reaction
C 10 a colourless gas is given off
D 10 no reaction

© UCLES 2020 0653/22/O/N/20 [Turn over

324/693
Combined By Nesrine
8
2023-2017

22 Acid X reacts with metal Y.

A colourless gas is given off and a pale green solution is produced.

Two tests are carried out on the solution.

test reagent(s) added result

1 aqueous silver nitrate and nitric acid white precipitate


2 aqueous sodium hydroxide green precipitate

What are acid X and metal Y?

acid metal

A hydrochloric iron
B hydrochloric zinc
C sulfuric iron
D sulfuric zinc

23 Substance X is a coloured solid.

Substance X acts as a catalyst for the reaction between zinc and dilute sulfuric acid.

Molten X can be electrolysed.

What is X?

A a Group I compound
B a Group I metal
C a transition metal compound
D a transition metal

24 The elements in Group II of the Periodic Table show a similar trend in reactivity to the elements in
Group I.

Which statement about Group II elements is correct?

A Barium atoms lose electrons more readily than magnesium atoms.


B Calcium reacts with water more rapidly than strontium reacts with water.
C Magnesium displaces strontium ions from aqueous solution.
D Strontium oxide is reduced by heating with carbon.

© UCLES 2020 0653/22/O/N/20

325/693
Combined By Nesrine
9
2023-2017

25 Which method is used to extract copper from copper(II) oxide?

A dissolving copper(II) oxide in hydrochloric acid and then filtering

B dissolving copper(II) oxide in water and then filtering

C heating the copper(II) oxide

D heating the copper(II) oxide mixed with carbon

26 Which process does not produce carbon dioxide?

A complete combustion of methane


B cracking of large alkane molecules
C reaction between an acid and magnesium carbonate
D thermal decomposition of calcium carbonate

27 Which statement describes a hydrocarbon?

A a compound that burns to form carbon dioxide and hydrogen


B a compound that contains carbon and hydrogen only
C a compound that only contains ionic bonds
D a compound that reacts easily with metals

28 The diagram shows the speed–time graph for an object.

speed 20
m/s

10

0
0 30
time / s

What is the distance travelled by the object in 30 s?

A 150 m B 300 m C 450 m D 600 m

© UCLES 2020 0653/22/O/N/20 [Turn over

326/693
Combined By Nesrine
10
2023-2017

29 A measuring cylinder contains water. Five identical metal screws are added to the water as
shown.

cm3 cm3
10 10
9 9
8 8
7 7
6 6
5 5
4 4
3 3
2 2
1 1

before after
screws added screws added

The mass of each screw is 3.8 g.

What is the density of the metal of the screws?

A 1.6 g / cm3 B 2.2 g / cm3 C 7.9 g / cm3 D 8.6 g / cm3

© UCLES 2020 0653/22/O/N/20

327/693
Combined By Nesrine
11
2023-2017

30 A spring is stretched by a load that is gradually increased until the spring extends beyond its limit
of proportionality.

Which graph shows the relationship between the load and the extension produced?

A B

extension extension

0 0
0 load 0 load

C D

extension extension

0 0
0 load 0 load

31 A device uses 0.50 kJ of energy in 25 minutes.

What is the power of the device?

A 0.33 W B 12.5 W C 20 W D 750 W

32 The molecules in a substance vibrate about fixed positions.

The substance is now cooled.

Which row gives the state of the substance and the effect of cooling on the distance between its
molecules?

state of effect on distance


substance between molecules

A solid decreases
B solid increases
C liquid decreases
D liquid increases

© UCLES 2020 0653/22/O/N/20 [Turn over

328/693
Combined By Nesrine
12
2023-2017

33 In which states of matter can convection occur?

in a solid in a liquid in a gas

A no no yes
B no yes yes
C yes no no
D yes yes no

34 The diagram shows a section of a rope.

Four wave crests pass a point on the rope every second.

Each wave crest travels 80 cm in one second.

wave crest
20 cm
direction
5.0 cm of wave

What is the speed of the wave?

A 4.0 cm / s B 5.0 cm / s C 20 cm / s D 80 cm / s

35 A source of light is placed in front of a plane mirror.

Which labelled point shows the position of the image of the source?

A
B

C
D
source
of light

© UCLES 2020 0653/22/O/N/20

329/693
Combined By Nesrine
13
2023-2017

36 Radio waves, visible light and X-rays all travel in a vacuum.

Which wave travels at the greatest speed?

A radio waves
B visible light
C X-rays
D they all travel at the same speed

37 The diagram represents a wave in air. Molecules are closer together in region P than they are in
region Q.

region P region Q

What are the names of regions P and Q, and which type of wave is represented?

region P region Q type of wave

A compression rarefaction longitudinal


B compression rarefaction transverse
C rarefaction compression longitudinal
D rarefaction compression transverse

38 A power supply causes a current in a circuit.

The electromotive force (e.m.f.) of the power supply and the resistance of the circuit are both
changed.

Which pair of changes must result in a smaller current in the circuit?

e.m.f. resistance

A decreased decreased
B decreased increased
C increased decreased
D increased increased

© UCLES 2020 0653/22/O/N/20 [Turn over

330/693
Combined By Nesrine
14
2023-2017

39 There is a current of 2.0 A in a 4.0  resistor for 20 s.

What is the charge that flows through the resistor in this time and what is the p.d. across it?

charge / C p.d. / V

A 10 2.0
B 10 8.0
C 40 2.0
D 40 8.0

40 The current in the starter motor of a car is 400 A when it is connected to a 12 V battery.

How much energy is delivered to the starter motor in 2.0 seconds?

A 0.060 J B 67 J C 2400 J D 9600 J

© UCLES 2020 0653/22/O/N/20

331/693
Combined By Nesrine
2023-2017

Cambridge IGCSE™

COMBINED SCIENCE 0653/23


Paper 2 Multiple Choice (Extended) October/November 2020
45 minutes

You must answer on the multiple choice answer sheet.


*9781381446*

You will need: Multiple choice answer sheet


Soft clean eraser
Soft pencil (type B or HB is recommended)

INSTRUCTIONS
 There are forty questions on this paper. Answer all questions.
 For each question there are four possible answers A, B, C and D. Choose the one you consider correct
and record your choice in soft pencil on the multiple choice answer sheet.
 Follow the instructions on the multiple choice answer sheet.
 Write in soft pencil.
 Write your name, centre number and candidate number on the multiple choice answer sheet in the
spaces provided unless this has been done for you.
 Do not use correction fluid.
 Do not write on any bar codes.
 You may use a calculator.

INFORMATION
 The total mark for this paper is 40.
 Each correct answer will score one mark. A mark will not be deducted for a wrong answer.
 Any rough working should be done on this question paper.
 The Periodic Table is printed in the question paper.

This document has 16 pages. Blank pages are indicated.

IB20 11_0653_23/4RP
© UCLES 2020 [Turn over

332/693
Combined By Nesrine
2
2023-2017

1 The diagram shows two red blood cells inside a capillary and two tissue cells near this capillary.

red blood cell

tissue cell

How does the oxygen in the red blood cells reach the tissue cells?

A by absorption
B by diffusion
C by respiration
D by transpiration

2 The diagram shows how the activity of an enzyme changes with temperature.

enzyme activity
/ arbitrary units

X temperature / °C

This enzyme works in the human body.

What is the most likely value of temperature X?

A 10 C B 40 C C 70 C D 100 C

3 Which row shows the results of mechanical digestion?

food is broken down into


smaller soluble
smaller pieces
molecules molecules

A   
B   
C   
D   

© UCLES 2020 0653/23/O/N/20

333/693
Combined By Nesrine
3
2023-2017

4 Protease breaks down protein.

What is the protein broken down into?

A amino acids
B fatty acids
C glycerol
D starch

5 The graph shows the uptake of water by root hair cells over many hours during a day.

water uptake
by root hair cells

0
0 time

What could have caused the change in the rate of uptake at T?

A decrease in temperature
B decrease in humidity
C increase in light intensity
D increase in temperature

6 How does mucus benefit the gas exchange system?

A It absorbs carbon monoxide before it reaches the alveoli.


B It prevents friction between the air and the trachea.
C It removes the nicotine in cigarette smoke.
D It traps pathogens.

7 Which shows the balanced chemical equation for aerobic respiration?

A 6CO2 + C6H12O6  6O2 + 6H2O

B 6CO2 + 6H2O  C6H12O6 + 6O2

C C6H12O6 + 6O2  6CO2 + 6H2O

D 6O2 + 6H2O  6CO2 + C6H12O6

© UCLES 2020 0653/23/O/N/20 [Turn over

334/693
Combined By Nesrine
4
2023-2017

8 A person's body secretes adrenaline in response to a frightening experience.

Which statement is correct?

A The person's blood glucose concentration decreases.


B The person's breathing rate does not change.
C The person's pulse rate increases.
D The person's pupils become narrower.

© UCLES 2020 0653/23/O/N/20

335/693
Combined By Nesrine
5
2023-2017

9 The diagram shows a germinating seed.

light

soil

What does the germinating seed show?

shoot root

A negative phototropism negative gravitropism


B negative phototropism positive gravitropism
C positive phototropism negative gravitropism
D positive phototropism positive gravitropism

10 Which row describes asexual reproduction?

number of a zygote is offspring identical


parents produced to the parent

A 1 no yes
B 1 yes no
C 2 no yes
D 2 yes no

© UCLES 2020 0653/23/O/N/20 [Turn over

336/693
Combined By Nesrine
6
2023-2017

11 The diagram shows a section through a buttercup flower.

Which structure produces pollen grains?

D
B

12 The diagram shows a male gamete and a female gamete.

Which label is correct?

A B D
central vacuole jelly coat nucleus containing
chromosomes
C
from mother
food store

13 Some stages in the process of eutrophication are listed.

1 reduction in dissolved oxygen


2 increased aerobic respiration by decomposers
3 increased availability of nitrates
4 death of organisms requiring dissolved oxygen
5 increased growth of producers and increased decomposition after death of
producers

In which sequence do these stages take place?

A 14352

B 31524

C 35214

D 43125

© UCLES 2020 0653/23/O/N/20

337/693
Combined By Nesrine
7
2023-2017

14 The temperature and pressure of oxygen in two different containers are shown.

temperature pressure
/ C kN / m2

container 1 20 200
container 2 50 150

Which statement about the oxygen molecules in container 1 compared to container 2 is correct?

A In container 1 they are closer together and moving faster.


B In container 1 they are closer together and moving slower.
C In container 1 they are further apart and moving faster.
D In container 1 they are further apart and moving slower.

15 A pure sample of a coloured dye is tested using chromatography.

The chromatogram obtained is shown.

solvent front

P
Q
R

baseline

How is the Rf value of the dye calculated?

A R B R C RS D RS
P Q QS PS

16 Which statement describes a mixture?

A It contains molecules made from the same type of atom.


B It contains only one type of atom.
C It contains two different types of atom joined by chemical bonds.
D It contains two different types of atom that can be separated by physical processes.

© UCLES 2020 0653/23/O/N/20 [Turn over

338/693
Combined By Nesrine
8
2023-2017

17 Which equation is not correctly balanced?

A Ca + 2H2O  Ca(OH)2 + H2

B CaCO3 + 2HCl  CaCl 2 + H2O + CO2

C CaO + 2HCl  CaCl 2 + H2O

D Ca(OH)2 + 2HCl  CaCl 2 + H2O

18 What happens to cations during electrolysis?

A They gain electrons.


B They gain oxygen.
C They lose electrons.
D They lose oxygen.

19 The equation and the energy level diagram for the reaction between hydrogen and chlorine are
shown.

H2(g) + Cl 2(g)  2HCl (g)

H2(g) + Cl 2(g)
energy X
2HCl (g)

progress of reaction

Which statement about this reaction is correct?

A The reaction is endothermic.


B The products have less energy than the reactants.
C X is the activation energy.
D More bonds are being broken than are being formed.

© UCLES 2020 0653/23/O/N/20

339/693
Combined By Nesrine
9
2023-2017

20 Dilute hydrochloric acid reacts with solid calcium carbonate.

Decreasing the temperature and diluting the acid both decrease the rate of reaction.

Which statement explains why these changes cause the rate of reaction to decrease?

A Both result in the acid particles having less energy.


B Both result in a lower proportion of collisions between reacting particles being successful.
C Both result in fewer acid particles per cm3 of solution.
D Both result in a lower frequency of collisions between reacting particles.

21 The equation for the reaction between zinc oxide and copper is shown.

ZnO + Cu  Zn + CuO

Which statement about this reaction is correct?

A Copper is the oxidising agent.


B Copper oxide is being oxidised.
C Zinc is the reducing agent.
D Zinc oxide is being reduced.

22 Which two substances both react with dilute sulfuric acid to make the salt magnesium sulfate?

A magnesium carbonate and magnesium chloride


B magnesium chloride and magnesium nitrate
C magnesium oxide and magnesium carbonate
D magnesium oxide and magnesium nitrate

© UCLES 2020 0653/23/O/N/20 [Turn over

340/693
Combined By Nesrine
10
2023-2017

23 Acid X reacts with metal Y.

A colourless gas is given off and a pale green solution is produced.

Two tests are carried out on the solution.

test reagent(s) added result

1 aqueous silver nitrate and nitric acid white precipitate


2 aqueous sodium hydroxide green precipitate

What are acid X and metal Y?

acid metal

A hydrochloric iron
B hydrochloric zinc
C sulfuric iron
D sulfuric zinc

24 X, Y and Z are elements in Group VII.

X reacts with potassium iodide but not with potassium bromide.

Y reacts with potassium bromide but not with sodium chloride.

Z does not react with potassium bromide or with potassium iodide.

What are X, Y and Z?

X Y Z

A bromine chlorine iodine


B bromine iodine chlorine
C chlorine bromine iodine
D iodine chlorine bromine

© UCLES 2020 0653/23/O/N/20

341/693
Combined By Nesrine
11
2023-2017

25 Some physical properties of four elements are shown.

Which element can act as a catalyst?

melting point conductivity density


/ C as a solid g / cm3

A 98 good 0.97
B 113 poor 2.07
C 1455 good 8.9
D 1683 poor 2.32

26 Which method is used to extract copper from copper(II) oxide?

A dissolving copper(II) oxide in hydrochloric acid and then filtering

B dissolving copper(II) oxide in water and then filtering

C heating the copper(II) oxide

D heating the copper(II) oxide mixed with carbon

27 Which statement describes a hydrocarbon?

A a compound that burns to form carbon dioxide and hydrogen


B a compound that contains carbon and hydrogen only
C a compound that only contains ionic bonds
D a compound that reacts easily with metals

28 The diagram shows a speed–time graph for an object.

10
speed
m/s

0
0 1 2 3
time / s

What is the average speed of the object?

A 2.0 m / s B 4.0 m / s C 7.0 m / s D 10 m / s

© UCLES 2020 0653/23/O/N/20 [Turn over

342/693
Combined By Nesrine
12
2023-2017

29 A load is hung from a spring. Measurements are taken to determine the spring constant of the
spring.

Which calculation is used to obtain the spring constant?

extension of spring
A
mass of load

extension of spring
B
weight of load

mass of load
C
extension of spring

weight of load
D
extension of spring

30 A container has a square base of side 2.0 m.

The pressure due to the water on the base of the container is 20 000 N / m2.

container

water

2.0 m

2.0 m

What is the force due to the water on the base of the container?

A 5000 N B 10 000 N C 40 000 N D 80 000 N

31 A crane raises a mass of 200 kg through a vertical distance of 12 m.

The gravitational field strength g is 10 N / kg.

How much work is done on the mass?

A 17 J B 170 J C 2400 J D 24 000 J

32 A car of mass 1200 kg travels at a speed of 15 m / s.

The speed of the car now increases to 25 m / s.

What is the increase in the kinetic energy of the car?

A 60 000 J B 135 000 J C 240 000 J D 375 000 J

© UCLES 2020 0653/23/O/N/20

343/693
Combined By Nesrine
13
2023-2017

33 For which energy resource is the Sun the only source?

A geothermal
B natural gas
C nuclear
D tidal

34 The molecules in a substance vibrate about fixed positions.

The substance is now cooled.

Which row gives the state of the substance and the effect of cooling on the distance between its
molecules?

state of effect on distance


substance between molecules

A solid decreases
B solid increases
C liquid decreases
D liquid increases

35 In which states of matter can convection occur?

in a solid in a liquid in a gas

A no no yes
B no yes yes
C yes no no
D yes yes no

© UCLES 2020 0653/23/O/N/20 [Turn over

344/693
Combined By Nesrine
14
2023-2017

36 The diagram shows a section of a rope.

Four wave crests pass a point on the rope every second.

Each wave crest travels 80 cm in one second.

wave crest
20 cm
direction
5.0 cm of wave

What is the speed of the wave?

A 4.0 cm / s B 5.0 cm / s C 20 cm / s D 80 cm / s

37 The diagram represents a wave in air. Molecules are closer together in region P than they are in
region Q.

region P region Q

What are the names of regions P and Q, and which type of wave is represented?

region P region Q type of wave

A compression rarefaction longitudinal


B compression rarefaction transverse
C rarefaction compression longitudinal
D rarefaction compression transverse

38 A power supply causes a current in a circuit.

The electromotive force (e.m.f.) of the power supply and the resistance of the circuit are both
changed.

Which pair of changes must result in a smaller current in the circuit?

e.m.f. resistance

A decreased decreased
B decreased increased
C increased decreased
D increased increased

© UCLES 2020 0653/23/O/N/20

345/693
Combined By Nesrine
15
2023-2017

39 A circuit includes a lamp, a switch and an ammeter. The switch is open.

The switch is now closed and the ammeter displays the current reading shown.

4 6
2 8

10
0

The switch remains closed for 20 seconds before it is opened again.

What is the charge that flows while the switch is closed?

A 0.25 C B 4.0 C C 90 C D 100 C

40 What is the purpose of a fuse in an electrical appliance?

A to maintain the correct current in the appliance


B to maintain the correct voltage across the appliance
C to prevent the insulation around the cables from becoming too thin
D to protect the wires from overheating when the current is too large

Permission to reproduce items where third-party owned material protected by copyright is included has been sought and cleared where possible. Every
reasonable effort has been made by the publisher (UCLES) to trace copyright holders, but if any items requiring clearance have unwittingly been included, the
publisher will be pleased to make amends at the earliest possible opportunity.

To avoid the issue of disclosure of answer-related information to candidates, all copyright acknowledgements are reproduced online in the Cambridge
Assessment International Education Copyright Acknowledgements Booklet. This is produced for each series of examinations and is freely available to download
at www.cambridgeinternational.org after the live examination series.

Cambridge Assessment International Education is part of the Cambridge Assessment Group. Cambridge Assessment is the brand name of the University of
Cambridge Local Examinations Syndicate (UCLES), which itself is a department of the University of Cambridge.

© UCLES 2020 0653/23/O/N/20

346/693
The Periodic Table of Elements
Group
I II III IV V VI VII VIII

© UCLES 2020
1 2

H He
hydrogen helium
Key 1 4
3 4 atomic number 5 6 7 8 9 10

Li Be atomic symbol B C N O F Ne
lithium beryllium name boron carbon nitrogen oxygen fluorine neon
7 9 relative atomic mass 11 12 14 16 19 20
11 12 13 14 15 16 17 18
Na Mg Al Si P S Cl Ar
sodium magnesium aluminium silicon phosphorus sulfur chlorine argon
23 24 27 28 31 32 35.5 40
19 20 21 22 23 24 25 26 27 28 29 30 31 32 33 34 35 36
K Ca Sc Ti V Cr Mn Fe Co Ni Cu Zn Ga Ge As Se Br Kr
potassium calcium scandium titanium vanadium chromium manganese iron cobalt nickel copper zinc gallium germanium arsenic selenium bromine krypton
39 40 45 48 51 52 55 56 59 59 64 65 70 73 75 79 80 84
37 38 39 40 41 42 43 44 45 46 47 48 49 50 51 52 53 54

Rb Sr Y Zr Nb Mo Tc Ru Rh Pd Ag Cd In Sn Sb Te I Xe
rubidium strontium yttrium zirconium niobium molybdenum technetium ruthenium rhodium palladium silver cadmium indium tin antimony tellurium iodine xenon
85 88 89 91 93 96 – 101 103 106 108 112 115 119 122 128 127 131
16

55 56 57–71 72 73 74 75 76 77 78 79 80 81 82 83 84 85 86
lanthanoids

347/693
Cs Ba Hf Ta W Re Os Ir Pt Au Hg Tl Pb Bi Po At Rn

0653/23/O/N/20
caesium barium hafnium tantalum tungsten rhenium osmium iridium platinum gold mercury thallium lead bismuth polonium astatine radon
133 137 178 181 184 186 190 192 195 197 201 204 207 209 – – –
87 88 89–103 104 105 106 107 108 109 110 111 112 114 116
actinoids
Fr Ra Rf Db Sg Bh Hs Mt Ds Rg Cn Fl Lv
francium radium rutherfordium dubnium seaborgium bohrium hassium meitnerium darmstadtium roentgenium copernicium flerovium livermorium
– – – – – – – – – – – – –

57 58 59 60 61 62 63 64 65 66 67 68 69 70 71
lanthanoids La Ce Pr Nd Pm Sm Eu Gd Tb Dy Ho Er Tm Yb Lu
lanthanum cerium praseodymium neodymium promethium samarium europium gadolinium terbium dysprosium holmium erbium thulium ytterbium lutetium
139 140 141 144 – 150 152 157 159 163 165 167 169 173 175
89 90 91 92 93 94 95 96 97 98 99 100 101 102 103
actinoids Ac Th Pa U Np Pu Am Cm Bk Cf Es Fm Md No Lr
actinium thorium protactinium uranium neptunium plutonium americium curium berkelium californium einsteinium fermium mendelevium nobelium lawrencium
– 232 231 238 – – – – – – – – – – –
2023-2017

The volume of one mole of any gas is 24 dm3 at room temperature and pressure (r.t.p.).
Combined By Nesrine
Combined By Nesrine
2023-2017

Cambridge Assessment International Education


Cambridge International General Certificate of Secondary Education

COMBINED SCIENCE 0653/22


Paper 2 Multiple Choice (Extended) February/March 2019
45 minutes
Additional Materials: Multiple Choice Answer Sheet
Soft clean eraser
*3842696802*

Soft pencil (type B or HB is recommended)

READ THESE INSTRUCTIONS FIRST

Write in soft pencil.


Do not use staples, paper clips, glue or correction fluid.
Write your name, centre number and candidate number on the Answer Sheet in the spaces provided
unless this has been done for you.
DO NOT WRITE IN ANY BARCODES.

There are forty questions on this paper. Answer all questions. For each question there are four possible
answers A, B, C and D.
Choose the one you consider correct and record your choice in soft pencil on the separate Answer Sheet.

Read the instructions on the Answer Sheet very carefully.

Each correct answer will score one mark. A mark will not be deducted for a wrong answer.
Any rough working should be done in this booklet.
A copy of the Periodic Table is printed on page 16.
Electronic calculators may be used.

This document consists of 14 printed pages and 2 blank pages.

IB19 03_0653_22/3RP
© UCLES 2019 [Turn over

348/693
Combined By Nesrine
2
2023-2017

1 The following are features of palisade mesophyll cells:

1 column shaped
2 have a nucleus
3 have large vacuoles
4 have many chloroplasts

Which features of these cells help them to absorb maximum light and carry out photosynthesis?

A 1, 2, 3 and 4
B 1 and 4 only
C 2 and 4 only
D 4 only

2 In an experiment, an enzyme from the human alimentary canal is found to work slowly at 20 °C.

What is the optimum temperature for enzymes working in the human alimentary canal?

A 17 °C B 27 °C C 37 °C D 77 °C

3 Which graph shows the effect of light intensity on the rate of photosynthesis?

A B

rate of rate of
photosynthesis photosynthesis

0 0
0 light intensity 0 light intensity

C D

rate of rate of
photosynthesis photosynthesis

0 0
0 light intensity 0 light intensity

© UCLES 2019 0653/22/F/M/19

349/693
Combined By Nesrine
3
2023-2017

4 What is caused by an iron deficiency in the diet of a human?

A bleeding gums
B rickets
C cannot form white blood cells
D anaemia

5 The diagram shows a transverse section through a plant root.

In which tissue is water transported from the root to the leaves?

A 1 and 2 B 1 only C 2 only D neither 1 or 2

6 What will give the lowest rate of transpiration?

1 high temperature
2 high humidity in the atmosphere
3 high rate of movement of water molecules

A 1 only B 2 only C 1 and 3 D 2 and 3

7 What are the reactants in aerobic respiration?

A carbon dioxide and oxygen


B carbon dioxide and water
C glucose and oxygen
D glucose and water

8 What controls phototropism and gravitropism in the shoot of a plant?

A auxin in the cells


B carbon dioxide in the air
C minerals in the soil
D water in the cells

© UCLES 2019 0653/22/F/M/19 [Turn over

350/693
Combined By Nesrine
4
2023-2017

9 Which row is correct for sexual reproduction?

genetically different one zygote


offspring produced parent produced

A   
B   
C   
D   

10 Four students are comparing the human male and female gametes.

Which student has the correct comparison?

size movement number

A egg bigger sperm mobile usually one egg


B sperm bigger sperm not mobile many eggs
C egg bigger sperm not mobile one sperm
D sperm bigger sperm mobile many sperm

11 The graph shows the energy content of organisms at each trophic level in a food chain.

Which letter represents the primary consumers?

1000

100
energy
/ arbitrary 10
units
1

0
A B C D

© UCLES 2019 0653/22/F/M/19

351/693
Combined By Nesrine
5
2023-2017

12 A farmer chops down a tree to provide firewood. He gets warm when chopping down the tree.
The farmer then burns the wood to keep warm.

What is the original source of the energy that warms the farmer in both cases?

A photosynthesis by the tree growing the wood


B respiration
C the match used to light the fire
D the Sun

13 Eutrophication causes the death of organisms in water.

Which row shows the changes that take place during eutrophication?

oxygen
decomposition respiration
concentration

A decreases increases decreases


B increases decreases decreases
C decreases decreases increases
D increases increases decreases

14 Which statement describes oxygen molecules at room temperature and pressure?

A They are closely packed and move around slowly.


B They are closely packed and vibrate about a fixed point.
C They are loosely packed and move around rapidly.
D They are loosely packed and vibrate about a fixed point.

15 Which piece of equipment can be used to measure exactly 21.6 cm3 of dilute sulfuric acid?

A B C D

beaker burette measuring pipette


cylinder

© UCLES 2019 0653/22/F/M/19 [Turn over

352/693
Combined By Nesrine
6
2023-2017

16 Which statement about the compound formed between a metal and a non-metal is correct?

A It forms strong bonds by sharing electrons.


B It has strong bonds between its atoms.
C It has strong bonds between metal ions and delocalised electrons.
D It has strong bonds between oppositely charged ions.

17 Which statement about the electrolysis of molten lead(II) bromide is correct?

A Bromide ions gain electrons to form bromine at the cathode.


B Bromine loses electrons to form bromide ions at the anode.
C Lead atoms lose electrons to form lead ions at the anode.
D Lead ions accept electrons to form lead at the cathode.

18 Four statements about reactions are listed.

1 Burning a fuel is an exothermic reaction.


2 Endothermic reactions heat up the surroundings.
3 Endothermic reactions take in energy.
4 When exothermic reactions take place the reactants gain energy.

Which statements are correct?

A 1 and 2 B 1 and 3 C 2 and 4 D 3 and 4

19 Which statement about the rate of a reaction is not correct?

A Decreasing the concentration of a reactant solution decreases the frequency of collisions


between particles.
B Decreasing the temperature of a reaction mixture decreases the frequency of collisions
between particles.
C Increasing the particle size of a solid reactant increases the rate of the reaction.
D Increasing the temperature of a reaction mixture increases the rate of the reaction.

20 A solution is tested for the presence of cations.

test result

add excess aqueous ammonia green precipitate

Which cation is present?

A Cu2+ B Fe2+ C Fe3+ D Zn2+

© UCLES 2019 0653/22/F/M/19

353/693
Combined By Nesrine
7
2023-2017

21 Chlorine, bromine and iodine are elements in Group VII of the Periodic Table.

Which trend is observed going down Group VII?

A Each element has the same physical state.


B The colour of the element becomes lighter.
C The reactivity of the element decreases.
D The state of the element changes from solid to liquid to gas.

22 Hydrogen reacts very slowly with nitrogen to form ammonia.

Metal X is a catalyst for this reaction.

What is another property of metal X?

A It forms coloured compounds.


B It forms covalent compounds.
C It has a low density.
D It has a low melting point.

23 The reactivity series for some metals and carbon is shown.

potassium sodium calcium magnesium aluminium carbon zinc copper


most reactive least reactive

Which process is used to extract calcium from its ore?

A reducing the ore with carbon


B electrolysis of the molten ore
C heating the ore with aluminium
D heating the ore in an inert atmosphere

24 A colourless liquid turns blue cobalt chloride paper to pink.

The colourless liquid boils at 78 °C.

Which statement about the colourless liquid is correct?

A It does not contain water.


B It is a hydrocarbon.
C It contains some water.
D It is pure water.

© UCLES 2019 0653/22/F/M/19 [Turn over

354/693
Combined By Nesrine
8
2023-2017

25 Some statements about gases in the air are listed.

1 The amount of carbon dioxide in the atmosphere is increased by burning fossil fuels.
2 Methane is a greenhouse gas.
3 Increasing carbon dioxide in the atmosphere decreases the greenhouse effect.
4 Methane is a product of respiration.

Which statements describe factors that contribute to climate change?

A 1 and 2 B 1 and 4 C 2 and 3 D 3 and 4

26 Which of hydrogen, petroleum and wood are fossil fuels?

hydrogen petroleum wood

A   
B   
C   
D   

27 Which statement describes compounds in the same homologous series?

A They have different general formulae and different chemical properties.


B They have different general formulae and similar chemical properties.
C They have the same general formula and different chemical properties.
D They have the same general formula and similar chemical properties.

28 The speed-time graph represents part of the journey of a car.

20
speed
m/s
15

10

0
0 10 20
time / s

How far does the car travel between 0 s and 20 s?

A 150 m B 200 m C 250 m D 400 m

© UCLES 2019 0653/22/F/M/19

355/693
Combined By Nesrine
9
2023-2017

29 A vehicle moves in a straight line.

The table shows how its speed varies over a time of 40 s.

time / s 0 10 20 30 40
speed
26 24 18 10 2
m/ s

What describes the motion of the vehicle during the 40 s?

A constant acceleration
B constant deceleration
C non-constant deceleration
D positive acceleration

30 The diagram shows a cyclist riding along a hilly road.

At which position does the cyclist have the least gravitational potential energy?

D
A

31 Which row gives thermal properties of air and aluminium?

air aluminium

A a bad thermal conductor a bad thermal conductor


B a bad thermal conductor a good thermal conductor
C a good thermal conductor a bad thermal conductor
D a good thermal conductor a good thermal conductor

© UCLES 2019 0653/22/F/M/19 [Turn over

356/693
Combined By Nesrine
10
2023-2017

32 The diagram shows the direction of a wave passing from medium 1 into medium 2.

medium 1

medium 2

How do the speed and the wavelength of the wave in medium 2 compare with the speed and the
wavelength of the wave in medium 1?

A In medium 2, both the speed and the wavelength are greater.


B In medium 2, both the speed and the wavelength are smaller.
C In medium 2, the speed is greater but the wavelength stays the same.
D In medium 2, the speed is smaller but the wavelength stays the same.

33 A thin converging lens forms a real image.

In the diagrams F indicates each principal focus of the lens.

Which diagram shows how a real image of the object is formed?

A B

lens lens

object
F object image
F image F F

C D

lens lens

object object image


F
F image F F

© UCLES 2019 0653/22/F/M/19

357/693
Combined By Nesrine
11
2023-2017

34 Which statement about light and infra-red radiation is correct?

A Their wavelengths in a vacuum are equal.


B They are longitudinal waves.
C They need a medium through which to travel.

D They travel at 3.0 × 108 m / s in a vacuum.

35 There is a current of 6.0 A in a wire.

How much charge flows through the wire in 2.0 minutes?

A 0.050 coulomb
B 3.0 coulomb
C 12 coulomb
D 720 coulomb

36 A resistance wire of length l has cross-sectional area A and resistance R.

A second resistance wire of the same material has length 0.50 l and cross-sectional area 2.0 A.

What is the resistance of the second wire?

A 0.25 R B 0.50 R C R D 2.0 R

© UCLES 2019 0653/22/F/M/19 [Turn over

358/693
Combined By Nesrine
12
2023-2017

37 A variable power supply is connected to a resistor and there is a current in the resistor.

variable
power supply

current

The potential difference across the resistor is decreased.

The temperature of the resistor does not change.

What happens to the current in the resistor and what happens to the resistance of the resistor?

current resistance

A decreases increases
B decreases stays the same
C increases decreases
D increases stays the same

38 The diagram shows a circuit with three switches P, Q and R.

Which switches must be closed so that both lamps light?

A P and Q only
B P and R only
C Q and R only
D P, Q and R

© UCLES 2019 0653/22/F/M/19

359/693
Combined By Nesrine
13
2023-2017

39 The diagram shows three identical lamps connected in series to a battery.

Each lamp is labelled 0.60 V, 0.30 A. The lamps are working at normal brightness.

What is the potential difference across the battery and the current in the battery?

potential difference / V current / A

A 0.60 0.30
B 0.60 0.90
C 1.80 0.30
D 1.80 0.90

40 The diagram shows four 4.0 Ω resistors connected to a power supply.

+ –

4.0 Ω 4.0 Ω

4.0 Ω 4.0 Ω

What is the resistance of the circuit?

A 1.0 Ω B 2.0 Ω C 4.0 Ω D 16 Ω

© UCLES 2019 0653/22/F/M/19

360/693
Combined By Nesrine
2023-2017

Cambridge Assessment International Education


Cambridge International General Certificate of Secondary Education

COMBINED SCIENCE 0653/21


Paper 2 Multiple Choice (Extended) May/June 2019
45 minutes
Additional Materials: Multiple Choice Answer Sheet
Soft clean eraser
*1585307531*

Soft pencil (type B or HB is recommended)

READ THESE INSTRUCTIONS FIRST

Write in soft pencil.


Do not use staples, paper clips, glue or correction fluid.
Write your name, centre number and candidate number on the Answer Sheet in the spaces provided
unless this has been done for you.
DO NOT WRITE IN ANY BARCODES.

There are forty questions on this paper. Answer all questions. For each question there are four possible
answers A, B, C and D.
Choose the one you consider correct and record your choice in soft pencil on the separate Answer Sheet.

Read the instructions on the Answer Sheet very carefully.

Each correct answer will score one mark. A mark will not be deducted for a wrong answer.
Any rough working should be done in this booklet.
A copy of the Periodic Table is printed on page 20.
Electronic calculators may be used.

This document consists of 17 printed pages and 3 blank pages.

IB19 06_0653_21/2RP
© UCLES 2019 [Turn over

361/693
Combined By Nesrine
2
2023-2017

1 A student is reading a text book. He finds the following definition about how substances move in
and out of cells.

The net movement of water molecules from a


region of higher water potential to a region of
lower water potential through a partially
permeable membrane is called

The corner of the page has been torn.

What is the missing word at the end of the sentence?

A diffusion
B dissolving
C evaporation
D osmosis

2 The graph shows how the activity of an enzyme varies.

enzyme activity

0
0

Which label for the x-axis of this graph is correct?

A enzyme activity
B pH
C temperature
D time

3 What is defined as the breakdown of food into smaller pieces, without chemically changing the
molecules?

A absorption
B chemical digestion
C egestion
D mechanical digestion

© UCLES 2019 0653/21/M/J/19

362/693
Combined By Nesrine
3
2023-2017

4 The diagram shows an experiment at the start and one hour later.

beaker

water water and


partially permeable simple sugars
membrane

starch solution
plus Q

start of experiment one hour later

What is Q?

A amylase
B lipase
C protease
D water

5 The rate of water absorption into a plant is increased by the large surface area of which type of
cell?

A mesophyll
B root cortex
C root hair
D xylem

© UCLES 2019 0653/21/M/J/19 [Turn over

363/693
Combined By Nesrine
4
2023-2017

6 The graphs P, Q and R show the changes in the volume of air in the lungs of the same person,
measured after different levels of activities.

P Q
2 2

volume volume
of air in 1 of air in 1
lungs / dm3 lungs / dm3

0 0
0 10 20 0 10 20
time / s time / s

R
2

volume
of air in 1
lungs / dm3

0
0 10 20
time / s

Which row shows the correct graph for each level of activity?

immediately immediately
at rest after 10 minutes after 10 minutes
of running of walking

A P Q R
B P R Q
C R Q P
D R P Q

7 Which word equation represents aerobic respiration?

A carbon dioxide + glucose → oxygen + water

B glucose + oxygen → carbon dioxide + water

C oxygen + water → carbon dioxide + glucose

D water + carbon dioxide → glucose + oxygen

© UCLES 2019 0653/21/M/J/19

364/693
Combined By Nesrine
5
2023-2017

8 How does adrenaline affect blood glucose concentration and pulse rate?

blood glucose
pulse rate
concentration

A decreases decreases
B decreases increases
C increases decreases
D increases increases

9 Diagram 1 shows a germinating bean seed placed horizontally.

pin

diagram 1

Diagram 2 shows the same seed after three days. The shoot has grown upwards because of the
action of an auxin.

Where is the auxin produced?

C A

pin

diagram 2

10 What are the features of sexual reproduction?

fusion
nature of offspring
of nuclei

A no genetically dissimilar
B yes genetically identical
C no genetically identical
D yes genetically dissimilar

© UCLES 2019 0653/21/M/J/19 [Turn over

365/693
Combined By Nesrine
6
2023-2017

11 The diagram shows a section through an insect-pollinated flower.

Which labels are correct?

anther petal sepal stigma

A 1 3 4 2
B 1 4 3 2
C 2 3 4 1
D 2 4 3 1

© UCLES 2019 0653/21/M/J/19

366/693
Combined By Nesrine
7
2023-2017

12 The diagram shows a food web from the African grasslands.

hawk

leopard
baboon
snake
tick bird

scorpion tick

impala

locust seed-eating bird

grass

Which row correctly identifies the positions of the organisms in the food web?

primary secondary tertiary


consumer consumer consumer

A grass seed-eating bird locust


B impala tick leopard
C locust scorpion tick bird
D seed-eating bird tick bird baboon

13 Which changes to the composition of the atmosphere are caused by cutting down forests?

carbon dioxide gas oxygen gas

A decreases decreases
B decreases increases
C increases decreases
D increases increases

© UCLES 2019 0653/21/M/J/19 [Turn over

367/693
Combined By Nesrine
8
2023-2017

14 The diagram shows apparatus used for filtration.

Why can sugar and salt not be separated by using this apparatus?

A They are both compounds.


B They are both white.
C They both dissolve in water.
D They both have the same size particles.

15 Copper sulfate crystals dissolve in water.

Which word describes the role of the water?

A filtrate
B solute
C solution
D solvent

16 Magnesium chloride is an ionic compound.

Which row describes the formation of magnesium chloride and the strength of the attraction
between its ions?

strength of the
formation of magnesium chloride
attraction between ions

A electrons are shared between magnesium and chlorine strong


B electrons are shared between magnesium and chlorine weak
C electrons are transferred from magnesium to chlorine strong
D electrons are transferred from magnesium to chlorine weak

© UCLES 2019 0653/21/M/J/19

368/693
Combined By Nesrine
9
2023-2017

17 Which process occurs at the anode during the electrolysis of concentrated aqueous
sodium chloride?

A Chloride ions lose electrons to form chlorine.


B Hydrogen ions gain electrons to form hydrogen.
C Oxide ions lose electrons to form oxygen.
D Sodium ions gain electrons to form sodium.

18 When an excess of zinc is added to dilute hydrochloric acid, a gas is released.

Which pieces of apparatus are needed to investigate the rate of this reaction?

1 balance
2 gas syringe
3 stop watch
4 thermometer

A 1 and 2 B 1 and 4 C 2 and 3 D 3 and 4

19 Calcium chloride is a soluble salt.

It is made by adding calcium carbonate to substance X.

Solid calcium chloride is obtained from the reaction mixture by process Y.

What are substance X and process Y?

substance X process Y

A hydrochloric acid crystallisation


B hydrochloric acid filtration
C sodium chloride crystallisation
D sodium chloride filtration

20 Which aqueous ion gives a white precipitate with aqueous sodium hydroxide and with aqueous
ammonia?

A Cu2+ B Fe2+ C Fe3+ D Zn2+

© UCLES 2019 0653/21/M/J/19 [Turn over

369/693
Combined By Nesrine
10
2023-2017

21 Which pair of substances react together?

A bromine and potassium chloride


B bromine and potassium iodide
C iodine and potassium bromide
D iodine and potassium chloride

22 Iron obtained from the blast furnace contains small amounts of carbon and silicon.

Which statement describes this iron?

A It is a covalent compound.
B It is an alloy.
C It is an ionic compound.
D It is slag.

23 P, Q, R and S are four metallic elements.

An atom of S forms an ion by losing only one electron.

When Q is added to a solution of R2+ ions, metal R is produced.

P reacts with cold water to form hydrogen.

What are P, Q, R and S?

P Q R S

A calcium magnesium copper sodium


B copper magnesium iron potassium
C potassium copper zinc sodium
D sodium zinc iron magnesium

24 Which statement about water is not correct?

A A water molecule consists of three atoms covalently bonded together.


B The water supply is treated with chlorine to kill the bacteria in it.
C Water changes the colour of cobalt chloride paper from blue to pink.
D Water has a low melting point because covalent bonds are weak.

© UCLES 2019 0653/21/M/J/19

370/693
Combined By Nesrine
11
2023-2017

25 Which statement shows that petroleum is a mixture?

A Petroleum can be burned as a fuel.


B Petroleum can be separated into fractions by distillation.
C Petroleum is a fossil fuel formed over millions of years.
D Petroleum is a thick, black liquid.

26 Which statement about alkanes is not correct?

A Alkanes are unsaturated hydrocarbons.


B Alkanes burn to release heat energy.
C Alkanes form carbon dioxide and water when they burn.
D Alkane molecules contain only single bonds.

27 Which reaction equation represents cracking?

A CH4 + 2O2 → CO2 + 2H2O

B C2H4 + Br2 → C2H4Br2

C nC2H4 → ( C2H2 ) n

D C2H6 → C2H4 + H2

28 Which labelled part of the electromagnetic spectrum is often involved in thermal energy transfer
by radiation?

radio gamma
A B C D
waves rays

visible
light

© UCLES 2019 0653/21/M/J/19 [Turn over

371/693
Combined By Nesrine
12
2023-2017

29 The diagrams show two speed–time graphs and two distance–time graphs.

Which graph represents the motion of a train with a positive acceleration that is not constant?

A B

speed speed

0 0
0 time 0 time

C D

distance distance

0 0
0 time 0 time

© UCLES 2019 0653/21/M/J/19

372/693
Combined By Nesrine
13
2023-2017

30 The diagram shows an extension-load graph for a spring.

12

11
extension / cm
10

0
0 2 4 6 8 10 12 14 16 18 20 22 24

load / N

The unstretched length of the spring is 10.0 cm.

What is the length of the spring when a load of 8.0 N is suspended from it?

A 4.0 cm B 14.0 cm C 16.0 cm D 26.0 cm

31 Which statement describes the process of convection in a liquid?

A Heated liquid becomes less dense and falls.


B Heated liquid becomes less dense and rises.
C Heated liquid becomes more dense and falls.
D Heated liquid becomes more dense and rises.

© UCLES 2019 0653/21/M/J/19 [Turn over

373/693
Combined By Nesrine
14
2023-2017

32 A balloon contains helium. The balloon is released and rises through the atmosphere. Its volume
increases and the temperature of the helium inside it decreases.

What happens to the average distance between the helium molecules and what happens to their
average speed?

average distance average speed


between molecules of molecules

A decreases decreases
B decreases increases
C increases decreases
D increases increases

33 The diagram shows a vacuum flask containing a hot liquid in a cold room.

X and Y are points on the inside surfaces of the walls of the flask.

cold room

X Y
hot liquid

vacuum

How is thermal energy transferred through the vacuum between X and Y?

A by conduction and convection


B by conduction only
C by radiation and convection
D by radiation only

© UCLES 2019 0653/21/M/J/19

374/693
Combined By Nesrine
15
2023-2017

34 The diagram represents a wave at one moment.

Q
R S
P

Which labelled arrows represent the amplitude and the wavelength of the wave?

amplitude wavelength

A P R
B P S
C Q R
D Q S

35 Which row describes what happens to sound waves as they travel from air into water, and from
water into rock?

sound travelling from sound travelling from


air into water water into rock

A slows down slows down


B slows down speeds up
C speeds up slows down
D speeds up speeds up

36 The amplitude of a sound wave decreases and its frequency increases.

What happens to the sound heard?

A It becomes louder and its pitch becomes higher.


B It becomes louder and its pitch becomes lower.
C It becomes quieter and its pitch becomes higher.
D It becomes quieter and its pitch becomes lower.

37 What is the unit of electric charge?

A ampere
B coulomb
C volt
D watt

© UCLES 2019 0653/21/M/J/19 [Turn over

375/693
Combined By Nesrine
16
2023-2017

38 There is a current of 2.0 A in a resistor. The power produced in the resistor is 8.0 W.

What is the potential difference across the resistor?

A 0.25 V B 4.0 V C 10 V D 16 V

39 Three resistors, one of resistance 4.0 Ω and two of resistance 2.0 Ω, are connected in different
arrangements.

Which arrangement has a total resistance of 5.0 Ω?

A B
2.0 Ω 2.0 Ω
4.0 Ω 2.0 Ω
2.0 Ω 4.0 Ω

C D
2.0 Ω 2.0 Ω
2.0 Ω 2.0 Ω 4.0 Ω
4.0 Ω

40 A mains circuit can safely supply a current of up to 40 A.

The current in a hairdryer is 2 A when it is operating normally. The hairdryer is connected to the
mains by a lead which can safely carry up to 5 A.

What is the correct fuse to protect the hairdryer?

A 1 A fuse
B 3 A fuse
C 10 A fuse
D 50 A fuse

© UCLES 2019 0653/21/M/J/19

376/693
Combined By Nesrine
2023-2017

Cambridge Assessment International Education


Cambridge International General Certificate of Secondary Education

COMBINED SCIENCE 0653/22


Paper 2 Multiple Choice (Extended) May/June 2019
45 minutes
Additional Materials: Multiple Choice Answer Sheet
Soft clean eraser
*3434792860*

Soft pencil (type B or HB is recommended)


MODIFIED LANGUAGE

READ THESE INSTRUCTIONS FIRST

Write in soft pencil.


Do not use staples, paper clips, glue or correction fluid.
Write your name, centre number and candidate number on the Answer Sheet in the spaces provided
unless this has been done for you.
DO NOT WRITE IN ANY BARCODES.

There are forty questions on this paper. Answer all questions. For each question there are four possible
answers A, B, C and D.
Choose the one you consider correct and record your choice in soft pencil on the separate Answer Sheet.

Read the instructions on the Answer Sheet very carefully.

Each correct answer will score one mark. A mark will not be deducted for a wrong answer.
Any rough working should be done in this booklet.
A copy of the Periodic Table is printed on page 16.
Electronic calculators may be used.

This document consists of 15 printed pages and 1 blank page.

IB09 06_0653_22_ML/FP
© UCLES 2019 [Turn over

377/693
Combined By Nesrine
2
2023-2017

1 A student is reading a text book. He finds the following definition about how substances move in
and out of cells.

The net movement of water molecules from a


region of higher water potential to a region of
lower water potential through a partially
permeable membrane is called

The corner of the page has been torn.

What is the missing word at the end of the sentence?

A diffusion
B dissolving
C evaporation
D osmosis

2 An incomplete symbol equation for photosynthesis is given below.

xCOy + xH2O → CxHzOx + xOy

Which row shows the numbers that should replace x, y and z to make the equation balanced?

x y z

A 6 2 6
B 6 12 2
C 6 2 12
D 12 6 2

3 A person has bleeding gums.

Which vitamin could be deficient in his diet, and which food should he eat to provide this vitamin?

vitamin food

A C fish
B C oranges
C D fish
D D oranges

© UCLES 2019 0653/22/M/J/19

378/693
Combined By Nesrine
3
2023-2017

4 What is the purpose of mechanical digestion?

A to break down food into smaller pieces


B to break down insoluble foods into soluble particles
C to change food so that it can be absorbed into the blood
D to digest insoluble food components using enzymes

5 A student uses a microscope to observe an artery.

Which feature is present in an artery?

A a wide lumen
B valves
C walls with a thick layer of muscle
D wall only a single cell in thickness

6 What is the route for carbon dioxide passing out of the body?

A alveoli → capillaries → bronchioles → bronchi → trachea → larynx

B alveoli → capillaries → bronchi → bronchioles → larynx → trachea

C capillaries → alveoli → bronchi → bronchioles → trachea → larynx

D capillaries → alveoli → bronchioles → bronchi → trachea → larynx

© UCLES 2019 0653/22/M/J/19 [Turn over

379/693
Combined By Nesrine
4
2023-2017

7 The diagram shows the female reproductive system.

X Y

What are the functions of the parts labelled X, Y, and Z?

X Y Z
A development release of ring of muscle at
of fetus female gametes opening of uterus
B development site of receives penis
of fetus fertilisation during intercourse
C receives penis release of ring of muscle at
during intercourse female gametes opening of uterus
D receives penis site of development
during intercourse fertilisation of fetus

8 How does adrenaline affect blood glucose concentration and pulse rate?

blood glucose
pulse rate
concentration

A decreases decreases
B decreases increases
C increases decreases
D increases increases

© UCLES 2019 0653/22/M/J/19

380/693
Combined By Nesrine
5
2023-2017

9 Diagram 1 shows a germinating bean seed placed horizontally.

pin

diagram 1

Diagram 2 shows the same seed after three days. The shoot has grown upwards because of the
action of an auxin.

Where is the auxin produced?

C A

pin

diagram 2

10 What are the features of sexual reproduction?

fusion
nature of offspring
of nuclei

A no genetically dissimilar
B yes genetically identical
C no genetically identical
D yes genetically dissimilar

© UCLES 2019 0653/22/M/J/19 [Turn over

381/693
Combined By Nesrine
6
2023-2017

11 The diagram represents several food chains in a food web.

kestrel cat
wolf
small bird rat
sheep
ant grasshopper

grass

How many different food chains are there in the food web shown?

A 3 B 4 C 5 D 9

12 Which row shows the substances that diffuse from the mother to the fetus at the placenta?

carbon waste
nutrients oxygen
dioxide products

A    
B    
C    
D    

13 Which term is defined as a unit containing all of the organisms and their environment, interacting
with each other?

A carbon cycle
B ecosystem
C food chain
D food web

14 A molecule of hydrogen has the formula H2.

A molecule of a protein contains several different elements.

Which statement about these molecules is correct?

A They both contain cations and anions bonded together.


B They both contain different types of atom.
C They both contain more than one atom bonded together.
D They both contain only one type of atom.

© UCLES 2019 0653/22/M/J/19

382/693
Combined By Nesrine
7
2023-2017

15 The diagram shows apparatus used for filtration.

Why can this apparatus not be used to separate sugar and salt?

A They are both compounds.


B They are both white.
C They both dissolve in water.
D They both have the same size particles.

16 An atom of an element has the proton number 16 and the nucleon number 36.

Which row shows the number of neutrons in the atom and the group number of the element in the
Periodic Table?

number of neutrons
group number
in the atom

A 20 VI
B 20 VIII
C 36 VI
D 36 VIII

17 Which statement about sodium ions and chloride ions in sodium chloride is not correct?

A They are strongly attracted to each other.


B They both have noble gas electronic structures.
C They are arranged in a regular lattice.
D They share pairs of electrons.

© UCLES 2019 0653/22/M/J/19 [Turn over

383/693
Combined By Nesrine
8
2023-2017

18 Which statement about the electrolysis of molten magnesium chloride is correct?

A Cations form atoms by losing electrons.


B Chlorine atoms gain electrons to form chloride ions.
C Magnesium ions gain electrons to form magnesium atoms.
D Magnesium is produced at the anode and chlorine is formed at the cathode.

19 The graph shows the volume of hydrogen gas produced when dilute hydrochloric acid reacts with
zinc.

At which point is the rate of reaction greatest?

D
C

volume
/ cm3

time / s

20 Which aqueous ion gives a white precipitate with aqueous sodium hydroxide and with aqueous
ammonia?

A Cu2+ B Fe2+ C Fe3+ D Zn2+

21 Some properties of three metals in Group I are shown.

melting
relative softness reaction with water
point / °C

sodium 98 soft reacts rapidly


potassium 64 very soft burns on contact with water
caesium 29 very, very soft violently explosive

Rubidium is below potassium in Group I.

What is a property of rubidium?

A It explodes on contact with water.


B It is a hard solid.

C It is a liquid at 20 °C.
D It is less dense than potassium.

© UCLES 2019 0653/22/M/J/19

384/693
Combined By Nesrine
9
2023-2017

22 Which diagram represents brass?

A B C D

– + – +
+ – + –
– + – +
+ – + –

23 Zinc reacts with chromium oxide. The reaction equation is shown.

3Zn + Cr2O3 → 2Cr + 3ZnO

Which statement about this reaction is correct?

A Chromium is above zinc in the reactivity series.


B Chromium oxide is the oxidising agent because it gains oxygen.
C Chromium oxide is the reducing agent because it loses oxygen.
D Zinc is the reducing agent because it gains oxygen.

24 Which statement about water is not correct?

A A water molecule consists of three atoms covalently bonded together.


B The water supply is treated with chlorine to kill the bacteria in it.
C Water changes the colour of cobalt chloride paper from blue to pink.
D Water has a low melting point because covalent bonds are weak.

25 Which statement shows that petroleum is a mixture?

A Petroleum can be burned as a fuel.


B Petroleum can be separated into fractions by distillation.
C Petroleum is a fossil fuel formed over millions of years.
D Petroleum is a thick, black liquid.

26 Which statement about alkanes is correct?

A Alkanes are compounds containing carbon, hydrogen and oxygen.


B Alkanes are saturated hydrocarbons.
C Ethane is used to make poly(ethene).
D Methane is the only alkane that does not contain a double bond.

© UCLES 2019 0653/22/M/J/19 [Turn over

385/693
Combined By Nesrine
10
2023-2017

27 Which row identifies the temperature used and describes the change to the alkane molecules
during the cracking process?

temperature change to the


/ °C alkane molecules

A 100 become larger


B 100 become smaller
C 500 become larger
D 500 become smaller

28 The speed of a car is measured at 1.0 s intervals. The results are shown in the table.

time / s 0 1.0 2.0 3.0 4.0


speed
20 21 23 26 30
km / h

Which is a description of the motion of the car?

A at rest
B constant acceleration
C constant speed
D non-constant acceleration

29 A metal ball of mass 0.050 kg is released from rest and falls to the ground. It hits the ground with
kinetic energy 1.2 J.

The gravitational field strength is 10 N / kg. Air resistance can be ignored.

From what height above the ground is the ball released?

A 0.042 m B 0.42 m C 2.4 m D 24 m

30 Which mode of transport uses a renewable energy source?

A a coal-fired steam train


B a nuclear-powered submarine
C a petrol-engined car
D a sailing boat moved by the wind

© UCLES 2019 0653/22/M/J/19

386/693
Combined By Nesrine
11
2023-2017

31 A solid is heated.

Which two properties of the solid both change as a result?

A density and volume


B density and weight
C mass and volume
D mass and weight

32 Non-metal solids conduct heat but not as well as metals.

Which row describes how non-metal solids conduct heat?

molecular heat transfer


vibration by electrons

A no no
B no yes
C yes no
D yes yes

© UCLES 2019 0653/22/M/J/19 [Turn over

387/693
Combined By Nesrine
12
2023-2017

33 The diagram shows a vacuum flask containing a hot liquid in a cold room.

X and Y are points on the inside surfaces of the walls of the flask.

cold room

X Y
hot liquid

vacuum

How is thermal energy transferred through the vacuum between X and Y?

A by conduction and convection


B by conduction only
C by radiation and convection
D by radiation only

34 The diagram represents a wave at one moment.

Q
R S
P

Which labelled arrows represent the amplitude and the wavelength of the wave?

amplitude wavelength

A P R
B P S
C Q R
D Q S

© UCLES 2019 0653/22/M/J/19

388/693
Combined By Nesrine
13
2023-2017

35 Which wave is longitudinal?

A microwave
B light wave
C radio wave
D sound wave

36 Which statement about gamma rays and radio waves is correct?

A In a vacuum, gamma rays and radio waves travel at 300 m / s.

B In a vacuum, gamma rays and radio waves travel at 3.0 × 108 m / s.


C In a vacuum, gamma rays travel faster than radio waves.
D In a vacuum, radio waves travel faster than gamma rays.

37 Four wires are made of the same material. They have different lengths and different
cross-sectional areas.

Which wire has the smallest resistance?

cross-sectional
length
area

A
A l
2
B l A
A
C 2l
2
D 2l A

© UCLES 2019 0653/22/M/J/19 [Turn over

389/693
Combined By Nesrine
14
2023-2017

38 Three resistors, one resistor has resistance 4.0 Ω and two have resistance 2.0 Ω, are connected
in different arrangements.

Which arrangement has a total resistance of 5.0 Ω?

A B
2.0 Ω 2.0 Ω
4.0 Ω 2.0 Ω
2.0 Ω 4.0 Ω

C D
2.0 Ω 2.0 Ω
2.0 Ω 2.0 Ω 4.0 Ω
4.0 Ω

39 There is a current of 2.0 A in a resistor. The power produced in the resistor is 8.0 W.

What is the potential difference across the resistor?

A 0.25 V B 4.0 V C 10 V D 16 V

40 A mains circuit can safely supply a current of up to 40 A.

The current in a hairdryer is 2 A when it is operating normally. The hairdryer is connected to the
mains by a lead which can safely carry up to 5 A.

What is the correct fuse to protect the hairdryer?

A 1 A fuse
B 3 A fuse
C 10 A fuse
D 50 A fuse

© UCLES 2019 0653/22/M/J/19

390/693
Combined By Nesrine
2023-2017

Cambridge Assessment International Education


Cambridge International General Certificate of Secondary Education

COMBINED SCIENCE 0653/23


Paper 1 Multiple Choice (Extended) May/June 2019
45 minutes
Additional Materials: Multiple Choice Answer Sheet
Soft clean eraser
*8109103470*

Soft pencil (type B or HB is recommended)

READ THESE INSTRUCTIONS FIRST

Write in soft pencil.


Do not use staples, paper clips, glue or correction fluid.
Write your name, centre number and candidate number on the Answer Sheet in the spaces provided
unless this has been done for you.
DO NOT WRITE IN ANY BARCODES.

There are forty questions on this paper. Answer all questions. For each question there are four possible
answers A, B, C and D.
Choose the one you consider correct and record your choice in soft pencil on the separate Answer Sheet.

Read the instructions on the Answer Sheet very carefully.

Each correct answer will score one mark. A mark will not be deducted for a wrong answer.
Any rough working should be done in this booklet.
A copy of the Periodic Table is printed on page 16.
Electronic calculators may be used.

This document consists of 14 printed pages and 2 blank pages.

IB19 06_0653_23/2RP
© UCLES 2019 [Turn over

391/693
Combined By Nesrine
2
2023-2017

1 Which row has a correct structural adaptation for red blood cells and some of the cells lining the
trachea?

red blood cells cells lining the trachea

A nucleus absent has cilia


B nucleus present has cilia
C nucleus absent large surface area
D nucleus present large surface area

2 A student is reading a text book. He finds the following definition about how substances move in
and out of cells.

The net movement of water molecules from a


region of higher water potential to a region of
lower water potential through a partially
permeable membrane is called

The corner of the page has been torn.

What is the missing word at the end of the sentence?

A diffusion
B dissolving
C evaporation
D osmosis

3 The enzyme salivary amylase starts digesting starchy foods in the mouth.

This stops when the food reaches the stomach.

Why does this happen?

A The acid in the stomach slows down all reactions.


B The shape of the active site of the enzyme is altered by the low pH.
C The kinetic energy of molecules is reduced by acids.
D The shape of the substrate molecules is changed.

© UCLES 2019 0653/23/M/J/19

392/693
Combined By Nesrine
3
2023-2017

4 Which condition is caused by a lack of vitamin C in the diet?

A breathlessness
B rickets
C scurvy
D constipation

5 The diagram shows the alimentary canal and some associated organs.

In which structure is bile stored?

6 Physical activity affects our rate and depth of breathing.

What happens during increased physical activity?

rate of breathing depth of breathing

A decreases decreases
B decreases increases
C increases decreases
D increases increases

7 Which substances are used and produced during photosynthesis?

substances used substances produced

A carbon dioxide and glucose oxygen and water


B carbon dioxide and water glucose and oxygen
C glucose and oxygen carbon dioxide and water
D oxygen and water carbon dioxide and glucose

© UCLES 2019 0653/23/M/J/19 [Turn over

393/693
Combined By Nesrine
4
2023-2017

8 How does adrenaline affect blood glucose concentration and pulse rate?

blood glucose
pulse rate
concentration

A decreases decreases
B decreases increases
C increases decreases
D increases increases

9 Diagram 1 shows a germinating bean seed placed horizontally.

pin

diagram 1

Diagram 2 shows the same seed after three days. The shoot has grown upwards because of the
action of an auxin.

Where is the auxin produced?

C A

pin

diagram 2

10 What are the features of sexual reproduction?

fusion
nature of offspring
of nuclei

A no genetically dissimilar
B yes genetically identical
C no genetically identical
D yes genetically dissimilar

© UCLES 2019 0653/23/M/J/19

394/693
Combined By Nesrine
5
2023-2017

11 Which process is the transfer of pollen grains from the anther to the stigma?

A fertilisation
B germination
C pollination
D transpiration

12 The diagram shows part of a placenta.

vein
artery
mothers blood

placenta
membrane
barrier

umbilical vein umbilical artery

Why do nutrients in the mother’s blood enter the blood in the umbilical vein?

A A net movement of nutrient particles occurs from a region of high concentration to a lower
concentration.
B Nutrients move from a region of higher water potential to a region of lower water potential.
C Pressure in the maternal blood forces nutrients into the umbilical vein.
D The nutrients travel into the umbilical vein, across a partially permeable membrane by
osmosis.

© UCLES 2019 0653/23/M/J/19 [Turn over

395/693
Combined By Nesrine
6
2023-2017

13 The diagram shows a food web.

eagle

fox
stoat

rabbit grasshopper

grass

Which type of organism is not represented in this food web?

A carnivore
B consumer
C decomposer
D herbivore

14 The diagram shows apparatus used for filtration.

Why can sugar and salt not be separated by using this apparatus?

A They are both compounds.


B They are both white.
C They both dissolve in water.
D They both have the same size particles.

15 Which description of the named substance is correct?

substance element or mixture

A air mixture
B brass element
C carbon dioxide element
D hydrogen chloride mixture

© UCLES 2019 0653/23/M/J/19

396/693
Combined By Nesrine
7
2023-2017

16 Which statement explains why sodium chloride has a much higher melting point than carbon
dioxide?

A Ionic bonding is weaker than covalent bonding.


B Ionic bonding is stronger than covalent bonding.
C The attractive forces between ions are stronger than the attractive forces between
molecules.
D The attractive forces between ions are weaker than the attractive forces between molecules.

17 Molten sodium chloride is electrolysed.

What are the products at the electrodes?

product at anode product at cathode

A chlorine hydrogen
B chlorine sodium
C hydrogen chlorine
D sodium chlorine

18 Zinc reacts with excess dilute sulfuric acid to form hydrogen gas.

Copper sulfate can act as a catalyst for this reaction.

Which statement is not correct?

A If more concentrated sulfuric acid is used the rate of the reaction increases.
B If the temperature is increased it takes less time for the zinc to react completely.
C Larger pieces of zinc produce more hydrogen every ten seconds than the same mass of
powdered zinc.
D When copper sulfate is added to the mixture more hydrogen is formed every second.

19 Magnesium reacts with zinc oxide to make magnesium oxide and zinc.

Which substance is the oxidising agent in this reaction?

A magnesium
B magnesium oxide
C zinc
D zinc oxide

© UCLES 2019 0653/23/M/J/19 [Turn over

397/693
Combined By Nesrine
8
2023-2017

20 Which aqueous ion gives a white precipitate with aqueous sodium hydroxide and with aqueous
ammonia?

A Cu2+ B Fe2+ C Fe3+ D Zn2+

21 An element has the electronic structure 2,8,1.

Which row describes this element?

group number in
metal / non-metal
the Periodic Table

A I metal
B I non-metal
C II metal
D II non-metal

22 Which diagram represents an alloy?

A B C D
+ – + – + – +
– + – + – + –
+ – + – + – +
– + – + – + –
+ – + – + – +

23 In which mixture does the metal displace the aqueous metal ion?

A copper and magnesium sulfate solution


B iron and zinc sulfate solution
C magnesium and copper sulfate solution
D zinc and magnesium sulfate solution

24 Which statement about water is not correct?

A A water molecule consists of three atoms covalently bonded together.


B The water supply is treated with chlorine to kill the bacteria in it.
C Water changes the colour of cobalt chloride paper from blue to pink.
D Water has a low melting point because covalent bonds are weak.

© UCLES 2019 0653/23/M/J/19

398/693
Combined By Nesrine
9
2023-2017

25 Which statement shows that petroleum is a mixture?

A Petroleum can be burned as a fuel.


B Petroleum can be separated into fractions by distillation.
C Petroleum is a fossil fuel formed over millions of years.
D Petroleum is a thick, black liquid.

26 Which substances react together?

1 ethene and methane


2 ethene and bromine
3 ethene and oxygen

A 1, 2 and 3 B 1 and 2 only C 1 and 3 only D 2 and 3 only

27 Which statement about cracking is not correct?

A A high temperature and a catalyst are used.


B Alkenes are made.
C Hydrogen can be made.
D Larger alkanes are made from smaller alkanes.

© UCLES 2019 0653/23/M/J/19 [Turn over

399/693
Combined By Nesrine
10
2023-2017

28 Which speed–time graph represents the motion of an object that travels a distance of 24 m?

A B

speed speed
m/s m/s

4.0
3.0

0 0
0 6.0 0 8.0
time / s time / s

C D

speed speed
m/s m/s

4.0

2.0

0 0
0 3.0 0 12
time / s time / s

29 Which property of a body is the effect of a gravitational field acting on the mass of the body?

A density
B surface area
C volume
D weight

30 What is the expression for density?


mass volume volume weight
A B C D
volume mass weight volume

31 A body moving with a speed of 2.0 m / s has a kinetic energy of 8.0 J.

What is the mass of the body?

A 1.0 kg B 2.0 kg C 4.0 kg D 8.0 kg

© UCLES 2019 0653/23/M/J/19

400/693
Combined By Nesrine
11
2023-2017

32 The diagram shows a vacuum flask containing a hot liquid in a cold room.

X and Y are points on the inside surfaces of the walls of the flask.

cold room

X Y
hot liquid

vacuum

How is thermal energy transferred through the vacuum between X and Y?

A by conduction and convection


B by conduction only
C by radiation and convection
D by radiation only

33 The diagram represents a wave at one moment.

Q
R S
P

Which labelled arrows represent the amplitude and the wavelength of the wave?

amplitude wavelength

A P R
B P S
C Q R
D Q S

© UCLES 2019 0653/23/M/J/19 [Turn over

401/693
Combined By Nesrine
12
2023-2017

34 Which electromagnetic radiation has the lowest frequency?

A gamma
B infrared
C radio
D ultraviolet

35 A converging lens is placed in the position shown in the diagram.

Each principal focus is marked F, and two points that are two focal lengths from the lens are
marked 2F.

At which labelled point is an object placed so that the lens acts as a magnifying glass?

converging lens

A B C D
2F F F 2F

36 Where does sound travel at the greatest speed?

A in a gas
B in a liquid
C in a solid
D in a vacuum

37 There is a current of 2.0 A in a resistor. The power produced in the resistor is 8.0 W.

What is the potential difference across the resistor?

A 0.25 V B 4.0 V C 10 V D 16 V

© UCLES 2019 0653/23/M/J/19

402/693
Combined By Nesrine
13
2023-2017

38 How is the resistance R of a wire related to its length l and to its cross-sectional area A?

(∝ means proportional to)

A R ∝ 1 and R ∝ A
l

B R ∝ 1 and R ∝ 1
l A

C R ∝ l and R ∝ A

D R ∝ l and R ∝ 1
A

39 Three resistors, one of resistance 4.0 Ω and two of resistance 2.0 Ω, are connected in different
arrangements.

Which arrangement has a total resistance of 5.0 Ω?

A B
2.0 Ω 2.0 Ω
4.0 Ω 2.0 Ω
2.0 Ω 4.0 Ω

C D
2.0 Ω 2.0 Ω
2.0 Ω 2.0 Ω 4.0 Ω
4.0 Ω

40 A mains circuit can safely supply a current of up to 40 A.

The current in a hairdryer is 2 A when it is operating normally. The hairdryer is connected to the
mains by a lead which can safely carry up to 5 A.

What is the correct fuse to protect the hairdryer?

A 1 A fuse
B 3 A fuse
C 10 A fuse
D 50 A fuse

© UCLES 2019 0653/23/M/J/19

403/693
Combined By Nesrine
2023-2017

Cambridge Assessment International Education


Cambridge International General Certificate of Secondary Education

COMBINED SCIENCE 0653/21


Paper 2 Multiple Choice (Extended) October/November 2019
45 minutes
Additional Materials: Multiple Choice Answer Sheet
Soft clean eraser
*8584537552*

Soft pencil (type B or HB is recommended)

READ THESE INSTRUCTIONS FIRST

Write in soft pencil.


Do not use staples, paper clips, glue or correction fluid.
Write your name, centre number and candidate number on the Answer Sheet in the spaces provided
unless this has been done for you.
DO NOT WRITE IN ANY BARCODES.

There are forty questions on this paper. Answer all questions. For each question there are four possible
answers A, B, C and D.
Choose the one you consider correct and record your choice in soft pencil on the separate Answer Sheet.

Read the instructions on the Answer Sheet very carefully.

Each correct answer will score one mark. A mark will not be deducted for a wrong answer.
Any rough working should be done in this booklet.
A copy of the Periodic Table is printed on page 16.
Electronic calculators may be used.

This document consists of 15 printed pages and 1 blank page.

IB19 11_0653_21/2RP
© UCLES 2019 [Turn over

404/693
Combined By Nesrine
2
2023-2017

1 A biologist keeps a potted plant in a laboratory.

Which feature of the potted plant shows that it is a living organism?

A It grows larger over time.


B It has green leaves.
C The compost in the pot dries after he waters it.
D The stems contain xylem.

2 The diagram shows a section through a leaf.

Which row correctly identifies the labelled parts of the leaf section?

X Y Z

A cuticle vascular bundle palisade mesophyll


B palisade mesophyll vascular bundle spongy mesophyll
C palisade mesophyll cuticle spongy mesophyll
D spongy mesophyll cuticle vascular bundle

© UCLES 2019 0653/21/O/N/19

405/693
Combined By Nesrine
3
2023-2017

3 1 cm³ of substance X is added to 10 cm³ starch suspension and mixed. Food tests are carried out
immediately after mixing and again after an hour.

The results of the tests are shown in the table.

colour of solution colour of solution


test reagent
after mixing after one hour

Benedict’s solution blue orange


iodine solution blue / black brown

What is substance X?

A amylase
B protease
C lipase
D sugar

4 How are root hair cells adapted for absorption of water?

A large surface area


B thick cell wall
C many chloroplasts
D no nucleus

5 The diagram shows a double circulatory system.

lung

heart

body

In which direction does the blood flow in this type of system?

A heart → body → heart → lung

B body → heart → body → lung

C heart → body → lung → heart

D lung → heart → lung → body

© UCLES 2019 0653/21/O/N/19 [Turn over

406/693
Combined By Nesrine
4
2023-2017

6 The concentrations of carbon dioxide and oxygen in expired air differ from the concentrations in
inspired air.

concentration in
gas
expired air

1 carbon dioxide higher


2 carbon dioxide lower
3 oxygen higher
4 oxygen lower

Which rows correctly show the difference?

A 1 and 3 B 1 and 4 C 2 and 3 D 2 and 4

7 Which statement about aerobic respiration is correct?

A It exchanges gases through the walls of the alveoli.


B It expels carbon dioxide from the lungs.
C It only produces carbon dioxide and energy.
D It uses oxygen to release energy from glucose.

8 What is the effect of adrenaline on the rate of breathing and pulse rate?

rate of breathing pulse rate

A decreases decreases
B decreases increases
C increases decreases
D increases increases

9 Auxins are produced in the tip of a shoot.

In an experiment, auxin was applied to a plant shoot just below its tip.

Which row describes the change to the shoot and explains this change?

shoot explanation

A becomes longer cells divide


B becomes longer cells elongate
C becomes longer cells elongate and then divide
D did not change length the auxin has no effect

© UCLES 2019 0653/21/O/N/19

407/693
Combined By Nesrine
5
2023-2017

10 Which statement about sexual reproduction is always correct?

A It involves only one parent.


B It involves the fusion of nuclei.
C It produces genetically identical offspring.
D It takes place only in animals.

11 The table gives comparisons between insect-pollinated and wind-pollinated flowers.

Which row is correct?

surface of stigma surface of pollen

insect wind insect wind

A feathery sticky smooth spiky


B feathery sticky spiky smooth
C sticky feathery smooth spiky
D sticky feathery spiky smooth

12 The following are adaptive features of some gametes.

1 a flagellum
2 a jelly coat
3 a sac of enzymes at one end
4 a store of energy

Which are features of a human male gamete?

A 1 and 3 B 1 and 4 C 2 and 3 D 2 and 4

13 An increased volume of nitrate ions is washed into a lake.

Why does this result in a decrease in the number of fish in the lake?

A There is a decrease in the decomposition of producers.


B There is a decrease in the growth of producers.
C There is an increase in aerobic respiration by decomposers.
D There is an increase in dissolved oxygen.

© UCLES 2019 0653/21/O/N/19 [Turn over

408/693
Combined By Nesrine
6
2023-2017

14 A chromatogram of substance Q is shown.

solvent front
20 cm
18 cm
16 cm
14 cm
12 cm
10 cm
8 cm
6 cm
4 cm
2 cm
base line
0 cm
Q

What is the Rf value of Q?

A 0.2 B 0.4 C 0.8 D 1.6

15 Four processes are listed.

1 melting of ice

2 electrolysis of molten lead(II) bromide


3 combustion of carbon
4 rusting of iron

Which processes are chemical changes?


A 1 and 3 only B 1, 2 and 3 C 2 and 4 only D 2, 3 and 4

16 Which diagram represents a mixture of two different elements?

A B C D

© UCLES 2019 0653/21/O/N/19

409/693
Combined By Nesrine
7
2023-2017

17 The fertiliser ammonium sulfate has the formula (NH4)2SO4.

How many atoms of each element are present in the formula?

number of number of number of number of


hydrogen atoms nitrogen atoms oxygen atoms sulfur atoms

A 4 1 1 1
B 4 2 4 1
C 8 1 4 1
D 8 2 4 1

18 Element X is a non-metal used in the treatment of the water supply.

It is made during the electrolysis of a metal salt.

What is the colour of X and at which electrode is it made?

colour electrode

A red anode
B red cathode
C yellow-green anode
D yellow-green cathode

19 The energy level diagram for the reaction between P and Q to produce R and S is shown.

40

R+S
30

energy P+Q
20
kJ / mol

10

0
progress of reaction

Which statement about this reaction is correct?

A Energy is required to form new bonds.


B The activation energy is 10 kJ / mol.
C The energy required to break bonds is greater than the energy given out when bonds form.
D The reaction is exothermic because the energy of the products is greater than the energy of
the reactants.

© UCLES 2019 0653/21/O/N/19 [Turn over

410/693
Combined By Nesrine
8
2023-2017

20 Calcium carbonate reacts with dilute hydrochloric acid.

The time taken to collect 10 cm3 of carbon dioxide is recorded.

The experiment is repeated at a different temperature. The results are shown.

temperature time taken


experiment
/ °C /s

1 20 55
2 80 30

The rate of reaction in each experiment is different.

Which statement about the rate of reaction of experiment 1, compared with experiment 2, is
correct?

A It is greater because at the lower temperature the particles move more slowly so they have
more time to react.
B It is greater because the particles collide more frequently.
C It is lower because the particles collide at the same frequency and fewer of them have the
minimum energy to react.
D It is lower because the particles collide less frequently and fewer of them have the minimum
energy to react.

21 The equation for the reaction of carbon monoxide with copper oxide is shown.

CO + CuO → Cu + CO2

Which statement about this reaction is not correct?

A Carbon dioxide is the oxidising agent.


B Carbon monoxide is the reducing agent.
C Carbon monoxide is being oxidised.
D Copper oxide is the oxidising agent.

22 Which two substances are used to make copper sulfate?

A copper and dilute sulfuric acid


B copper and sulfur
C copper oxide and dilute sulfuric acid
D copper oxide and sulfur

© UCLES 2019 0653/21/O/N/19

411/693
Combined By Nesrine
9
2023-2017

23 Which statement about elements in the Periodic Table is not correct?

A Elements in Group I react by gaining electrons to form ions.


B Elements in Group II have two electrons in their outer shell.
C Elements in Group VIII are unreactive because they have full outer shells of electrons.
D Elements with seven outer shell electrons are non-metals.

24 Which statement about alloys is correct?

A They are made from metals because metals are poor electrical conductors.
B They are mixtures of compounds that contain metals.
C They have all the same properties as the metals from which they are made.
D They have different properties to the metals from which they are made.

25 Which gas is not present in clean air?

A carbon monoxide
B neon
C nitrogen
D water vapour

26 Which two gases cause an enhanced greenhouse effect when their concentrations in the
atmosphere increase?

A carbon monoxide and carbon dioxide


B carbon dioxide and methane
C methane and sulfur dioxide
D sulfur dioxide and carbon monoxide

27 Which statement about fractions obtained from petroleum is not correct?

A Different fractions have different boiling points.


B Gasoline contains molecules of only one hydrocarbon.
C Molecules in diesel oil are smaller than molecules in bitumen.
D Naphtha is used as a feedstock for making chemicals.

© UCLES 2019 0653/21/O/N/19 [Turn over

412/693
Combined By Nesrine
10
2023-2017

28 The graph shows how the speed of a car changes with time. The car travels at constant speed,
then accelerates, and finally brakes to a stop.

speed

0
0 5 20 30
time / s

The car travels 60 m while it brakes to a stop.

What is the average speed of the car while it is braking?

A 3.0 m / s B 4.0 m / s C 6.0 m / s D 12 m / s

29 The gravitational field strength on Mars is less than that on Earth.

An object is taken from Earth to Mars.

Which statement describes the object when it is on Mars?

A It has greater mass than on Earth.


B It has less mass than on Earth.
C It has less weight than on Earth.
D It has the same weight as on Earth.

© UCLES 2019 0653/21/O/N/19

413/693
Combined By Nesrine
11
2023-2017

30 A student wants to determine the density of an irregularly shaped stone. He has a measuring
cylinder, water, a balance and a thermometer.

These are his measurements:

P initial volume of water in measuring cylinder


Q mass of measuring cylinder
R mass of stone
S mass of water in measuring cylinder
T reading on measuring cylinder with stone fully immersed in the water
U temperature of water

Which three measurements are needed to determine the density of the stone?

A P, R and T
B P, T and U
C Q, R and S
D Q, S and U

31 Which device uses a non-renewable energy source?

A diesel engine
B solar cell
C water turbine
D windmill

32 A metal pan containing water is heated on a hot stove. Energy is transferred thermally from the
stove to the water.

How is the energy transferred through the pan and then throughout the water?

through the pan throughout the water

A conduction conduction
B conduction convection
C convection conduction
D convection convection

© UCLES 2019 0653/21/O/N/19 [Turn over

414/693
Combined By Nesrine
12
2023-2017

33 On a hot, sunny day a boy finds that his head stays cooler when he wears a white hat than when
he wears an otherwise identical black hat.

Why does the white hat keep his head cooler?

A It absorbs less radiation from the Sun than the black hat.
B It conducts less heat energy than the black hat.
C It conducts more heat energy than the black hat.
D It emits more radiation from his head than the black hat.

34 Which wave is longitudinal?

A infrared
B radio
C sound
D ultraviolet

35 The diagram shows light striking a plane mirror.

plane mirror

50°

ray of light
normal

What is the angle of reflection of the ray when it is reflected from the mirror?

A 40° B 50° C 80° D 100°

© UCLES 2019 0653/21/O/N/19

415/693
Combined By Nesrine
13
2023-2017

36 A circuit contains a battery of e.m.f. E, two lamps and two voltmeters, connected as shown.

The voltmeter readings V1 and V2, and the current in three parts of the circuit I, I1 and I2 are
labelled.

e.m.f. E

reading = V1
I
V
I1

I2
V
reading = V2

Which row gives expressions for current I and e.m.f. E?

current I e.m.f. E

A I = I1 = I2 E = V1 = V2
B I = I1 = I2 E = V1 + V2
C I = I1 + I2 E = V1 = V2
D I = I1 + I2 E = V1 + V2

37 A student measures the speed of sound. He claps his hands and the sound reflects from a wall
that is 100 m away from him.

wall

student

electronic
timer

100 m

An electronic timer next to the student detects the echo of the sound 0.60 s after it is made.

Which calculation gives the speed of sound?

A 200 m / s B 200 m / s C 100 m / s D 100 m / s


0.30 0.60 0.60 1. 2

© UCLES 2019 0653/21/O/N/19 [Turn over

416/693
Combined By Nesrine
14
2023-2017

38 A piece of wire has a resistance of 8.0 Ω.

The length of the wire is doubled and the diameter of the wire is halved.

What is the new resistance of the wire?

A 2.0 Ω B 4.0 Ω C 8.0 Ω D 64 Ω

39 Four ammeters V, W, X and Y are connected in the circuit shown.

V W
A A

Y X
A A

Which ammeters have the same reading as each other?

A V and W only
B V and Y only
C X and Y only
D V, W, X and Y

40 A 20 V power supply provides a current of 5.0 A for 1.0 minute.

How much energy does the power supply deliver?

A 4.0 J B 100 J C 240 J D 6000 J

© UCLES 2019 0653/21/O/N/19

417/693
Combined By Nesrine
2023-2017

Cambridge Assessment International Education


Cambridge International General Certificate of Secondary Education

COMBINED SCIENCE 0653/22


Paper 2 Multiple Choice (Extended) October/November 2019
45 minutes
Additional Materials: Multiple Choice Answer Sheet
Soft clean eraser
*0713601905*

Soft pencil (type B or HB is recommended)

READ THESE INSTRUCTIONS FIRST

Write in soft pencil.


Do not use staples, paper clips, glue or correction fluid.
Write your name, centre number and candidate number on the Answer Sheet in the spaces provided
unless this has been done for you.
DO NOT WRITE IN ANY BARCODES.

There are forty questions on this paper. Answer all questions. For each question there are four possible
answers A, B, C and D.
Choose the one you consider correct and record your choice in soft pencil on the separate Answer Sheet.

Read the instructions on the Answer Sheet very carefully.

Each correct answer will score one mark. A mark will not be deducted for a wrong answer.
Any rough working should be done in this booklet.
A copy of the Periodic Table is printed on page 16.
Electronic calculators may be used.

This document consists of 16 printed pages.

IB19 11_0653_22/2RP
© UCLES 2019 [Turn over

418/693
Combined By Nesrine
2
2023-2017

1 A biologist keeps a potted plant in a laboratory.

Which feature of the potted plant shows that it is a living organism?

A It grows larger over time.


B It has green leaves.
C The compost in the pot dries after he waters it.
D The stems contain xylem.

2 Which statement about human gametes is correct?

A Egg cells are much smaller than sperm cells.


B Egg cells are produced in larger numbers than sperm cells.
C Egg cells have a jelly coating that changes after fertilisation.
D The flagellum is an adaptive feature of an egg cell.

3 The diagram shows a cross section of a stem.

tissue 1

tissue 2

Which row shows the correct names and functions of the tissues?

tissue 1 tissue 2
name function name function

A phloem support only phloem transport only


B phloem transport only xylem support and transport
C xylem transport only phloem support and transport
D xylem support only xylem transport only

© UCLES 2019 0653/22/O/N/19

419/693
Combined By Nesrine
3
2023-2017

4 1 cm³ of substance X is added to 10 cm³ starch suspension and mixed. Food tests are carried out
immediately after mixing and again after an hour.

The results of the tests are shown in the table.

colour of solution colour of solution


test reagent
after mixing after one hour

Benedict’s solution blue orange


iodine solution blue / black brown

What is substance X?

A amylase
B protease
C lipase
D sugar

5 The diagram represents the human heart and four blood vessels.

Which blood vessel contains blood at the highest pressure?

B
C

D
A

© UCLES 2019 0653/22/O/N/19 [Turn over

420/693
Combined By Nesrine
4
2023-2017

6 The diagram shows an alveolus and a blood capillary.

2
1

4
3

Which two arrows represent gas exchange by diffusion only?

A 1 and 2 B 1 and 3 C 2 and 4 D 3 and 4

7 Which statement about aerobic respiration is correct?

A It exchanges gases through the walls of the alveoli.


B It expels carbon dioxide from the lungs.
C It only produces carbon dioxide and energy.
D It uses oxygen to release energy from glucose.

8 Nitrates in the soil are taken up by the roots of a plant.

What are the nitrates used to make?

A fat
B glucose
C protein
D starch

9 Which statement about sexual reproduction is always correct?

A It involves only one parent.


B It involves the fusion of nuclei.
C It produces genetically identical offspring.
D It takes place only in animals.

© UCLES 2019 0653/22/O/N/19

421/693
Combined By Nesrine
5
2023-2017

10 Which row gives the most suitable characteristics of a wind-pollinated flower?

pollen grains anthers stigma

A smooth few small


B smooth many large
C sticky few large
D sticky many small

11 A developing fetus is connected to its mother by an umbilical cord and placenta.

What is the function of the placenta?

A to allow the mixing of the mother’s blood with the blood of the fetus
B to exchange nutrients and waste
C to keep the fetus warm
D to stop the fetus from moving

12 The diagram shows a food web.

cat
sparrowhawk

thrush
robin

frog

snail slug
caterpillar

delphinium
nasturtium

cabbage

How could the frog be classed?

A second trophic level and secondary consumer


B second trophic level and tertiary consumer
C third trophic level and secondary consumer
D third trophic level and tertiary consumer

© UCLES 2019 0653/22/O/N/19 [Turn over

422/693
Combined By Nesrine
6
2023-2017

13 The table shows the possible effects of two processes on the concentration of two gases in the
atmosphere.

concentration of gases in atmosphere


process
carbon dioxide oxygen

1 combustion of fossil fuels decrease increase


2 combustion of fossil fuels increase decrease
3 deforestation decrease increase
4 deforestation increase decrease

Which rows show the effects of deforestation and combustion of fossil fuels on the concentration
of carbon dioxide and oxygen in the atmosphere?

A 1 and 3 B 1 and 4 C 2 and 3 D 2 and 4

14 The chromatogram of a black ink containing three dyes is shown.

solvent front

blue

purple
20 cm
16 cm
12 cm
yellow
baseline 4 cm

What is the Rf value of the purple ink?

A 0.2 B 0.4 C 0.6 D 1.67

15 A white solid X is formed when magnesium reacts with oxygen.

What is X?

A a compound
B a mixture
C an alloy
D an element

© UCLES 2019 0653/22/O/N/19

423/693
Combined By Nesrine
7
2023-2017

16 The fertiliser ammonium sulfate has the formula (NH4)2SO4.

How many atoms of each element are present in the formula?

number of number of number of number of


hydrogen atoms nitrogen atoms oxygen atoms sulfur atoms

A 4 1 1 1
B 4 2 4 1
C 8 1 4 1
D 8 2 4 1

17 Element X is a non-metal used in the treatment of the water supply.

It is made during the electrolysis of a metal salt.

What is the colour of X and at which electrode is it made?

colour electrode

A red anode
B red cathode
C yellow-green anode
D yellow-green cathode

18 The energy level diagram for a reaction is shown.

X
energy
Y

progress of reaction

Which statement is correct?

A X is the energy change of the reaction.


B Y is the activation energy of the reaction.
C The energy change of the reaction is larger than the activation energy of the reaction.
D The reaction is exothermic.

© UCLES 2019 0653/22/O/N/19 [Turn over

424/693
Combined By Nesrine
8
2023-2017

19 Dilute hydrochloric acid reacts with excess calcium carbonate.

The amount of carbon dioxide made in one minute is recorded.

The experiment is repeated using the same volume of more concentrated hydrochloric acid.

How does the volume of carbon dioxide collected in one minute and the frequency of collisions of
reacting particles change?

volume of frequency
carbon dioxide of collisions

A decreases decreases
B decreases increases
C increases decreases
D increases increases

20 Copper sulfate is a soluble salt.

How are pure crystals of copper sulfate prepared?

A Mix copper chloride solution with sodium sulfate solution, filter, rinse and dry.
B React copper oxide with excess dilute sulfuric acid, evaporate, cool, filter, rinse and dry.
C React excess copper carbonate, with dilute sulfuric acid, filter, evaporate, cool, filter, rinse
and dry.
D React excess copper with dilute sulfuric acid, filter, evaporate, cool, filter, rinse and dry.

21 Which row describes the reactivity and the electronic structure of a noble gas?

reactivity electronic structure

A reactive full outer shell


B reactive incomplete outer shell
C unreactive incomplete outer shell
D unreactive full outer shell

22 Which statement about alloys is correct?

A They are made from metals because metals are poor electrical conductors.
B They are mixtures of compounds that contain metals.
C They have all the same properties as the metals from which they are made.
D They have different properties to the metals from which they are made.

© UCLES 2019 0653/22/O/N/19

425/693
Combined By Nesrine
9
2023-2017

23 Which statement about the extraction of iron in a blast furnace is not correct?

A Carbon dioxide reduces iron oxide.


B Carbon monoxide is oxidised by iron oxide.
C Carbon reduces carbon dioxide.
D The high temperatures required are produced by reacting carbon with oxygen.

24 What is the composition of clean air?

A 78% nitrogen, 21% carbon dioxide and small amounts of other gases
B 78% nitrogen, 21% oxygen and small amounts of other gases
C 78% oxygen, 21% carbon dioxide and small amounts of other gases
D 78% oxygen, 21% nitrogen and small amounts of other gases

25 Which two gases cause an enhanced greenhouse effect when their concentrations in the
atmosphere increase?

A carbon monoxide and carbon dioxide


B carbon dioxide and methane
C methane and sulfur dioxide
D sulfur dioxide and carbon monoxide

26 Gasoline is one fraction obtained from petroleum.

Which row describes the boiling point of the compounds and the molecules in this fraction?

boiling point molecules


A they have different boiling points they contain different
numbers of carbon atoms
B they have different boiling points they contain the same
number of carbon atoms
C they have the same boiling point they contain different
numbers of carbon atoms
D they have the same boiling point they contain the same
number of carbon atoms

© UCLES 2019 0653/22/O/N/19 [Turn over

426/693
Combined By Nesrine
10
2023-2017

27 Which hydrocarbons belong to the same homologous series?

A C2H2, C2H4, C2H6


B CH4, C2H4, C3H4
C C2H4, C3H6, C4H8
D C2H4, C3H8, C4H10

28 A measuring cylinder contains liquid.

More liquid is now poured into the measuring cylinder.

The diagrams show the measuring cylinder before and after the liquid is poured into it.

10 10
9 9
cm3 8 cm3 8
7 7
6 6
5 5
4 4
3 3
2 2
1 1

before after

What volume of liquid is poured into the measuring cylinder?

A 3.5 cm3 B 4.0 cm3 C 4.5 cm3 D 8.0 cm3

© UCLES 2019 0653/22/O/N/19

427/693
Combined By Nesrine
11
2023-2017

29 The graph shows how the speed of a car changes with time. The car travels at constant speed,
then accelerates, and finally brakes to a stop.

speed

0
0 5 20 30
time / s

The car travels 60 m while it brakes to a stop.

What is the average speed of the car while it is braking?

A 3.0 m / s B 4.0 m / s C 6.0 m / s D 12 m / s

30 A spring obeys Hooke’s law until it reaches its limit of proportionality.

A load is hung from the spring. The load is gradually increased and the spring is stretched
beyond its limit of proportionality.

Which is the extension-load graph for the spring?

A B

extension extension

0 0
0 load 0 load

C D

extension extension

0 0
0 load 0 load

© UCLES 2019 0653/22/O/N/19 [Turn over

428/693
Combined By Nesrine
12
2023-2017

31 The diagram shows a load attached to a spring.

spring

load

The load is pulled down and then released so that it oscillates between point P (highest point)
and point Q (lowest point).

Which form of energy does the load have at point P?

A gravitational potential energy only


B kinetic energy only
C kinetic energy and gravitational potential energy
D neither kinetic energy nor gravitational potential energy

32 Liquids consist of molecules that are constantly moving.

Which row describes the motion of the molecules in a liquid and compares the forces between
them to the forces between molecules in a gas?

motion of molecules forces between molecules

A random stronger than in a gas


B random weaker than in a gas
C vibrating about fixed positions stronger than in a gas
D vibrating about fixed positions weaker than in a gas

© UCLES 2019 0653/22/O/N/19

429/693
Combined By Nesrine
13
2023-2017

33 A circular bowl in a room contains water.

Which two factors both ensure that the smallest quantity of water evaporates in a day?

temperature diameter
of room of bowl

A high large
B high small
C low large
D low small

34 In which process is thermal energy transferred by molecular vibrations?

A conduction
B convection
C evaporation
D radiation

35 The diagram shows light striking a plane mirror.

plane mirror

50°

ray of light
normal

What is the angle of reflection of the ray when it is reflected from the mirror?

A 40° B 50° C 80° D 100°

© UCLES 2019 0653/22/O/N/19 [Turn over

430/693
Combined By Nesrine
14
2023-2017

36 A student measures the speed of sound. He claps his hands and the sound reflects from a wall
that is 100 m away from him.

wall

student

electronic
timer

100 m

An electronic timer next to the student detects the echo of the sound 0.60 s after it is made.

Which calculation gives the speed of sound?

A 200 m / s B 200 m / s C 100 m / s D 100 m / s


0.30 0.60 0.60 1. 2

37 The diagram represents a sound wave travelling in air.

wave direction

1 2 3 4 5 6

Which numbered points are at the centre of a compression and which numbered points are at the
centre of a rarefaction?

centre of a centre of a
compression rarefaction

A 1 and 5 2 and 4
B 1 and 5 3 and 6
C 3 and 6 1 and 5
D 3 and 6 2 and 4

© UCLES 2019 0653/22/O/N/19

431/693
Combined By Nesrine
15
2023-2017

38 A piece of wire has a resistance of 8.0 Ω.

The length of the wire is doubled and the diameter of the wire is halved.

What is the new resistance of the wire?

A 2.0 Ω B 4.0 Ω C 8.0 Ω D 64 Ω

39 Four ammeters V, W, X and Y are connected in the circuit shown.

V W
A A

Y X
A A

Which ammeters have the same reading as each other?

A V and W only
B V and Y only
C X and Y only
D V, W, X and Y

40 There is a current I in a lamp. The potential difference across the lamp is V and the power
produced by the lamp is P.

In a second lamp, the current is 2I and the potential difference across it is V .


2

What is the power produced by this other lamp?

A P B P C P D 2P
4 2

Permission to reproduce items where third-party owned material protected by copyright is included has been sought and cleared where possible. Every
reasonable effort has been made by the publisher (UCLES) to trace copyright holders, but if any items requiring clearance have unwittingly been included, the
publisher will be pleased to make amends at the earliest possible opportunity.

To avoid the issue of disclosure of answer-related information to candidates, all copyright acknowledgements are reproduced online in the Cambridge
Assessment International Education Copyright Acknowledgements Booklet. This is produced for each series of examinations and is freely available to download
at www.cambridgeinternational.org after the live examination series.

Cambridge Assessment International Education is part of the Cambridge Assessment Group. Cambridge Assessment is the brand name of the University of
Cambridge Local Examinations Syndicate (UCLES), which itself is a department of the University of Cambridge.

© UCLES 2019 0653/22/O/N/19

432/693
Combined By Nesrine
2023-2017

Cambridge Assessment International Education


Cambridge International General Certificate of Secondary Education

COMBINED SCIENCE 0653/23


Paper 2 Multiple Choice (Extended) October/November 2019
45 minutes
Additional Materials: Multiple Choice Answer Sheet
Soft clean eraser
*9416393999*

Soft pencil (type B or HB is recommended)

READ THESE INSTRUCTIONS FIRST

Write in soft pencil.


Do not use staples, paper clips, glue or correction fluid.
Write your name, centre number and candidate number on the Answer Sheet in the spaces provided
unless this has been done for you.
DO NOT WRITE IN ANY BARCODES.

There are forty questions on this paper. Answer all questions. For each question there are four possible
answers A, B, C and D.
Choose the one you consider correct and record your choice in soft pencil on the separate Answer Sheet.

Read the instructions on the Answer Sheet very carefully.

Each correct answer will score one mark. A mark will not be deducted for a wrong answer.
Any rough working should be done in this booklet.
A copy of the Periodic Table is printed on page 20.
Electronic calculators may be used.

This document consists of 17 printed pages and 3 blank pages.

IB19 11_0653_23/2RP
© UCLES 2019 [Turn over

433/693
Combined By Nesrine
2
2023-2017

1 A biologist keeps a potted plant in a laboratory.

Which feature of the potted plant shows that it is a living organism?

A It grows larger over time.


B It has green leaves.
C The compost in the pot dries after he waters it.
D The stems contain xylem.

2 The diagram shows a ciliated cell.

cilia

Which row shows where ciliated cells are found in the human gas exchange system and their
correct function?

location of ciliated cells function of ciliated cells


move mucus move mucus
bronchi trachea
away from lungs towards lungs

A    
B    
C    
D    

3 What is the word equation for photosynthesis?

A carbon dioxide + oxygen → glucose + water

B carbon dioxide + water → glucose + oxygen

C glucose + oxygen → carbon dioxide + water

D glucose + water → carbon dioxide + oxygen

© UCLES 2019 0653/23/O/N/19

434/693
Combined By Nesrine
3
2023-2017

4 1 cm³ of substance X is added to 10 cm³ starch suspension and mixed. Food tests are carried out
immediately after mixing and again after an hour.

The results of the tests are shown in the table.

colour of solution colour of solution


test reagent
after mixing after one hour

Benedict’s solution blue orange


iodine solution blue / black brown

What is substance X?

A amylase
B protease
C lipase
D sugar

5 Which diagram is correctly labelled?

A B
trachea bronchus
bronchus trachea

rib rib

diaphragm diaphragm

C D
trachea trachea
bronchus rib

diaphragm bronchus

rib diaphragm

© UCLES 2019 0653/23/O/N/19 [Turn over

435/693
Combined By Nesrine
4
2023-2017

6 Which statement about aerobic respiration is correct?

A It exchanges gases through the walls of the alveoli.


B It expels carbon dioxide from the lungs.
C It only produces carbon dioxide and energy.
D It uses oxygen to release energy from glucose.

7 Which are absorbed from the alimentary canal into the blood?

1 fibre
2 glucose
3 vitamin C

A 1 and 2 only B 1 and 3 only C 2 and 3 only D 1, 2 and 3

8 Shoots were grown in different light conditions.

Some shoots had their tips covered with foil.

shoot tip direction of light

1 covered from all around


2 covered from one direction
3 uncovered from all around
4 uncovered from one direction

Which shoots would grow straight upwards?

A 1, 2 and 3 B 1 and 3 only C 3 and 4 only D 3 only

9 Which statement about sexual reproduction is always correct?

A It involves only one parent.


B It involves the fusion of nuclei.
C It produces genetically identical offspring.
D It takes place only in animals.

© UCLES 2019 0653/23/O/N/19

436/693
Combined By Nesrine
5
2023-2017

10 The diagram shows four pollen grains.

Which pollen grain is most likely to be distributed by an animal?

A B C D

11 Which statement about human gametes is correct?

A Sperm cells are much larger than egg cells.


B Sperm cells are produced in smaller numbers than egg cells.
C Sperm cells have a jelly coating that changes after fertilisation.
D The flagellum is an adaptive feature of a sperm cell.

12 The diagram shows a food web.

seal gull

octopus crab starfish

periwinkle limpet

seaweed

Which organism is found in more than one trophic level?

A crab
B gull
C octopus
D starfish

© UCLES 2019 0653/23/O/N/19 [Turn over

437/693
Combined By Nesrine
6
2023-2017

13 The flow diagram shows some stages in the eutrophication of a pond.

more ......1...... into water

more ......2......

more aerobic respiration by ......3......

reduced dissolved oxygen in water

Which words complete gaps 1, 2 and 3?

1 2 3

A decomposers nitrates producers


B decomposers producers nitrates
C nitrates producers decomposers
D nitrates decomposers producers

© UCLES 2019 0653/23/O/N/19

438/693
Combined By Nesrine
7
2023-2017

14 Chromatography is carried out on three solutions P, Q and R.

The chromatogram obtained is shown.

solvent front

base line
P Q R

Which statement is not correct?

A P contains at least two substances.


B Q contains the substance with the highest Rf value.
C R is insoluble in the solvent.
D P, Q and R together may contain only three substances.

15 Which substance is a single compound?

A air
B oxygen
C petroleum
D water

16 The fertiliser ammonium sulfate has the formula (NH4)2SO4.

How many atoms of each element are present in the formula?

number of number of number of number of


hydrogen atoms nitrogen atoms oxygen atoms sulfur atoms

A 4 1 1 1
B 4 2 4 1
C 8 1 4 1
D 8 2 4 1

© UCLES 2019 0653/23/O/N/19 [Turn over

439/693
Combined By Nesrine
8
2023-2017

17 Element X is a non-metal used in the treatment of the water supply.

It is made during the electrolysis of a metal salt.

What is the colour of X and at which electrode is it made?

colour electrode

A red anode
B red cathode
C yellow-green anode
D yellow-green cathode

18 An energy level diagram for a reaction is shown.

reactants
energy

products

progress of reaction

Which row describes the energy transfer and the type of energy change for this reaction?

energy transfer energy change

A energy is absorbed by reactants endothermic


B energy is absorbed by reactants exothermic
C energy is released to surroundings endothermic
D energy is released to surroundings exothermic

© UCLES 2019 0653/23/O/N/19

440/693
Combined By Nesrine
9
2023-2017

19 Calcium carbonate reacts with 50 cm3 hydrochloric acid.

The carbon dioxide produced is collected in a gas syringe.

The experiment is done four times using concentrated or dilute hydrochloric acid and using 5 g
calcium carbonate in powder or lump form.

Which experiment takes the longest time to collect 10 cm3 of gas?

calcium carbonate hydrochloric acid

A lumps concentrated
B lumps dilute
C powder concentrated
D powder dilute

20 The equation for a reaction is shown.

CuO + CO → Cu + CO2

Which statement about this reaction is correct?

A CO acts as a reducing agent.


B CO2 is reduced.
C Cu is oxidised.
D CuO acts as a reducing agent.

21 Copper sulfate is a soluble salt which is prepared by reacting insoluble copper oxide with dilute
sulfuric acid.

Which statement about the preparation of copper sulfate crystals is not correct?

A After the reaction, the mixture is filtered and copper sulfate solution is collected.
B Excess copper oxide is used to ensure that all the acid is used up.
C The final solution is heated so that all the water boils off.
D The mixture of copper oxide and dilute sulfuric acid is heated to speed up the reaction.

22 Which statement about alloys is correct?

A They are made from metals because metals are poor electrical conductors.
B They are mixtures of compounds that contain metals.
C They have all the same properties as the metals from which they are made.
D They have different properties to the metals from which they are made.

© UCLES 2019 0653/23/O/N/19 [Turn over

441/693
Combined By Nesrine
10
2023-2017

23 Which equation does not represent a reaction that takes place in the blast furnace?

A C + O2 → CO2

B C + CO2 → 2CO

C 2Fe + CO2 → 2FeO + C

D Fe2O3 + 3CO → 2Fe + 3CO2

24 Which row describes the percentage composition of clean air?

carbon dioxide nitrogen noble gases oxygen

A less than 1 78 less than 1 21


B less than 1 78 21 less than 1
C 21 less than 1 less than 1 78
D 78 less than 1 less than 1 21

25 Which two gases cause an enhanced greenhouse effect when their concentrations in the
atmosphere increase?

A carbon monoxide and carbon dioxide


B carbon dioxide and methane
C methane and sulfur dioxide
D sulfur dioxide and carbon monoxide

© UCLES 2019 0653/23/O/N/19

442/693
Combined By Nesrine
11
2023-2017

26 A simple fractionating column is shown.

fraction 1

fraction 2

fraction 3

fraction 4
petroleum

Which statement about the fractions is correct?

A Fraction 1 contains compounds with the highest boiling points.


B Fraction 2 contains larger hydrocarbon molecules than fraction 3.
C Fraction 3 is more viscous than fraction 4.
D Fraction 4 is the least flammable.

27 What is a typical property of alkanes?

A They are catalysts.


B They burn in air.
C They can be neutralised.
D They react endothermically.

© UCLES 2019 0653/23/O/N/19 [Turn over

443/693
Combined By Nesrine
12
2023-2017

28 The graph shows how the speed of a car changes with time. The car travels at constant speed,
then accelerates, and finally brakes to a stop.

speed

0
0 5 20 30
time / s

The car travels 60 m while it brakes to a stop.

What is the average speed of the car while it is braking?

A 3.0 m / s B 4.0 m / s C 6.0 m / s D 12 m / s

29 Which of these bodies has a resultant force acting on it?

A a book at rest on a table


B a car travelling up a hill in a straight line at constant speed
C a football moving upwards freely after being kicked
D a parachutist descending vertically at constant speed

© UCLES 2019 0653/23/O/N/19

444/693
Combined By Nesrine
13
2023-2017

30 The force acting on a spring is gradually increased from 0 N.

The spring eventually passes its limit of proportionality.

Which graph shows how the extension of the spring changes as the force increases?

A B

extension extension

0 0
0 force 0 force

C D

extension extension

0 0
0 force 0 force

31 Some energy resources are less reliable than others.

Which type of power station cannot produce electricity at all times?

A coal-fired power station


B geothermal power station
C hydroelectric power station
D solar power station

32 Which statement about the molecules in a gas is correct?

A They are closer together than those in solids.


B They are further apart than those in liquids.
C They are not free to move around.
D They are packed together in a regular pattern.

© UCLES 2019 0653/23/O/N/19 [Turn over

445/693
Combined By Nesrine
14
2023-2017

33 The equipment shown is used to demonstrate convection in air. Point X is labelled.

burning
candle

Which row describes and explains the movement of the air at X?

movement of air at X explanation

A downwards air becomes less dense when heated


B downwards air becomes more dense when heated
C upwards air becomes less dense when heated
D upwards air becomes more dense when heated

34 What type of wave is a sound wave and in which direction do air particles vibrate as the wave
passes through the air?

type of wave direction of vibration

A longitudinal parallel to wave direction


B longitudinal perpendicular to wave direction
C transverse parallel to wave direction
D transverse perpendicular to wave direction

35 A boy plays a series of musical notes of increasing frequency on a violin. As the frequency of the
note increases, he plays the notes more loudly.

How do the amplitude and the wavelength of the sound waves change?

amplitude wavelength

A decreases decreases
B decreases increases
C increases decreases
D increases increases

© UCLES 2019 0653/23/O/N/19

446/693
Combined By Nesrine
15
2023-2017

36 The diagram shows light striking a plane mirror.

plane mirror

50°

ray of light
normal

What is the angle of reflection of the ray when it is reflected from the mirror?

A 40° B 50° C 80° D 100°

37 A student measures the speed of sound. He claps his hands and the sound reflects from a wall
that is 100 m away from him.

wall

student

electronic
timer

100 m

An electronic timer next to the student detects the echo of the sound 0.60 s after it is made.

Which calculation gives the speed of sound?

A 200 m / s B 200 m / s C 100 m / s D 100 m / s


0.30 0.60 0.60 1. 2

38 A piece of wire has a resistance of 8.0 Ω.

The length of the wire is doubled and the diameter of the wire is halved.

What is the new resistance of the wire?

A 2.0 Ω B 4.0 Ω C 8.0 Ω D 64 Ω

© UCLES 2019 0653/23/O/N/19 [Turn over

447/693
Combined By Nesrine
16
2023-2017

39 Four ammeters V, W, X and Y are connected in the circuit shown.

V W
A A

Y X
A A

Which ammeters have the same reading as each other?

A V and W only
B V and Y only
C X and Y only
D V, W, X and Y

40 There is a current I in a resistor and a potential difference V across it.

Which equation gives the energy E transferred by the resistor in a time t ?

I V t
A E= B E= C E= D E = IVt
Vt It VI

© UCLES 2019 0653/23/O/N/19

448/693
Combined By Nesrine
2023-2017

Cambridge International Examinations


Cambridge International General Certificate of Secondary Education

COMBINED SCIENCE 0653/02


*0123456789*

Paper 2 Multiple Choice (Extended) For Examination from 2019


SPECIMEN PAPER
45 minutes
Additional Materials: Multiple Choice Answer Sheet
Soft clean eraser
Soft pencil (type B or HB is recommended)

READ THESE INSTRUCTIONS FIRST

Write in soft pencil.


Do not use staples, paper clips, glue or correction fluid.
Write your name, Centre number and candidate number on the Answer Sheet in the spaces provided unless
this has been done for you.
DO NOT WRITE IN ANY BARCODES.

There are forty questions on this paper. Answer all questions. For each question there are four possible
answers A, B, C and D.
Choose the one you consider correct and record your choice in soft pencil on the separate Answer Sheet.

Read the instructions on the Answer Sheet very carefully.

Each correct answer will score one mark. A mark will not be deducted for a wrong answer.
Any rough working should be done in this booklet.
A copy of the Periodic Table is printed on page 18.
Electronic calculators may be used.

This document consists of 18 printed pages.

© UCLES 2016 [Turn over

449/693
Combined By Nesrine
2023-2017
2

1 The width of the plant cell in the diagram is 30 mm when it is magnified by a microscope
(magnification shown in brackets).

(×1000)

What is the actual width of the cell?

A 0.003 mm

B 0.03 mm

C 0.3 mm

D 30 mm

2 The diagram shows an experiment using an uncooked potato. The skin of the potato was removed
as shown.

concentrated
sugar solution

potato
skin has been
water removed

Which diagram shows the result of the experiment after 24 hours?

A B C D

© UCLES 2016 0653/02/SP/19

450/693
Combined By Nesrine
2023-2017
3

3 The graph shows how the rate of an enzyme-controlled reaction between starch and amylase
changes with temperature. What explains the shape of the graph within the temperature range
marked X?

rate of
reaction

temperature X

A The higher temperature breaks down the enzyme’s substrate.

B The higher temperature decreases the kinetic energy of the enzyme.

C The higher temperature denatures the enzyme.

D The higher temperature helps the enzyme to function as a biological catalyst.

4 A healthy plant has been in the light. A leaf is taken from the plant, decolourised and then tested
with iodine solution.

What colour does the iodine solution change to?

A black

B brick red

C pale blue

D yellow

5 A man reduces the amount of salt, saturated fat and fibre in his diet.

How could these changes affect the risk of developing the following conditions?

constipation coronary heart disease


A increased risk increased risk
B increased risk reduced risk
C reduced risk increased risk
D reduced risk reduced risk

© UCLES 2016 0653/02/SP/19 [Turn over

451/693
Combined By Nesrine
2023-2017
4

6 The diagram shows a section through the human heart.

Which structure is the ventricle?

D A

C B

7 One of the effects of tobacco smoke on the gas exchange system is that haemoglobin carries
oxygen around the body less efficiently.

Which component of tobacco smoke is responsible for this effect?

A carbon monoxide

B nicotine

C smoke particles

D tar

© UCLES 2016 0653/02/SP/19

452/693
Combined By Nesrine
2023-2017
5

8 The diagram shows a light from a lamp shining from one direction only onto a shoot.

The shoot was left in the light for 48 hours.

Which diagram shows how the shoot would look after 48 hours under the influence of auxin?

A B C D

© UCLES 2016 0653/02/SP/19 [Turn over

453/693
Combined By Nesrine
2023-2017
6

9 The diagram shows a section through a flower.

In which parts of the flower are pollen grains produced and received?

pollen grains pollen grains


produced received
A Q P
B Q S
C S P
D S Q

10 Which row in the table describes female gametes compared to male gametes?

number
size mobility
produced
A larger fewer less mobile
B larger greater more mobile
C smaller fewer more mobile
D smaller greater less mobile

© UCLES 2016 0653/02/SP/19

454/693
Combined By Nesrine
2023-2017
7

11 A food chain is shown below. The numbers show the amount of energy, measured in kJ, that
passes from one organism to another.

500 kJ 75 kJ
grass rabbit fox

Calculate how much energy is lost from this food chain at X.

A 25 kJ

B 75 kJ

C 425 kJ

D 575 kJ

12 The diagram shows a food web.

fox

hawk snake shrew

bird frog spider

beetle

wood beetle slug caterpillar

tree daisy grass

How many producers and how many consumers are shown in this food web?

number of number of
producers consumers
A 3 3
B 3 11
C 11 3
D 13 1

© UCLES 2016 0653/02/SP/19 [Turn over

455/693
Combined By Nesrine
2023-2017
8

13 Eutrophication is one of the consequences of water pollution.

Some of the stages of eutrophication are listed in the wrong order.

1 Increased aerobic respiration by decomposers.

2 Death of organisms requiring dissolved oxygen in water.

3 Increased availability of nitrate.

4 Increased growth of producers.

What is the correct order of these stages of eutrophication?

A 1→4→2→3

B 1→3→2→4

C 3→4→1→2

D 3→1→4→2

14 Which row in the table correctly describes the three substances?

air brass iron

A compound compound element


B element mixture compound
C mixture element compound
D mixture mixture element

15 Sodium chloride is an ionic crystalline substance with a high melting point.

Which statement describes the oppositely charged ions in sodium chloride crystals?

A There is strong attraction between them and a random arrangement.

B There is strong attraction between them and a regular arrangement.

C There is weak attraction between them and a random arrangement.

D There is weak attraction between them and a regular arrangement.

© UCLES 2016 0653/02/SP/19

456/693
Combined By Nesrine
2023-2017
9

16 What is the dot-and-cross diagram for a molecule of ethene?

H H H H
C C C C
H H H H
A B

H H H H
H C C H H C C H
H H H H
C D

17 Hexane, C6H14, burns in an excess of oxygen, forming carbon dioxide and water.

What is the equation for this reaction?

A C6H14 + 9O2 → 6CO2 + 7H2O


B C6H14 + 19O2 → 12CO2 + 14H2O

C 2C6H14 + 19O2 → 6CO2 + 7H2O

D 2C6H14 + 19O2 → 12CO2 + 14H2O

18 Molten copper bromide is electrolysed. The products are collected and cooled to room temperature.

Which row describes the cooled products?

anode product cathode product


A brown liquid reddish-brown solid
B reddish-brown solid brown liquid
C colourless gas reddish-brown solid
D silvery solid colourless gas

© UCLES 2016 0653/02/SP/19 [Turn over

457/693
Combined By Nesrine
2023-2017
10

19 Which statement describes bond breaking?

A It is an endothermic process which results in a temperature decrease.

B It is an endothermic process which results in a temperature increase.

C It is an exothermic process which results in a temperature decrease.

D It is an exothermic process which results in a temperature increase.

20 Marble and chalk are two forms of calcium carbonate.

Equal masses of marble lumps and powdered chalk are added to dilute hydrochloric acid.

dilute hydrochloric acid dilute hydrochloric acid

marble lumps powdered chalk

The marble takes longer than the chalk to fully react.

Why is this?

A Marble is more reactive than chalk.

B Marble is more soluble than chalk.

C The marble has a smaller surface area than chalk.

D The marble is more basic than chalk.

21 Which row in the table describes an alkali?

solubility in reaction with an


water acid
A insoluble does not react
B insoluble reacts
C soluble does not react
D soluble reacts

© UCLES 2016 0653/02/SP/19

458/693
Combined By Nesrine
2023-2017
11

22 The table shows the results of tests on an aqueous solution of compound X.

test result
blue litmus paper turns red
aqueous silver nitrate white precipitate formed

What is X?

A HCl

B HNO3
C NaCl

D NaOH

23 Metal X reacts rapidly with cold water.

Metal Y does not react with dilute hydrochloric acid.

Which row describes the reactivities of X and Y?

reactivity of metal reactivity compared to hydrogen

A X is more reactive than Y X is less reactive than hydrogen


B X is more reactive than Y X is more reactive than hydrogen
C Y is more reactive than X Y is less reactive than hydrogen
D Y is more reactive than X Y is more reactive than hydrogen

24 Carbon is used to extract copper from copper oxide.

Which statement about the process is correct?

A Carbon is more reactive than copper.

B Carbon oxidises copper oxide.

C Copper is more reactive than carbon.

D Copper oxide reduces carbon.

25 Which statement describes the reactivity of potassium?

A It forms negative ions very easily.

B It forms positive ions more readily than lithium does.

C It is displaced from its salts by copper.

D It is displaced from its salts by sodium.

© UCLES 2016 0653/02/SP/19 [Turn over

459/693
Combined By Nesrine
2023-2017
12

26 Which row in the table describes the method of extraction of aluminium, and the reason for using
this method?

method of
reason
extraction
A heat with carbon aluminium is less reactive than carbon
B heat with carbon aluminium is more reactive than carbon
C electrolysis aluminium is more reactive than carbon
D electrolysis aluminium is resistant to corrosion

27 Petroleum is separated into useful fractions by fractional distillation.

Which row in the table describes the properties of the compounds in the fraction obtained from the
top of the fractionating column?

intermolecular
boiling point molecular size attractive
forces
A high large weak
B high small strong
C low large strong
D low small weak

28 The speed-time graph for a car journey is shown.

speed 2

3
1 4

0
0 time

During which two parts of the journey is the car moving at constant speed?

A 1 and 3

B 1 and 5

C 2 and 4

D 3 and 5

© UCLES 2016 0653/02/SP/19

460/693
Combined By Nesrine
2023-2017
13

29 The strength of the gravitational field on the Moon is less than the strength of the gravitational field
on Earth.

An object has mass M and weight W on the Moon.

What is the mass and what is the weight of the object on Earth?

mass weight
A M more than W
B M W
C more than M more than W
D more than M W

30 The equation for Hooke’s Law relates the extension of a spring to the load applied to it.

In an experiment, loads are applied to a spring and the spring extends. The table shows the
results.

load / N 0 12 24 36 48 60 72
length of spring / cm 15 18 21 24 27 30 33

What is the value of the spring constant k for this spring, and has the spring been loaded past its
limit of proportionality?

spring constant k loaded past limit of


N/ cm proportionality?
A 4.0 no
B 4.0 yes
C 12 no
D 12 yes

31 A ball rolls along a frictionless, horizontal track at an initial speed of 8.0 m / s. It reaches a sloping
section of the track and continues to roll up the slope.

8.0 m / s

What is the maximum vertical height that the ball reaches up the slope?

The acceleration of free fall g is 10 m / s2. Ignore air resistance.

A 0.80 m

B 3.2 m

C 32 m

D 80 m

© UCLES 2016 0653/02/SP/19 [Turn over

461/693
Combined By Nesrine
2023-2017
14

32 In which pair of energy resources is the Sun not the original source of energy?

A coal and oil

B geothermal and nuclear

C hydroelectricity and natural gas

D wind and waves

33 A gas is trapped in a sealed container of constant volume. The gas is heated.

What effect does this have on the gas molecules?

A The average distance between the molecules increases.

B The average mass of the molecules increases.

C The molecules expand.

D The molecules move more quickly.

34 The diagram shows an ice cube surrounded by air. The ice cube cools the air around it. This
cooling changes the density of the air and causes the air to move.

ice cube

Which row in the table shows the change in density of the air and the direction in which the air
moves?

density change direction of movement


A decreases downwards
B decreases upwards
C increases downwards
D increases upwards

35 A water wave with a wavelength of 2.0 cm moves a distance of 900 cm in 1.0 minute.

What is its frequency?

A 7.5 Hz

B 30 Hz

C 450 Hz

D 1800 Hz

© UCLES 2016 0653/02/SP/19

462/693
Combined By Nesrine
2023-2017
15

36 Which ray diagram shows how an image is formed by a magnifying glass?

A B

C D

37 Which electromagnetic waves are found immediately either side of the visible region of the
electromagnetic spectrum?

A infra-red and ultraviolet

B microwaves and infra-red

C microwaves and X-rays

D ultraviolet and X-rays

© UCLES 2016 0653/02/SP/19 [Turn over

463/693
Combined By Nesrine
2023-2017
16

38 The diagrams represent two sound waves. The diagrams are drawn to the same scale.

time time

P Q

Which statement correctly compares the pitch and the loudness of the two sounds?

A P has a higher pitch and is louder than Q.

B P has a higher pitch and is quieter than Q.

C P has a lower pitch and is louder than Q.

D P has a lower pitch and is quieter than Q.

39 The circuit shows a 3.0 V battery connected to a resistor of resistance R. There is a current I in the
resistor.

3.0 V

Which row in the table shows a possible pair of values of I and R?

I /A R/Ω

A 1.5 1.5
B 1.5 2.0
C 4.0 12
D 6.0 2.0

© UCLES 2016 0653/02/SP/19

464/693
Combined By Nesrine
2023-2017
17

40 The diagram shows two resistors connected to a battery. The currents in different parts of the
circuit are indicated.

6.0 A 6.0 A
P

2.0 A

What is the current at point P?

A 2.0 A

B 4.0 A

C 8.0 A

D 14 A

© UCLES 2016 0653/02/SP/19

465/693
Combined By Nesrine
© UCLES 2016 2023-2017
Examinations Syndicate (UCLES), which is itself a department of the University of Cambridge.
Cambridge International Examinations is part of the Cambridge Assessment Group. Cambridge Assessment is the brand name of University of Cambridge Local

publisher will be pleased to make amends at the earliest possible opportunity.


reasonable effort has been made by the publisher (UCLES) to trace copyright holders, but if any items requiring clearance have unwittingly been included, the
Permission to reproduce items where third-party owned material protected by copyright is included has been sought and cleared where possible. Every
The Periodic Table of Elements
Group
I II III IV V VI VII VIII
1 2

H He
hydrogen helium
Key 1 4
3 4 atomic number 5 6 7 8 9 10

Li Be atomic symbol B C N O F Ne
lithium beryllium name boron carbon nitrogen oxygen fluorine neon
7 9 relative atomic mass 11 12 14 16 19 20
11 12 13 14 15 16 17 18
Na Mg Al Si P S Cl Ar
sodium magnesium aluminium silicon phosphorus sulfur chlorine argon
23 24 27 28 31 32 35.5 40
19 20 21 22 23 24 25 26 27 28 29 30 31 32 33 34 35 36
K Ca Sc Ti V Cr Mn Fe Co Ni Cu Zn Ga Ge As Se Br Kr
potassium calcium scandium titanium vanadium chromium manganese iron cobalt nickel copper zinc gallium germanium arsenic selenium bromine krypton
39 40 45 48 51 52 55 56 59 59 64 65 70 73 75 79 80 84
0653/02/SP/19

37 38 39 40 41 42 43 44 45 46 47 48 49 50 51 52 53 54

Rb Sr Y Zr Nb Mo Tc Ru Rh Pd Ag Cd In Sn Sb Te I Xe

18
rubidium strontium yttrium zirconium niobium molybdenum technetium ruthenium rhodium palladium silver cadmium indium tin antimony tellurium iodine xenon
85 88 89 91 93 96 – 101 103 106 108 112 115 119 122 128 127 131
55 56 57–71 72 73 74 75 76 77 78 79 80 81 82 83 84 85 86
Cs Ba lanthanoids
Hf Ta W Re Os Ir Pt Au Hg Tl Pb Bi Po At Rn
caesium barium hafnium tantalum tungsten rhenium osmium iridium platinum gold mercury thallium lead bismuth polonium astatine radon
133 137 178 181 184 186 190 192 195 197 201 204 207 209 – – –
87 88 89–103 104 105 106 107 108 109 110 111 112 114 116

Fr Ra actinoids
Rf Db Sg Bh Hs Mt Ds Rg Cn Fl Lv
francium radium rutherfordium dubnium seaborgium bohrium hassium meitnerium darmstadtium roentgenium copernicium flerovium livermorium
– – – – – – – – – – – – –

57 58 59 60 61 62 63 64 65 66 67 68 69 70 71
lanthanoids La Ce Pr Nd Pm Sm Eu Gd Tb Dy Ho Er Tm Yb Lu
lanthanum cerium praseodymium neodymium promethium samarium europium gadolinium terbium dysprosium holmium erbium thulium ytterbium lutetium
139 140 141 144 – 150 152 157 159 163 165 167 169 173 175
89 90 91 92 93 94 95 96 97 98 99 100 101 102 103
actinoids Ac Th Pa U Np Pu Am Cm Bk Cf Es Fm Md No Lr
actinium thorium protactinium uranium neptunium plutonium americium curium berkelium californium einsteinium fermium mendelevium nobelium lawrencium
– 232 231 238 – – – – – – – – – – –

The volume of one mole of any gas is 24 dm3 at room temperature and pressure (r.t.p.).

466/693
Combined By Nesrine
2023-2017

Cambridge International Examinations


Cambridge International General Certificate of Secondary Education

COMBINED SCIENCE 0653/22


Paper 2 Multiple Choice (Extended) February/March 2018
45 minutes
Additional Materials: Multiple Choice Answer Sheet
Soft clean eraser
*8468472230*

Soft pencil (type B or HB is recommended)

READ THESE INSTRUCTIONS FIRST

Write in soft pencil.


Do not use staples, paper clips, glue or correction fluid.
Write your name, Centre number and candidate number on the Answer Sheet in the spaces provided
unless this has been done for you.
DO NOT WRITE IN ANY BARCODES.

There are forty questions on this paper. Answer all questions. For each question there are four possible
answers A, B, C and D.
Choose the one you consider correct and record your choice in soft pencil on the separate Answer Sheet.

Read the instructions on the Answer Sheet very carefully.

Each correct answer will score one mark. A mark will not be deducted for a wrong answer.
Any rough working should be done in this booklet.
A copy of the Periodic Table is printed on page 16.
Electronic calculators may be used.

This document consists of 15 printed pages and 1 blank page.

IB18 03_0653_22/2RP
© UCLES 2018 [Turn over

467/693
Combined By Nesrine
2
2023-2017

1 Which is a characteristic of all living organisms?

A breathing
B eating
C egestion
D movement

2 The diagram shows a typical animal cell.

What are the functions of structures X, Y and Z?

X Y Z
A traps light contains controls entry and
genetic material exit of materials
B traps light site of chemical provides support
reactions
C contains site of chemical controls entry and
genetic material reactions exit of materials
D contains controls entry and provides support
genetic material exit of materials

3 The statements explain the activity of a human enzyme as the temperature increases from 20 °C
to 50 °C. The statements are in the wrong order.

1 The enzyme is working at its optimum rate.


2 The kinetic energy of the enzyme molecules begins to increase.
3 The enzyme begins to change shape.
4 The enzyme is completely denatured.

What is the correct order of the statements?

A 1→3→2→4

B 1→4→3→2

C 2→1→3→4

D 3→2→4→1

© UCLES 2018 0653/22/F/M/18

468/693
Combined By Nesrine
3
2023-2017

4 Tests were performed on four samples of food. The results are shown in the table.

Which food contains protein only?

results of food tests


Benedict’s test biuret test iodine test

A blue blue blue/black


B blue purple brown
C red blue blue/black
D red purple brown

5 A student is investigating how light affects photosynthesis.

gas measuring cylinder

beaker

water glass funnel

aquatic plant
X Y

The student shines a light from point Y and measures the volume of gas produced in five
minutes.

Which gas is produced and how does the rate of gas production change when the light is moved
from Y to X?

rate of gas
gas produced
production

A carbon dioxide decreases


B carbon dioxide increases
C oxygen decreases
D oxygen increases

© UCLES 2018 0653/22/F/M/18 [Turn over

469/693
Combined By Nesrine
4
2023-2017

6 The diagram shows the alimentary canal.

Which label shows where bile is stored?

7 The graph shows the rate of water loss from a plant during daylight hours.

rate of water
loss / g per hour

Y
0
0 time / hours

What could cause the change in the rate of water loss between point X and point Y?

A The air becomes cooler.


B The air becomes drier.
C The day becomes sunnier.
D The stomata open wider.

© UCLES 2018 0653/22/F/M/18

470/693
Combined By Nesrine
5
2023-2017

8 What are possible causes of coronary heart disease?

exercise smoking stress

A   
B   
C   
D   

9 During aerobic respiration of glucose, oxygen is used up and water is produced.

How many molecules of oxygen are used and how many molecules of water are produced when
one molecule of glucose is respired?

number of number of
molecules of molecules of
oxygen used water produced

A 1 1
B 1 6
C 6 1
D 6 6

10 Light shines on a shoot tip from the direction shown.

light
X Y

After three days the shoot tip has bent towards the light.

What is the reason for this change?

A Auxin moves away from the light causing cell elongation in area Y.
B Auxin moves away from the light preventing cell elongation in area Y.
C Auxin moves towards the light causing cell elongation in area X.
D Auxin moves towards the light preventing cell elongation in area X.

© UCLES 2018 0653/22/F/M/18 [Turn over

471/693
Combined By Nesrine
6
2023-2017

11 How do male gametes compare with female gametes?

move
size
independently

A larger 
B larger 
C smaller 
D smaller 

12 The diagram shows part of the carbon cycle.

Which arrow represents a process that releases oxygen into the atmosphere?

carbon dioxide
in the air
B

A carbohydrates in
dead organic matter

C D
death
carbohydrates
in animals death

carbohydrates
in green plants

13 Eutrophication occurs after fertiliser is washed into a lake.

What is not true of eutrophication?

A Algae population in the lake decreases.


B Bacteria population in the lake increases.
C Nitrate concentration in the lake increases.
D Oxygen concentration in the lake decreases.

© UCLES 2018 0653/22/F/M/18

472/693
Combined By Nesrine
7
2023-2017

14 Which statement about atoms and molecules is correct?

A Atoms gain or lose electrons to become molecules.


B Atoms of the same element contain the same number of molecules.
C Molecules are the simplest unit of an atom.
D Molecules contain atoms which are covalently bonded.

15 Which diagram represents a mixture of two elements?

A B C D

16 Which statement about atoms and ions is not correct?

A A chlorine atom loses one electron to obtain a noble gas electronic structure.
B A magnesium atom has two valency electrons.
C A sodium ion, Na+, has eight electrons in its outer shell.
D Oxygen atoms and oxide ions each have two occupied electron shells.

17 Which substance contains a multiple covalent bond?

A hydrogen
B methane
C nitrogen
D water

18 Which equation represents the reaction at the cathode during the electrolysis of aqueous
copper(II) chloride?

A 2Cl – → Cl 2 + 2e−

B Cu2+ + 2e− → Cu

C 2H+ + 2e− → H2

D 4OH– → O2 + 2H2O + 4e−

© UCLES 2018 0653/22/F/M/18 [Turn over

473/693
Combined By Nesrine
8
2023-2017

19 In the reaction between an acid and a metal, the rate of reaction decreases as the reaction
proceeds.

A student suggests three reasons why the rate of this reaction decreases.

1 The concentration of the acid decreases as it gets used up.


2 The energy needed to break bonds is used up as the product forms.
3 The surface area of the metal increases as it gets smaller.

Which reasons are correct?

A 1, 2 and 3 B 1 and 2 only C 1 only D 3 only

20 The equation shows the reaction of copper oxide with carbon.

copper oxide + carbon → copper + carbon dioxide

In the reaction, the carbon is the ««1«« agent and is ««2«« during the reaction.

Which words complete gaps 1 and 2?

1 2

A oxidising oxidised
B oxidising reduced
C reducing oxidised
D reducing reduced

21 Magnesium, magnesium oxide and magnesium carbonate are insoluble in water.

Which method is used to make pure crystals of magnesium sulfate?

A Add an excess of magnesium carbonate to dilute sulfuric acid, filter and evaporate the filtrate
to dryness.
B Add an excess of magnesium oxide to dilute sulfuric acid and leave overnight to crystallise.
C Add magnesium oxide to an excess of dilute sulfuric acid and evaporate to dryness.
D Add magnesium ribbon to an excess of dilute sulfuric acid, filter and evaporate to dryness.

© UCLES 2018 0653/22/F/M/18

474/693
Combined By Nesrine
9
2023-2017

22 Solid X is warmed with dilute sodium hydroxide. A gas, which turns moist red litmus paper to
blue, is given off.

Dilute hydrochloric acid is added to solid X. A gas, which turns limewater cloudy, is given off.

What is X?

A ammonium carbonate
B ammonium chloride
C sodium carbonate
D sodium chloride

23 Astatine is at the bottom of Group VII of the Periodic Table.

What happens if astatine is added to aqueous potassium chloride?

A A black precipitate is formed.


B Chlorine is formed.
C No reaction takes place.
D The colour of the solution becomes darker.

24 The noble gases make up Group VIII of the Periodic Table.

Which statement is correct?

A Argon exists as non-bonded atoms.


B Krypton is very poisonous.
C Neon burns in pure oxygen with a red flame.
D The chemical formula of helium is He2.

25 Why is drinking water treated with chlorine?

A to improve the taste


B to kill bacteria
C to remove colour
D to remove insoluble impurities

© UCLES 2018 0653/22/F/M/18 [Turn over

475/693
Combined By Nesrine
10
2023-2017

26 A gas that causes climate change is formed during the extraction of iron from iron ore.

Which solution reacts with this gas?

A aqueous sodium chloride


B hydrochloric acid
C dilute sulfuric acid
D limewater

27 Which statement about the rusting of iron is correct?

A Iron becomes lighter when it rusts.


B Iron is reduced when it rusts.
C Rusting is a reaction involving iron, oxygen and water.
D Rusting is a reaction involving iron and water only.

28 Diagrams 1, 2 and 3 each show either a distance-time graph or a speed-time graph.

distance speed speed

0 0 0
0 time 0 time 0 time
diagram 1 diagram 2 diagram 3

Which of the diagrams represent the motion of an object moving with a non-zero constant
acceleration?

A 1 and 3 B 1 only C 2 only D 3 only

© UCLES 2018 0653/22/F/M/18

476/693
Combined By Nesrine
11
2023-2017

29 Two objects on Earth each have a mass of 20 kg.

One object is moved to a planet larger than Earth. The other object is moved into deep space.

What is the mass of the objects in these new positions?

mass of object on mass of object in


the other planet / kg deep space / kg

A 20 0
B 20 20
C more than 20 0
D more than 20 20

30 A spring that obeys Hooke’s law has no load attached to it. The length of the spring is 8.0 cm and
it has a spring constant k of 5.0 N / cm.

A load is now hung from the spring, and the length of the spring increases to 18 cm. The limit of
proportionality is not reached.

What is the weight of the load?

A 2.0 N B 40 N C 50 N D 90 N

31 Which energy resource is non-renewable?

A geothermal energy
B hydroelectric energy
C nuclear energy
D wave energy

32 A force of 20 N does 10 J of work when it moves an object through a distance d in the direction of
the force.

What is distance d?

A 0.50 m B 2.0 m C 10 m D 200 m

© UCLES 2018 0653/22/F/M/18 [Turn over

477/693
Combined By Nesrine
12
2023-2017

33 The molecules of a substance are far apart and move at high speed in straight lines until they hit
something.

The temperature of the substance is changed and this causes the molecules to move more
quickly.

What is the state of the substance, and how has its temperature changed?

state of how temperature


substance has changed

A gas decreased
B gas increased
C liquid decreased
D liquid increased

34 The diagram shows a cold gas in a tank. The tank contains a heater that is switched off.

cold gas
heater

The heater is now switched on.

What happens to the density of the gas near the heater, and in which direction does the heated
gas start to move?

direction of
density
movement

A decreases downwards
B decreases upwards
C increases downwards
D increases upwards

© UCLES 2018 0653/22/F/M/18

478/693
Combined By Nesrine
13
2023-2017

35 The diagram represents a wave.

6.0 cm
distance

8.0 cm

What is the wavelength of the wave?

A 3.0 cm B 4.0 cm C 6.0 cm D 8.0 cm

36 Which diagram shows how a ray of light passes from air into a glass block, and shows the angle
of incidence labelled i ?

A B C D

i i
i i

glass glass
block block

37 A sound wave travels in substance P. The sound wave then passes into a different substance Q
and the speed of the sound wave decreases.

What are possible substances for P and Q?

P Q

A air steel
B air water
C water air
D water steel

© UCLES 2018 0653/22/F/M/18 [Turn over

479/693
Combined By Nesrine
14
2023-2017

38 A lamp is labelled 12 V, 25 W.

How much electrical energy does the lamp convert in 4.0 minutes when lit at its normal
brightness?

A 100 J B 1200 J C 6000 J D 72 000 J

39 Why is the electricity supply to a house fitted with a fuse?

A to increase the current in the circuit


B to increase the resistance of the circuit
C to maintain a constant current in the circuit
D to prevent overheating of the cables in the circuit

40 The diagrams show four circuits, each containing an ammeter and two lamps with different
resistances.

Which circuit shows an ammeter with a reading equal to the current in each lamp?

A B C D

A
A A
A

© UCLES 2018 0653/22/F/M/18

480/693
Combined By Nesrine
2023-2017

Cambridge International Examinations


Cambridge International General Certificate of Secondary Education

COMBINED SCIENCE 0653/21


Paper 2 Multiple Choice (Extended) May/June 2018
45 minutes
Additional Materials: Multiple Choice Answer Sheet
Soft clean eraser
*7603036318*

Soft pencil (type B or HB is recommended)

READ THESE INSTRUCTIONS FIRST

Write in soft pencil.


Do not use staples, paper clips, glue or correction fluid.
Write your name, Centre number and candidate number on the Answer Sheet in the spaces provided
unless this has been done for you.
DO NOT WRITE IN ANY BARCODES.

There are forty questions on this paper. Answer all questions. For each question there are four possible
answers A, B, C and D.
Choose the one you consider correct and record your choice in soft pencil on the separate Answer Sheet.

Read the instructions on the Answer Sheet very carefully.

Each correct answer will score one mark. A mark will not be deducted for a wrong answer.
Any rough working should be done in this booklet.
A copy of the Periodic Table is printed on page 16.
Electronic calculators may be used.

This document consists of 16 printed pages.

IB18 06_0653_21/2RP
© UCLES 2018 [Turn over

481/693
Combined By Nesrine
2
2023-2017

1 Most cars burn fossil fuels to release energy for their movement.

Which characteristic of living organisms is similar to this?

A excretion
B growth
C nutrition
D respiration

2 Which structure controls the passage of substances into and out of a cell?

A cell membrane
B cell wall
C nucleus
D vacuole

© UCLES 2018 0653/21/M/J/18

482/693
Combined By Nesrine
3
2023-2017

3 Amylase is an enzyme that digests starch.

Identical mixtures of starch and amylase are kept at different temperatures.

The percentage of starch digested in 20 minutes is recorded.

The results are shown in the graph.

100

percentage of
starch digested
in 20 minutes 50

0
0 10 20 30 40 50 60 70
temperature / °C

The mixtures that were kept at 0 °C and 70 °C are then kept at a temperature of 40 °C for one
hour.

What are the results after this hour?

percentage of starch digested


sample originally sample originally
kept at 0 °C kept at 70 °C

A 0 0
B 0 100
C 100 0
D 100 100

4 Which two chemical substances are required for photosynthesis?

A carbon dioxide and glucose


B glucose and oxygen
C oxygen and water
D water and carbon dioxide

© UCLES 2018 0653/21/M/J/18 [Turn over

483/693
Combined By Nesrine
4
2023-2017

5 The diagram shows a plant in a controlled environment.

controlled
environment

damp
soil

The list gives three ways in which the environment can be altered.

1 humidity increased
2 light intensity increased
3 temperature increased

Which changes will cause an increase in the rate of transpiration of the plant?

1 2 3

A   
B   
C   
D   

© UCLES 2018 0653/21/M/J/18

484/693
Combined By Nesrine
5
2023-2017

6 The diagram shows a section through the human heart.

valve 4
valve 3

valve 2 valve 1

What happens to the valves as blood is being pumped to the lungs?

valve 1 valve 2 valve 3 valve 4

A closed closed open closed


B closed closed open open
C open open closed closed
D open open closed open

7 What is the maximum number of carbon dioxide molecules produced when four glucose
molecules are used in aerobic respiration?

A 6 B 12 C 24 D 48

8 Why is tar in cigarette smoke a harmful chemical?

A It causes addiction.
B It causes cancer.
C It makes platelets stick together.
D It sticks to blood vessel walls.

© UCLES 2018 0653/21/M/J/18 [Turn over

485/693
Combined By Nesrine
6
2023-2017

9 The diagram shows a seedling with its shoot horizontal.

shoot P

Gravity is the stimulus acting on the seedling.

Where will the greatest concentrations of auxin be found in the shoot and what effect will this
have on the rate of cell elongation?

greatest concentration effect of auxin on rate


of auxin of cell elongation

A P increases
B P decreases
C Q increases
D Q decreases

10 Why are many flowers brightly coloured?

A to attract insects to pollinate the flower


B to encourage birds to eat insects on the flower
C to frighten animals away from the flower
D to help with wind-pollination of the flower

11 The diagrams show the human male and female reproductive systems.

2 4

Which numbered parts produce gametes?

A 1 and 3 B 1 and 4 C 2 and 3 D 2 and 4

© UCLES 2018 0653/21/M/J/18

486/693
Combined By Nesrine
7
2023-2017

12 At which trophic level in a food chain does transpiration occur?

A trophic level 1
B trophic level 2
C trophic level 3
D trophic level 4

13 Which statement about greenhouse gases is correct?

A They are caused by acid rain.


B They are produced by photosynthesis.
C They generate heat when they react with sunlight.
D They reduce the loss of heat from the Earth.

14 The diagrams represent different substances.

P Q R S T

Which row describes the substances?

only separate mixture of atoms


only molecules
atoms and molecules

A P Q S
B Q T R
C T P R
D T Q P

15 Which row describes the method used to obtain salt from salt water and petrol from petroleum?

salt from salt water petrol from petroleum

A crystallisation distillation
B crystallisation fractional distillation
C filtration distillation
D filtration fractional distillation

© UCLES 2018 0653/21/M/J/18 [Turn over

487/693
Combined By Nesrine
8
2023-2017

16 Some changes are listed.

1 boiling
2 decomposing
3 evaporating
4 oxidising

Which changes are physical changes?

A 1 and 2 B 1 and 3 C 2 and 4 D 3 and 4

17 The formula of a sodium ion is Na+. The formula of an oxide ion is O2–.

What is the formula of sodium oxide?

A NaO B NaO2 C Na2O D Na2O3

18 The diagram shows apparatus for electrolysis.

Only one label is correct.

electrode
+ –

cathode anode

electrolyte

Which label on the diagram is correct?

A anode
B cathode
C electrode
D electrolyte

© UCLES 2018 0653/21/M/J/18

488/693
Combined By Nesrine
9
2023-2017

19 Molten sodium chloride is electrolysed.

Which row shows the product and the equation for the reaction at the stated electrode?

equation for reaction


electrode product
at the electrode

A negative chlorine 2Cl – → Cl 2 + 2e–


B negative sodium Na+ + e– → Na
C positive chlorine 2Cl 2– → Cl 2 + 4e–
D positive sodium Na+ + 2e– → Na

20 Which change must take place in an endothermic reaction?

A Bubbles of gas are released.


B The mass decreases.
C The temperature decreases.
D The temperature increases.

21 The volume of carbon dioxide produced in a reaction is measured.

The results are plotted on a graph.

At which time is the rate of reaction greatest?

volume of
carbon dioxide

A B C D
time

© UCLES 2018 0653/21/M/J/18 [Turn over

489/693
Combined By Nesrine
10
2023-2017

22 Four reaction equations involving oxides of iron are listed.

1 Fe2O3 + 2Al → Al 2O3 + 2Fe

2 4FeO + O2 → 2Fe2O3

3 FeO + H2 → Fe + H2O

4 2FeO + C → 2Fe + CO2

Which statement is correct?

A In reaction 1, Al is being oxidised by Fe2O3.


B In reaction 2, FeO is being reduced by O2.
C In reaction 3, H2 is being reduced by FeO.
D In reaction 4, FeO is being oxidised by carbon.

23 Excess solid copper oxide is added to warm dilute sulfuric acid and stirred.

How are pure copper sulfate crystals obtained from the mixture?

A distil the mixture → wash the solid → dry the solid

B filter the mixture → distil the filtrate

C filter the mixture → heat the filtrate to saturation → cool and filter → dry the solid

D heat the mixture to saturation → cool and filter → dry the solid

24 Which statement describes the metallic character of elements in Period 2 of the Periodic Table?

A The first and last elements in this period are metals.


B The metallic elements are in the centre of this period.
C The metallic elements are on the left of this period.
D The metallic elements are on the right of this period.

25 The arrangement of particles in four substances is shown.

Which diagram represents an alloy?

A B C D

+ – + –
– + – +
+ – + –

© UCLES 2018 0653/21/M/J/18

490/693
Combined By Nesrine
11
2023-2017

26 Which process produces a gas that contributes to climate change?

A the electrolysis of molten lead(II) bromide

B the reaction of calcium with water


C the reaction of copper oxide with dilute sulfuric acid
D the thermal decomposition of calcium carbonate

27 Butane, C4H10, and decane, C10H22, are alkanes.

Molecules of decane are larger than molecules of butane.

Which row describes the properties of decane compared to those of butane?

intermolecular
boiling point
attractive forces

A higher stronger
B higher weaker
C lower stronger
D lower weaker

28 Diagrams 1, 2 and 3 show two distance-time graphs and one speed-time graph.

distance distance speed

0 0 0
0 time 0 time 0 time
diagram 1 diagram 2 diagram 3

Which of the diagrams represent the motion of an object that is accelerating?

A 1 and 2 B 1 only C 2 only D 3 only

© UCLES 2018 0653/21/M/J/18 [Turn over

491/693
Combined By Nesrine
12
2023-2017

29 A student takes an object from one place on Earth to another place where the gravitational field is
weaker.

Which property of the object has a smaller value at the second location?

A density
B mass
C volume
D weight

30 An unstretched spring obeys Hooke’s law and has a length of 10 cm. A load with a mass of 2.0 kg
is hung from it, and its length becomes 14 cm.

The load is now increased to 6.0 kg, and the new length of the spring is Y. The limit of
proportionality is not reached.

10 cm
14 cm
Y

2.0 kg

6.0 kg

What is Y?

A 22 cm B 26 cm C 30 cm D 42 cm

31 Electricity can be obtained from the energy in water behind a hydroelectric dam.

Is this energy resource renewable, and in which form is its energy stored?

renewable form of energy

A no chemical
B no gravitational potential
C yes chemical
D yes gravitational potential

© UCLES 2018 0653/21/M/J/18

492/693
Combined By Nesrine
13
2023-2017

32 A machine does 6.0 kJ of useful work in 20 minutes.

How much useful power does it produce?

A 0.30 W B 5.0 W C 120 W D 300 W

33 On a summer’s day, hot air rises above hot roofs.

What is the name of this process?

A concentration
B condensation
C conduction
D convection

34 A radio station transmits signals at a frequency of 9.1 × 107 Hz.

What is the wavelength of the radio waves?

Light travels at a speed of 3.0 × 108 m / s.

A 0.30 m B 0.33 m C 3.0 m D 3.3 m

35 The diagram shows a ray of light as it enters a glass block.

Which labelled angle is the angle of refraction?

ray of
light

A
B
D
C glass block

© UCLES 2018 0653/21/M/J/18 [Turn over

493/693
Combined By Nesrine
14
2023-2017

36 Optical fibres are used to transmit telephone conversations.

What is transmitted by the optical fibres?

A electromagnetic waves with constant amplitude


B pulses of electromagnetic waves
C pulses of sound waves
D sound waves with constant amplitude

37 A student writes two sentences about sound waves.

‘A sound wave travels through the air as compressions and ......X...... .’

‘The air at the compressions has a different ......Y...... from the air at ......X...... .’

What are the missing words, X and Y?

X Y

A rarefactions density
B rarefactions state
C refractions density
D refractions state

38 A negative ion X is close to a positive ion and another negative ion. Electrical forces act on ion X
because of the charges in the other two ions.

Which diagram shows the directions of the two forces acting on ion X?

A B C D
positive
ion + + + +
– ion X – ion X – ion X – ion X
– – – –
negative
ion

39 A 240 V mains supply causes a current of 4.00 A in a heater.

How much energy is transferred in the heater in 5.00 minutes?

A 192 J B 4800 J C 18 000 J D 288 000 J

© UCLES 2018 0653/21/M/J/18

494/693
Combined By Nesrine
15
2023-2017

40 An electric kettle has the following label attached to its base.

current: 7.5 A
frequency: 50 Hz
power: 1800 W
voltage: 240 V

How is an appropriate fuse for the kettle labelled?

A 10 A B 60 Hz C 2000 W D 300 V

Permission to reproduce items where third-party owned material protected by copyright is included has been sought and cleared where possible. Every
reasonable effort has been made by the publisher (UCLES) to trace copyright holders, but if any items requiring clearance have unwittingly been included, the
publisher will be pleased to make amends at the earliest possible opportunity.

To avoid the issue of disclosure of answer-related information to candidates, all copyright acknowledgements are reproduced online in the Cambridge
International Examinations Copyright Acknowledgements Booklet. This is produced for each series of examinations and is freely available to download at
www.cie.org.uk after the live examination series.

Cambridge International Examinations is part of the Cambridge Assessment Group. Cambridge Assessment is the brand name of University of Cambridge Local
Examinations Syndicate (UCLES), which is itself a department of the University of Cambridge.

© UCLES 2018 0653/21/M/J/18

495/693
Combined By Nesrine
2023-2017

Cambridge International Examinations


Cambridge International General Certificate of Secondary Education

COMBINED SCIENCE 0653/22


Paper 2 Multiple Choice (Extended) May/June 2018
45 minutes
Additional Materials: Multiple Choice Answer Sheet
Soft clean eraser
*3054014789*

Soft pencil (type B or HB is recommended)

READ THESE INSTRUCTIONS FIRST

Write in soft pencil.


Do not use staples, paper clips, glue or correction fluid.
Write your name, Centre number and candidate number on the Answer Sheet in the spaces provided
unless this has been done for you.
DO NOT WRITE IN ANY BARCODES.

There are forty questions on this paper. Answer all questions. For each question there are four possible
answers A, B, C and D.
Choose the one you consider correct and record your choice in soft pencil on the separate Answer Sheet.

Read the instructions on the Answer Sheet very carefully.

Each correct answer will score one mark. A mark will not be deducted for a wrong answer.
Any rough working should be done in this booklet.
A copy of the Periodic Table is printed on page 20.
Electronic calculators may be used.

This document consists of 17 printed pages and 3 blank pages.

IB18 06_0653_22/4RP
© UCLES 2018 [Turn over

496/693
Combined By Nesrine
2
2023-2017

1 Which pair of structures is found in a typical plant cell but not in a typical animal cell?

A cell membrane and chloroplasts


B cell membrane and cytoplasm
C cell wall and chloroplasts
D cell wall and cytoplasm

2 Amylase is an enzyme that digests starch.

Identical mixtures of starch and amylase are kept at different temperatures.

The percentage of starch digested in 20 minutes is recorded.

The results are shown in the graph.

100

percentage of
starch digested
in 20 minutes 50

0
0 10 20 30 40 50 60 70
temperature / °C

The mixtures that were kept at 0 °C and 70 °C are then kept at a temperature of 40 °C for one
hour.

What are the results after this hour?

percentage of starch digested


sample originally sample originally
kept at 0 °C kept at 70 °C

A 0 0
B 0 100
C 100 0
D 100 100

© UCLES 2018 0653/22/M/J/18

497/693
Combined By Nesrine
3
2023-2017

3 Which two chemical substances are required for photosynthesis?

A carbon dioxide and glucose


B glucose and oxygen
C oxygen and water
D water and carbon dioxide

4 The diagram shows a section through the human heart.

valve 4
valve 3

valve 2 valve 1

What happens to the valves as blood is being pumped to the lungs?

valve 1 valve 2 valve 3 valve 4

A closed closed open closed


B closed closed open open
C open open closed closed
D open open closed open

5 Two students exercised for five minutes. Immediately afterwards, each student measured her
own pulse rate.

The pulse rate for one student was lower than the pulse rate for the other student.

Why might her pulse rate be lower?

A Her exercise was harder.


B Her heart rate was higher.
C She had secreted more adrenaline.
D She takes regular exercise.

© UCLES 2018 0653/22/M/J/18 [Turn over

498/693
Combined By Nesrine
4
2023-2017

6 What is not a feature of a gas exchange surface in animals?

A moist
B permeable
C small surface area
D thin

7 What is the maximum number of carbon dioxide molecules produced when four glucose
molecules are used in aerobic respiration?

A 6 B 12 C 24 D 48

8 Adrenaline is sometimes called the ‘fight or flight’ hormone.

Which is an effect of adrenaline that helps prepare the body to fight or to take flight when
frightened?

A It increases blood glucose concentration.


B It increases the rate of digestion.
C It maintains a constant body temperature.
D It slows down the heart rate.

9 Which row describes sexual reproduction?

number of offspring genetically involves zygote


parents identical to parents production

A 1  
B 1  
C 2  
D 2  

10 Which features are correct for a wind-pollinated flower?

nectar petals

A absent small
B absent large
C present small
D present large

© UCLES 2018 0653/22/M/J/18

499/693
Combined By Nesrine
5
2023-2017

11 In a food chain, the energy transferred from the first trophic level to the second trophic level is
greater than the energy transferred from the second trophic level to the third trophic level.

Which process is not a reason for this difference?

A egestion
B excretion
C movement
D photosynthesis

12 Fertilisers help crop plants grow.

How might the overuse of fertilisers damage the ecosystem?

A Animals feeding on the crop plants will die.


B It will lead to flooding
C Nutrients will not be available to plants.
D Waterways will be polluted.

13 What are the effects of acid rain?

damage to
damage to trees
limestone buildings

A  
B  
C  
D  

© UCLES 2018 0653/22/M/J/18 [Turn over

500/693
Combined By Nesrine
6
2023-2017

14 The diagrams represent different substances.

P Q R S T

Which row describes the substances?

only separate mixture of atoms


only molecules
atoms and molecules

A P Q S
B Q T R
C T P R
D T Q P

15 Which method is used to separate a mixture of two liquids?

A chromatography
B crystallisation
C filtration
D fractional distillation

16 Which process involves a physical change?

A adding magnesium to nitric acid


B burning methane
C evaporating petroleum
D rusting iron

17 An ion of element X has 8 protons, 8 neutrons and 10 electrons.

An ion of element Y has 11 protons, 12 neutrons and 10 electrons.

What is the formula of the ionic compound formed between X and Y?

A XY B XY2 C X2Y D X2Y2

© UCLES 2018 0653/22/M/J/18

501/693
Combined By Nesrine
7
2023-2017

18 The diagram shows apparatus for electrolysis.

Only one label is correct.

electrode
+ –

cathode anode

electrolyte

Which label on the diagram is correct?

A anode
B cathode
C electrode
D electrolyte

19 When aqueous copper chloride is electrolysed, ions move to the electrodes.

Which ions move to the positive electrode?

A chloride
B copper
C hydrogen
D oxide

20 Which change must take place in an endothermic reaction?

A Bubbles of gas are released.


B The mass decreases.
C The temperature decreases.
D The temperature increases.

© UCLES 2018 0653/22/M/J/18 [Turn over

502/693
Combined By Nesrine
8
2023-2017

21 Magnesium reacts with dilute hydrochloric acid in four experiments.

The same mass of magnesium and the same volume and concentration of the acid are used.

Which conditions produce the greatest rate of reaction?

magnesium temperature / °C

A powder 10
B powder 20
C ribbon 10
D ribbon 20

22 The equation for the combustion of ethane is shown.

2C2H6 + 7O2 → 4CO2 + 6H2O

Which statement about this reaction is correct?

A Ethane is oxidised because it gains oxygen.


B Ethane is reduced because carbon to hydrogen bonds are broken.
C It is endothermic because chemical energy is transformed to heat.
D It is exothermic because heat is transformed to chemical energy.

23 Zinc sulfate is a soluble salt.

It is made by reacting excess zinc with dilute sulfuric acid.

Which row describes how zinc sulfate is obtained from the mixture after the reaction has
finished?

step 1 step 2 step 3


A evaporate all wash the solid dry the solid
of the water that is left
B filter the mixture collect the residue wash and
from the filter paper dry the residue
C filter the mixture saturate the solution filter, wash and
and crystallise dry the crystals
D saturate the solution allow the solution filter, wash and
to crystallise dry the crystals

© UCLES 2018 0653/22/M/J/18

503/693
Combined By Nesrine
9
2023-2017

24 Part of the Periodic Table is shown.

I II III IV V VI VII VIII

V W X

Which element forms an anion and which element forms a cation?

forms an anion forms a cation

A V W
B V X
C W V
D X W

25 Which statement about metals is not correct?

A Calcium is extracted from its ore by electrolysis.


B Compounds containing metals are ionic and the metal is always the positive ion.
C Magnesium is more reactive than iron because it forms positive ions less easily than iron.
D Some metals react with dilute hydrochloric acid to give hydrogen.

26 Which process produces a gas that contributes to climate change?

A the electrolysis of molten lead(II) bromide

B the reaction of calcium with water


C the reaction of copper oxide with dilute sulfuric acid
D the thermal decomposition of calcium carbonate

© UCLES 2018 0653/22/M/J/18 [Turn over

504/693
Combined By Nesrine
10
2023-2017

27 The fractional distillation of petroleum is shown.

fraction
refinery gas

gasoline

paraffin

light gas oil

diesel

lubricating oil

petroleum

bitumen

Which fraction contains molecules with the largest intermolecular attractive forces?

A bitumen
B diesel
C gasoline
D refinery gas

28 The graph shows how the speed of a bicycle varies with time.

At which labelled point is the acceleration of the bicycle the greatest?

A
speed

B
D
C
0
0 time

© UCLES 2018 0653/22/M/J/18

505/693
Combined By Nesrine
11
2023-2017

29 An unstretched spring obeys Hooke’s law and has a length of 10 cm. A load with a mass of 2.0 kg
is hung from it, and its length becomes 14 cm.

The load is now increased to 6.0 kg, and the new length of the spring is Y. The limit of
proportionality is not reached.

10 cm
14 cm
Y

2.0 kg

6.0 kg

What is Y?

A 22 cm B 26 cm C 30 cm D 42 cm

30 A train is travelling along a straight, horizontal track at constant speed.

The work done by the train is recorded as it travels through a measured distance.

Which quantity can be calculated using only these two pieces of information?

A force exerted by the train


B speed of the train
C time taken to travel this distance
D weight of the train

31 The molecules of a substance are close together. They are vibrating and constantly changing
places within the substance.

The substance now loses energy and this causes it to change state.

Which change of state has occurred?

A gas to liquid
B liquid to gas
C liquid to solid
D solid to liquid

© UCLES 2018 0653/22/M/J/18 [Turn over

506/693
Combined By Nesrine
12
2023-2017

32 On a summer’s day, hot air rises above hot roofs.

What is the name of this process?

A concentration
B condensation
C conduction
D convection

33 A wave has a frequency of 120 Hz and a wavelength of 50 cm.

What is the speed of the wave?

A 2.4 m / s B 60 m / s C 240 m / s D 6000 m / s

34 The diagram shows a ray of light as it enters a glass block.

Which labelled angle is the angle of refraction?

ray of
light

A
B
D
C glass block

© UCLES 2018 0653/22/M/J/18

507/693
Combined By Nesrine
13
2023-2017

35 Light travels along a glass optical fibre by total internal reflection. The light enters the fibre at right
angles to the end.

Which diagram shows the path of the light in the fibre?

A B

C D

36 Gamma rays, radio waves and visible light all travel in a vacuum.

How do the speeds of these waves compare?

A Gamma rays have the highest speed.


B Radio waves have the highest speed.
C The waves all have the same speed.
D Visible light waves have the highest speed.

© UCLES 2018 0653/22/M/J/18 [Turn over

508/693
Combined By Nesrine
14
2023-2017

37 A student writes two sentences about sound waves.

‘A sound wave travels through the air as compressions and ......X...... .’

‘The air at the compressions has a different ......Y...... from the air at ......X...... .’

What are the missing words, X and Y?

X Y

A rarefactions density
B rarefactions state
C refractions density
D refractions state

38 A negative ion X is close to a positive ion and another negative ion. Electrical forces act on ion X
because of the charges in the other two ions.

Which diagram shows the directions of the two forces acting on ion X?

A B C D
positive
ion + + + +
– ion X – ion X – ion X – ion X
– – – –
negative
ion

© UCLES 2018 0653/22/M/J/18

509/693
Combined By Nesrine
15
2023-2017

39 A lamp can be dimmed or switched off. Its circuit is protected by a fuse.

Which diagram shows this circuit?

A B

C D

© UCLES 2018 0653/22/M/J/18 [Turn over

510/693
Combined By Nesrine
16
2023-2017

40 The diagram shows three identical resistors R1, R2 and R3 connected to a battery.

The current in R1 is I. The potential difference (p.d.) across R2 is V.

I R1

R2 R3

Which row gives the current in R3 and the p.d. across R3?

current in R3 p.d. across R3


I V
A
2 2

B I V
2
V
C I
2
D I V

© UCLES 2018 0653/22/M/J/18

511/693
Combined By Nesrine
2023-2017

Cambridge International Examinations


Cambridge International General Certificate of Secondary Education

COMBINED SCIENCE 0653/23


Paper 2 Multiple Choice (Extended) May/June 2018
45 minutes
Additional Materials: Multiple Choice Answer Sheet
Soft clean eraser
*0756182778*

Soft pencil (type B or HB is recommended)

READ THESE INSTRUCTIONS FIRST

Write in soft pencil.


Do not use staples, paper clips, glue or correction fluid.
Write your name, Centre number and candidate number on the Answer Sheet in the spaces provided
unless this has been done for you.
DO NOT WRITE IN ANY BARCODES.

There are forty questions on this paper. Answer all questions. For each question there are four possible
answers A, B, C and D.
Choose the one you consider correct and record your choice in soft pencil on the separate Answer Sheet.

Read the instructions on the Answer Sheet very carefully.

Each correct answer will score one mark. A mark will not be deducted for a wrong answer.
Any rough working should be done in this booklet.
A copy of the Periodic Table is printed on page 16.
Electronic calculators may be used.

This document consists of 16 printed pages.

IB18 06_0653_23/2RP
© UCLES 2018 [Turn over

512/693
Combined By Nesrine
2
2023-2017

1 Which structure found in plant cells traps light energy for photosynthesis?

A cell wall
B chloroplast
C nucleus
D vacuole

2 Amylase is an enzyme that digests starch.

Identical mixtures of starch and amylase are kept at different temperatures.

The percentage of starch digested in 20 minutes is recorded.

The results are shown in the graph.

100

percentage of
starch digested
in 20 minutes 50

0
0 10 20 30 40 50 60 70
temperature / °C

The mixtures that were kept at 0 °C and 70 °C are then kept at a temperature of 40 °C for one
hour.

What are the results after this hour?

percentage of starch digested


sample originally sample originally
kept at 0 °C kept at 70 °C

A 0 0
B 0 100
C 100 0
D 100 100

© UCLES 2018 0653/23/M/J/18

513/693
Combined By Nesrine
3
2023-2017

3 Which substance is broken down to produce lactic acid during the manufacture of yoghurt?

A glucose
B lactose
C protein
D starch

4 Which two chemical substances are required for photosynthesis?

A carbon dioxide and glucose


B glucose and oxygen
C oxygen and water
D water and carbon dioxide

5 A person eats a diet high in carbohydrate and fat and low in fibre.

What might be the long-term consequences of this diet?

constipation obesity starvation

A   
B   
C   
D   

© UCLES 2018 0653/23/M/J/18 [Turn over

514/693
Combined By Nesrine
4
2023-2017

6 The diagram shows a section through the human heart.

valve 4
valve 3

valve 2 valve 1

What happens to the valves as blood is being pumped to the lungs?

valve 1 valve 2 valve 3 valve 4

A closed closed open closed


B closed closed open open
C open open closed closed
D open open closed open

7 The diagram shows cross-sections of two different blood vessels.

P Q

What type of blood vessel is Q and what is the relative blood pressure in Q compared with P?

relative blood
blood vessel Q
pressure in Q

A artery higher
B artery lower
C vein higher
D vein lower

© UCLES 2018 0653/23/M/J/18

515/693
Combined By Nesrine
5
2023-2017

8 What is the maximum number of carbon dioxide molecules produced when four glucose
molecules are used in aerobic respiration?

A 6 B 12 C 24 D 48

9 What are the functions of the cilia and mucus in the gas exchange system?

cilia mucus

A make mucus trap pathogens


B make mucus move cilia
C move mucus trap pathogens
D move mucus move cilia

10 The diagram shows a seedling with its root horizontal.

Y root

Gravity is the stimulus acting on the seedling.

Where will the greatest concentration of auxin be found in the root and what effect will this have
on the rate of cell elongation?

greatest effect of auxin


concentration on rate of cell
of auxin elongation

A X decreases
B X increases
C Y decreases
D Y increases

© UCLES 2018 0653/23/M/J/18 [Turn over

516/693
Combined By Nesrine
6
2023-2017

11 The table shows some features of flowers.

Which features are typical of wind-pollinated flowers?

petals position of anther nectaries

A large inside flower absent


B large outside flower present
C small inside flower present
D small outside flower absent

12 What is a disadvantage of breast-feeding compared with bottle-feeding using formula milk?

A difficult to measure the amount of milk given


B milk contains antibodies
C milk is always immediately available
D milk is at the optimum temperature

13 The diagram shows how energy flows through a food chain.

Sun Y Y Y

X
Z Z
producer herbivore carnivore

What are the main energy transfers shown by arrows X, Y and Z?

X Y Z

A chemical → chemical chemical → heat chemical → light


B chemical → chemical chemical → light chemical → light
C light → chemical chemical → heat chemical → chemical
D light → chemical chemical → light chemical → chemical

© UCLES 2018 0653/23/M/J/18

517/693
Combined By Nesrine
7
2023-2017

14 The diagrams represent different substances.

P Q R S T

Which row describes the substances?

only separate mixture of atoms


only molecules
atoms and molecules

A P Q S
B Q T R
C T P R
D T Q P

15 A mixture of salt water and sand is filtered.

Which statement is correct?

A The salt and the sand are trapped by the filter paper.
B The salt is dissolved in the water and passes through the filter paper.
C The sand is insoluble in water and passes through the filter paper.
D The sand is trapped by the filter paper and pure water is obtained.

16 Which process is not a chemical change?

A the electrolysis of molten lead bromide


B the fractional distillation of petroleum
C the oxidation of copper
D the rusting of iron

17 Chromium oxide contains chromium ions, Cr3+, and oxide ions, O2–.

What is the formula of chromium oxide?

A CrO B CrO2 C Cr2O3 D Cr3O2

© UCLES 2018 0653/23/M/J/18 [Turn over

518/693
Combined By Nesrine
8
2023-2017

18 The diagram shows apparatus for electrolysis.

Only one label is correct.

electrode
+ –

cathode anode

electrolyte

Which label on the diagram is correct?

A anode
B cathode
C electrode
D electrolyte

19 Molten calcium chloride is electrolysed.

What happens at the negative electrode?

A Calcium atoms lose electrons to form calcium ions.


B Calcium ions gain electrons to form calcium atoms.
C Chloride ions lose electrons to form chlorine molecules.
D Chlorine molecules gain electrons to form chloride ions.

20 Which change must take place in an endothermic reaction?

A Bubbles of gas are released.


B The mass decreases.
C The temperature decreases.
D The temperature increases.

© UCLES 2018 0653/23/M/J/18

519/693
Combined By Nesrine
9
2023-2017

21 Dilute hydrochloric acid is reacted with calcium carbonate at 20 °C.

The reaction is repeated at 30 °C.

Which statement about the second reaction is correct?

A It is faster because there are fewer collisions per second between reacting particles.
B It is faster because there are more collisions per second between reacting particles.
C It is slower because there are fewer collisions per second between reacting particles.
D It is slower because there are more collisions per second between reacting particles.

22 Which reaction is a redox reaction?

A 2HCl + CaCO3 → CaCl 2 + CO2 + H2O

B AgNO3 + KCl → AgCl + KNO3

C Ca(OH)2 + CO2 → CaCO3 + H2O

D 2H2 + O2 → 2H2O

23 What is the trend shown by the elements across a period in the Periodic Table, from left to right?

A metals → non-metals

B metals → non-metals → metals

C non-metals → metals

D non-metals → metals → non-metals

24 Which diagram represents the arrangement of atoms in an alloy?

A B C D

25 Which reaction does not take place in the blast furnace?

A calcium carbonate → calcium oxide + carbon dioxide

B calcium oxide + silicon dioxide → calcium silicate

C iron oxide + carbon monoxide → iron + carbon dioxide

D iron oxide + carbon dioxide → iron + carbon monoxide

© UCLES 2018 0653/23/M/J/18 [Turn over

520/693
Combined By Nesrine
10
2023-2017

26 Which process produces a gas that contributes to climate change?

A the electrolysis of molten lead(II) bromide

B the reaction of calcium with water


C the reaction of copper oxide with dilute sulfuric acid
D the thermal decomposition of calcium carbonate

27 Which statement about hydrocarbons is not correct?

A Alkenes are made by cracking alkanes.


B Butene decolourises bromine.
C C2H4 is an alkene.
D Ethanol is an alkane.

28 The diagram shows a speed-time graph for a falling object.

20
speed
m/s
10

0
0 1.0 2.0
time / s

How far does the object fall in 2.0 seconds?

A 5.0 m B 20 m C 30 m D 40 m

© UCLES 2018 0653/23/M/J/18

521/693
Combined By Nesrine
11
2023-2017

29 An unstretched spring obeys Hooke’s law and has a length of 10 cm. A load with a mass of 2.0 kg
is hung from it, and its length becomes 14 cm.

The load is now increased to 6.0 kg, and the new length of the spring is Y. The limit of
proportionality is not reached.

10 cm
14 cm
Y

2.0 kg

6.0 kg

What is Y?

A 22 cm B 26 cm C 30 cm D 42 cm

30 A copper block is pulled down a rough slope at a constant speed.

Which form of energy of the block increases?

A chemical
B gravitational
C kinetic
D thermal

31 A weightlifter lifts 150 kg through a distance of 2.0 m in a time of 1.5 s.

The acceleration of free fall g is 10 m / s2.

How much power does she produce?

A 200 W B 450 W C 2000 W D 4500 W

© UCLES 2018 0653/23/M/J/18 [Turn over

522/693
Combined By Nesrine
12
2023-2017

32 On a summer’s day, hot air rises above hot roofs.

What is the name of this process?

A concentration
B condensation
C conduction
D convection

33 The diagram shows part of the electromagnetic spectrum. Two regions are labelled P and Q.

infra-red ultraviolet gamma


P Q
waves waves rays

What type of radiation is P, and which radiation has the lower frequency, P or Q?

lower
radiation P
frequency

A visible light P
B visible light Q
C X-rays P
D X-rays Q

34 The diagram represents a water wave moving into a region where the depth of water is different.

region Y
region X

How do the speed and wavelength of the waves in region Y compare with their values in
region X?

speed in Y wavelength in Y

A greater smaller
B greater the same
C smaller smaller
D smaller the same

© UCLES 2018 0653/23/M/J/18

523/693
Combined By Nesrine
13
2023-2017

35 The diagram shows a ray of light as it enters a glass block.

Which labelled angle is the angle of refraction?

ray of
light

A
B
D
C glass block

36 A student writes two sentences about sound waves.

‘A sound wave travels through the air as compressions and ......X...... .’

‘The air at the compressions has a different ......Y...... from the air at ......X...... .’

What are the missing words, X and Y?

X Y

A rarefactions density
B rarefactions state
C refractions density
D refractions state

37 A negative ion X is close to a positive ion and another negative ion. Electrical forces act on ion X
because of the charges in the other two ions.

Which diagram shows the directions of the two forces acting on ion X?

A B C D
positive
ion + + + +
– ion X – ion X – ion X – ion X
– – – –
negative
ion

© UCLES 2018 0653/23/M/J/18 [Turn over

524/693
Combined By Nesrine
14
2023-2017

38 The diagram shows four wires at the same temperature, made from the same metal. The diagram
is drawn to scale.

Which wire has the least electrical resistance between its ends?

A B

C D

39 A resistor is connected in a circuit as shown.

The reading on the ammeter is 2.0 A and the reading on the voltmeter is 4.0 V.

What is the power produced in the resistor?

A 0.50 W B 2.0 W C 6.0 W D 8.0 W

© UCLES 2018 0653/23/M/J/18

525/693
Combined By Nesrine
15
2023-2017

40 A circuit consists of two identical resistors, R1 and R2, and a battery.

IB

I1 R1

I2 R2

The current in the battery is IB. The current in R1 is I1 and the current in R2 is I2.

How are IB, I1 and I2 related?

A IB = I1 = I2

B IB > I1 and I1 = I2

C IB < I2 and I1 = I2

D IB > I1 > I2

Permission to reproduce items where third-party owned material protected by copyright is included has been sought and cleared where possible. Every
reasonable effort has been made by the publisher (UCLES) to trace copyright holders, but if any items requiring clearance have unwittingly been included, the
publisher will be pleased to make amends at the earliest possible opportunity.

To avoid the issue of disclosure of answer-related information to candidates, all copyright acknowledgements are reproduced online in the Cambridge
International Examinations Copyright Acknowledgements Booklet. This is produced for each series of examinations and is freely available to download at
www.cie.org.uk after the live examination series.

Cambridge International Examinations is part of the Cambridge Assessment Group. Cambridge Assessment is the brand name of University of Cambridge Local
Examinations Syndicate (UCLES), which is itself a department of the University of Cambridge.

© UCLES 2018 0653/23/M/J/18

526/693
Combined By Nesrine
2023-2017

Cambridge International Examinations


Cambridge International General Certificate of Secondary Education

COMBINED SCIENCE 0653/21


Paper 2 Multiple Choice (Extended) October/November 2018
45 minutes
Additional Materials: Multiple Choice Answer Sheet
Soft clean eraser
*6198045591*

Soft pencil (type B or HB is recommended)

READ THESE INSTRUCTIONS FIRST

Write in soft pencil.


Do not use staples, paper clips, glue or correction fluid.
Write your name, Centre number and candidate number on the Answer Sheet in the spaces provided
unless this has been done for you.
DO NOT WRITE IN ANY BARCODES.

There are forty questions on this paper. Answer all questions. For each question there are four possible
answers A, B, C and D.
Choose the one you consider correct and record your choice in soft pencil on the separate Answer Sheet.

Read the instructions on the Answer Sheet very carefully.

Each correct answer will score one mark. A mark will not be deducted for a wrong answer.
Any rough working should be done in this booklet.
A copy of the Periodic Table is printed on page 16.
Electronic calculators may be used.

This document consists of 14 printed pages and 2 blank pages.

IB18 11_0653_21/2RP
© UCLES 2018 [Turn over

527/693
Combined By Nesrine
2
2023-2017

1 Which structure found in plant cells is matched to its function?

structure function

A cell membrane provides strength and support


B chloroplast absorbs light energy
C cytoplasm filled with cell sap for strengthening
D permanent vacuole site of chemical reactions

2 Which process depends on diffusion?

A circulation
B digestion
C gaseous exchange
D phagocytosis

3 Biological catalysts speed up reactions in the body.

What is another name for biological catalysts?

A antibodies
B enzymes
C fatty acids
D hormones

4 Microorganisms are used to make yoghurt.

Which acid is produced when microorganisms break down lactose in the milk?

A amino acid
B fatty acid
C hydrochloric acid
D lactic acid

© UCLES 2018 0653/21/O/N/18

528/693
Combined By Nesrine
3
2023-2017

5 The diagram shows a leaf that was tested for starch using iodine solution.

green area

white area

Which row shows the results for this leaf and explains the results?

green area of white area of


explanation
leaf after test leaf after test

A blue-black blue-black chlorophyll is found in all parts of the leaf


B blue-black brown chlorophyll is found in only part of the leaf
C brown brown chlorophyll is found in all parts of the leaf
D brown blue-black chlorophyll is found in only part of the leaf

6 The diagram shows part of the human alimentary canal.

Where is bile made?

D B

© UCLES 2018 0653/21/O/N/18 [Turn over

529/693
Combined By Nesrine
4
2023-2017

7 Which diagram shows the double circulatory system of a human?

A B

heart lungs heart body

lungs body body lungs

C D

heart lungs heart lungs

body heart heart body

8 Aerobic respiration is the release of a relatively ......X...... amount of energy by the breakdown of
food substances in the presence of ......Y...... .

Which words complete the gaps X and Y?

X Y

A large carbon dioxide


B large oxygen
C small carbon dioxide
D small oxygen

9 The diagram shows structures called lamellae. They are found in the gills of fish.

lamellae

Lamellae increase the surface area of the gills. The gills are the site of gaseous exchange in fish.

What is the effect of this increased surface area?

A decreased rate of carbon dioxide diffusion into the blood


B decreased rate of oxygen diffusion into the blood
C increased rate of carbon dioxide diffusion into the blood
D increased rate of oxygen diffusion into the blood

© UCLES 2018 0653/21/O/N/18

530/693
Combined By Nesrine
5
2023-2017

10 Which statement about adrenaline is not correct?

A Adrenaline is transported in the blood plasma.


B Adrenaline lowers the blood glucose concentration.
C The heart is one of the target organs for adrenaline.
D The liver destroys adrenaline.

11 What is the function of the amniotic sac?

A It surrounds the fetus in the uterus and contains amniotic fluid.


B It surrounds the fetus in the uterus and provides essential nutrients for the fetus.
C It surrounds the fetus in the vagina and contains amniotic fluid.
D It surrounds the fetus in the vagina and provides essential nutrients for the fetus.

12 What is the definition of a trophic level?

A It shows how an organism loses energy.


B It shows the position of an organism in a food chain.
C It shows the consumers of an organism.
D It shows the food eaten by an organism.

13 Which are possible harmful effects of deforestation?

global warming species extinction

A  
B  
C  
D  

14 Sucrose is a covalent compound.

It is a solid at room temperature.

Which statement about sucrose is correct?

A It is made of atoms that are close together and in continuous random motion.
B It is made of atoms that are far apart and vibrating about a fixed point.
C It is made of molecules that are close together and vibrating about a fixed point.
D It is made of molecules that are far apart and in continuous random motion.

© UCLES 2018 0653/21/O/N/18 [Turn over

531/693
Combined By Nesrine
6
2023-2017

15 Four dyes are separated using chromatography.

The results are shown.

1 2 3 4

Which dyes contain two colours that are present in both dyes?

A 1 and 2 B 1 and 4 C 2 and 3 D 2 and 4

16 Which diagram represents the bonding in a molecule of ethene?

A B C D

H H H H H H
H C C H H C C H C C C C
H H H H H H

17 On which label does the formula match the name of the acid?

A B C D

hydrochloric hydrochloric sulfuric sulfuric


acid acid acid acid
HCl H2SO4 HNO3 Na2SO4

© UCLES 2018 0653/21/O/N/18

532/693
Combined By Nesrine
7
2023-2017

18 Some chemical compounds are broken down by electrolysis using inert electrodes.

Which row identifies the electrode products for the stated electrolyte?

electrolyte product at anode product at cathode

A aqueous copper chloride hydrogen copper


B molten aluminium oxide aluminium oxygen
C molten copper chloride chlorine copper
D molten potassium bromide potassium bromine

19 When concentrated sulfuric acid is added to water, the temperature of the mixture increases.

Which row describes the type of reaction and the energy change for this process?

type of reaction energy change

A endothermic chemical to thermal


B endothermic thermal to chemical
C exothermic chemical to thermal
D exothermic thermal to chemical

20 Which diagram shows apparatus used to investigate the rate of a reaction in which a gas is given
off?

A B

gas

water

C D

gas

water water

© UCLES 2018 0653/21/O/N/18 [Turn over

533/693
Combined By Nesrine
8
2023-2017

21 Iron oxide reacts with carbon monoxide.

The word equation for the reaction is:

iron oxide + carbon monoxide → iron + carbon dioxide

Which statement is not correct?

A Carbon is neither oxidised nor reduced.


B Carbon is oxidised.
C Iron is reduced.
D This is a redox reaction.

22 Which element in a period of the Periodic Table has the greatest metallic character?

A the element which most readily forms an anion


B the element with the fewest outer-shell electrons
C the element with the highest atomic number
D the element with the largest group number

23 The positions of four elements are shown in the outline of the Periodic Table.

Which element has a high melting point and forms coloured compounds?

A B
C D

24 Which gas is used to provide an inert atmosphere in lamps?

A argon
B helium
C neon
D nitrogen

© UCLES 2018 0653/21/O/N/18

534/693
Combined By Nesrine
9
2023-2017

25 Which statement about metals is not correct?

A Copper is below hydrogen in the reactivity series.


B Lithium produces a flame when a small piece is added to cold water.
C Magnesium reacts with steam to produce hydrogen.
D Zinc reacts with copper ions to form zinc ions and copper.

26 Gasoline is a hydrocarbon fuel obtained from petroleum.

Which statement is correct?

A Gasoline burns to form carbon dioxide and water.


B Gasoline contains the elements carbon, hydrogen and oxygen.
C Gasoline is used as a fuel in diesel engines.
D The combustion of gasoline is an endothermic reaction.

27 P, Q and R are three fractions obtained from petroleum by fractional distillation.

Molecules of R are larger than molecules of P.

The intermolecular forces in Q are weaker than those in P.

What is the order of boiling points?

lowest highest

A P Q R
B Q P R
C R P Q
D R Q P

28 A gold block is taken from the surface of the Earth to the surface of the Moon.

The gravitational field is weaker on the Moon than it is on the Earth.

Which property of the gold block changes?

A density
B mass
C volume
D weight

© UCLES 2018 0653/21/O/N/18 [Turn over

535/693
Combined By Nesrine
10
2023-2017

29 A cube of aluminium has sides of length 1.0 cm.

1.0 cm
1.0 cm
1.0 cm

Compared with this cube, which statement about a cube of aluminium with sides of 2.0 cm is
correct?

A It has the same density.


B It has the same mass.
C It has twice the density.
D It has twice the mass.

30 A spring that obeys Hooke’s law is 20 cm long when unstretched.

A load of 10 N is hung from the spring and its length increases to 25 cm.

The 10 N load is removed and replaced with a 30 N load.

What is the new length of the spring?

A 15 cm B 35 cm C 40 cm D 60 cm

31 A brick of mass 4.0 kg rests on a window ledge. It falls off the window ledge and drops through a
height of 5.0 m to the ground. The acceleration of free fall g is 10 m / s2.

Air resistance can be ignored.

Which row states the kinetic energy and the speed of the brick just before it hits the ground?

kinetic energy speed of brick


of brick / J m/s

A 20 2.2
B 20 3.2
C 200 7.1
D 200 10

© UCLES 2018 0653/21/O/N/18

536/693
Combined By Nesrine
11
2023-2017

32 A scientist investigates two different substances, P and Q.

Substance P completely fills its container but can be compressed.

Substance Q is not in a container but has a definite shape.

In which state is each substance?

substance P substance Q

A gas liquid
B gas solid
C liquid gas
D liquid solid

33 A liquid evaporates when molecules leave its surface.

Which molecules leave the surface, and what happens to the temperature of the remaining
liquid?

A The more energetic molecules leave and the temperature falls.


B The more energetic molecules leave and the temperature rises.
C The less energetic molecules leave and the temperature falls.
D The less energetic molecules leave and the temperature rises.

34 How is heat transferred in solids?

A Heated molecules become less dense and travel to colder areas.


B Heated molecules become more dense and travel to colder areas.
C Heated molecules vibrate more quickly and cause neighbouring molecules to vibrate more
quickly.
D Heated molecules vibrate more slowly and cause neighbouring molecules to vibrate more
quickly.

© UCLES 2018 0653/21/O/N/18 [Turn over

537/693
Combined By Nesrine
12
2023-2017

35 The diagram shows a water wave travelling at 0.56 m / s.

0.10 m

What is the frequency of the wave?

A 0.11 Hz B 0.36 Hz C 2.8 Hz D 5.6 Hz

36 Which list shows electromagnetic waves in order of decreasing wavelength (largest to smallest)?

A gamma rays → radio waves → infra-red → microwaves

B microwaves → visible light → X-rays → infra-red

C radio waves → visible light → ultraviolet → X-rays

D X-rays → infra-red → microwaves → visible light

37 The diagrams represent four different sound waves. The scales are the same in all the diagrams.

Which sound has the lowest pitch?

A B

time time

C D

time time

© UCLES 2018 0653/21/O/N/18

538/693
Combined By Nesrine
13
2023-2017

38 The diagram shows a negatively charged oil drop between two metal plates. The plates are
connected by an open switch to a power supply. The oil drop is falling at a steady speed.

metal plate


oil drop
+

metal plate

The switch is now closed.

What happens to the oil drop?

A It moves downwards at an increasing speed.


B It moves upwards at an increasing speed.
C It moves to the left at a constant speed.
D It moves to the right at a constant speed.

39 Three pieces of resistance wire X, Y and Z are made of the same metal.

The diagram shows the lengths and the diameters of the wires.

X Y Z

l
l
d 2l
2d
d

What is the order of the wires when they are placed in order of increasing resistance, least
resistance first?

A Y→X→Z B Y→Z→X C Z→X→Y D Z→Y→X

40 An electric motor is connected to a 120 V mains supply.

The motor transfers 72 000 J of energy in 2.0 minutes.

What is the current in the motor?

A 0.20 A B 5.0 A C 10 A D 300 A

© UCLES 2018 0653/21/O/N/18

539/693
Combined By Nesrine
2023-2017

Cambridge International Examinations


Cambridge International General Certificate of Secondary Education

COMBINED SCIENCE 0653/22


Paper 2 Multiple Choice (Extended) October/November 2018
45 minutes
Additional Materials: Multiple Choice Answer Sheet
Soft clean eraser
*5895915064*

Soft pencil (type B or HB is recommended)

READ THESE INSTRUCTIONS FIRST

Write in soft pencil.


Do not use staples, paper clips, glue or correction fluid.
Write your name, Centre number and candidate number on the Answer Sheet in the spaces provided
unless this has been done for you.
DO NOT WRITE IN ANY BARCODES.

There are forty questions on this paper. Answer all questions. For each question there are four possible
answers A, B, C and D.
Choose the one you consider correct and record your choice in soft pencil on the separate Answer Sheet.

Read the instructions on the Answer Sheet very carefully.

Each correct answer will score one mark. A mark will not be deducted for a wrong answer.
Any rough working should be done in this booklet.
A copy of the Periodic Table is printed on page 16.
Electronic calculators may be used.

This document consists of 15 printed pages and 1 blank page.

IB18 11_0653_22/2RP
© UCLES 2018 [Turn over

540/693
Combined By Nesrine
2
2023-2017

1 The diagram shows a typical plant cell.

Which part of the cell controls the movement of materials into and out of the cell?

D
B

2 Which process depends on diffusion?

A circulation
B digestion
C gaseous exchange
D phagocytosis

3 Enzymes are used in digestion to break down larger molecules into smaller molecules.

Which row matches the large molecules with the small molecules they are broken down into?

large molecules small molecules

A fat glycerol and fatty acids


B glycogen glycerol and amino acids
C protein simple sugars
D starch amino acids

4 Which two nutrients are needed for healthy bone and tooth development?

A calcium and iron


B iron and vitamin C
C vitamin C and vitamin D
D vitamin D and calcium

© UCLES 2018 0653/22/O/N/18

541/693
Combined By Nesrine
3
2023-2017

5 The diagram shows a leaf that was tested for starch using iodine solution.

green area

white area

Which row shows the results for this leaf and explains the results?

green area of white area of


explanation
leaf after test leaf after test

A blue-black blue-black chlorophyll is found in all parts of the leaf


B blue-black brown chlorophyll is found in only part of the leaf
C brown brown chlorophyll is found in all parts of the leaf
D brown blue-black chlorophyll is found in only part of the leaf

6 The diagram shows a cross-section of a root hair cell.

Y
X

Which row identifies the part of the cell with the larger surface area and its function?

part of cell function

A X water and glucose uptake


B X water and ion uptake
C Y water and glucose uptake
D Y water and ion uptake

© UCLES 2018 0653/22/O/N/18 [Turn over

542/693
Combined By Nesrine
4
2023-2017

7 The diagram shows the external surface of the heart.

Which letter identifies a coronary artery?

B
C

8 Six molecules of glucose are aerobically respired in an animal cell.

How many molecules of carbon dioxide are released in this process?

A 1 B 6 C 12 D 36

9 Which features will maximise the rate of gas exchange across the alveoli?

large surface area small blood supply thin membrane

A   
B   
C   
D   

10 What happens to the blood glucose concentration and pulse rate when adrenaline is released
into a person’s bloodstream?

blood glucose
pulse rate
concentration

A decreases increases
B decreases decreases
C increases increases
D increases decreases

© UCLES 2018 0653/22/O/N/18

543/693
Combined By Nesrine
5
2023-2017

11 Which row describes the net diffusion of substances between the fetus and the mother across the
placenta?

from fetus to mother from mother to fetus

A carbon dioxide and glucose oxygen and amino acids


B carbon dioxide and waste products oxygen and glucose
C oxygen and glucose carbon dioxide and amino acid
D oxygen and waste products carbon dioxide and glucose

12 What is the definition of a trophic level?

A It shows how an organism loses energy.


B It shows the position of an organism in a food chain.
C It shows the consumers of an organism.
D It shows the food eaten by an organism.

13 Which are possible harmful effects of deforestation?

global warming species extinction

A  
B  
C  
D  

14 Sucrose is a covalent compound.

It is a solid at room temperature.

Which statement about sucrose is correct?

A It is made of atoms that are close together and in continuous random motion.
B It is made of atoms that are far apart and vibrating about a fixed point.
C It is made of molecules that are close together and vibrating about a fixed point.
D It is made of molecules that are far apart and in continuous random motion.

© UCLES 2018 0653/22/O/N/18 [Turn over

544/693
Combined By Nesrine
6
2023-2017

15 The apparatus used for chromatography is shown.

beaker
chromatography
paper

sample pencil line


spots
solvent

Which statement about the method used for chromatography is not correct?

A The beaker is swirled to help the solvent to rise.


B The chromatography paper is placed in the beaker after the solvent has been added.
C The chromatography paper is removed before the solvent reaches the top of the paper.
D The sample spots are placed on the pencil line above the level of the solvent.

16 Which molecule contains a double covalent bond?

A ethene
B methane
C nitrogen
D water

17 Which formula does not represent an acid?

A H2SO4 B HCl C HNO3 D NH3

© UCLES 2018 0653/22/O/N/18

545/693
Combined By Nesrine
7
2023-2017

18 During the electrolysis of aqueous copper chloride, ions move.

A gas is produced at one of the electrodes.

Which diagram shows the movement of ions and the electrode at which the gas is produced?

A B
power supply power supply

– + – +

gas bubbles
gas bubbles anions anions

cations cations

C D
power supply power supply

– + – +

gas bubbles
gas bubbles anions anions

cations cations

19 Which statement describes an endothermic reaction?

A Chemical energy is transformed into thermal energy and the temperature falls.
B Chemical energy is transformed into thermal energy and the temperature rises.
C Thermal energy is transformed into chemical energy and the temperature falls.
D Thermal energy is transformed into chemical energy and the temperature rises.

© UCLES 2018 0653/22/O/N/18 [Turn over

546/693
Combined By Nesrine
8
2023-2017

20 Hydrogen peroxide decomposes to form oxygen and water.

A catalyst is added to the hydrogen peroxide.

Which row describes the change in the rate of reaction and the mass of catalyst left at the end of
the reaction?

mass of catalyst left


rate of reaction
at end of reaction

A decrease less
B decrease no change
C increase less
D increase no change

21 Iron oxide reacts with carbon monoxide.

The word equation for the reaction is:

iron oxide + carbon monoxide → iron + carbon dioxide

Which statement is not correct?

A Carbon is neither oxidised nor reduced.


B Carbon is oxidised.
C Iron is reduced.
D This is a redox reaction.

22 Francium is the element at the bottom of Group I.

What would happen if a sample of francium is added to water?

A Francium reacts rapidly to produce bubbles of carbon dioxide.


B Francium reacts to form a precipitate of francium oxide.
C Francium reacts violently and produces a flammable gas.
D Francium sinks and no reaction occurs.

© UCLES 2018 0653/22/O/N/18

547/693
Combined By Nesrine
9
2023-2017

23 The positions of four elements are shown in the outline of the Periodic Table.

Which element has a high melting point and forms coloured compounds?

A B
C D

24 Which statement about noble gases is not correct?

A A neon atom has a full outer shell of electrons.


B Helium is used to fill balloons.
C Noble gases are very unreactive.
D Noble gases exist as molecules containing two atoms.

25 Which metal most readily forms positive ions?

A calcium
B copper
C iron
D zinc

26 Gasoline is a hydrocarbon fuel obtained from petroleum.

Which statement is correct?

A Gasoline burns to form carbon dioxide and water.


B Gasoline contains the elements carbon, hydrogen and oxygen.
C Gasoline is used as a fuel in diesel engines.
D The combustion of gasoline is an endothermic reaction.

© UCLES 2018 0653/22/O/N/18 [Turn over

548/693
Combined By Nesrine
10
2023-2017

27 Petroleum is separated into fractions by fractional distillation.

Which labelled fraction contains molecules with the largest intermolecular attractive forces?

D
petroleum

28 A train travels between two stations.

The distance-time graph for the train is shown.

At which time is the train travelling the fastest?

distance

0
0 A B C D time

29 What name is given to the gravitational force acting on a mass?

A density
B power
C weight
D work

© UCLES 2018 0653/22/O/N/18

549/693
Combined By Nesrine
11
2023-2017

30 A cube of aluminium has sides of length 1.0 cm.

1.0 cm
1.0 cm
1.0 cm

Compared with this cube, which statement about a cube of aluminium with sides of 2.0 cm is
correct?

A It has the same density.


B It has the same mass.
C It has twice the density.
D It has twice the mass.

31 A brick of mass 4.0 kg rests on a window ledge. It falls off the window ledge and drops through a
height of 5.0 m to the ground. The acceleration of free fall g is 10 m / s2.

Air resistance can be ignored.

Which row states the kinetic energy and the speed of the brick just before it hits the ground?

kinetic energy speed of brick


of brick / J m/s

A 20 2.2
B 20 3.2
C 200 7.1
D 200 10

32 Gases are easier to compress than either solids or liquids.

Which statement about gas molecules is correct?

A They are closer together and the forces between them are stronger.
B They are closer together and the forces between them are weaker.
C They are further apart and the forces between them are stronger.
D They are further apart and the forces between them are weaker.

© UCLES 2018 0653/22/O/N/18 [Turn over

550/693
Combined By Nesrine
12
2023-2017

33 A liquid evaporates when molecules leave its surface.

Which molecules leave the surface, and what happens to the temperature of the remaining
liquid?

A The more energetic molecules leave and the temperature falls.


B The more energetic molecules leave and the temperature rises.
C The less energetic molecules leave and the temperature falls.
D The less energetic molecules leave and the temperature rises.

34 The diagram represents a wave.

Which labelled distance gives the amplitude of the wave?

B
D

35 A student is watching television. He uses a remote controller to change the programme.

The remote controller uses electromagnetic waves. Electromagnetic waves are also used to
transmit the television signals from a satellite.

Which row shows the type of wave used for each purpose?

remote controller satellite

A infra-red microwaves
B infra-red radio waves
C ultraviolet microwaves
D ultraviolet radio waves

36 A girl stands in front of a plane mirror. She then walks towards the mirror at a speed of 1.0 m / s.

At what combined speed do the girl and her image appear to approach each other?

A 0m/s B 0.50 m / s C 1.0 m / s D 2.0 m / s

© UCLES 2018 0653/22/O/N/18

551/693
Combined By Nesrine
13
2023-2017

37 The diagrams represent four different sound waves. The scales are the same in all the diagrams.

Which sound has the lowest pitch?

A B

time time

C D

time time

38 Three pieces of resistance wire X, Y and Z are made of the same metal.

The diagram shows the lengths and the diameters of the wires.

X Y Z

l
l
d 2l
2d
d

What is the order of the wires when they are placed in order of increasing resistance, least
resistance first?

A Y→X→Z B Y→Z→X C Z→X→Y D Z→Y→X

© UCLES 2018 0653/22/O/N/18 [Turn over

552/693
Combined By Nesrine
14
2023-2017

39 The diagram shows two identical lamps connected in parallel to a 12 V power supply. A current of
3.0 A is delivered by the power supply.

12 V

3.0 A

What is the power produced by each lamp?

A 4.0 W B 8.0 W C 18 W D 36 W

40 The diagram shows two 6.0 Ω resistors and one 12 Ω resistor connected in series to a power
supply. The voltmeter connected across one 6.0 Ω resistor reads 2.0 V.

power supply

6.0 Ω 6.0 Ω 12 Ω

What is the potential difference across the power supply?

A 6.0 V B 8.0 V C 12 V D 48 V

© UCLES 2018 0653/22/O/N/18

553/693
Combined By Nesrine
2023-2017

Cambridge International Examinations


Cambridge International General Certificate of Secondary Education

COMBINED SCIENCE 0653/23


Paper 2 Multiple Choice (Extended) October/November 2018
45 minutes
Additional Materials: Multiple Choice Answer Sheet
Soft clean eraser
*1618482987*

Soft pencil (type B or HB is recommended)

READ THESE INSTRUCTIONS FIRST

Write in soft pencil.


Do not use staples, paper clips, glue or correction fluid.
Write your name, Centre number and candidate number on the Answer Sheet in the spaces provided
unless this has been done for you.
DO NOT WRITE IN ANY BARCODES.

There are forty questions on this paper. Answer all questions. For each question there are four possible
answers A, B, C and D.
Choose the one you consider correct and record your choice in soft pencil on the separate Answer Sheet.

Read the instructions on the Answer Sheet very carefully.

Each correct answer will score one mark. A mark will not be deducted for a wrong answer.
Any rough working should be done in this booklet.
A copy of the Periodic Table is printed on page 16.
Electronic calculators may be used.

This document consists of 16 printed pages.

IB18 11_0653_23/2RP
© UCLES 2018 [Turn over

554/693
Combined By Nesrine
2
2023-2017

1 The diagram shows a typical plant cell.

Which part of the cell contains the genetic information of the cell?

D
B

2 Which process depends on diffusion?

A circulation
B digestion
C gaseous exchange
D phagocytosis

© UCLES 2018 0653/23/O/N/18

555/693
Combined By Nesrine
3
2023-2017

3 A human enzyme breaks down starch into simple sugars.

A solution of this human enzyme was heated to 90 °C for 30 minutes.

2 cm3 of this human enzyme solution was added to starch solution in several different test-tubes.
The test-tubes were kept at different temperatures for 15 minutes.

Which graph shows the amount of sugar produced in the test-tubes?

A B

amount amount
of sugar of sugar

temperature / °C temperature / °C

C D

amount amount
of sugar of sugar

temperature / °C temperature / °C

© UCLES 2018 0653/23/O/N/18 [Turn over

556/693
Combined By Nesrine
4
2023-2017

4 The diagram shows a leaf that was tested for starch using iodine solution.

green area

white area

Which row shows the results for this leaf and explains the results?

green area of white area of


explanation
leaf after test leaf after test

A blue-black blue-black chlorophyll is found in all parts of the leaf


B blue-black brown chlorophyll is found in only part of the leaf
C brown brown chlorophyll is found in all parts of the leaf
D brown blue-black chlorophyll is found in only part of the leaf

5 A doctor advises a man to change his diet and reduce his intake of saturated fat and salt.

Which condition is the man most likely to be suffering from?

A constipation
B coronary heart disease
C dental decay
D starvation

6 The diagram shows a cross-section of a root hair cell.

Which row describes the root hair cell and its function?

animal cell or plant cell function

A animal cell water and glucose uptake


B animal cell water and ion uptake
C plant cell water and glucose uptake
D plant cell water and ion uptake

© UCLES 2018 0653/23/O/N/18

557/693
Combined By Nesrine
5
2023-2017

7 Which blood vessel contains blood at the highest pressure?

lungs

B C

A D

rest of body

8 The diagram shows two different types of cell which line the trachea in the gas exchange system.

What is the role of X and Y?

X Y

A produces mucus traps pathogens


B produces mucus moves pathogens towards the mouth
C moves pathogens towards the mouth traps pathogens
D moves pathogens towards the mouth moves pathogens towards the mouth

© UCLES 2018 0653/23/O/N/18 [Turn over

558/693
Combined By Nesrine
6
2023-2017

9 Which statement about adrenaline is not correct?

A It decreases blood glucose concentration.


B It is carried by the blood.
C It is produced by a gland.
D The heart is one of its target organs.

10 In an investigation, four test-tubes containing seeds were set up as shown in the diagram.

After several days, which test-tube will contain the most germinated seeds?

A B C D
dry
cotton
wool black
box

seeds seeds
gauze
damp damp solution dry
cotton cotton that cotton
wool wool absorbs wool
1 °C 20 °C oxygen 20 °C 1 °C

11 During pregnancy in humans, gas exchange occurs between a mother and her fetus.

Where does this gas exchange occur?

A amniotic fluid
B amniotic sac
C placenta
D umbilical cord

12 What is the definition of a trophic level?

A It shows how an organism loses energy.


B It shows the position of an organism in a food chain.
C It shows the consumers of an organism.
D It shows the food eaten by an organism.

© UCLES 2018 0653/23/O/N/18

559/693
Combined By Nesrine
7
2023-2017

13 Which are possible harmful effects of deforestation?

global warming species extinction

A  
B  
C  
D  

14 Sucrose is a covalent compound.

It is a solid at room temperature.

Which statement about sucrose is correct?

A It is made of atoms that are close together and in continuous random motion.
B It is made of atoms that are far apart and vibrating about a fixed point.
C It is made of molecules that are close together and vibrating about a fixed point.
D It is made of molecules that are far apart and in continuous random motion.

15 Which diagram shows how apparatus is used to separate the different colours in an ink?

A B
chromatography
paper
chromatography
paper

pencil line ink spot

ink spot
pencil line
water water

C D

chromatography chromatography
paper paper

ink line ink spot pencil line ink spot

water water

© UCLES 2018 0653/23/O/N/18 [Turn over

560/693
Combined By Nesrine
8
2023-2017

16 Which is the electronic structure of a noble gas?

A 2,6 B 2,8 C 2,8,1 D 2,8,7

17 What is the formula of nitric acid?

A HCl B HNO3 C NaOH D NH3

18 During the electrolysis of molten potassium chloride, which particles in the electrolyte move
towards the cathode?

A electrons
B chloride ions
C chlorine molecules
D potassium ions

19 The temperature of some water is recorded.

Sodium chloride is dissolved in the water and the temperature of the solution is recorded.

temperature of water = 20 °C

temperature of solution = 18 °C

Which statement about the process is correct?

A It is endothermic because chemical energy is changed into heat energy.


B It is endothermic because heat energy is changed into chemical energy.
C It is exothermic because chemical energy is changed into heat energy.
D It is exothermic because heat energy is changed into chemical energy.

20 Substance X increases the rate of a chemical reaction, but it remains unchanged at the end of
the reaction.

Which word describes substance X?

A catalyst
B electrolyte
C product
D unreactive

© UCLES 2018 0653/23/O/N/18

561/693
Combined By Nesrine
9
2023-2017

21 Iron oxide reacts with carbon monoxide.

The word equation for the reaction is:

iron oxide + carbon monoxide → iron + carbon dioxide

Which statement is not correct?

A Carbon is neither oxidised nor reduced.


B Carbon is oxidised.
C Iron is reduced.
D This is a redox reaction.

22 Rubidium is an element below potassium in Group I of the Periodic Table.

Which property of rubidium is not correct?

A Rubidium is a soft metal.


B Rubidium is less reactive than potassium.
C Rubidium melts at a lower temperature than potassium.
D Rubidium reacts with water forming hydrogen.

23 The positions of four elements are shown in the outline of the Periodic Table.

Which element has a high melting point and forms coloured compounds?

A B
C D

24 Which gas is used to fill weather balloons?

A argon
B carbon dioxide
C helium
D nitrogen

© UCLES 2018 0653/23/O/N/18 [Turn over

562/693
Combined By Nesrine
10
2023-2017

25 What is observed when magnesium ribbon is placed into aqueous copper sulfate?

A The blue solution gets darker and a brown solid appears.


B The blue solution gets darker and no solid is visible.
C The blue solution gets paler and a brown solid appears.
D The blue solution gets paler and no solid is visible.

26 Gasoline is a hydrocarbon fuel obtained from petroleum.

Which statement is correct?

A Gasoline burns to form carbon dioxide and water.


B Gasoline contains the elements carbon, hydrogen and oxygen.
C Gasoline is used as a fuel in diesel engines.
D The combustion of gasoline is an endothermic reaction.

27 Which substance rapidly decolourises bromine?

A B C D
H H H H H H H H H

H C H H C C H H C C C H C C C OH

H H H H H H H H

28 The graph shows the motion of a cyclist over a period of 30 s.

7
speed
m/s

0
0 20 30
time / s

Which distance does she travel?

A 90 m B 105 m C 115 m D 120 m

© UCLES 2018 0653/23/O/N/18

563/693
Combined By Nesrine
11
2023-2017

29 A cube of aluminium has sides of length 1.0 cm.

1.0 cm
1.0 cm
1.0 cm

Compared with this cube, which statement about a cube of aluminium with sides of 2.0 cm is
correct?

A It has the same density.


B It has the same mass.
C It has twice the density.
D It has twice the mass.

30 A stretching force is applied to a copper wire. The wire obeys Hooke’s Law until it reaches the
limit of proportionality.

Which is the extension-load graph for the wire and shows the limit of proportionality labelled P?

A B

extension extension P
P

0 0
0 load 0 load

C D

extension extension P
P

0 0
0 load 0 load

© UCLES 2018 0653/23/O/N/18 [Turn over

564/693
Combined By Nesrine
12
2023-2017

31 A brick of mass 4.0 kg rests on a window ledge. It falls off the window ledge and drops through a
height of 5.0 m to the ground. The acceleration of free fall g is 10 m / s2.

Air resistance can be ignored.

Which row states the kinetic energy and the speed of the brick just before it hits the ground?

kinetic energy speed of brick


of brick / J m/s

A 20 2.2
B 20 3.2
C 200 7.1
D 200 10

32 A liquid evaporates when molecules leave its surface.

Which molecules leave the surface, and what happens to the temperature of the remaining
liquid?

A The more energetic molecules leave and the temperature falls.


B The more energetic molecules leave and the temperature rises.
C The less energetic molecules leave and the temperature falls.
D The less energetic molecules leave and the temperature rises.

33 Convection is a process by which thermal energy is transferred from one place to another.

Where can convection take place?

A in a gas and in a vacuum


B in a liquid and in a gas
C in a liquid and in a solid
D in a solid and in a vacuum

© UCLES 2018 0653/23/O/N/18

565/693
Combined By Nesrine
13
2023-2017

34 A musical instrument produces a note of frequency 170 Hz. The sound wave produced travels
through the air at a speed of 340 m / s.

Which row describes the sound wave?

nature of
wavelength / m
wave

A longitudinal 0.50
B longitudinal 2.0
C transverse 0.50
D transverse 2.0

35 A girl stands in front of a plane mirror and observes her image. She walks 2.0 m towards the
mirror.

What is the change in the distance between the girl and her image?

A 0m B 1.0 m C 2.0 m D 4.0 m

36 The diagrams represent four different sound waves. The scales are the same in all the diagrams.

Which sound has the lowest pitch?

A B

time time

C D

time time

© UCLES 2018 0653/23/O/N/18 [Turn over

566/693
Combined By Nesrine
14
2023-2017

37 Three pieces of resistance wire X, Y and Z are made of the same metal.

The diagram shows the lengths and the diameters of the wires.

X Y Z

l
l
d 2l
2d
d

What is the order of the wires when they are placed in order of increasing resistance, least
resistance first?

A Y→X→Z B Y→Z→X C Z→X→Y D Z→Y→X

38 A lamp has a potential difference V across it that causes a current I in it.

Which equation gives the power P produced by the lamp?

A P= I B P= V C P = IV D P=I+V
V I

39 A kettle is connected to a power supply as shown.

power kettle
supply

If too much current flows, a component connected at X automatically disconnects the power
supply.

Which symbol represents the component at X?

A B C D

© UCLES 2018 0653/23/O/N/18

567/693
Combined By Nesrine
15
2023-2017

40 A student sets up the circuit shown in the diagram. The ammeter reads 0.60 A and the voltmeter
reads 3.0 V.

A 0.60 A

V
3.0 V

The student now takes two resistors that are identical to the original resistor. She connects them
in series with the original resistor.

What are the new readings on the ammeter and the voltmeter?

ammeter / A voltmeter / V

A 0.20 1.0
B 0.20 3.0
C 0.60 1.0
D 0.60 3.0

Permission to reproduce items where third-party owned material protected by copyright is included has been sought and cleared where possible. Every
reasonable effort has been made by the publisher (UCLES) to trace copyright holders, but if any items requiring clearance have unwittingly been included, the
publisher will be pleased to make amends at the earliest possible opportunity.

To avoid the issue of disclosure of answer-related information to candidates, all copyright acknowledgements are reproduced online in the Cambridge
International Examinations Copyright Acknowledgements Booklet. This is produced for each series of examinations and is freely available to download at
www.cie.org.uk after the live examination series.

Cambridge International Examinations is part of the Cambridge Assessment Group. Cambridge Assessment is the brand name of University of Cambridge Local
Examinations Syndicate (UCLES), which is itself a department of the University of Cambridge.

© UCLES 2018 0653/23/O/N/18

568/693
The Periodic Table of Elements
Group
I II III IV V VI VII VIII

© UCLES 2018
1 2

H He
hydrogen helium
Key 1 4
3 4 atomic number 5 6 7 8 9 10

Li Be atomic symbol B C N O F Ne
lithium beryllium name boron carbon nitrogen oxygen fluorine neon
7 9 relative atomic mass 11 12 14 16 19 20
11 12 13 14 15 16 17 18
Na Mg Al Si P S Cl Ar
sodium magnesium aluminium silicon phosphorus sulfur chlorine argon
23 24 27 28 31 32 35.5 40
19 20 21 22 23 24 25 26 27 28 29 30 31 32 33 34 35 36
K Ca Sc Ti V Cr Mn Fe Co Ni Cu Zn Ga Ge As Se Br Kr
potassium calcium scandium titanium vanadium chromium manganese iron cobalt nickel copper zinc gallium germanium arsenic selenium bromine krypton
39 40 45 48 51 52 55 56 59 59 64 65 70 73 75 79 80 84
37 38 39 40 41 42 43 44 45 46 47 48 49 50 51 52 53 54

Rb Sr Y Zr Nb Mo Tc Ru Rh Pd Ag Cd In Sn Sb Te I Xe
rubidium strontium yttrium zirconium niobium molybdenum technetium ruthenium rhodium palladium silver cadmium indium tin antimony tellurium iodine xenon
85 88 89 91 93 96 – 101 103 106 108 112 115 119 122 128 127 131
16

55 56 57–71 72 73 74 75 76 77 78 79 80 81 82 83 84 85 86
lanthanoids

569/693
Cs Ba Hf Ta W Re Os Ir Pt Au Hg Tl Pb Bi Po At Rn

0653/23/O/N/18
caesium barium hafnium tantalum tungsten rhenium osmium iridium platinum gold mercury thallium lead bismuth polonium astatine radon
133 137 178 181 184 186 190 192 195 197 201 204 207 209 – – –
87 88 89–103 104 105 106 107 108 109 110 111 112 114 116
actinoids
Fr Ra Rf Db Sg Bh Hs Mt Ds Rg Cn Fl Lv
francium radium rutherfordium dubnium seaborgium bohrium hassium meitnerium darmstadtium roentgenium copernicium flerovium livermorium
– – – – – – – – – – – – –

57 58 59 60 61 62 63 64 65 66 67 68 69 70 71
lanthanoids La Ce Pr Nd Pm Sm Eu Gd Tb Dy Ho Er Tm Yb Lu
lanthanum cerium praseodymium neodymium promethium samarium europium gadolinium terbium dysprosium holmium erbium thulium ytterbium lutetium
139 140 141 144 – 150 152 157 159 163 165 167 169 173 175
89 90 91 92 93 94 95 96 97 98 99 100 101 102 103
actinoids Ac Th Pa U Np Pu Am Cm Bk Cf Es Fm Md No Lr
actinium thorium protactinium uranium neptunium plutonium americium curium berkelium californium einsteinium fermium mendelevium nobelium lawrencium
– 232 231 238 – – – – – – – – – – –
2023-2017

The volume of one mole of any gas is 24 dm3 at room temperature and pressure (r.t.p.).
Combined By Nesrine
Combined By Nesrine
2023-2017

Cambridge International Examinations


Cambridge International General Certificate of Secondary Education

COMBINED SCIENCE 0653/22


Paper 2 Multiple Choice (Extended) February/March 2017
45 minutes
Additional Materials: Multiple Choice Answer Sheet
Soft clean eraser
*6441820168*

Soft pencil (type B or HB is recommended)

READ THESE INSTRUCTIONS FIRST

Write in soft pencil.


Do not use staples, paper clips, glue or correction fluid.
Write your name, Centre number and candidate number on the Answer Sheet in the spaces provided
unless this has been done for you.
DO NOT WRITE IN ANY BARCODES.

There are forty questions on this paper. Answer all questions. For each question there are four possible
answers A, B, C and D.
Choose the one you consider correct and record your choice in soft pencil on the separate Answer Sheet.

Read the instructions on the Answer Sheet very carefully.

Each correct answer will score one mark. A mark will not be deducted for a wrong answer.
Any rough working should be done in this booklet.
A copy of the Periodic Table is printed on page 16.
Electronic calculators may be used.

This document consists of 15 printed pages and 1 blank page.

IB17 03_0653_22/2RP
© UCLES 2017 [Turn over

570/693
Combined By Nesrine
2
2023-2017

1 The diagram shows a palisade cell as seen under a light microscope.

Which structure converts light energy to chemical energy?

A
C

2 The graph shows the effect of pH on the activity of four different enzymes.

Which enzyme is most active in the stomach?

D
B

enzyme A
activity

0 1 2 3 4 5 6 7 8 9 10 11 12 13 14
pH

© UCLES 2017 0653/22/F/M/17

571/693
Combined By Nesrine
3
2023-2017

3 Microorganisms are used to convert milk into yoghurt.

The pH of the milk is regularly recorded as it turns into yoghurt.

Which graph shows the change in pH as the yoghurt is made?

A B

pH pH

time time

C D

pH pH

time time

4 What is the equation for photosynthesis?

A 6H2O + 6CO2 → C6H12O6 + 6O2

B 6H2O + 6O2 → C6H12O6 + 6CO2

C C6H12O6 + 6CO2 → 6O2 + 6H2O

D C6H12O6 + 6O2 → 6CO2 + 6H2O

5 Which statement describes transpiration?

A evaporation of water from leaf mesophyll cells


B intake of water from the atmosphere through the stomata
C transport of water through xylem tissue to the leaves
D uptake of water by root hairs in the soil

© UCLES 2017 0653/22/F/M/17 [Turn over

572/693
Combined By Nesrine
4
2023-2017

6 Oxygenated blood returns to the heart from the lungs in vessel X and leaves the heart to circulate
around the body in vessel Y.

What are X and Y?

X Y

A aorta pulmonary vein


B pulmonary artery vena cava
C pulmonary vein aorta
D vena cava pulmonary artery

7 Which row is correct for aerobic respiration?

relative amount of
gas used
energy released

A large carbon dioxide


B large oxygen
C small carbon dioxide
D small oxygen

8 Which component of cigarette smoke reduces the oxygen-carrying capacity of the blood?

A carbon monoxide
B nicotine
C smoke particles
D tar

9 The diagram shows a seedling with its root and shoot horizontal.

The seedling is kept moist for three days.

Where will the greatest concentration of auxin be found?

D C
shoot root

© UCLES 2017 0653/22/F/M/17

573/693
Combined By Nesrine
5
2023-2017

10 The diagram shows a section through a flower.

Which row correctly identifies structure P and the method of pollination in the flower?

method of
structure P
pollination

A anther insect
B anther wind
C stigma insect
D stigma wind

11 How does HIV affect the immune system?

A It decreases the number of platelets.


B It destroys red blood cells.
C It increases the number of white blood cells.
D It reduces antibody formation.

12 Energy flows along a food chain.

What does every food chain start with?

A carnivore
B consumer
C herbivore
D producer

© UCLES 2017 0653/22/F/M/17 [Turn over

574/693
Combined By Nesrine
6
2023-2017

13 Which two gases contribute most to global warming?

A carbon dioxide and methane


B carbon monoxide and carbon dioxide
C methane and oxygen
D oxygen and carbon monoxide

14 Which diagram represents molecules of hydrogen gas?

A B C D

15 Magnesium reacts with sulfur, to form magnesium sulfide.

Magnesium sulfide is an ionic compound.

Which statement is not correct?

A Electrons are shared between sulfur and magnesium.


B Electrons are transferred from magnesium to sulfur.
C The magnesium ions are positively charged.
D The sulfur atoms gain electrons.

16 Which row shows the formulae of sodium hydroxide and of potassium hydroxide?

sodium potassium
hydroxide hydroxide

A NaOH KOH
B NaOH POH
C SOH KOH
D SOH POH

© UCLES 2017 0653/22/F/M/17

575/693
Combined By Nesrine
7
2023-2017

17 Molten sodium chloride can be electrolysed.

Which row describes what happens at each electrode?

anode cathode

A chloride ions gain electrons sodium ions lose electrons


B chloride ions lose electrons sodium ions gain electrons
C sodium ions gain electrons chloride ions lose electrons
D sodium ions lose electrons chloride ions gain electrons

18 In which change is chemical energy transformed into heat (thermal energy)?

A combustion of refinery gas


B cracking of alkanes into alkenes
C distillation of seawater

D electrolysis of molten lead(II) bromide

19 The rate of reaction between magnesium and dilute hydrochloric acid is measured. The reaction
is repeated at a different temperature and the rate of reaction increases.

Which statement describes the second reaction?

A A higher temperature is used and the particles collide less often.


B A higher temperature is used and the particles collide more often.
C A lower temperature is used and the particles collide less often.
D A lower temperature is used and the particles collide more often.

20 Copper oxide is ……1…… in water. Excess copper oxide reacts with warm dilute sulfuric acid.
When the reaction is complete, the mixture is ……2…… to obtain copper sulfate solution.

Which words correctly complete gaps 1 and 2?

1 2

A insoluble distilled
B insoluble filtered
C soluble distilled
D soluble filtered

© UCLES 2017 0653/22/F/M/17 [Turn over

576/693
Combined By Nesrine
8
2023-2017

21 Which aqueous reagents give a white precipitate when added to aqueous zinc chloride?

sodium barium silver


hydroxide nitrate nitrate

A   
B   
C   
D   

22 Which statements about elements in the Periodic Table are correct?

1 The higher the group number, the more metallic an element is.
2 The group number is equal to the number of electrons in the outer shell.
3 Elements on the left of the Periodic Table form ions by losing electrons.

A 1 only B 1 and 2 C 1 and 3 D 2 and 3

23 Element X is a very soft solid.

It reacts violently with water.

A purple flame is seen as it reacts with water.

What is X?

A iodine
B potassium
C sodium
D zinc

24 Which diagrams represent alloys?

1 2 3 4

– + – + – +
+ – + – + –
– + – + – +
+ – + – + –

A 1 and 2 B 1 and 4 C 2 and 3 D 3 and 4

© UCLES 2017 0653/22/F/M/17

577/693
Combined By Nesrine
9
2023-2017

25 Iron occurs in the ground as iron oxide.

Gold occurs in the ground as the element.

Which statement explains this observation?

A Gold is more reactive than iron.


B Gold oxide is more reactive than iron oxide.
C Iron is more reactive than gold.
D Iron oxide is more reactive than gold oxide.

26 Which chemical test shows the presence of water?

A Water has a boiling point of 100 °C.

B Water has a freezing point of 0 °C.


C Water turns anhydrous cobalt chloride from blue to pink.
D Water turns anhydrous copper sulfate from blue to white.

27 Which pair of hydrocarbons can be distinguished from each other by the addition of bromine
water?

H H

A C C and butane

H H

H H H

B H C C C and butene

H H

H H

C H C C H and methane

H H

H H

D C C and propene

H H

© UCLES 2017 0653/22/F/M/17 [Turn over

578/693
Combined By Nesrine
10
2023-2017

28 Diagrams 1 and 2 show a distance-time graph and a speed-time graph.

distance speed

0 0
0 time 0 time

diagram 1 diagram 2

Which of the diagrams represents the motion of an object that is moving with constant
acceleration and then with constant speed?

A diagram 1 and diagram 2


B diagram 1 only
C diagram 2 only
D neither of the diagrams

29 A solid rectangular metal block has the dimensions shown. The density of the metal is 8.0 g / cm3.

5.0 cm

4.0 cm

10.0 cm

What is the mass of the metal block?

A 160 g B 320 g C 400 g D 1600 g

© UCLES 2017 0653/22/F/M/17

579/693
Combined By Nesrine
11
2023-2017

30 The extension-load graph for a rubber band is shown.

extension

0
0 5 10
load / N

Which statement about the rubber band is correct?

A An increase in the load from 1.0 N to 2.0 N has a smaller effect on the extension than an
increase from 5.0 N to 6.0 N.
B The extension is directly proportional to the load for loads greater than 5.0 N.
C The rubber band could be used as a spring balance with a linear (evenly spaced) scale.
D The rubber band is more difficult to stretch as the load becomes greater.

31 A lifting system contains an electric motor with an input power of 500 W. The system has an
efficiency of 60%.

How much power is wasted by the lifting system?

A 200 W B 300 W C 20 000 W D 30 000 W

32 A container with fixed volume is filled with a substance. The atoms in the substance are far apart
and move in straight lines until they strike something.

The substance is now heated without changing state.

Which row gives the state of the substance and the effect of heating on its atoms?

effect on atoms
state of substance
when heated

A gas expand
B gas move more quickly
C liquid expand
D liquid move more quickly

© UCLES 2017 0653/22/F/M/17 [Turn over

580/693
Combined By Nesrine
12
2023-2017

33 Diagram 1 shows a force F lifting a weight through a height h.

Diagram 2 shows the same force F lifting the same weight through a height 2h.

In both cases, air resistance and friction are negligible.

force F force F

2h
h
weight weight

diagram 1 diagram 2

Each lift can take either 1 s or 10 s.

Which row shows the greatest power being developed when the weight is lifted?

height time taken


lifted for the lift / s

A h 1
B h 10
C 2h 1
D 2h 10

34 In which states of matter is convection the main heat transfer process?

A gases and solids only


B liquids and gases only
C solids and liquids only
D solids, liquids and gases

35 A vibrating object hits the surface of a liquid every 0.050 s. This causes a wave to spread out over
the surface at a speed of 30 cm / s.

What is the wavelength of the wave?

A 0.0017 cm B 0.67 cm C 1.5 cm D 600 cm

© UCLES 2017 0653/22/F/M/17

581/693
Combined By Nesrine
13
2023-2017

36 A man walks towards a large plane mirror at a constant speed of 2.0 m / s. An image of the man is
produced by reflection.

Which statement about the man and his image is correct?

A The distance between them decreases at 2.0 m / s.


B The distance between them decreases at 4.0 m / s.
C The distance between them remains constant.
D They approach each other at an increasing speed.

37 A distant star emits a short pulse containing light waves, microwaves and radio waves. The
waves travel to Earth in vacuo (in a vacuum).

Which waves reach the Earth first?

A the light waves


B the microwaves
C the radio waves
D they all arrive at the same time

38 Four loudspeakers each vibrate at the frequencies shown.

Which loudspeaker produces the lowest-pitched sound that can be heard by a human?

A 5.0 Hz B 10 Hz C 5.0 × 103 Hz D 10 × 103 Hz

© UCLES 2017 0653/22/F/M/17 [Turn over

582/693
Combined By Nesrine
14
2023-2017

39 The diagrams show four circuits.

Which circuit can be used to find the resistance of resistor R?

A B

R R
V A

A V

C D

R
A V
R

V A

40 A 12 volt battery is connected to two resistors.

Currents P, Q and R, and potential differences S and T across the two resistors are labelled.

12 volt

R
S
P

T
Q

Which row gives the relationship between the three currents, and gives the values of S and T ?

currents S / volt T / volt

A P=Q=R 6.0 6.0


B P=Q=R 12 12
C P+Q=R 6.0 6.0
D P+Q=R 12 12

© UCLES 2017 0653/22/F/M/17

583/693
Combined By Nesrine
2023-2017

Cambridge International Examinations


Cambridge International General Certificate of Secondary Education

COMBINED SCIENCE 0653/21


Paper 2 Multiple Choice (Extended) May/June 2017
45 minutes
Additional Materials: Multiple Choice Answer Sheet
Soft clean eraser
*4667062358*

Soft pencil (type B or HB is recommended)

READ THESE INSTRUCTIONS FIRST

Write in soft pencil.


Do not use staples, paper clips, glue or correction fluid.
Write your name, Centre number and candidate number on the Answer Sheet in the spaces provided
unless this has been done for you.
DO NOT WRITE IN ANY BARCODES.

There are forty questions on this paper. Answer all questions. For each question there are four possible
answers A, B, C and D.
Choose the one you consider correct and record your choice in soft pencil on the separate Answer Sheet.

Read the instructions on the Answer Sheet very carefully.

Each correct answer will score one mark. A mark will not be deducted for a wrong answer.
Any rough working should be done in this booklet.
A copy of the Periodic Table is printed on page 20.
Electronic calculators may be used.

This document consists of 17 printed pages and 3 blank pages.

IB17 06_0653_21/3RP
© UCLES 2017 [Turn over

584/693
Combined By Nesrine
2
2023-2017

1 Process Q happens in cells.

carbohydrates → process Q → energy released

What is process Q?

A growth
B nutrition
C respiration
D sensitivity

2 Which row shows the site of chemical reactions in a cell and identifies the selectively permeable
structure in a cell?

selectively
site of chemical
permeable
reactions
structure

A cytoplasm cell membrane


B cytoplasm cell wall
C vacuole cell membrane
D vacuole cell wall

3 Which statements about enzymes are correct?

1 Enzymes are proteins.


2 Some enzymes carry out chemical digestion.
3 Enzymes speed up the rate of chemical reactions.
4 All enzymes work fastest at pH 7.

A 1, 2 and 3 B 1 and 2 only C 1 and 3 only D 2, 3 and 4

4 Which substance in leaves traps light energy for use in photosynthesis?

A carbohydrate
B carbon
C carbon dioxide
D chlorophyll

© UCLES 2017 0653/21/M/J/17

585/693
Combined By Nesrine
3
2023-2017

5 The statements show how a person’s diet can be unbalanced.

1 eating too much fibre


2 eating too much saturated fat
3 eating too much salt

Which of these increase the risk of coronary heart disease?

A 1, 2 and 3 B 1 and 2 only C 1 and 3 only D 2 and 3 only

6 Which row matches the adaptation of a root hair cell to its function?

adaptation function

A large surface area uptake of water and glucose


B large surface area uptake of water and ions
C small surface area uptake of water and glucose
D small surface area uptake of water and ions

7 The diagram shows a section through the heart.

The ventricles contract and blood is forced into the arteries.

What is the state of valves 1 and 2 when this happens?

valve 1 valve 2

A closed closed
B closed open
C open closed
D open open

© UCLES 2017 0653/21/M/J/17 [Turn over

586/693
Combined By Nesrine
4
2023-2017

8 Which molecule contains the energy that is released in aerobic respiration?

A C6H12O6 B CO2 C H2O D O2

9 The diagram shows apparatus at the start of a breathing experiment.

mouthpiece

limewater

tube X tube Y

A person breathes in and out through the mouthpiece for a short time.

Which row shows the results?

limewater in tube X limewater in tube Y

A stays clear stays clear


B stays clear turns cloudy
C turns cloudy stays clear
D turns cloudy turns cloudy

10 A shoot is illuminated from one side only.

What collects on the dark side of the shoot?

A auxin
B chlorophyll
C glucose
D starch

© UCLES 2017 0653/21/M/J/17

587/693
Combined By Nesrine
5
2023-2017

11 Materials are exchanged between a mother and her fetus across the placenta.

Which row shows the overall direction of movement of these materials?

mother to fetus fetus to mother

A amino acids glucose


B amino acids urea
C carbon dioxide glucose
D carbon dioxide urea

12 Which type of organism makes its own organic nutrients?

A carnivore
B consumer
C herbivore
D producer

13 What is an undesirable effect of overuse of fertilisers in agriculture?

A acid rain
B deforestation
C eutrophication
D global warming

© UCLES 2017 0653/21/M/J/17 [Turn over

588/693
Combined By Nesrine
6
2023-2017

14 Which diagram shows how a mixture of dyes in a food colouring are separated?

A B

filter paper filter paper

spot of food
colouring
food colouring
water solution

C D
spot of food
colouring
filter paper

filter paper spot of food


colouring

water water

15 Which statement describes a mixture?

A It contains molecules made from the same type of atom.


B It contains only one type of atom.
C It contains two different types of atom joined by chemical bonds.
D It contains two different types of atom that can be separated by physical processes.

© UCLES 2017 0653/21/M/J/17

589/693
Combined By Nesrine
7
2023-2017

16 The atomic (proton) number of magnesium is 12.

Which diagram shows the electronic structure of a magnesium atom?

A B
key
= electron
= nucleus

C D

17 Aluminium ions have the formula Al 3+.

Oxide ions have the formula O2–.

What is the formula of aluminium oxide?

A Al O B Al O2 C Al 2O3 D Al 3O2

18 Molten sodium chloride is electrolysed.

Which equations represent the reactions at the electrodes?

anode cathode

A 2Cl – → Cl 2 + 2e– Na+ + e– → Na


B Cl 2 + 2e– → 2Cl – Na → Na+ + e–
C Na → Na+ + e– Cl 2 + 2e– → 2Cl –
D Na+ + e– → Na 2Cl – → Cl 2 + 2e–

© UCLES 2017 0653/21/M/J/17 [Turn over

590/693
Combined By Nesrine
8
2023-2017

19 Which statement about chemical reactions is not correct?

A A higher temperature increases the rate of an endothermic reaction.


B Chemical energy is converted into thermal energy in an endothermic reaction.
C Temperature decreases in an endothermic reaction and there is an increase in chemical
energy.
D Temperature increases in an exothermic reaction because there is an increase in thermal
energy.

20 Hydrogen peroxide decomposes to form water and oxygen.

Which changes in temperature and in concentration both reduce the rate of this reaction?

temperature of concentration of
hydrogen peroxide hydrogen peroxide

A decrease decrease
B decrease increase
C increase decrease
D increase increase

21 In which word equation is copper reduced?

A anhydrous copper sulfate + water → hydrated copper sulfate

B copper carbonate + hydrochloric acid → copper chloride + water + carbon dioxide

C copper oxide + hydrogen → copper + water

D copper + oxygen → copper oxide

22 Acidified barium nitrate solution is added to solution X. A white precipitate forms.

What is X?

A hydrochloric acid
B limewater
C potassium chloride
D sulfuric acid

© UCLES 2017 0653/21/M/J/17

591/693
Combined By Nesrine
9
2023-2017

23 Which element is a non-metallic solid at room temperature?

melting point number of electrons


/ °C in outer shell

A –210 5
B –7 7
C 98 1
D 3730 4

24 What is an alloy?

A a compound containing two metallic elements


B a compound containing two non-metallic elements
C a mixture containing two metallic elements
D a mixture containing two non-metallic elements

25 X, Y and Z are three metallic elements.

When Z is heated with the oxide of X, the element X is formed.

When X is added to a solution of Y2+ ions no reaction takes place.

What is the order of reactivity of the metals?

least most
reactive reactive

A X Y Z
B Y X Z
C Y Z X
D Z Y X

© UCLES 2017 0653/21/M/J/17 [Turn over

592/693
Combined By Nesrine
10
2023-2017

26 Which pie chart shows the proportions of gases in clean air?

A B C D

N2 N2 O2
N2
N2 O2
other
O2 CO2 gases
O2 CO2

CO2
other other CO2 other
gases gases gases

27 Which statement about the products of the fractional distillation of petroleum is not correct?

A Fractions obtained from high up the fractional distillation column have low boiling points.
B Fractions obtained from low down the fractional distillation column contain large molecules.
C Large molecules have weak intermolecular attractive forces.
D Refinery gas is used for heating and cooking.

28 The diagrams show two distance-time graphs and two speed-time graphs.

Which graph represents the motion of an object that is moving with constant acceleration?

A B

distance distance

0 0
0 time 0 time

C D

speed speed

0 0
0 time 0 time

© UCLES 2017 0653/21/M/J/17

593/693
Combined By Nesrine
11
2023-2017

29 Which row shows the unit for force, the unit for mass and the unit for weight?

force mass weight

A kg kg N
B kg N kg
C N kg N
D N N kg

30 A spring obeys Hooke’s law. A load of 10 N hangs from the spring and causes the spring to
extend by 12 mm.

Two springs, identical to the first one, are now joined as shown. A load of 5.0 N is hung from the
springs.

springs

5.0 N

What is the total extension of the combination of the two springs?

A 3.0 mm B 6.0 mm C 12 mm D 24 mm

© UCLES 2017 0653/21/M/J/17 [Turn over

594/693
Combined By Nesrine
12
2023-2017

31 A brick of mass of 3.0 kg rests on a shelf. The brick drops off the shelf. The brick hits the ground
at a speed of 8.0 m / s. Air resistance can be ignored.

The acceleration of free fall g is 10 m / s2.

How much kinetic energy did the brick have just before it hit the ground, and how much potential
energy did the brick have when it was on the shelf?

kinetic energy potential


before hitting energy on shelf
ground / J /J

A 24 24
B 24 96
C 96 0
D 96 96

32 A liquid changes into a gas and this causes the temperature of the liquid to change.

What is the name of this process, and how does the temperature change?

name of temperature
process change

A condensation decreases
B condensation increases
C evaporation decreases
D evaporation increases

© UCLES 2017 0653/21/M/J/17

595/693
Combined By Nesrine
13
2023-2017

33 Four identical metal tanks in a room each contain the same amount of water.

The water is at the same temperature as the room.

Two of the tanks are insulated, and two of the tanks are not insulated.

A cooling unit is placed in each of the tanks, in the position shown.

In which tank does all the water become cool the most quickly?

A B C D
insulation

cooling water cooling water


unit unit
water water

34 A wave travels through a substance from point X to point Y. The diagram shows the direction in
which particles of the substance vibrate.

X Y

Which row states the type of wave involved, and gives an example of this type of wave?

type of wave example

A longitudinal radio
B longitudinal sound
C transverse radio
D transverse sound

© UCLES 2017 0653/21/M/J/17 [Turn over

596/693
Combined By Nesrine
14
2023-2017

35 A ray of light is travelling in glass. The ray reaches a boundary with air and splits into two rays as
shown.

50°
air
glass

30° 30°

original
ray

What has happened to the original ray?

A It has been partially internally reflected.


B It has been partially internally refracted.
C It has been totally internally reflected.
D It has been totally internally refracted.

36 A space telescope is fitted with an infra-red detector, an ultraviolet detector and a visible light
detector.

An explosion on the surface of the Sun emits infra-red, ultraviolet and visible light at the same
time.

Which detector is the first to detect the explosion?

(Space is a vacuum.)

A the infra-red detector


B the ultraviolet detector
C the visible light detector
D all three detect it simultaneously

37 An electronic circuit in a fire alarm makes a loudspeaker vibrate alternately at two different
frequencies.

Which pair of frequencies is suitable to use in the alarm to alert people to the danger of fire?

A 1.5 Hz and 15 Hz
B 15 Hz and 150 000 Hz
C 150 Hz and 15 000 Hz
D 150 000 Hz and 15 000 000 Hz

© UCLES 2017 0653/21/M/J/17

597/693
Combined By Nesrine
15
2023-2017

38 The diagram shows a wire of length l and cross-sectional area X.

l
X

Which two changes must decrease the resistance of the wire?

A decrease l and decrease X

B decrease l and increase X


C increase l and decrease X
D increase l and increase X

39 An 800 W microwave oven and a 2500 W conventional electric oven are both designed to operate
from a 230 V supply.

Which row shows the rating of the fuse that should be fitted to each device?

microwave conventional
oven electric oven

A 5A 5A
B 5A 13 A
C 13 A 5A
D 13 A 13 A

© UCLES 2017 0653/21/M/J/17 [Turn over

598/693
Combined By Nesrine
16
2023-2017

40 The diagram shows a circuit containing a battery and four resistors. One resistor is labelled R.
Some values of p.d. and current are shown.

12 V

4.0 A
7.0 V R 2.0 V

A
3.0 A

What is the p.d. across resistor R, and what is the current in resistor R?

p.d. / V current / A

A 3.0 1.0
B 3.0 4.0
C 12 1.0
D 12 4.0

© UCLES 2017 0653/21/M/J/17

599/693
Combined By Nesrine
2023-2017

Cambridge International Examinations


Cambridge International General Certificate of Secondary Education

COMBINED SCIENCE 0653/22


Paper 2 Multiple Choice (Extended) May/June 2017
45 minutes
Additional Materials: Multiple Choice Answer Sheet
Soft clean eraser
*1153441284*

Soft pencil (type B or HB is recommended)

READ THESE INSTRUCTIONS FIRST

Write in soft pencil.


Do not use staples, paper clips, glue or correction fluid.
Write your name, Centre number and candidate number on the Answer Sheet in the spaces provided
unless this has been done for you.
DO NOT WRITE IN ANY BARCODES.

There are forty questions on this paper. Answer all questions. For each question there are four possible
answers A, B, C and D.
Choose the one you consider correct and record your choice in soft pencil on the separate Answer Sheet.

Read the instructions on the Answer Sheet very carefully.

Each correct answer will score one mark. A mark will not be deducted for a wrong answer.
Any rough working should be done in this booklet.
A copy of the Periodic Table is printed on page 16.
Electronic calculators may be used.

This document consists of 16 printed pages.

IB17 06_0653_22/2RP
© UCLES 2017 [Turn over

600/693
Combined By Nesrine
2
2023-2017

1 A person moves their hand away from a hot object.

Which characteristic of living organisms is this?

A growth
B nutrition
C reproduction
D sensitivity

2 The diagram shows an animal cell under a light microscope.

What is the function of part X?

A to carry out photosynthesis


B to let molecules in and out of the cell
C to store and pass on cell information
D to support and protect the cell

3 The diagram shows an image of a plant cell that has been magnified.

100 mm

The magnification is × 200.

What is the length of the actual cell?

A 0.2 mm B 0.5 mm C 2 mm D 20 000 mm

© UCLES 2017 0653/22/M/J/17

601/693
Combined By Nesrine
3
2023-2017

4 Which statement about enzymes is correct?

A They are killed by high temperatures.


B They are made from amino acids.
C They are unaffected by pH.
D They are used up in biological reactions.

5 Which row matches the adaptation of a root hair cell to its function?

adaptation function

A large surface area uptake of water and glucose


B large surface area uptake of water and ions
C small surface area uptake of water and glucose
D small surface area uptake of water and ions

6 The diagram shows a section through the heart.

The ventricles contract and blood is forced into the arteries.

What is the state of valves 1 and 2 when this happens?

valve 1 valve 2

A closed closed
B closed open
C open closed
D open open

© UCLES 2017 0653/22/M/J/17 [Turn over

602/693
Combined By Nesrine
4
2023-2017

7 The diagram shows apparatus at the start of a breathing experiment.

mouthpiece

limewater

tube X tube Y

A person breathes in and out through the mouthpiece for a short time.

Which row shows the results?

limewater in tube X limewater in tube Y

A stays clear stays clear


B stays clear turns cloudy
C turns cloudy stays clear
D turns cloudy turns cloudy

8 How does mucus benefit the gas exchange system?

A It absorbs carbon monoxide before it reaches the alveoli.


B It prevents friction between the air and the trachea.
C It removes the nicotine in cigarette smoke.
D It traps pathogens.

© UCLES 2017 0653/22/M/J/17

603/693
Combined By Nesrine
5
2023-2017

9 The diagram shows a seedling with its root and shoot horizontal.

The seedling is kept moist for three days.

Where will the greatest concentration of auxin be found?

B
root

A D
shoot

10 Which substance is at a higher concentration in the blood on the fetal side of the placenta than in
the blood on the mother’s side of the placenta?

A amino acids
B carbon dioxide
C glucose
D oxygen

11 Which type of blood cell is affected by the human immuno-deficiency virus (HIV) and what effect
does the virus have on those cells?

type of blood cell effect on the blood cell

A red prevents them carrying oxygen


B red stops them from making the blood clot
C white stops them from performing phagocytosis
D white reduces antibody production

12 What is an ecosystem?

A a network of inter-connected food chains


B a specific area in which a plant or an animal lives
C all the plants and animals living within a specific area
D the interactions between all living organisms, in a specific area, and their environment

© UCLES 2017 0653/22/M/J/17 [Turn over

604/693
Combined By Nesrine
6
2023-2017

13 What is an undesirable effect of overuse of fertilisers in agriculture?

A acid rain
B deforestation
C eutrophication
D global warming

14 Which diagram shows how a mixture of dyes in a food colouring are separated?

A B

filter paper filter paper

spot of food
colouring
food colouring
water solution

C D
spot of food
colouring
filter paper

filter paper spot of food


colouring

water water

15 A mixture contains hydrogen, helium, neon and oxygen.

What describes this mixture?

A elements and compounds


B elements only
C molecules and compounds
D molecules only

© UCLES 2017 0653/22/M/J/17

605/693
Combined By Nesrine
7
2023-2017

16 Which row describes the electronic structures of a chlorine atom and of a sodium ion?

chlorine atom sodium ion

A 2,8,7 2,8
B 2,8,7 2,8,8
C 2,8,8 2,8
D 2,8,8 2,8,8

17 Aluminium ions have the formula Al 3+.

Oxide ions have the formula O2–.

What is the formula of aluminium oxide?

A Al O B Al O2 C Al 2O3 D Al 3O2

18 Aqueous copper chloride is electrolysed using inert electrodes.

Which row describes what happens at each electrode?

cathode anode
A chloride ions gain copper ions lose
electrons to form chlorine electrons to form copper
B chloride ions lose copper ions gain
electrons to form chlorine electrons to form copper
C copper ions gain chloride ions lose
electrons to form copper electrons to form chlorine
D copper ions lose chloride ions gain
electrons to form copper electrons to form chlorine

19 Methane reacts with oxygen, releasing heat.

Which statement explains the energy changes in this reaction?

A Chemical energy is changed into thermal energy.


B Energy is made in the reaction.
C The heat released increases the temperature of the surroundings.
D The reaction is endothermic.

© UCLES 2017 0653/22/M/J/17 [Turn over

606/693
Combined By Nesrine
8
2023-2017

20 Magnesium ribbon reacts with dilute hydrochloric acid to form hydrogen gas.

Which change increases the rate of the reaction?

A adding water to the mixture


B trapping the hydrogen gas
C using a lower temperature
D using powdered magnesium

21 In which reactions is the underlined substance oxidised?

1 iron when it rusts


2 methane when it burns in air
3 copper oxide when it reacts with carbon

A 1, 2 and 3 B 1 and 2 only C 1 and 3 only D 2 and 3 only

22 Substance X is warmed with aqueous sodium hydroxide and aluminium.

A gas is produced which turns damp red litmus paper blue.

Which anion is present in X?

A carbonate
B hydroxide
C nitrate
D sulfate

© UCLES 2017 0653/22/M/J/17

607/693
Combined By Nesrine
9
2023-2017

23 The elements in Group VII of the Periodic Table are shown.

fluorine
chlorine
bromine
iodine
astatine

Which statement is not correct?

A Astatine has a lower melting point than iodine.


B Chlorine can displace iodine from an iodide solution.
C Fluorine is more reactive than astatine.
D Iodine vapour has a darker colour than fluorine gas.

24 What is an alloy?

A a compound containing two metallic elements


B a compound containing two non-metallic elements
C a mixture containing two metallic elements
D a mixture containing two non-metallic elements

25 Which pair of substances produces a metal when they are mixed together?

A copper and aqueous iron(II) ions


B iron and aqueous zinc ions

C magnesium and aqueous copper(II) ions

D zinc and aqueous magnesium ions

© UCLES 2017 0653/22/M/J/17 [Turn over

608/693
Combined By Nesrine
10
2023-2017

26 Which pie chart shows the proportions of gases in clean air?

A B C D

N2 N2 O2
N2
N2 O2
other
O2 CO2 gases
O2 CO2

CO2
other other CO2 other
gases gases gases

27 Petroleum is separated into different fractions by fractional distillation.

Which statement about fractional distillation is not correct?

A Larger molecules have stronger covalent bonds between their atoms.


B The boiling point of hydrocarbons increases with the size of the molecules.
C The different fractions have different boiling points.
D The smaller molecules have weaker intermolecular attractive forces between them.

© UCLES 2017 0653/22/M/J/17

609/693
Combined By Nesrine
11
2023-2017

28 The diagram shows the speed-time graph for a vehicle.

12.0
speed
m/s

6.0

0
0 30
time / s

What is the acceleration of the vehicle, and how far does it travel in 30 s?

acceleration distance
m / s2 travelled / m

A 0.20 180
B 0.20 270
C 0.40 180
D 0.40 270

29 Which row shows the unit for force, the unit for mass and the unit for weight?

force mass weight

A kg kg N
B kg N kg
C N kg N
D N N kg

© UCLES 2017 0653/22/M/J/17 [Turn over

610/693
Combined By Nesrine
12
2023-2017

30 A spring obeys Hooke’s law. A load of 10 N hangs from the spring and causes the spring to
extend by 12 mm.

Two springs, identical to the first one, are now joined as shown. A load of 5.0 N is hung from the
springs.

springs

5.0 N

What is the total extension of the combination of the two springs?

A 3.0 mm B 6.0 mm C 12 mm D 24 mm

31 A brick of mass of 3.0 kg rests on a shelf. The brick drops off the shelf. The brick hits the ground
at a speed of 8.0 m / s. Air resistance can be ignored.

The acceleration of free fall g is 10 m / s2.

How much kinetic energy did the brick have just before it hit the ground, and how much potential
energy did the brick have when it was on the shelf?

kinetic energy potential


before hitting energy on shelf
ground / J /J

A 24 24
B 24 96
C 96 0
D 96 96

© UCLES 2017 0653/22/M/J/17

611/693
Combined By Nesrine
13
2023-2017

32 Two cylinders contain the same type of gas.

In which case must the gas in one cylinder be at a higher temperature than the gas in the other
cylinder?

A In one cylinder the gas molecules are moving faster.


B In one cylinder the gas occupies a smaller volume.
C In one cylinder there is a greater number of gas molecules.
D In one cylinder there is a greater spacing between the gas molecules.

33 Which surface is a better absorber of infra-red radiation, and which surface is a better emitter of
infra-red radiation?

better absorber better emitter

A black surface black surface


B black surface white surface
C white surface black surface
D white surface white surface

34 What can cause the speed of a wave to change?

A either reflection or refraction


B reflection only
C refraction only
D neither reflection nor refraction

35 The diagram represents the surface of a transparent liquid. Two rays of light are travelling within
the liquid. They both reach the surface. The path of each ray is shown.

air
liquid 50° 55°

What is the critical angle for this liquid?

A 35° B 40° C 50° D 55°

© UCLES 2017 0653/22/M/J/17 [Turn over

612/693
Combined By Nesrine
14
2023-2017

36 Which type of electromagnetic wave is used in airport security scanners?

A gamma-rays
B microwaves
C radio waves
D X-rays

37 An electronic circuit in a fire alarm makes a loudspeaker vibrate alternately at two different
frequencies.

Which pair of frequencies is suitable to use in the alarm to alert people to the danger of fire?

A 1.5 Hz and 15 Hz
B 15 Hz and 150 000 Hz
C 150 Hz and 15 000 Hz
D 150 000 Hz and 15 000 000 Hz

38 Four wires are made from the same material but have different lengths and diameters.

Which wire has the least resistance?

length diameter
/ cm / mm

A 50 0.10
B 50 0.20
C 100 0.10
D 100 0.20

39 A 12 V power supply is connected to a 6.0 Ω resistor. This causes a current in the resistor.

How much thermal energy is produced in the resistor in 5.0 minutes?

A 120 J B 600 J C 7200 J D 21 600 J

© UCLES 2017 0653/22/M/J/17

613/693
Combined By Nesrine
15
2023-2017

40 The diagram shows a circuit containing four resistors and four ammeters.

Which ammeter has the smallest reading?

A A A D

10 Ω 40 Ω
B
30 Ω
A

20 Ω
A
C

Permission to reproduce items where third-party owned material protected by copyright is included has been sought and cleared where possible. Every
reasonable effort has been made by the publisher (UCLES) to trace copyright holders, but if any items requiring clearance have unwittingly been included, the
publisher will be pleased to make amends at the earliest possible opportunity.

To avoid the issue of disclosure of answer-related information to candidates, all copyright acknowledgements are reproduced online in the Cambridge
International Examinations Copyright Acknowledgements Booklet. This is produced for each series of examinations and is freely available to download at
www.cie.org.uk after the live examination series.

Cambridge International Examinations is part of the Cambridge Assessment Group. Cambridge Assessment is the brand name of University of Cambridge Local
Examinations Syndicate (UCLES), which is itself a department of the University of Cambridge.

© UCLES 2017 0653/22/M/J/17

614/693
Combined By Nesrine
2023-2017

Cambridge International Examinations


Cambridge International General Certificate of Secondary Education

COMBINED SCIENCE 0653/23


Paper 2 Multiple Choice (Extended) May/June 2017
45 minutes
Additional Materials: Multiple Choice Answer Sheet
Soft clean eraser
*5348792529*

Soft pencil (type B or HB is recommended)

READ THESE INSTRUCTIONS FIRST

Write in soft pencil.


Do not use staples, paper clips, glue or correction fluid.
Write your name, Centre number and candidate number on the Answer Sheet in the spaces provided
unless this has been done for you.
DO NOT WRITE IN ANY BARCODES.

There are forty questions on this paper. Answer all questions. For each question there are four possible
answers A, B, C and D.
Choose the one you consider correct and record your choice in soft pencil on the separate Answer Sheet.

Read the instructions on the Answer Sheet very carefully.

Each correct answer will score one mark. A mark will not be deducted for a wrong answer.
Any rough working should be done in this booklet.
A copy of the Periodic Table is printed on page 16.
Electronic calculators may be used.

This document consists of 16 printed pages.

IB17 06_0653_23/2RP
© UCLES 2017 [Turn over

615/693
Combined By Nesrine
2
2023-2017

1 Which are characteristics of all living organisms?

A excretion, breathing and sensitivity


B excretion, movement and respiration
C gas exchange and muscle contraction
D muscle contraction and sensitivity

2 The diagram shows a plant cell and an animal cell.

plant cell animal cell

What is structure P and what is one of its functions?

structure function
A cell membrane controls the entry of
glucose into the cell
B cell membrane supports the cell
C cell wall controls the entry of
glucose into the cell
D cell wall supports the cell

© UCLES 2017 0653/23/M/J/17

616/693
Combined By Nesrine
3
2023-2017

3 The graph shows the effect of one variable on amylase activity.

What are the labels X and Y?

X Y

A amylase activity pH
B amylase activity temperature
C pH amylase activity
D temperature amylase activity

4 The equation summarises a process that occurs in living organisms.

6H2O + 6CO2 → C6H12O6 + 6O2

Which molecule contains the greatest amount of chemical energy?

A H2O B CO2 C C6H12O6 D O2

5 Which row matches the part of the alimentary canal to its function?

part of the alimentary canal function of part

A colon absorption of digested food


B ileum egestion
C mouth mechanical digestion
D pancreas production of bile

© UCLES 2017 0653/23/M/J/17 [Turn over

617/693
Combined By Nesrine
4
2023-2017

6 Which row matches the adaptation of a root hair cell to its function?

adaptation function

A large surface area uptake of water and glucose


B large surface area uptake of water and ions
C small surface area uptake of water and glucose
D small surface area uptake of water and ions

7 The diagram shows a section through the heart.

The ventricles contract and blood is forced into the arteries.

What is the state of valves 1 and 2 when this happens?

valve 1 valve 2

A closed closed
B closed open
C open closed
D open open

© UCLES 2017 0653/23/M/J/17

618/693
Combined By Nesrine
5
2023-2017

8 The diagram shows apparatus at the start of a breathing experiment.

mouthpiece

limewater

tube X tube Y

A person breathes in and out through the mouthpiece for a short time.

Which row shows the results?

limewater in tube X limewater in tube Y

A stays clear stays clear


B stays clear turns cloudy
C turns cloudy stays clear
D turns cloudy turns cloudy

9 Which component of tobacco smoke increases the risk of lung cancer?

A carbon dioxide
B carbon monoxide
C nicotine
D tar

© UCLES 2017 0653/23/M/J/17 [Turn over

619/693
Combined By Nesrine
6
2023-2017

10 The diagram shows a section through a flower.

Which row correctly identifies structure Q and the method of pollination in the flower?

method of
structure Q
pollination

A anther insect
B anther wind
C stigma insect
D stigma wind

11 What effect does HIV have on the components of blood?

A Blood does not clot as quickly.


B Plasma can no longer carry hormones.
C Red blood cells carry less oxygen.
D White blood cells make fewer antibodies.

12 Which statement about decomposers is not correct?

A They are the final stage of food chains.


B They break down dead organic matter.
C They produce oxygen.
D They release heat energy into the environment.

© UCLES 2017 0653/23/M/J/17

620/693
Combined By Nesrine
7
2023-2017

13 What is an undesirable effect of overuse of fertilisers in agriculture?

A acid rain
B deforestation
C eutrophication
D global warming

14 Which diagram shows how a mixture of dyes in a food colouring are separated?

A B

filter paper filter paper

spot of food
colouring
food colouring
water solution

C D
spot of food
colouring
filter paper

filter paper spot of food


colouring

water water

15 Which elements react together to give positive ions and negative ions that have the same
electronic structure as argon?

A calcium and chlorine


B calcium and fluorine
C magnesium and chlorine
D magnesium and fluorine

© UCLES 2017 0653/23/M/J/17 [Turn over

621/693
Combined By Nesrine
8
2023-2017

16 Which dot-and-cross diagram represents the arrangement of outer-shell electrons in a molecule


of nitrogen?

A B

N N N N

C D

N N N N

17 Aluminium ions have the formula Al 3+.

Oxide ions have the formula O2–.

What is the formula of aluminium oxide?

A Al O B Al O2 C Al 2O3 D Al 3O2

18 Molten sodium chloride is electrolysed.

What are the electrode products?

at the anode at the cathode

A chlorine hydrogen
B chlorine sodium
C hydrogen chlorine
D sodium chlorine

19 Which statement describes an exothermic reaction?

A It gives out thermal energy.


B It needs energy to start it.
C It neither gives out nor takes in energy.
D It takes in energy.

© UCLES 2017 0653/23/M/J/17

622/693
Combined By Nesrine
9
2023-2017

20 Apparatus used to measure the rate of a reaction, which produces a gas, is shown.

cotton wool

acid
marble

51.2 g balance

Which other piece of apparatus is needed?

A beaker
B gas syringe
C stopclock
D thermometer

21 Iron is extracted from its ore using carbon monoxide.

The word equation is shown.

iron(III) oxide + carbon monoxide → iron + carbon dioxide

Which statement is correct?

A Carbon monoxide is oxidised by gaining oxygen.


B Carbon monoxide is reduced by losing oxygen.

C Iron(III) oxide is oxidised by losing oxygen.

D Iron(III) oxide is reduced by gaining oxygen.

22 Which method can be used to make pure solid sodium nitrate, NaNO3?

A Add aqueous sodium hydroxide to a conical flask, titrate with dilute nitric acid, then
crystallise.
B Dissolve solid sodium chloride in dilute nitric acid, leave for 10 minutes and then crystallise.
C Heat sodium with nitrogen and oxygen. Let the mixture cool, then collect the solid that is
made.
D Mix copper nitrate and sodium chloride solutions then filter the mixture and collect the sodium
nitrate from the filter paper.

© UCLES 2017 0653/23/M/J/17 [Turn over

623/693
Combined By Nesrine
10
2023-2017

23 Information about an element in the Periodic Table is shown.

Which row describes an element in the Periodic Table?

number of
group number electrons in metal / non-metal
outer shell

A I 1 metal
B II 2 non-metal
C VI 2 non-metal
D VIII 8 metal

24 What is an alloy?

A a compound containing two metallic elements


B a compound containing two non-metallic elements
C a mixture containing two metallic elements
D a mixture containing two non-metallic elements

25 Which pie chart shows the proportions of gases in clean air?

A B C D

N2 N2 O2
N2
N2 O2
other
O2 CO2 gases
O2 CO2

CO2
other other CO2 other
gases gases gases

26 Which process does not contribute to increasing levels of carbon dioxide in the air?

A burning petrol and diesel in cars


B combustion of the sulfur compounds in petrol and diesel
C destroying rainforests
D releasing waste gases from coal-fired power stations

© UCLES 2017 0653/23/M/J/17

624/693
Combined By Nesrine
11
2023-2017

27 Which substance rapidly turns bromine from orange to colourless?

A ethane
B ethanol
C ethene
D methane

28 A car is travelling on a straight road at a speed of 2.0 m / s. It starts to accelerate constantly at


3.0 m / s2.

How long does it take for the speed of the car to reach 8.0 m / s?

A 0.50 s B 2.0 s C 2.7 s D 18 s

29 Which row shows the unit for force, the unit for mass and the unit for weight?

force mass weight

A kg kg N
B kg N kg
C N kg N
D N N kg

© UCLES 2017 0653/23/M/J/17 [Turn over

625/693
Combined By Nesrine
12
2023-2017

30 A spring obeys Hooke’s law. A load of 10 N hangs from the spring and causes the spring to
extend by 12 mm.

Two springs, identical to the first one, are now joined as shown. A load of 5.0 N is hung from the
springs.

springs

5.0 N

What is the total extension of the combination of the two springs?

A 3.0 mm B 6.0 mm C 12 mm D 24 mm

31 A brick of mass of 3.0 kg rests on a shelf. The brick drops off the shelf. The brick hits the ground
at a speed of 8.0 m / s. Air resistance can be ignored.

The acceleration of free fall g is 10 m / s2.

How much kinetic energy did the brick have just before it hit the ground, and how much potential
energy did the brick have when it was on the shelf?

kinetic energy potential


before hitting energy on shelf
ground / J /J

A 24 24
B 24 96
C 96 0
D 96 96

© UCLES 2017 0653/23/M/J/17

626/693
Combined By Nesrine
13
2023-2017

32 The molecules in a substance vibrate about fixed positions.

The substance is now cooled.

Which row gives the state of the substance and the effect of cooling on the distance between its
molecules?

state of effect on distance


substance between molecules

A liquid decreases
B liquid increases
C solid decreases
D solid increases

33 A solid is heated and it melts. The liquid that is produced is then heated and it boils.

What happens to the temperature of the solid while it is melting, and what happens to the
temperature of the liquid while it is boiling?

temperature of solid temperature of liquid

A increases increases
B increases remains constant
C remains constant increases
D remains constant remains constant

34 A cooling unit is to be fitted in a tank of water to cool all the water.

What is the best position for the unit to be fitted, and what is the main method of thermal energy
transfer in the water?

position to fit main method of


cooling unit thermal energy transfer

A at the bottom conduction


B at the bottom convection
C at the top conduction
D at the top convection

© UCLES 2017 0653/23/M/J/17 [Turn over

627/693
Combined By Nesrine
14
2023-2017

35 A microwave oven emits radiation with a frequency of 2.5 × 109 Hz.

What is the wavelength of these waves? The speed of light is 3.0 × 108 m / s.

A 0.0075 m B 0.12 m C 7.5 m D 120 m

36 The diagram shows part of the electromagnetic spectrum.

gamma-rays P ultraviolet Q infra-red

Which row shows the missing types of radiation at P and Q?

at P at Q

A radio waves microwaves


B radio waves visible light
C X-rays microwaves
D X-rays visible light

37 An electronic circuit in a fire alarm makes a loudspeaker vibrate alternately at two different
frequencies.

Which pair of frequencies is suitable to use in the alarm to alert people to the danger of fire?

A 1.5 Hz and 15 Hz
B 15 Hz and 150 000 Hz
C 150 Hz and 15 000 Hz
D 150 000 Hz and 15 000 000 Hz

38 The table gives the lengths and the diameters of four different wires made from the same metal.

Which wire has the smallest resistance?

length of diameter of
wire / m wire / mm

A 3.0 3.0
B 3.0 4.0
C 4.0 3.0
D 4.0 4.0

© UCLES 2017 0653/23/M/J/17

628/693
Combined By Nesrine
15
2023-2017

39 There is a current of 20 mA in an electrical component when there is a p.d. of 10 V across it.

How much energy is transferred by the component in 30 minutes?

A 6.0 J B 360 J C 6000 J D 360 000 J

40 The diagram shows a circuit containing a battery, three resistors and four voltmeters.

Which voltmeter has the greatest reading?

V
B
A
V

V V
D C

Permission to reproduce items where third-party owned material protected by copyright is included has been sought and cleared where possible. Every
reasonable effort has been made by the publisher (UCLES) to trace copyright holders, but if any items requiring clearance have unwittingly been included, the
publisher will be pleased to make amends at the earliest possible opportunity.

To avoid the issue of disclosure of answer-related information to candidates, all copyright acknowledgements are reproduced online in the Cambridge
International Examinations Copyright Acknowledgements Booklet. This is produced for each series of examinations and is freely available to download at
www.cie.org.uk after the live examination series.

Cambridge International Examinations is part of the Cambridge Assessment Group. Cambridge Assessment is the brand name of University of Cambridge Local
Examinations Syndicate (UCLES), which is itself a department of the University of Cambridge.

© UCLES 2017 0653/23/M/J/17

629/693
Combined By Nesrine
2023-2017

Cambridge International Examinations


Cambridge International General Certificate of Secondary Education

COMBINED SCIENCE 0653/21


Paper 2 Multiple Choice (Extended) October/November 2017
45 minutes
Additional Materials: Multiple Choice Answer Sheet
Soft clean eraser
*7877868817*

Soft pencil (type B or HB is recommended)

READ THESE INSTRUCTIONS FIRST

Write in soft pencil.


Do not use staples, paper clips, glue or correction fluid.
Write your name, Centre number and candidate number on the Answer Sheet in the spaces provided
unless this has been done for you.
DO NOT WRITE IN ANY BARCODES.

There are forty questions on this paper. Answer all questions. For each question there are four possible
answers A, B, C and D.
Choose the one you consider correct and record your choice in soft pencil on the separate Answer Sheet.

Read the instructions on the Answer Sheet very carefully.

Each correct answer will score one mark. A mark will not be deducted for a wrong answer.
Any rough working should be done in this booklet.
A copy of the Periodic Table is printed on page 16.
Electronic calculators may be used.

This document consists of 16 printed pages.

IB17 11_0653_21/3RP
© UCLES 2017 [Turn over

630/693
Combined By Nesrine
2
2023-2017

1 Which characteristics help to define a living organism?

A diffusion, movement, respiration


B excretion, nutrition, sensitivity
C excretion, reproduction, transpiration
D growth, inspiration, nutrition

2 The diagram shows a palisade cell.

Which structure converts energy from light into chemical energy?

C
A

3 Why does the rate of enzyme activity change when the temperature rises above the optimum
temperature?

A The enzyme has been denatured.


B The enzyme has been used up.
C The enzyme molecules are moving too slowly.
D The enzyme speeds up the rate of the reaction.

4 Which chemical is used to test for a food substance that contains the elements carbon, hydrogen,
nitrogen and oxygen?

A Benedict’s solution
B biuret solution
C ethanol
D iodine solution

© UCLES 2017 0653/21/O/N/17

631/693
Combined By Nesrine
3
2023-2017

5 Which letters from the list represent the balanced equation for photosynthesis?

P C6H12O6 T H2O
Q 6C6H12O6 U 6H2O
R CO2 V O2
S 6CO2 W 6O2

A P + U → R + V

B Q + T → S + U

C R + T → W + P

D U + S → P + W

6 In which order does food pass through parts of the alimentary canal?

A oesophagus → colon → small intestine

B small intestine → oesophagus → rectum

C small intestine → rectum → anus

D stomach → colon → small intestine

7 The diagram shows a plant cell.

What does structure X do?

A decreases the surface area of the cell for water and ion absorption
B decreases the surface area of the cell for water and sugar absorption
C increases the surface area of the cell for water and ion absorption
D increases the surface area of the cell for water and sugar absorption

© UCLES 2017 0653/21/O/N/17 [Turn over

632/693
Combined By Nesrine
4
2023-2017

8 The diagram shows the double circulation of blood around the human body.

Which blood vessel contains blood at the highest pressure?

lungs

A B

C D

body

9 The photomicrograph shows a sample of human blood.

What is the function of the cells marked X?

A antibody formation
B clotting of blood
C phagocytosis
D transport of oxygen

© UCLES 2017 0653/21/O/N/17

633/693
Combined By Nesrine
5
2023-2017

10 Which component of tobacco smoke reduces the ability of haemoglobin to carry oxygen?

A carbon monoxide
B nicotine
C smoke particles
D tar

11 During pregnancy, the fetus is contained within the amniotic sac. The amniotic sac contains
amniotic fluid.

What is the function of the amniotic fluid?

A It protects the fetus against knocks and bumps.


B It provides the fetus with oxygen and nutrients.
C It removes the fetal waste products.
D It supplies the fetus with blood.

12 The diagram represents part of the carbon cycle.

1
atmospheric
green plants
carbon dioxide
2

animals
4

microorganisms

Which arrows show where respiration takes place?

A 1, 3 and 4 B 1 and 3 only C 2, 3 and 4 D 2 and 3 only

13 Which gas dissolves in water vapour to produce acid rain?

A methane
B nitrogen
C oxygen
D sulfur dioxide

© UCLES 2017 0653/21/O/N/17 [Turn over

634/693
Combined By Nesrine
6
2023-2017

14 The formulae of three substances are shown.

substance formula

methane CH4
water H2O
oxygen O2

Which statement is correct?

A Methane is made from five different types of atom.


B Methane, water and oxygen are molecules.
C Only methane and water are molecules.
D Oxygen is made from two different types of atom.

15 Which process is used to separate petroleum?

A crystallisation
B distillation
C filtration
D fractional distillation

16 What is the electronic structure of a chlorine atom, Cl, and of a chloride ion, Cl –?

chlorine atom chloride ion

A 2,8,6 2,8,8
B 2,8,7 2,8,6
C 2,8,7 2,8,8
D 2,8,8 2,8,7

© UCLES 2017 0653/21/O/N/17

635/693
Combined By Nesrine
7
2023-2017

17 Element Q and element R combine to form a covalent compound, Q2R.

The arrangement of the outer-shell electrons in the compound is shown.

Q R Q

Which compound has the same arrangement of outer shell electrons as Q2R?

A carbon dioxide
B hydrogen chloride
C methane
D water

18 Aluminium sulfate contains aluminium ions, Al 3+, and sulfate ions, SO42–.

Iron(II) nitride contains iron(II) ions, Fe2+, and nitride ions, N3–.

What are the formulae of aluminium sulfate and of iron(II) nitride?

aluminium sulfate iron(II) nitride

A Al 2(SO4)3 Fe2N3
B Al 2(SO4)3 Fe3N2
C Al 3(SO4)2 Fe2N3
D Al 3(SO4)2 Fe3N2

19 What is produced at the anode during the electrolysis of molten lead(II) bromide?

A bromide ions
B bromine
C lead
D lead(II) ions

© UCLES 2017 0653/21/O/N/17 [Turn over

636/693
Combined By Nesrine
8
2023-2017

20 The diagram shows gas X burning and heating a liquid.

liquid

flame

gas X

Which row is correct?

the burning of gas X


gas X
is exothermic

A hydrogen 
B hydrogen 
C oxygen 
D oxygen 

21 Gases X and Y react together to form gas Z.

The equation for the reaction is shown.

2X(g) + Y(g) → Z(g)

The total volume of gas is measured as the reaction occurs. The dotted line in the graph shows
the results.

The reaction is repeated using the same volumes of X and Y under the same conditions but with
the addition of a catalyst.

Which line shows the results for the second experiment?

A
total B
volume
of gas C

time / s

© UCLES 2017 0653/21/O/N/17

637/693
Combined By Nesrine
9
2023-2017

22 Carbon reacts with carbon dioxide at high temperatures.

carbon + carbon dioxide → carbon monoxide

Which statement about the reaction is correct?

A Both carbon and carbon dioxide are oxidised.


B Both carbon and carbon dioxide are reduced.
C The carbon is oxidised and the carbon dioxide is reduced.
D The carbon is reduced and the carbon dioxide is oxidised.

23 Excess aqueous barium nitrate is added to dilute sulfuric acid to produce barium sulfate.

How is barium sulfate obtained from the reaction mixture?

A electrolysis
B evaporation
C filtration
D fractional distillation

24 Which statement about elements in the Periodic Table is correct?

A Barium is a non-metal in Group II and its atoms have two electrons in their outer shells.
B Chlorine is a non-metal in Group VII and its atoms have seven electrons in their outer shells.
C Fluorine is a non-metal in Group VII and its atoms have one electron in their outer shells.
D Sodium is a metal in Group II and its atoms have one electron in their outer shells.

25 Which substance is added to the blast furnace to remove acidic impurities during the extraction of
iron?

A calcium silicate
B carbon monoxide
C coke
D limestone

© UCLES 2017 0653/21/O/N/17 [Turn over

638/693
Combined By Nesrine
10
2023-2017

26 P, Q, R and S are four gases found in clean air.

P is very unreactive.

Q makes up 21% of the air.

R makes up 78% of the air.

S is formed when fossil fuels are burned.

Which row is correct?

P Q R S

A argon nitrogen oxygen carbon dioxide


B argon oxygen nitrogen carbon dioxide
C carbon dioxide oxygen nitrogen argon
D carbon dioxide nitrogen oxygen argon

27 Which process is an example of thermal decomposition?

A cracking an alkane

B electrolysis of molten lead(II) bromide


C extraction of iron in a blast furnace
D fractional distillation of petroleum

© UCLES 2017 0653/21/O/N/17

639/693
Combined By Nesrine
11
2023-2017

28 The diagram is a speed-time graph for a moving object.

10.0
speed
m/s

0
0 2.0
time / s

What is the acceleration of the object and what distance does it travel in 2.0 s?

acceleration distance
m / s2 travelled / m

A 5.0 10
B 5.0 20
C 20 10
D 20 20

29 A piece of scientific equipment is taken on a space ship from Earth to a distant planet.

Which property or properties of the equipment must remain the same on the distant planet?

mass weight

A   key
B   = must be the same
C   = does not have to be the same
D  

30 A student stretches a steel spring by hanging a load on it. The measurements for the extension of
the spring are shown in the table.

load / N 1.0 2.0 3.0 4.0 5.0 6.0


extension / cm 0.5 1.0 1.5 2.0 2.5 3.0

What is the value for the spring constant k of the spring?

A 0.50 N / cm B 1.0 N / cm C 2.0 N / cm D 18 N / cm

© UCLES 2017 0653/21/O/N/17 [Turn over

640/693
Combined By Nesrine
12
2023-2017

31 A panel of solar cells is 15% efficient. The power supplied by the Sun to the panel is 40 kW.

What is the output power of the panel?

A 2.7 kW B 6.0 kW C 25 kW D 34 kW

32 When a liquid evaporates, which molecules escape and what happens, if anything, to the
temperature of the remaining liquid?

molecules temperature of
escaping remaining liquid
A less energetic decreases
molecules
B less energetic stays the same
molecules
C more energetic decreases
molecules
D more energetic stays the same
molecules

33 A teacher explains about transfer of thermal energy.

When air is ......X......, it becomes less dense and rises.


This helps to explain transfer of thermal energy by «...Y«... .

Which words complete gaps X and Y?

X Y

A cooled conduction
B cooled convection
C heated conduction
D heated convection

© UCLES 2017 0653/21/O/N/17

641/693
Combined By Nesrine
13
2023-2017

34 The diagram shows a section of a rope.

Four wave crests pass a point on the rope every second.

Each wave crest travels 80 cm in one second.

wave crest
20 cm
direction
5 cm of wave

What is the speed of the wave?

A 4.0 cm / s B 5.0 cm / s C 20 cm / s D 80 cm / s

35 The diagram shows a ray of light striking a plane mirror X.

Plane mirror Y is at 90° to mirror X.

mirror Y

NOT TO
SCALE

mirror X

60°

What is the angle of reflection at mirror Y?

A 30° B 60° C 90° D 120°

36 Electromagnetic waves are used to scan passengers’ luggage before they board an aeroplane.

Electromagnetic waves are also used in a television remote controller.

Which type of electromagnetic wave is used for each of these purposes?

scanning television
luggage remote controller

A radio waves infra-red waves


B radio waves ultraviolet waves
C X-rays infra-red waves
D X-rays ultraviolet waves

© UCLES 2017 0653/21/O/N/17 [Turn over

642/693
Combined By Nesrine
14
2023-2017

37 The diagram represents a wave in air. Molecules are closer together in region P than they are in
region Q.

region P region Q

What are the names of regions P and Q, and which type of wave is represented?

region P region Q type of wave

A compression rarefaction longitudinal


B compression rarefaction transverse
C rarefaction compression longitudinal
D rarefaction compression transverse

38 The resistance of a wire depends on its length and on its diameter.

Which row shows two changes that both increase the resistance of the wire?

change 1 change 2

A decrease the length decrease the diameter


B decrease the length increase the diameter
C increase the length decrease the diameter
D increase the length increase the diameter

© UCLES 2017 0653/21/O/N/17

643/693
Combined By Nesrine
15
2023-2017

39 The device Z in this circuit is designed to cut off the electricity supply automatically if too much
current flows.

What is device Z?

A a fuse
B a resistor
C a switch
D an ammeter

40 The diagram shows a circuit containing a 12 V battery, four identical resistors, an ammeter and a
voltmeter. Two values of current are shown.

12 V

A
3.0 A

3.0 A

What is the reading on the ammeter and what is the reading on the voltmeter?

reading on reading on
ammeter / A voltmeter / V

A 3.0 6.0
B 3.0 12
C 6.0 6.0
D 6.0 12

To avoid the issue of disclosure of answer-related information to candidates, all copyright acknowledgements are reproduced online in the Cambridge
International Examinations Copyright Acknowledgements Booklet. This is produced for each series of examinations and is freely available to download at
www.cie.org.uk after the live examination series.

© UCLES 2017 0653/21/O/N/17

644/693
Combined By Nesrine
2023-2017

Cambridge International Examinations


Cambridge International General Certificate of Secondary Education

COMBINED SCIENCE 0653/22


Paper 2 Multiple Choice (Extended) October/November 2017
45 minutes
Additional Materials: Multiple Choice Answer Sheet
Soft clean eraser
*4541115173*

Soft pencil (type B or HB is recommended)

READ THESE INSTRUCTIONS FIRST

Write in soft pencil.


Do not use staples, paper clips, glue or correction fluid.
Write your name, Centre number and candidate number on the Answer Sheet in the spaces provided
unless this has been done for you.
DO NOT WRITE IN ANY BARCODES.

There are forty questions on this paper. Answer all questions. For each question there are four possible
answers A, B, C and D.
Choose the one you consider correct and record your choice in soft pencil on the separate Answer Sheet.

Read the instructions on the Answer Sheet very carefully.

Each correct answer will score one mark. A mark will not be deducted for a wrong answer.
Any rough working should be done in this booklet.
A copy of the Periodic Table is printed on page 20.
Electronic calculators may be used.

This document consists of 17 printed pages and 3 blank pages.

IB17 11_0653_22/2RP
© UCLES 2017 [Turn over

645/693
Combined By Nesrine
2
2023-2017

1 Which characteristics help to define a living organism?

A diffusion, movement, respiration


B excretion, nutrition, sensitivity
C excretion, reproduction, transpiration
D growth, inspiration, nutrition

2 The diagram shows a palisade cell.

Which structure converts energy from light into chemical energy?

C
A

3 What is the role of microorganisms in the manufacture of yoghurt?

A to turn lactic acid into lactose sugar and lower the pH


B to turn lactic acid into lactose sugar and raise the pH
C to turn lactose sugar into lactic acid and lower the pH
D to turn lactose sugar into lactic acid and raise the pH

4 In which order does food pass through parts of the alimentary canal?

A oesophagus → colon → small intestine

B small intestine → oesophagus → rectum

C small intestine → rectum → anus

D stomach → colon → small intestine

© UCLES 2017 0653/22/O/N/17

646/693
Combined By Nesrine
3
2023-2017

5 These four conditions may be a result of malnutrition.

1 constipation
2 coronary heart disease
3 obesity
4 starvation

Which conditions are a direct result of an imbalance between energy intake and energy output?

A 1 and 2 B 1 and 4 C 2 and 3 D 3 and 4

6 What are the functions of root hairs?

uptake of ions uptake of sugar uptake of water

A   
B   
C   
D   

7 When we cut ourselves, blood comes out of the wound.

Which constituent of blood is most important in the formation of a blood clot?

A plasma
B platelets
C red blood cells
D white blood cells

8 What is the equation for aerobic respiration?

A 6CO2 + 6O2 → C6H12O6 + 6H2O

B 6H2O + 6CO2 → C6H12O6 + 6O2

C C6H12O6 + 6H2O → 6CO2 + 6O2

D C6H12O6 + 6O2 → 6CO2 + 6H2O

© UCLES 2017 0653/22/O/N/17 [Turn over

647/693
Combined By Nesrine
4
2023-2017

9 The depth and rate of breathing can be measured by a spirometer, and recorded in the form of a
graph.

Graph X shows the depth and rate of breathing of a person at rest.

graph X
4.0

3.5
volume
of air in 3.0
lungs / dm3
2.5

2.0
0 30
time / s

Which graph shows the depth and rate of breathing when the same person is running?

A B
4.0 4.0

3.5 3.5
volume volume
of air in 3.0 of air in 3.0
lungs / dm3 lungs / dm3
2.5 2.5

2.0 2.0
0 30 0 30
time / s time / s

C D
4.0 4.0

3.5 3.5
volume volume
of air in 3.0 of air in 3.0
lungs / dm3 lungs / dm3
2.5 2.5

2.0 2.0
0 30 0 30
time / s time / s

© UCLES 2017 0653/22/O/N/17

648/693
Combined By Nesrine
5
2023-2017

10 A student carried out an experiment to test geotropism.

A bean seed was pinned to a card with wet cotton wool, as shown.

card
wet cotton wool

bean seed
pin
gravity

Every day the card was turned 90° clockwise.

After a few days the student drew the results of the root growth.

Which diagram shows the student’s result?

A B

C D

11 During pregnancy, the fetus is contained within the amniotic sac. The amniotic sac contains
amniotic fluid.

What is the function of the amniotic fluid?

A It protects the fetus against knocks and bumps.


B It provides the fetus with oxygen and nutrients.
C It removes the fetal waste products.
D It supplies the fetus with blood.

© UCLES 2017 0653/22/O/N/17 [Turn over

649/693
Combined By Nesrine
6
2023-2017

12 The diagram shows a food web.

hawk

song bird

spider locust beetle

tree grass

Which statement about this food web is correct?

A Some of the energy from the grass eventually passes to the hawk.
B The producers get their energy from the soil.
C There are more carnivores shown than herbivores.
D There are six consumers shown.

© UCLES 2017 0653/22/O/N/17

650/693
Combined By Nesrine
7
2023-2017

13 Which graph shows the effect of large-scale deforestation on the changes in the concentrations
of oxygen and carbon dioxide in the air?

A B

O2

changes in changes in
concentration O2 concentration

CO2
CO2

time after deforestation time after deforestation

C D

O2
changes in O2 changes in
concentration concentration CO2
CO2

time after deforestation time after deforestation

14 The formulae of three substances are shown.

substance formula

methane CH4
water H2O
oxygen O2

Which statement is correct?

A Methane is made from five different types of atom.


B Methane, water and oxygen are molecules.
C Only methane and water are molecules.
D Oxygen is made from two different types of atom.

© UCLES 2017 0653/22/O/N/17 [Turn over

651/693
Combined By Nesrine
8
2023-2017

15 What is the correct sequence that takes place during fractional distillation?

A evaporate → condense → collect → heat

B evaporate → condense → heat → collect

C heat → condense → collect → evaporate

D heat → evaporate → condense → collect

16 Which substances react to produce a mixture of an element and a compound?

A copper oxide and carbon


B hydrochloric acid and sodium carbonate
C hydrogen and oxygen
D nitric acid and sodium hydroxide

17 The electronic structure of a sodium atom is 2,8,1.

The electronic structure of a sodium ion is 2,8.

Which statement is not correct?

A Sodium ions form metallic bonds.


B The electronic structure of a sodium ion is more stable than that of a sodium atom.
C The sodium atom loses one electron to become an ion.
D The sodium ion has a noble gas electronic structure.

18 The symbols for some ions are shown.

name of ion symbol

silver Ag+
nitrate NO3–
magnesium Mg2+
chloride Cl –

Which symbol equation is correct?

A AgNO3 + MgCl → AgCl + MgNO3

B Ag2NO3 + MgCl → Ag2Cl + MgNO3

C 2AgNO3 + MgCl2 → 2AgCl + Mg(NO3)2

D 2AgNO3 + Mg2Cl → 2AgCl + 2MgNO3

© UCLES 2017 0653/22/O/N/17

652/693
Combined By Nesrine
9
2023-2017

19 What is formed at the cathode during the electrolysis of aqueous copper chloride?

A chlorine
B copper
C hydrogen
D oxygen

20 The diagram shows gas X burning and heating a liquid.

liquid

flame

gas X

Which row is correct?

the burning of gas X


gas X
is exothermic

A hydrogen 
B hydrogen 
C oxygen 
D oxygen 

21 What is the effect of increasing the temperature on the collisions between reacting particles
during a chemical reaction?

number of collisions
energy of collisions
per second

A decreases decreases
B decreases increases
C increases decreases
D increases increases

© UCLES 2017 0653/22/O/N/17 [Turn over

653/693
Combined By Nesrine
10
2023-2017

22 The word equation for the reaction between hydrogen and copper oxide is shown.

hydrogen + copper oxide → copper + water

Which substance, shown in the word equation, is reduced in the reaction?

A copper
B copper oxide
C hydrogen
D water

23 Excess aqueous barium nitrate is added to dilute sulfuric acid to produce barium sulfate.

How is barium sulfate obtained from the reaction mixture?

A electrolysis
B evaporation
C filtration
D fractional distillation

24 Weather balloons are used to carry scientific instruments into the atmosphere.

Which gas is used to fill the balloons?

A argon
B helium
C krypton
D xenon

25 Which reaction does not take place in the blast furnace?

A Calcium carbonate decomposes to make calcium oxide.


B Carbon dioxide reacts with carbon to make carbon monoxide.
C Carbon monoxide reacts with iron oxide to make iron.
D Limestone reacts with iron oxide to make slag.

© UCLES 2017 0653/22/O/N/17

654/693
Combined By Nesrine
11
2023-2017

26 P, Q, R and S are four gases found in clean air.

P is very unreactive.

Q makes up 21% of the air.

R makes up 78% of the air.

S is formed when fossil fuels are burned.

Which row is correct?

P Q R S

A argon nitrogen oxygen carbon dioxide


B argon oxygen nitrogen carbon dioxide
C carbon dioxide oxygen nitrogen argon
D carbon dioxide nitrogen oxygen argon

27 In the fractional distillation of petroleum, fractions X and Y are removed at the positions shown.

fraction X

fraction Y

petroleum

Which row describes the molecular sizes and the intermolecular attractive forces in fractions X
and Y?

molecular sizes intermolecular attractive forces

A X larger than Y X greater than Y


B X larger than Y Y greater than X
C Y larger than X X greater than Y
D Y larger than X Y greater than X

© UCLES 2017 0653/22/O/N/17 [Turn over

655/693
Combined By Nesrine
12
2023-2017

28 The speed-time graph shown is for a bus travelling between stops.

Where on the graph is the acceleration of the bus greatest?

C
speed
B

A D
0
0 time

29 The table gives the volumes and masses of four objects.

Which object has the greatest density?

mass / g volume / cm3

A 5.4 2.0
B 13 3.0
C 15 6.0
D 18 5.0

30 A student stretches a steel spring by hanging a load on it. The measurements for the extension of
the spring are shown in the table.

load / N 1.0 2.0 3.0 4.0 5.0 6.0


extension / cm 0.5 1.0 1.5 2.0 2.5 3.0

What is the value for the spring constant k of the spring?

A 0.50 N / cm B 1.0 N / cm C 2.0 N / cm D 18 N / cm

31 A 600 W motor is 75% efficient. The motor is used to do 3600 J of useful work.

How long does it take the motor to do this work?

A 4.5 s B 6.0 s C 8.0 s D 24 s

© UCLES 2017 0653/22/O/N/17

656/693
Combined By Nesrine
13
2023-2017

32 Which description is correct for the molecules of a gas with a temperature that is rising?

force between average speed


molecules of molecules

A negligible decreasing
B negligible increasing
C strong decreasing
D strong increasing

33 The diagram shows a heater above a thermometer. The thermometer bulb is in the position
shown.

heater

air

thermometer thermometer bulb

Which row shows how the heat energy from the heater reaches the thermometer bulb?

conduction convection radiation

A no no yes
B no yes no
C no yes yes
D yes yes no

© UCLES 2017 0653/22/O/N/17 [Turn over

657/693
Combined By Nesrine
14
2023-2017

34 The diagram shows a section of a rope.

Four wave crests pass a point on the rope every second.

Each wave crest travels 80 cm in one second.

wave crest
20 cm
direction
5 cm of wave

What is the speed of the wave?

A 4.0 cm / s B 5.0 cm / s C 20 cm / s D 80 cm / s

35 Two plane mirrors are placed at 90° to each other. A ray of light strikes one mirror at an angle of
incidence of 60°.

Which diagram shows this ray and its path after reflection?

A B

60°
60° 60° 60°

C D

60° 60° 60°


60°

© UCLES 2017 0653/22/O/N/17

658/693
Combined By Nesrine
15
2023-2017

36 Electromagnetic waves are used to scan passengers’ luggage before they board an aeroplane.

Electromagnetic waves are also used in a television remote controller.

Which type of electromagnetic wave is used for each of these purposes?

scanning television
luggage remote controller

A radio waves infra-red waves


B radio waves ultraviolet waves
C X-rays infra-red waves
D X-rays ultraviolet waves

37 The diagram represents a wave in air. Molecules are closer together in region P than they are in
region Q.

region P region Q

What are the names of regions P and Q, and which type of wave is represented?

region P region Q type of wave

A compression rarefaction longitudinal


B compression rarefaction transverse
C rarefaction compression longitudinal
D rarefaction compression transverse

38 A piece of wire has electrical resistance.

The wire is stretched so that it becomes longer and thinner.

What, if anything, happens to its resistance?

A It could increase or decrease depending on how much it is stretched.


B It does not change because its smaller diameter cancels the effect of its greater length.
C It must decrease.
D It must increase.

© UCLES 2017 0653/22/O/N/17 [Turn over

659/693
Combined By Nesrine
16
2023-2017

39 The device Z in this circuit is designed to cut off the electricity supply automatically if too much
current flows.

What is device Z?

A a fuse
B a resistor
C a switch
D an ammeter

40 The diagram shows three resistors connected to a 12 V battery.

The current at two points in the circuit and the p.d. across one resistor are shown.

Another resistor is labelled R.

12 V
3.0 A

3.0 V R

2.0 A

What is the current in resistor R and what is the p.d. across resistor R?

current in p.d. across


resistor R / A resistor R / V

A 1.0 3.0
B 1.0 9.0
C 2.0 3.0
D 2.0 9.0

© UCLES 2017 0653/22/O/N/17

660/693
Combined By Nesrine
2023-2017

Cambridge International Examinations


Cambridge International General Certificate of Secondary Education

COMBINED SCIENCE 0653/23


Paper 2 Multiple Choice (Extended) October/November 2017
45 minutes
Additional Materials: Multiple Choice Answer Sheet
Soft clean eraser
*4058289354*

Soft pencil (type B or HB is recommended)

READ THESE INSTRUCTIONS FIRST

Write in soft pencil.


Do not use staples, paper clips, glue or correction fluid.
Write your name, Centre number and candidate number on the Answer Sheet in the spaces provided
unless this has been done for you.
DO NOT WRITE IN ANY BARCODES.

There are forty questions on this paper. Answer all questions. For each question there are four possible
answers A, B, C and D.
Choose the one you consider correct and record your choice in soft pencil on the separate Answer Sheet.

Read the instructions on the Answer Sheet very carefully.

Each correct answer will score one mark. A mark will not be deducted for a wrong answer.
Any rough working should be done in this booklet.
A copy of the Periodic Table is printed on page 20.
Electronic calculators may be used.

This document consists of 17 printed pages and 3 blank pages.

IB17 11_0653_23/3RP
© UCLES 2017 [Turn over

661/693
Combined By Nesrine
2
2023-2017

1 Which characteristics help to define a living organism?

A diffusion, movement, respiration


B excretion, nutrition, sensitivity
C excretion, reproduction, transpiration
D growth, inspiration, nutrition

2 The diagram shows a palisade cell.

Which structure converts energy from light into chemical energy?

C
A

3 Which substance is produced by respiration in microorganisms during yoghurt making?

A amino acids
B fatty acids
C hydrochloric acid
D lactic acid

4 The list shows chemicals that are important to a plant.

1 carbon dioxide
2 nitrates
3 oxygen
4 water

Which chemicals does a plant use in photosynthesis?

A 1, 2 and 4 B 1 and 2 only C 1 and 4 only D 3 and 4 only

© UCLES 2017 0653/23/O/N/17

662/693
Combined By Nesrine
3
2023-2017

5 In which order does food pass through parts of the alimentary canal?

A oesophagus → colon → small intestine

B small intestine → oesophagus → rectum

C small intestine → rectum → anus

D stomach → colon → small intestine

6 The graph shows the uptake of water by root hair cells over many hours during a day.

water uptake
by root hair cells

0
0 time

What could have caused the change in the rate of uptake at T?

A decrease in temperature
B decrease in humidity
C increase in light intensity
D increase in temperature

© UCLES 2017 0653/23/O/N/17 [Turn over

663/693
Combined By Nesrine
4
2023-2017

7 Which row correctly matches the cell to its function?

cell function

A blood clotting

B blood clotting

C oxygen transport

D oxygen transport

8 Dust particles and pathogens may be inhaled from the air. These can cause damage to the
airways if they are not removed.

Which features work together to remove them from the airways?

A cilia and enzymes


B mucus and cilia
C mucus and saliva
D saliva and enzymes

9 How does auxin cause a plant shoot to bend to the right?

A Cells elongate more on the left side of the shoot than on the right side.
B Cells elongate more on the right side of the shoot than on the left side.
C Cells shrink on the left side of the shoot.
D Cells shrink on the right side of the shoot.

© UCLES 2017 0653/23/O/N/17

664/693
Combined By Nesrine
5
2023-2017

10 The table shows features of four flowers.

Which row shows features of a typical wind-pollinated flower?

petals pollen grains shape of stigma


A large white petals large and hooked branched and feathery
and sweet scent
B large white petals small and light straight and enclosed
and sweet scent within the flower
C small green petals large and hooked straight and enclosed
and no scent within the flower
D small green petals small and light branched and feathery
and no scent

11 During pregnancy, the fetus is contained within the amniotic sac. The amniotic sac contains
amniotic fluid.

What is the function of the amniotic fluid?

A It protects the fetus against knocks and bumps.


B It provides the fetus with oxygen and nutrients.
C It removes the fetal waste products.
D It supplies the fetus with blood.

12 The diagram shows the trophic levels of a food chain.

producer herbivore carnivore carnivore


→ → →
1st trophic 2nd trophic 3rd trophic 4th trophic
level level level level

Why do most food chains not have more than four trophic levels?

A There are too many carnivores in the 3rd trophic level.


B There are too many herbivores in the 2nd trophic level.
C There is no energy transferred from the 2nd trophic level to the 3rd trophic level.
D There is not enough energy available to be transferred to a 5th trophic level.

© UCLES 2017 0653/23/O/N/17 [Turn over

665/693
Combined By Nesrine
6
2023-2017

13 The graph shows changes in the number of different species in the water flowing along a river.

At which point is untreated sewage released into the river?

number of
different
species

direction of
water flow
A B C D
distance along river

14 The formulae of three substances are shown.

substance formula

methane CH4
water H2O
oxygen O2

Which statement is correct?

A Methane is made from five different types of atom.


B Methane, water and oxygen are molecules.
C Only methane and water are molecules.
D Oxygen is made from two different types of atom.

© UCLES 2017 0653/23/O/N/17

666/693
Combined By Nesrine
7
2023-2017

15 Chromatography separates ink into different colours.

Which diagram shows how the apparatus is set up?

A B

beaker beaker
paper ink

ink paper

water water

C D

beaker beaker

paper paper

ink ink
water water

16 Which statement describes how sodium atoms and oxygen atoms combine to form sodium oxide,
Na2O?

A One sodium atom gains two electrons and two oxygen atoms lose one electron each.
B One sodium atom loses two electrons and two oxygen atoms gain one electron each.
C Two sodium atoms gain one electron each and one oxygen atom loses two electrons.
D Two sodium atoms lose one electron each and one oxygen atom gains two electrons.

© UCLES 2017 0653/23/O/N/17 [Turn over

667/693
Combined By Nesrine
8
2023-2017

17 Which dot-and-cross diagrams show the outer-shell electrons in molecules of water and of
carbon dioxide?

water carbon dioxide

A H O H O C O

B H O H O C O

C H O H O C O

D H O H O C O

18 The formulae of some ions are shown.

● ammonium, NH4+
● calcium, Ca2+
● nitrate, NO3–
● phosphate, PO43–

What is the formula of calcium nitrate and of ammonium phosphate?

calcium nitrate ammonium phosphate

A Ca(NO3)2 NH4(PO4)3
B Ca(NO3)2 (NH4)3PO4
C Ca2NO3 NH4(PO4)3
D Ca2NO3 (NH4)3PO4

© UCLES 2017 0653/23/O/N/17

668/693
Combined By Nesrine
9
2023-2017

19 Which row describes an ionic compound?

can be
melting point
electrolysed

A high no
B high yes
C low no
D low yes

20 The diagram shows gas X burning and heating a liquid.

liquid

flame

gas X

Which row is correct?

the burning of gas X


gas X
is exothermic

A hydrogen 
B hydrogen 
C oxygen 
D oxygen 

© UCLES 2017 0653/23/O/N/17 [Turn over

669/693
Combined By Nesrine
10
2023-2017

21 Dilute hydrochloric acid reacts with marble pieces to produce carbon dioxide.

The results of some experiments to investigate the rate of reaction are shown.

relative time taken to


size of
concentration of make 50 cm3 of
marble pieces
hydrochloric acid carbon dioxide / s

1 large 100
2 large 50
1 small 80
2 small 40

Which conclusion can be made from these results?

A When bigger marble pieces are used, the rate of reaction is greater.
B When smaller marble pieces are used, the rate of reaction is doubled.
C When the concentration is doubled, the rate of reaction is doubled.
D When the concentration is doubled, the rate of reaction is halved.

22 Copper is produced by heating copper oxide with carbon.

The word equation for this reaction is shown.

copper oxide + carbon → copper + carbon dioxide

Which statement explains why this is a redox reaction?

A Carbon dioxide contains oxygen.


B Carbon is a solid and carbon dioxide is a gas.
C Copper oxide is oxidised.
D Copper oxide loses oxygen and carbon gains oxygen.

23 Excess aqueous barium nitrate is added to dilute sulfuric acid to produce barium sulfate.

How is barium sulfate obtained from the reaction mixture?

A electrolysis
B evaporation
C filtration
D fractional distillation

© UCLES 2017 0653/23/O/N/17

670/693
Combined By Nesrine
11
2023-2017

24 What is a use for argon?

A as a fuel
B filling balloons
C providing an inert atmosphere
D the extraction of copper

25 Which diagram shows the arrangements of atoms in an alloy?

A B C D

– + – + – +
+ – + – + –
– + – + – +
+ – + – + –

26 P, Q, R and S are four gases found in clean air.

P is very unreactive.

Q makes up 21% of the air.

R makes up 78% of the air.

S is formed when fossil fuels are burned.

Which row is correct?

P Q R S

A argon nitrogen oxygen carbon dioxide


B argon oxygen nitrogen carbon dioxide
C carbon dioxide oxygen nitrogen argon
D carbon dioxide nitrogen oxygen argon

27 Which statement about the fractional distillation of petroleum is not correct?

A Fractions obtained from higher up the fractional distillation column have higher boiling point
ranges.
B Fractions obtained from lower down the fractional distillation column contain larger
molecules.
C Refinery gas is used for heating and cooking.
D Smaller molecules have weaker intermolecular attractive forces.

© UCLES 2017 0653/23/O/N/17 [Turn over

671/693
Combined By Nesrine
12
2023-2017

28 The diagram shows the speed-time graph for a car.

20
speed
m/s
10

0
0 5.0
time / s

What is the acceleration of the car?

A 2.0 m / s2 B 4.0 m / s2 C 50 m / s2 D 75 m / s2

29 Which conditions are necessary for an object to have weight?

must must be in a must be in an


have mass gravitational field electric field

A no yes no
B no yes yes
C yes no yes
D yes yes no

30 A student stretches a steel spring by hanging a load on it. The measurements for the extension of
the spring are shown in the table.

load / N 1.0 2.0 3.0 4.0 5.0 6.0


extension / cm 0.5 1.0 1.5 2.0 2.5 3.0

What is the value for the spring constant k of the spring?

A 0.50 N / cm B 1.0 N / cm C 2.0 N / cm D 18 N / cm

31 A motor is used to lift a load of 3000 N through a vertical height of 40 m in 2.0 minutes.

How much useful power does the motor produce?

A 1000 W B 9000 W C 60 000 W D 240 000 W

© UCLES 2017 0653/23/O/N/17

672/693
Combined By Nesrine
13
2023-2017

32 The molecules of a substance are far apart and moving at high speeds in straight lines. The
forces between them are negligible.

Two changes are now made to the substance.

change 1 The average speed of the molecules is reduced but they remain far apart.
change 2 The molecules move much closer together. There are much stronger forces
between them, but they can change places with each other.

What is the effect of each change?

change 1 change 2

A temperature decreases gas to liquid


B temperature decreases liquid to solid
C temperature increases gas to liquid
D temperature increases liquid to solid

33 The diagram shows an air-conditioning unit on the wall of a room. The unit draws in warm air
from the room and releases cold air into the room.

air-conditioning
unit

What happens to the cold air and what is the reason?

cold air reason

A falls it is less dense than warm air


B falls it is more dense than warm air
C rises it is less dense than warm air
D rises it is more dense than warm air

© UCLES 2017 0653/23/O/N/17 [Turn over

673/693
Combined By Nesrine
14
2023-2017

34 The diagram shows a section of a rope.

Four wave crests pass a point on the rope every second.

Each wave crest travels 80 cm in one second.

wave crest
20 cm
direction
5 cm of wave

What is the speed of the wave?

A 4.0 cm / s B 5.0 cm / s C 20 cm / s D 80 cm / s

35 A man stands a distance d in front of a plane mirror and views his own image in the mirror.

Is the image real or virtual, and what is the distance between the man and his image?

distance between
image
man and image

A real d
B real 2d
C virtual d
D virtual 2d

36 Electromagnetic waves are used to scan passengers’ luggage before they board an aeroplane.

Electromagnetic waves are also used in a television remote controller.

Which type of electromagnetic wave is used for each of these purposes?

scanning television
luggage remote controller

A radio waves infra-red waves


B radio waves ultraviolet waves
C X-rays infra-red waves
D X-rays ultraviolet waves

© UCLES 2017 0653/23/O/N/17

674/693
Combined By Nesrine
15
2023-2017

37 The diagram represents a wave in air. Molecules are closer together in region P than they are in
region Q.

region P region Q

What are the names of regions P and Q, and which type of wave is represented?

region P region Q type of wave

A compression rarefaction longitudinal


B compression rarefaction transverse
C rarefaction compression longitudinal
D rarefaction compression transverse

38 Which circuit is used to determine the resistance of the resistor R?

A B

V A

R R
A

C D

V A

R R
V

© UCLES 2017 0653/23/O/N/17 [Turn over

675/693
Combined By Nesrine
16
2023-2017

39 The device Z in this circuit is designed to cut off the electricity supply automatically if too much
current flows.

What is device Z?

A a fuse
B a resistor
C a switch
D an ammeter

40 The diagram shows a 12 V battery connected to a 2.0 Ω resistor, a 4.0 Ω resistor and resistor R.

The current at two points in the circuit and the p.d. across the 2.0 Ω resistor are shown.

12 V
6.0 A

2.0 Ω

2.0 A 4.0 Ω
6.0 V

What is the current in resistor R and what is the p.d. across resistor R?

current in p.d. across


resistor R / A resistor R / V

A 2.0 3.0
B 2.0 6.0
C 4.0 3.0
D 4.0 6.0

© UCLES 2017 0653/23/O/N/17

676/693
Combined By Nesrine
2023-2017

Cambridge International Examinations


Cambridge International General Certificate of Secondary Education

COMBINED SCIENCE 0653/02


*0123456789*

Paper 2 Multiple Choice (Extended) For Examination from 2017


SPECIMEN PAPER
45 minutes
Additional Materials: Multiple Choice Answer Sheet
Soft clean eraser
Soft pencil (type B or HB is recommended)

READ THESE INSTRUCTIONS FIRST

Write in soft pencil.


Do not use staples, paper clips, glue or correction fluid.
Write your name, Centre number and candidate number on the Answer Sheet in the spaces provided unless
this has been done for you.
DO NOT WRITE IN ANY BARCODES.

There are forty questions on this paper. Answer all questions. For each question there are four possible
answers A, B, C and D.
Choose the one you consider correct and record your choice in soft pencil on the separate Answer Sheet.

Read the instructions on the Answer Sheet very carefully.

Each correct answer will score one mark. A mark will not be deducted for a wrong answer.
Any rough working should be done in this booklet.
A copy of the Periodic Table is printed on page 18.
Electronic calculators may be used.

This
Thisdocument
documentconsists
consistsofofxx
17printed
printedpages
pagesand
andxx
1 blank pages.
page.

© UCLES 2015
2013 [Turn over

677/693
Combined By Nesrine
2023-2017
2

1 The width of the plant cell in the diagram is 30 mm when it is magnified by a microscope
(magnification shown in brackets).

(×1000)

What is the actual width of the cell?

A 0.003 mm B 0.03 mm C 0.3 mm D 30 mm

2 The graph shows how the rate of an enzyme-controlled reaction changes with temperature. What
explains the shape of the graph within the temperature range marked X?

rate of
reaction

temperature X

A The higher temperature breaks down the enzyme’s substrate.

B The higher temperature changes the shape of the enzyme.

C The higher temperature decreases the kinetic energy of the enzyme.

D The higher temperature helps the enzyme to function as a biological catalyst.

© UCLES 2015 0653/02/SP/17

678/693
Combined By Nesrine
2023-2017
3

3 A healthy plant has been in the light. A leaf is taken from it, decolourised and then tested with
iodine solution.

What colour does the iodine solution become?

A black

B brick red

C green

D pale blue

4 A man reduces the amount of salt, saturated fat and fibre in his diet.

How could these changes affect the risk of developing the following conditions?

constipation coronary heart disease obesity


A reduced risk increased risk reduced risk
B reduced risk reduced risk increased risk
C increased risk reduced risk reduced risk
D increased risk increased risk increased risk

5 The diagram shows a section through the human heart.

Which structure is the tricuspid valve?

D A

C B

© UCLES 2015 0653/02/SP/17 [Turn over

679/693
Combined By Nesrine
2023-2017
4

6 The table shows some of the features of respiration.

Which row in the table is correct for aerobic respiration?

amount of energy
occurs when releases carbon
released per
oxgyen is present dioxide
glucose molecule
A high yes always
B low yes sometimes
C high no sometimes
D low no always

7 One of the effects of tobacco smoke on the gas exchange system is that haemoglobin carries
oxygen around the body less efficiently.

Which component of tobacco smoke is responsible for this effect?

A carbon monoxide

B nicotine

C smoke particles

D tar

8 The diagram shows a plant that has been placed on its side. The shoot begins to grow upwards.

up
P

shoot

What causes the shoot to grow upwards?

A less auxin present at P and increased cell elongation at P

B more auxin present at P and increased cell elongation at P

C less auxin present at Q and increased cell elongation at Q

D more auxin present at Q and increased cell elongation at Q

© UCLES 2015 0653/02/SP/17

680/693
Combined By Nesrine
2023-2017
5

9 The diagram shows a section through a flower.

In which parts of the flower are pollen grains produced and received?

pollen grains pollen grains


produced received
A P S
B Q P
C R Q
D S R

10 Which row in the table correctly describes a female gamete compared with a male gamete?

number
size mobility
produced
A larger greater more mobile
B larger fewer less mobile
C smaller greater less mobile
D smaller fewer more mobile

© UCLES 2015 0653/02/SP/17 [Turn over

681/693
Combined By Nesrine
2023-2017
6

11 A food chain is shown below. The numbers show the amount of energy, measured in kJ, that
passes from one organism to another.

500 kJ 75 kJ
grass rabbit fox

Calculate how much energy is lost from this food chain at X.

A 25 kJ B 75 kJ C 425 kJ D 575 kJ

12 The diagram shows a food web.

fox

hawk snake shrew

bird frog spider

beetle

wood beetle slug caterpillar

tree daisy grass

How many types of consumer and how many types of producer are shown in this food web?

types of types of
consumer producer
A 3 3
B 3 14
C 11 3
D 11 14

© UCLES 2015 0653/02/SP/17

682/693
Combined By Nesrine
2023-2017
7

13 Which of these measures helps to reduce the effects of acid rain?

A avoiding the use of non-recyclable plastics

B reducing methane emissions in industry and agriculture

C removing sulfur dioxide from power station waste gases

D using alkaline fertilisers on fields

14 Which method of separation can be used to obtain pure water from aqueous potassium chloride?

A chromatography

B crystallisation

C distillation

D filtration

15 Which row in the table correctly describes the four substances?

sodium
air brass iron
chloride
A compound compound element mixture
B compound element mixture mixture
C mixture mixture element compound
D element mixture compound element

16 Magnesium fluoride, MgF2, is an ionic compound.

What are the electronic structures of the magnesium ion and the fluoride ion?

magnesium ion fluoride ion


A 2,8 2,6
B 2,8 2,8
C 2,8,1 2,6
D 2,8,1 2,8

© UCLES 2015 0653/02/SP/17 [Turn over

683/693
Combined By Nesrine
2023-2017
8

17 What is the dot-and-cross diagram for ethene?

H H H H
C C C C
H H H H
A B

H H H H
H C C H H C C H
H H H H
C D

18 Hexane, C6H14, burns in an excess of oxygen, forming carbon dioxide and water.

What is the equation for this reaction?

A C6H14 + 9O2 → 6CO2 + 7H2O


B C6H14 + 19O2 → 12CO2 + 14H2O

C 2C6H14 + 19O2 → 6CO2 + 7H2O

D 2C6H14 + 19O2 → 12CO2 + 14H2O

© UCLES 2015 0653/02/SP/17

684/693
Combined By Nesrine
2023-2017
9

19 Copper(II) bromide solution can be electrolysed in the same way as copper(II) chloride solution.

switch

inert inert
electrode electrode

copper(II) bromide
solution

Which row in the table correctly describes the products at the electrodes?

product at product at
anode cathode
A brown liquid brown-red solid
B brown-red solid brown liquid
C colourless gas brown-red solid
D silvery solid colourless gas

20 Which statement describes an endothermic reaction?

A Chemical energy is transformed to heat energy and the temperature decreases.

B Chemical energy is transformed to heat energy and the temperature increases.

C Heat energy is transformed to chemical energy and the temperature decreases.

D Heat energy is transformed to chemical energy and the temperature increases.

© UCLES 2015 0653/02/SP/17 [Turn over

685/693
Combined By Nesrine
2023-2017
10

21 Marble and chalk are two forms of calcium carbonate.

The diagram shows equal masses of lumps of marble and powdered chalk placed in dilute
hydrochloric acid.

dilute
hydrochloric acid
marble powdered
lumps chalk

The marble takes longer than the chalk to dissolve in the acid.

Why is this?

A Marble is more reactive than chalk.

B Marble is more soluble than chalk.

C The marble has the smaller surface area.


D The marble is more basic.

22 Metal X reacts rapidly with cold water.

Metal Y does not react with dilute hydrochloric acid.

Metal Y is obtained from its oxide by heating the oxide with carbon.

Which row in the table shows the more reactive metal and the type of reaction that metal Y oxide
undergoes when it is heated with carbon?

more reactive type of reaction


metal of metal oxide
A X oxidation
B X reduction
C Y oxidation
D Y reduction

© UCLES 2015 0653/02/SP/17

686/693
Combined By Nesrine
2023-2017
11

23 The table shows the properties of four substances.

Which substance is an alkali?

solubility in reaction with an


water acid
A insoluble reacts
B insoluble does not react
C soluble reacts
D soluble does not react

24 Which row in the table describes the physical state of some of the Group VII elements at room
temperature?

chlorine bromine iodine


A gas gas liquid
B gas liquid solid
C liquid liquid gas
D liquid solid solid

25 P, Q, R and S are four metals.

P reacts rapidly with dilute acid, but it slowly forms bubbles of gas in cold water.

S reacts slowly with dilute acid, but it displaces Q from an aqueous solution of its salt.

R reacts rapidly with cold water.

What is the order of reactivity of the metals?

least reactive most reactive


A Q S P R
B R P S Q
C R S P Q
D S Q P R

© UCLES 2015 0653/02/SP/17 [Turn over

687/693
Combined By Nesrine
2023-2017
12

26 Which row in the table describes the method of extraction of aluminium, and the reason for using
this method?

method of
reason
extraction
A heat with carbon aluminium is less reactive than carbon
B heat with carbon aluminium is more reactive than carbon
C electrolysis aluminium is more reactive than carbon
D electrolysis aluminium is resistant to corrosion

27 Petroleum is separated by fractional distillation.

Which row in the table describes the properties of the compounds collected at the top of the
fractionating column?

intermolecular
boiling point molecular size
forces
A high large strong
B high small weak
C low large strong
D low small weak

28 The speed / time graph for a car journey is shown.

speed 2

3
1 4

0
0 time

During which two parts of the journey is the car moving at constant speed?

A 1 and 3 B 1 and 5 C 2 and 4 D 3 and 5

© UCLES 2015 0653/02/SP/17

688/693
Combined By Nesrine
2023-2017
13

29 The strength of the gravitational field on the Moon is less than the strength of the gravitational field
on Earth.

An object has mass M and weight W on the Moon.

What is the mass and the weight of the object on Earth?

mass weight
A more than M W
B more than M more than W
C M W
D M more than W

30 The equation for Hooke’s Law relates the extension of a spring to the load applied to it.

In an experiment, loads are applied to a spring and the spring extends. The table shows the
results.

load/N 0 12 24 36 48 60 72
length of spring/cm 15 18 21 24 27 31 36

What is the value of the constant k for this spring, and has the spring been loaded past its limit of
proportionality?

spring constant k loaded past limit of


N / cm proportionality?
A 0.67 no
B 0.67 yes
C 4.0 no
D 4.0 yes

31 A ball rolls along a frictionless, horizontal track at a speed of 4.0 m / s. It reaches a sloping section
of the track and continues to roll up the slope.

4.0 m / s

What is the maximum vertical height that the ball reaches up the slope?

The acceleration of free fall g is 10 m / s2.

A 0.20 m B 0.40 m C 0.80 m D 2.5 m

© UCLES 2015 0653/02/SP/17 [Turn over

689/693
Combined By Nesrine
2023-2017
14

32 A gas is trapped in a sealed container of constant volume. The gas is heated.

What effect does this have on the gas molecules?

A The average distance between the molecules increases.

B The average mass of the molecules increases.

C The molecules expand.

D The molecules move more quickly.

33 Benzene and glycerine are two substances.

The table shows the melting point and the boiling point of benzene and of glycerine.

melting point / °C boiling point / °C


benzene 5.4 80

glycerine 18 290

At which temperature will both benzene and glycerine be liquid?

A 10 °C B 50 °C C 100 °C D 150 °C

34 The diagram shows an ice cube surrounded by air. The ice cube cools the air around it. This
cooling changes the density of the air and causes the air to move.

ice cube

air

Which row in the table shows the change in density of the air and the direction in which the air
moves?

density change direction of movement


A decreases downwards
B decreases upwards
C increases downwards
D increases upwards

© UCLES 2015 0653/02/SP/17

690/693
Combined By Nesrine
2023-2017
15

35 A water wave with a wavelength of 2.0 cm moves a distance of 900 cm in 1.0 minute.

What is its frequency?

A 7.5 Hz B 30 Hz C 450 Hz D 1800 Hz

36 Which electromagnetic waves are found immediately either side of the visible region of the
electromagnetic spectrum?

A infra-red and ultra-violet

B microwaves and infra-red

C microwaves and X-rays

D ultra-violet and X-rays

37 Two sound waves, P and Q, are displayed on an oscilloscope. The settings on the oscilloscope
are the same for P and Q.

P Q

Which statement correctly compares the pitch and the loudness of the two sounds?

A P has a higher pitch and is louder than Q.

B P has a higher pitch and is quieter than Q.

C P has a lower pitch and is louder than Q.

D P has a lower pitch and is quieter than Q.

38 Sound travels in liquids and solids.

Which row in the table shows the approximate speed of sound in a liquid, and the approximate
speed of sound in a solid?

speed in liquid speed in solid


m/s m/s
A 340 340
B 340 1500
C 1500 5000
D 5000 1500

© UCLES 2015 0653/02/SP/17 [Turn over

691/693
Combined By Nesrine
2023-2017
16

39 The circuit shows a current I in a resistor of resistance R.

3.0 V

Which row in the table shows possible pairs of values of I and R?

I /A R/Ω

A 1.5 1.5
B 1.5 2.0
C 6.0 2.0
D 4.0 12.0

40 The circuit shown contains four labelled switches.

When all the switches are closed, one switch can be opened without affecting the brightness of
either lamp.

Which switch is this?

© UCLES 2015 0653/02/SP/17

692/693
The Periodic Table of Elements
Group
I II III IV V VI VII VIII

© UCLES 2018
1 2

H He
hydrogen helium
Key 1 4
3 4 atomic number 5 6 7 8 9 10

Li Be atomic symbol B C N O F Ne
lithium beryllium name boron carbon nitrogen oxygen fluorine neon
7 9 relative atomic mass 11 12 14 16 19 20
11 12 13 14 15 16 17 18
Na Mg Al Si P S Cl Ar
sodium magnesium aluminium silicon phosphorus sulfur chlorine argon
23 24 27 28 31 32 35.5 40
19 20 21 22 23 24 25 26 27 28 29 30 31 32 33 34 35 36
K Ca Sc Ti V Cr Mn Fe Co Ni Cu Zn Ga Ge As Se Br Kr
potassium calcium scandium titanium vanadium chromium manganese iron cobalt nickel copper zinc gallium germanium arsenic selenium bromine krypton
39 40 45 48 51 52 55 56 59 59 64 65 70 73 75 79 80 84
37 38 39 40 41 42 43 44 45 46 47 48 49 50 51 52 53 54

Rb Sr Y Zr Nb Mo Tc Ru Rh Pd Ag Cd In Sn Sb Te I Xe
rubidium strontium yttrium zirconium niobium molybdenum technetium ruthenium rhodium palladium silver cadmium indium tin antimony tellurium iodine xenon
85 88 89 91 93 96 – 101 103 106 108 112 115 119 122 128 127 131
16

55 56 57–71 72 73 74 75 76 77 78 79 80 81 82 83 84 85 86
lanthanoids

693/693
Cs Ba Hf Ta W Re Os Ir Pt Au Hg Tl Pb Bi Po At Rn

0653/23/O/N/18
caesium barium hafnium tantalum tungsten rhenium osmium iridium platinum gold mercury thallium lead bismuth polonium astatine radon
133 137 178 181 184 186 190 192 195 197 201 204 207 209 – – –
87 88 89–103 104 105 106 107 108 109 110 111 112 114 116
actinoids
Fr Ra Rf Db Sg Bh Hs Mt Ds Rg Cn Fl Lv
francium radium rutherfordium dubnium seaborgium bohrium hassium meitnerium darmstadtium roentgenium copernicium flerovium livermorium
– – – – – – – – – – – – –

57 58 59 60 61 62 63 64 65 66 67 68 69 70 71
lanthanoids La Ce Pr Nd Pm Sm Eu Gd Tb Dy Ho Er Tm Yb Lu
lanthanum cerium praseodymium neodymium promethium samarium europium gadolinium terbium dysprosium holmium erbium thulium ytterbium lutetium
139 140 141 144 – 150 152 157 159 163 165 167 169 173 175
89 90 91 92 93 94 95 96 97 98 99 100 101 102 103
actinoids Ac Th Pa U Np Pu Am Cm Bk Cf Es Fm Md No Lr
actinium thorium protactinium uranium neptunium plutonium americium curium berkelium californium einsteinium fermium mendelevium nobelium lawrencium
– 232 231 238 – – – – – – – – – – –
2023-2017

The volume of one mole of any gas is 24 dm3 at room temperature and pressure (r.t.p.).
Combined By Nesrine

You might also like